Superpage
Bone marrow neoplastic

Authors: Nadine Aguilera, M.D., Raniah Al Amri, M.D., Zubaidah Al-Jumaili, M.D., Yazan Alhalaseh, M.D., Haley Amoth, M.D., Barina Aqil, M.D., Ahmed Samire Arfa, M.D., Jacob Armstrong, M.D., Meenakshi Garg Bansal, M.D., Rose Beck, M.D., Ph.D., Puneet Bedi, M.D., Marie Calaminici, M.D., Ph.D., Jui Choudhuri, M.D., Jessica Corean, M.D., Elizabeth Courville, M.D., Genevieve M. Crane, M.D., Ph.D., Ivan De La Riva-Morales, M.D., Courtney D. DiNardo, M.D., Arti Easwar, M.D., Mostafa Elhodaky, M.D., Ph.D., Julie Feldstein, M.D., Franklin Fuda, D.O., Ramya Gadde, M.D., Sharon Koorse Germans, M.D., Julia T. Geyer, M.D., Mark Girton, M.D., Swati Gite, M.D., Benjamin Graham, M.D., Sean Gu, M.D., Ph.D., Rita Gupta, M.D., Srishti Gupta, M.B.B.S., M.D., Christopher S. Hale, M.D., Hans Magne Hamnvåg, M.D., Mowafak Hamodat, M.B.Ch.B., M.Sc., Zarrin Hossein-Zadeh, M.D., Morgan Hrones, M.D., S. David Hudnall, M.D., Nourhan Ibrahim, M.D., Giovanni Insuasti-Beltran, M.D., Natasha Iranzad, M.D., Jesse Manuel Jaso, M.D., Sarvenaz Karamooz, M.D, Kristin Karner, M.D., Hatem Kaseb, M.D., Ph.D., M.P.H., Pallavi Khattar, M.D., Joseph Khoury, M.D., Ameet R. Kini, M.D., Ph.D., Shajo Kunnath-Velayudhan, M.B.B.S., M.M.S.T., Jingwei Li, M.D., Ph.D., Huifei Liu, M.D., Ph.D., Yen-Chun Liu, M.D., Ph.D., Sanam Loghavi, M.D. , Brenda Mai, M.D., Sucheta Malik, M.B.B.S., Rashmi Manur, M.D., M.P.H., Etan Marks, D.O., Daniela Mihova, M.D., Kamran M. Mirza, M.D., Ph.D., Angeli Mittal, M.D., William Patrick Morrow, M.D., Monika Nageshwar, M.B.B.S., M.D., Lynh Nguyen, M.D., Sara Niyazi, D.O., Oluwatobi "Tobi" Odetola, M.D., Nagehan Pakasticali, M.D., Raj Patel, M.D., Nat Pernick, M.D., Anamarija M. Perry, M.D., Maryam F. Raouf, M.D., Alexander Reese, D.O., Hamzah Rehan, D.O., Daniel Rivera, M.D., Mark Russell, M.D., Karan Saluja, M.D., Valentina Sangiorgio, M.D., Nikhil Sangle, M.D., Vignesh Shanmugam, M.D., Yang Shi, M.D., Ph.D., Alexa J. Siddon, M.D., Zeba N. Singh, M.B.B.S., M.D., Madina Sukhanova, Ph.D., Narittee Sukswai, M.D., Lee Bentley Syler, M.D., Zhenya Tang, M.D., Ph.D., Meryem Terzioglu, M.D., Beenu Thakral, M.D., Patricia Tsang, M.D., M.B.A., Tayler A. van den Akker, M.D., Daniela Vulpe, B.S., Ritika Walia, M.D., M.B.B.S., Maxwell Walters-Smith, B.S., B.A., Endi Wang, M.D., Ph.D., Mariko Yabe, M.D., Ph.D., Cecilia C.S. Yeung, M.D., Ismail Elbaz Younes, M.D., Ling Zhang, M.D., Shuchi Zinzuwadia, B.S., Shweta S. Zinzuwadia, B.S.
Resident / Fellow Advisory Boards: Frido Bruehl, M.D., Mario L. Marques-Piubelli, M.D.
Editorial Board Members: Elizabeth Courville, M.D., Genevieve M. Crane, M.D., Ph.D., Julie Feldstein, M.D., Mario L. Marques-Piubelli, M.D., Roberto N. Miranda, M.D., Anamarija M. Perry, M.D., Alexa J. Siddon, M.D., Patricia Tsang, M.D., M.B.A.
Deputy Editors-in-Chief: Genevieve M. Crane, M.D., Ph.D., Patricia Tsang, M.D., M.B.A.
Editor-in-Chief: Debra L. Zynger, M.D.

Copyright: 2001-2024, PathologyOutlines.com, Inc.

HemePath related: Jobs, Fellowships, Conferences, Cases, CME, Board Review, What's New

Related chapters: Bone marrow nonneoplastic, Hematology, Lymphoma & related disorders, Lymph nodes and spleen, nonlymphoid

Editorial Board oversight: Alexa J. Siddon, M.D. (last reviewed April 2023), Genevieve M. Crane, M.D., Ph.D. (last reviewed December 2022)
Page views in 2024 to date: 42

AMKL M7
Definition / general
  • Acute megakaryoblastic leukemia (AMKL, M7)
  • Up to 10% of AML in children, 5% or less of adult AML (Orphanet (May 2004): Acute megakaryoblastic leukemia [Accessed 6 April 2018])
  • See also Myeloid leukemia associated with Down syndrome
  • Associated with marrow fibrosis due to megakaryoblast secretion of fibrogenic cytokines, which makes marrow aspiration difficult
  • In adults, median age 57 years, 59% have prior hematologic disorder or myelodysplastic syndrome (Blood 2006;107:880)
  • 19% had prior chemotherapy, classify now as AML-MRC (myelodysplasia related changes) or t-AML
  • Survival: poor, median overall survival is 6 months
  • Peripheral blood: often contains micromegakaryocytes and atypical platelets
  • Down Syndrome (DS): 150x increased risk of AML compared to non-Down children age 0 - 4 years; 70% are AML M7 compared to 3 - 6% in non-Down children
  • DS children ages 0 - 3 years: ALL vs AML risk is 1:1.2 compared to 4:1 for non-DS children
Clinical features
  • Thrombocytopenia, may have thrombocytosis, dysplastic features in neutrophils, erythroids, megakaryocytes and platelets
  • Infrequent hepatosplenomegaly
  • Associated with germ cell tumors in young boys
Diagnosis
  • WHO 2008: 20%+ blasts
  • 50%+ blasts of megakaryocytic lineage are present in bone marrow
  • Must exclude AML-MRC (myelodysplasia related changes), AML with t(1;22)(p13;q13); inv(3)(q21;q26.2); t(3;3)(q21;q26.2) and Down syndrome related
  • Megakaryocytic lineage is based on CD41+, CD61+ or positive platelet peroxidase reaction on EM
Case reports
Microscopic (histologic) description
  • Megakaryoblasts (often better morphology on biopsy than smear) are medium / large cells with blue vacuolated, agranular, eosinophilic cytoplasm containing fine granules, cytoplasmic projections (blebs and pseudopods) resembling platelets, irregular cytoplasmic borders and cytoplasmic zoning; may occur in clusters
  • Nuclei are round or slightly indented with finely reticular, dense chromatin and 1 - 3 nucleoli
  • Myelofibrosis or increased marrow reticulin is common; may also have small lymphoid-like blasts
Microscopic (histologic) images

AFIP images
Missing Image

Abundant cytoplasm

Missing Image

Promegakaryocytes and large blasts

Missing Image

Touch prep shows 3 blasts

Missing Image

Extensive infiltration by blasts


Missing Image

Marked proliferation of megakaryocytes

Missing Image Missing Image

Reticulin stain shows marked increase in reticulin fibers

Missing Image

PAS+



Images hosted on other servers:
Missing Image

Bone marrow
biopsy: extensive
infiltration
by blasts

Missing Image

CD34: red, podocalyxin: brown

Cytology images

Images hosted on other servers:
Missing Image Missing Image

FNA and CSF

Electron microscopy description
  • Megakaryoblasts have demarcation membranes and “bulls-eye” alpha granules with peroxidase activity in nuclear envelope and endoplasmic reticulum, but not in granules and Golgi complex
Differential diagnosis

AML (megakaryoblastic) with t(1;22)(p13.3;q13.1); RBM15::MRTF1
Definition / general
Essential features
  • Rare subtype of AML with recurrent genetic abnormalities and differentiation along the megakaryocytic lineage
  • Defined by t(1:22)(p13.3;q13.1); RBM15-MKL1
  • Blasts are positive for platelet specific markers CD41a+, CD42b+, CD61+
  • Occurs most commonly de novo in infants and young children (aged ≤ 3 years) without Down syndrome
  • Severe disease often with poor prognosis
Terminology
  • Acute myeloid leukemia (megakaryoblastic) with t(1;22)(p13.3;q13.1); RBM15-MKL1
  • AMKL with t(1:22)(p13.3;q13.1); RBM15-MKL1
  • RBM15 and MKL1 are also referred to as OTT and MAL, respectively
  • Abbreviations RBM15, MKL1, OTT and MAL stand for RNA binding motif protein 15, megakaryoblastic leukemia 1, OneTwoTwo and megakaryoblastic acute leukemia
ICD coding
  • ICD-O: 9911/3 - Acute myeloid leukemia (megakaryoblastic) with t(1;22)(p13;q13); RBM15-MKL1
Epidemiology
Sites
  • Bone marrow, spleen and liver
  • Lymph node involvement can also occur
Etiology
  • Postulated normal counterpart: myeloid progenitor cell with predominant megakaryocytic differentiation
  • RBM15-MKL1 fusion gene arising from the t(1;22)(p13.3;q13.1) is thought to be the initiating event (Blood 2015;126:943, Nat Genet 2001;28:220)
  • RBM15 encodes a protein with RNA recognizing motifs and a split end paralogue and orthologue C terminal (SPOC) domain that interact with the SMRT and NCoR corepressor complexes and the transcription factor downstream of Notch signaling named RBPJ (EMBO J 2002;21:5417, Genes Dev 2003;17:1909)
  • Product of MKL1 acts as a transcriptional coactivator of serum response factor (SRP), a transcription factor that regulates genes involved in cell growth, proliferation, differentiation and genes controlling the actin cytoskeleton (Blood 2010;116:1942)
  • Fusion of RBM15 and MKL1 may influence chromatin organization, differentiation induced by the HOX pathway and extracellular signaling pathways (Proc Natl Acad Sci USA 2001;98:5776)
Clinical features
Diagnosis
  • Bone marrow biopsy
  • Cytogenetics
  • Molecular genetics
  • Immunophenotyping
  • Karyotype analysis showing evidence of t(1;22)(p13.3:q13.1) or molecular genetic evidence demonstrating RBM15-MKL1 fusion
Prognostic factors
Case reports
Treatment
  • Chemotherapy (combination of an anthracycline and cytarabine)
Microscopic (histologic) description
  • Mixture of small and large megakaryoblasts may be found in the bone marrow and peripheral blood together with more undifferentiated blast cells with high N:C ratio
  • Megakaryoblasts are most often medium sized to large (12 - 18 μm)
    • Nucleus is round, slightly irregular / indented with fine reticular chromatin and 1 - 3 nucleoli
    • Cytoplasm is basophilic, often agranular, with distinct blebs or pseudopod formation
  • Bone marrow is normocellular to hypercellular, with varying degrees of reticulin and fibrosis
  • Establishing the presence of ≥ 20% blasts on bone marrow aspirate may prove difficult due to extensive fibrosis; making correlation with bone marrow biopsy findings crucial
  • Micromegakaryocytes are commonly present
  • Dysplastic erythroid and granulocytic cells are usually not seen
Positive stains
  • Platelet glycoproteins:
    • CD41 (glycoprotein IIb / IIIa)
    • CD61 (glycoprotein IIIa)
    • CD42b (glycoprotein Ib)
  • Myeloid markers may also be positive
  • CD36
Negative stains
Flow cytometry description
  • Blasts are CD41a+, CD42b+, CD61+
Molecular / cytogenetics description
  • Translocation t(1;22)(p13.3;q13.1) is visible on karyotype (Nat Genet 2001;28:220)
  • Detection of RBM15-MRTFA (MKL1) fusion transcript via RT-PCR
Sample pathology report
  • Right posterior iliac crest, core biopsy, aspirate smear, touch imprint and clot particle:
    • Acute myeloid leukemia (megakaryoblastic) with t(1;22)(p13.3;q13.1); RBM15-MKL1 (see comment)
    • Comment: The bone core biopsy demonstrates extensive replacement of marrow by blast cells in areas arranged into clusters and aggregates surrounded by fibrotic stroma. The marrow aspirate smears reveal a heteromorphic population of blasts. Blast phenoptyping with immunohistochemical stains and flow cytometry analysis reveal positivity for markers of megakaryocytic maturation. Cytogenetics evaluation reveals t(1;22)(p13.3;q13.1), confirming above diagnosis.
Differential diagnosis
Board review style question #1
What is the resulting fusion gene product in t(1;22)(p13.3;q13.1) that is recurrent in one of the acute leukemic entities?

  1. DEK-NUP214
  2. KMT2A-MLLT3
  3. PML-RARA
  4. RBM15-MKL1
  5. RUNX1-RUNX1T1
Board review style answer #1
Board review style question #2
Which of the following populations have the highest incidence of acute megakaryoblastic leukemia (AMKL) with t(1;22)(p13.3;q13.1)?

  1. Children aged 5 - 10
  2. Elderly females
  3. Elderly males
  4. Infants with Down syndrome
  5. Infants without Down syndrome
Board review style answer #2
E. Infants without Down syndrome

Comment Here

Reference: AML (megakaryoblastic) with t(1;22)(p13.3;q13.1)
Board review style question #3
Which of the following markers is mostly likely to be negative in AMKL with t(1;22)(p13.3;q13.1)?

  1. CD13
  2. CD36
  3. CD41
  4. CD61
  5. Myeloperoxidase
Board review style answer #3

AML M5
Definition / general
Diagnosis
  • 80% or more nonerythroid bone marrow cells are monocyte lineage (monoblasts, promonocytes and monocytes)
  • A minor neutrophil component < 20%
Enzyme cytochemistry
  • Monoblast granules and monocytes are strongly positive for nonspecific esterase and lysozyme, but negative for myeloperoxidase
  • More mature monocyte lineage cells may be weakly myeloperoxidase positive
  • Note: if NSE negative, confirm monocyte lineage with immunostains
Positive stains
Molecular / cytogenetics description
  • 11q23 translocations in 19%, trisomy 8 in 17%

AML M6
Definition / general
  • Acute erythroid leukemia (AML-M6)
  • A neoplastic proliferation of immature cells (undifferentiated or proerythroblastic in appearance) committed exclusively to the erythroid lineage (> 80% of the bone marrow cells are erythroid, with ≥ 30% proerythroblasts), with no evidence of a significant myeloblastic component (Swerdlow: WHO Classification of Tumours of Haematopoietic and Lymphoid Tissues, 4th Edition, 2017)
  • Note: per WHO 2016, the category of acute erythroleukemia M6a no longer exists; cases previously classified as M6a are either myelodysplastic syndrome (MDS) or AML depending on the myeloblast percentage in bone marrow or peripheral blood
Essential features
  • A rare aggressive acute myeloid leukemia (Am J Hematol 2017;92:292)
  • Erythroid precursors > 80% and proerythroblasts ≥ 30% of the bone marrow cells
  • Myeloblasts are not increased
  • Therapy related cases should be diagnosed as therapy related myeloid neoplasms
Terminology
  • Acute myeloid leukemia, M6 type
  • Acute erythroid leukemia
  • Erythroleukemia
  • Pure erythroid leukemia M6b
  • Di Guglielmo disease
  • Acute erythraemic myelosis
  • Acute erythraemia
ICD coding
  • ICD-10: C94.00 - acute erythroid leukemia, not having achieved remission
Epidemiology
Sites
Clinical features
Diagnosis
  • Erythroblasts > 80% in bone marrow or peripheral blood with proerythroblasts ≥ 30%
  • Myeloblasts are not increased (< 5%)
  • For extramedullary mass lesion, sheets of erythroid precursor cells with proerythroblasts ≥ 30%
Prognostic factors
Case reports
Treatment
Microscopic (histologic) description
  • Bone marrow biopsy: sheets or clusters of immature cells, tumor cells may show intrasinusoidal growth pattern (Am J Hematol 2017;92:292)
  • Bone marrow aspirate: erythroid precursors > 80% and proerythroblasts ≥ 30% of bone marrow cells
    • Proerythroblasts have round nuclei, fine chromatin and one or more prominent nucleoli, deeply basophilic cytoplasm with vacuoles, often agranular
    • Ring sideroblasts are common
    • Myeloblasts are not increased
    • Dysmegakaryopoiesis is common and dysgranulopoiesis is infrequent
  • Mass in tissue: poorly differentiated tumor cells (Haematologica 2020 Jan 16 [Epub ahead of print])
Microscopic (histologic) images

Contributed by Huifei Liu, M.D., Ph.D.

Erythroblastic sarcoma

E-cadherin

CD43


AFIP images

Bone marrow smear (Wright-Giemsa)

Cytology images

Contributed by Huifei Liu, M.D., Ph.D.

CSF erythroblasts

Peripheral smear description
Peripheral smear images

Contributed by Huifei Liu, M.D., Ph.D.

Circulating erythroblasts

Positive stains
  • Erythroid specific markers: E-cadherin (positive in majority cases), hemoglobin A and glycophorin A (usually positive but can be negative in poorly differentiated erythroblasts) (Am J Hematol 2017;92:292)
  • Not lineage-specific markers: CD36, CD43, CD71, CD117, GATA1, Ferritin H
    • These markers are unusually positive; however, they can be positive in other type of hematopoietic neoplasms
    • CD36 is commonly positive on monocytic leukemia and acute megakaryocytic leukemia
    • CD43 is positive in most AML, T-ALL as well as some B-ALL; it is a very useful marker for extramedullary poorly differentiated tumor to confirm hematopoietic lineage
    • CD71 can be positive in other leukemic blasts although its expression levels in PEL should be very bright
    • CD117 is positive in most AML and some T-ALL
    • GATA1 is a transcription factor for erythroid development and it is positive in both PEL and acute megakaryocytic leukemia (Am J Clin Pathol 2017;147:420)
    • Ferritin H is expressed in erythroid precursors as well as macrophages
  • Special stains: PAS (usually in block-like pattern), alpha naphthyl acetate esterase, acid phosphatase
Negative stains
Molecular / cytogenetics description
Sample pathology report
  • Bone marrow, right, biopsy, clot section, aspirate smears, touch imprint and peripheral blood smear:
    • Pure erythroid leukemia, blasts 95% of total nucleated cells (see comment)
    • Comment:
      • Peripheral blood:
        • Red blood cells: Normocytic normochromic anemia with slight anisopoikilocytosis.
        • White blood cells: Leukopenia with few large proerythroblasts. The blasts have scant basophilic cytoplasm, round nuclei, open chromatin and few have prominent nucleoli. Cytoplasmic vacuoles are noted in some blasts.
        • Platelets: Decreased with rare giant forms.
      • Bone marrow biopsy:
        • Decalcification procedures performed to allow for histological assessment of submitted tissue.
        • Gross measurement: 0.5 cm
        • Quality: Adequate
        • Cellularity: 100%
        • Megakaryocytes: Rare
        • Infiltrate: The marrow is nearly completely replaced by sheets of immature cells with high nuclear to cytoplasmic ratio, open chromatin and some with prominent nucleoli.
      • Bone marrow clot:
        • Quality: Inadequate, blood only, no particles with many immature cells
      • Bone marrow smears:
        • Quality / cellularity: Adequate
        • Granulocytes: Markedly decreased
        • Erythrocytes: Markedly increased erythroid precursors with increased proerythroblasts.
        • Megakaryocytes: Not seen
        • Lymphocytes: Decreased, few small and mature cells.
        • bone marrow differential count: 500 total cells counted
          • Blasts percentage (0-5): 95
          • Promyelocytes percentage (2-8): 0
          • Myelocytes percentage (5-20): 0
          • Metamyelocytes percentage (13-32): 0
          • Granulocytes percentage (7-30): 2
          • Monocytes percentage (0-5): 0
          • Lymphocytes percentage (3-17): 3
          • Plasma cells percentage (0-2): 0
          • Eosinophils percentage (0-4): 0
          • Basophils percentage (0-1): 0
          • Pronormoblasts percentage (1-8): 80
          • Normoblasts percentage (7-32): 15
          • M:E ratio (2-4): N/A
      • Bone marrow touch imprint:
        • Quality: Adequate, similar to smear with increased erythroid blasts.
      • Flow cytometric analysis:
        • Aberrant erythroid blasts, 95.2% of total nucleated cells (F19-160).
      • Pending tests:
        • Cytogenetic karyotype and FISH.
Differential diagnosis
  • Other type of acute leukemia, especially acute megakaryocytic leukemia
  • Other poorly differentiated malignancy or small blue round cell tumor
  • Reactive erythroid hyperplasia
  • Pure erythroid leukemia should not be diagnosed as AML with myelodysplasia related change (AML with MRC) even if there is prior history of myeloid neoplasm, significant dysplasia of 2 lineages and a defining cytogenetic abnormality
    • AML with MRC requires minimal 20% myeloblasts while pure erythroid leukemia should not have a significant component of myeloblasts
Board review style question #1
Which of the following markers is most useful in diagnosing pure erythroid leukemia?

  1. CD117
  2. CD34
  3. Myeloperoxidase
  4. E-cadherin
  5. CD71
Board review style answer #1
Board review style question #2

A 7 year old boy presented with leukocytosis (55K), anemia (Hb 7.5 g/dl) and thrombocytopenia (52k). Peripheral blood smear (see photo) shows many mononucleated cells of variable sizes with high nuclear:cytoplasmic ratio, round nuclei, open chromatin and deep blue cytoplasm. Some large cells have conspicuous nucleolus. By flow cytometric immunophenotyping, these cells are positive for CD36, CD117 (heterogeneous) and negative for CD34, CD45, HLA-DR and all other lymphoid, myeloid and monocytic markers. Which of the following sentences is correct regarding this disease?

  1. These cells are positive for CD61
  2. These cells show reactivity with alpha-naphthyl acetate esterase, acid phosphatase and PAS
  3. These cells most likely have a normal karyotype (46, XY)
  4. A percentage of ≥ 20% of these cells in blood or bone marrow is required for diagnosis as pure erythroid leukemia
  5. The prognosis for this patient is good
Board review style answer #2
B. These cells show reactivity with alpha-naphthyl acetate esterase, acid phosphatase and PAS.

Comment Here

Reference: Acute erythroid leukemia (AML-M6)

AML general (pending)
[Pending]

AML hypocellular
Definition / general
  • Defined as < 20% bone marrow cellularity, 20% or more are blasts (WHO); other definitions range up to 40% cellularity
  • Up to 10% of AML cases
  • Median age 67 years (Leuk Res 1996;20:563)
  • Rare blasts in peripheral blood
  • Often has smoldering course although intensive chemotherapy may cause complete remission
Case reports
Microscopic (histologic) images

AFIP images
Missing Image

Markedly hypocellular marrow

Missing Image

Cells in interstitium are predominantly blasts

Positive stains
Differential diagnosis

AML with CEBPA mutation (WHO-HAEM5-biallelic or ICC bzip)
Definition / general
  • Subtype of acute myeloid leukemia (AML) with recurrent genetic abnormality
  • Required for diagnosis, the CEBPA mutations must be biallelic
  • Associated with more favorable prognosis
Essential features
  • CEBPA mutations must be biallelic, not just a single mutation, for diagnosis
  • May represent a germline predisposition syndrome and germline testing may be considered in patients with persistent CEBPA mutations following morphologic remission or in patients with family history of leukemia
ICD coding
  • ICD-10: C92.0 - acute myeloblastic leukemia
Epidemiology
  • Accounts for 4 - 9% of AML diagnoses in children and young adults
  • Less common in older patients
  • Reference: Blood 2009;113:6558
Sites
  • Peripheral blood and bone marrow
Pathophysiology
  • Hematopoietic progenitor cells require biallelic mutations of CEBPA
  • Possible continued clonal evolution
  • Reference: Sci Adv 2019;5:eaaw4304
Etiology
  • A subset of patients carry an underlying germline mutation, leading to predisposition to develop AML
Clinical features
  • Associated with lower frequency of lymphadenopathy and myeloid sarcoma
Diagnosis
  • Routine CBC with differential, bone marrow biopsy with flow cytometry, chromosome analysis and either targeted CEBPA molecular testing or next generation sequencing (massively parallel sequencing) for myeloid mutations
Laboratory
  • Typically present with relatively higher hemoglobin levels (still anemic), lower platelet counts and lower lactate dehydrogenase than CEBPA wildtype AML
Radiology description
  • PET scan identified hypermetabolic bone marrow
Prognostic factors
  • Favorable prognosis similar to AML with inv(16)(p13.1q22) or t(8;21)(q22;q22.1)
  • FLT3-ITD and GATA2 mutation status is of uncertain prognostic significance
  • Reference: Eur J Haematol 2015;94:439
Case reports
Treatment
  • Treated with similar induction and consolidation methods as other AMLs: 7+3 (cytarabine and anthracycline) and consolidation with cytarabine or azacitidine
  • May benefit from stem cell transplant (cannot use family member with CEBPA mutation if germline)
  • Relapsed patients have favorable prognosis as well (Blood 2013;122:1576)
Microscopic (histologic) description
  • No distinctive morphologic features
  • Typically, AML with or without maturation morphology
  • Multilineage dysplasia is present in 26% of cases of de novo AML with mutated CEBPA without an associated adverse prognosis; does not change classification
  • Reference: Haematologica 2017;102:529
Microscopic (histologic) images

Contributed by Kristin Karner, M.D.
Blasts infiltrating the marrow

Blasts infiltrating the marrow

Blasts on aspirate smear

Blasts on aspirate smear

Peripheral smear description
  • Varying amount of peripherally circulating blasts with or without maturation
Peripheral smear images

Contributed by Kristin Karner, M.D.
Blasts in peripheral blood

Blasts in peripheral blood

Positive stains
  • Depending on the blast immunophenotype, immunohistochemistry for CD34 is typically positive
Flow cytometry description
Flow cytometry images

Contributed by Jessica Corean, M.D.
AML with biallelic CEBPA

Typical immunophenotype

Molecular / cytogenetics description
  • Biallelic mutations of CEBPA required for diagnosis, disrupting the N and C terminus of CEBPA (J Clin Oncol 2010;28:570, Blood 2009;113:3088)
  • In familial cases, typically 1 mutation is germline, the other is somatic
  • CCAAT / enhancer binding protein alpha (CEBPA)
  • Mutation types include mutations in the encoding gene and promoter hypermethylation (Haematologica 2011;96:384)
  • Normal karyotype in 70% of cases
  • FLT3-ITD seen in 5 - 9% of cases
  • GATA2 seen in 39% of cases
  • Subset of cases have abnormal karyotype, del(9q) is common but does not make a diagnosis of AML with myelodysplasia related changes
    • Other reports of del(11q), which should be diagnosed as AML with myelodysplasia related changes
  • Patients should be evaluated for familial / germline syndromes (Blood 2015;126:1214)
Sample pathology report
  • Bone marrow aspirate, particle clot section and core biopsy:
    • Acute myeloid leukemia with biallelic mutations of CEBPA (84.3% blasts by morphology)
  • Microscopic description:
    • Peripheral blood smear:
      • Differential (100 cells)
        • Neutrophils: 8%
        • Lymphocytes: 7%
        • Monocytes: 1%
        • Eosinophils: 0%
        • Basophils: 0%
        • Metamyelocytes: 1%
        • Blasts: 83%
        • 1 nRBC/100 leukocytes
      • Erythrocyte number: decreased
      • Erythrocyte morphology: macrocytic, marked anisopoikilocytosis with ovalocytes, moderate polychromasia
      • Leukocyte number: increased
      • Leukocyte morphology: normal segmentation and granulation of neutrophils; blasts are small to intermediate in size with fine nuclear chromatin, nucleoli and scant basophilic cytoplasm
      • Platelet number: decreased
      • Platelet morphology: normal
    • Aspirate:
      • Aspiration differential count (300 cells)
        • Blasts: 84%
        • Promyelocytes: 2%
        • Myeloids: 6%
        • Erythroids: 4%
        • Lymphocytes: 4%
      • Specimen quality: adequate
      • Spicules: present
      • Trilineage hematopoiesis: scant residual
      • Erythroid maturation: decreased, full spectrum maturation
      • Myeloid maturation: blast morphology is similar to that described in the peripheral blood
      • Megakaryocyte morphology: 1 hyperlobated megakaryocyte identified in touch preparation
    • Core biopsy:
      • Bone trabeculae: normal
      • Cellularity: 80%
      • Trilineage hematopoiesis: scant residual
      • Erythroid maturation and localization: markedly decreased
      • Myeloid maturation and localization: sheets of blasts
      • Megakaryocyte number: rare
      • Megakaryocyte histotopography: rare hyperlobated megakaryocyte
  • Flow cytometry studies:
    • Interpretation: increased atypical CD34 positive myeloblasts (partial CD2, CD7 positive, CD11b positive, CD33 negative), representing approximately 89% of the leukocytes, consistent with acute myeloid leukemia
  • Chromosome results: 46,XY[20]
  • Myeloid mutation panel by NGS result:
    1. CEBPA c.332_339del, p.Ala111fs (NM_004364.4) variant frequency: 48.3%
    2. CEBPA c.759dup, p.Lys254fs (NM_004364.4) variant frequency: 47.3%
    3. TET2 c.2746C>T, p.Gln916* (NM_001127208.2) variant frequency: 48.9%
    4. TET2 c.4201G>T, p.Glu1401* (NM_001127208.2) variant frequency: 43.6%
    5. WT1 c.1410_1411ins34, p.Arg471fs (NM_024426.4) variant frequency: 1.0%
Differential diagnosis
  • Acute myeloid leukemia with myelodysplasia related changes:
    • Present: history of myelodysplastic syndrome or myelodysplastic / myeloproliferative neoplasm; myelodysplastic syndrome related cytogenetic abnormality or multilineage dysplasia
    • Absence of prior cytotoxic or radiation therapy for an unrelated disease and recurrent cytogenetic abnormality found in AML
  • Acute myeloid leukemia with (other) recurrent genetic abnormalities:
    • Through FISH, chromosome analysis and mutation analysis, AML defining recurrent genetic abnormalities can be identified
    • This includes t(8;21), inv(16) or t(16;16), PML-RARA, t(9;11), t(6;9), inv(3) or t(3;3), t(1;22), BCR-ABL1, mutated NPM1, mutated RUNX1
  • Therapy related myeloid neoplasms:
    • Onset of 2 - 10 years post cytotoxic chemotherapy or radiation administered for a previous neoplastic or nonneoplastic disease
  • Acute myeloid leukemia, NOS:
    • With the absence of pertinent history and recurrent cytogenetic abnormalities, AML is classified as NOS
Board review style question #1
Which of the following is true of AML with biallelic CEBPA mutation?

  1. The karyotype must not show any aberrations
  2. This has a poor prognosis
  3. This is typically a therapy related myeloid neoplasm
  4. This may be germline associated
Board review style answer #1
D. This entity may be germline associated and patients and their families are recommended to be tested for germline mutations. However, this is not always the case, as this leukemia typically presents de novo. The karyotype is most often normal but there are well documented karyotypic abnormalities identified, such as del(9q), which does not influence prognosis.

Comment Here

Reference: AML with biallelic mutation of CEBPA
Board review style question #2

Which of the following is the most frequently encountered flow cytometry immunophenotype for AML with biallelic CEBPA mutation?

  1. Positive for CD33 and CD117, negative for CD34 and HLA-DR
  2. Positive for CD41, CD61 and CD42b
  3. Positive for CD33, CD13 (low expression), CD117, CD123, CD4, CD36, CD64, negative for HLA-DR
  4. Positive for CD13, CD7, CD34 and HLA-DR
Board review style answer #2
D. Positive for CD13, CD7, CD34 and HLA-DR. This entity typically has expression of CD7 in 50 - 73% of cases. Additionally, the blasts usually show expression for CD34 and HLA-DR. Answer A is describing the immunophenotype typical of acute promyelocytic leukemia. Answer B is describing megakaryocytic lineage markers. Answer C is describing AML with monocytic differentiation.

Comment Here

Reference: AML with biallelic mutation of CEBPA

AML with BCR::ABL1
Definition / general
  • Acute myeloid leukemia (AML) with BCR::ABL1 fusion is a de novo AML in which BCR::ABL1 is detected at initial diagnosis without evidence of chronic myeloid leukemia (CML)
Essential features
  • BCR::ABL1 fusion is detected at the time of diagnosis in a de novo AML with BCR::ABL1
  • No CML morphologic features prior to or at diagnosis or posttherapy
  • Morphologic spectrum, includes AML with minimal differentiation, without maturation or with monocytic differentiation
  • Additional chromosomal aberrations are infrequently seen
  • Cryptic deletions within immunoglobulin and T cell receptor accompanied by losses of the IKZF1 or CDNK2A / B genes
Terminology
  • Acute myeloid leukemia with t(9;22)(q34.1;q11.2)
ICD coding
  • ICD-O: 9912/3 - acute myeloid leukemia with BCR::ABL1
  • ICD-11: 2A60.0 & XH6FZ7 - acute myeloid leukemia with recurrent genetic abnormalities & acute myeloid leukemia with BCR::ABL1
Epidemiology
Sites
  • Peripheral blood and bone marrow are mostly involved
  • Extramedullary site involvement is rare (Leuk Res 2008;32:1476)
Pathophysiology
  • Philadelphia (Ph) chromosome and the chimeric BCR::ABL1 fusion gene result from t(9;22)(q34;q11) or its variants, which encodes a constitutively active tyrosine kinase with oncogenic properties (Br J Haematol 2013;161:541)
  • 3 different breakpoint cluster regions in the BCR gene (M-bcr, m-bcr and µ-bcr) are reported: 8.5 kb hybrid mRNA (b2a2 or b3a2) encodes the 210 kDa protein (p210), 7.5 kb hybrid mRNA (e1a2) encodes the 190 kDa protein (p190) and 9 kb hybrid mRNA (c3a2) encodes the 230 kDa protein (p230) (Genomics 1995;27:67, Oncogene 2002;21:8652)
  • ~70 - 80% cases of AML with BCR::ABL1 harbor p210 transcripts (Am J Clin Pathol 2007;127:642)
Etiology
  • Unknown
Clinical features
  • Present with leukocytosis and anemia or thrombocytopenia
  • Patients with AML with BCR::ABL1 in comparison to CML myeloid blast phase (CML MBP) have lower peripheral blood basophil percentage, absolute basophil count and are, less commonly, found to have splenomegaly (Am J Clin Pathol 2007;127:642)
Diagnosis
  • Essential diagnostic criteria
    • Myeloid neoplasm with > 20% blasts expressing myeloid immunophenotype (based on immunohistochemistry or flow cytometry) in the peripheral blood or bone marrow
    • Detection of BCR::ABL1 at the time of initial diagnosis
    • Lack of CML features prior to or at diagnosis or after therapy
  • Desirable diagnostic criteria
    • Presence of t(9;22)(q34.1;q11.2) on conventional karyotyping
    • Determination of the BCR::ABL1 transcript subtype and establishment of a baseline level of BCR::ABL1 transcript for monitoring treatment response
Laboratory
  • Leukocytosis
  • Anemia or thrombocytopenia
Radiology description
Prognostic factors
Case reports
  • 28 year old woman presented with leukocytosis and was diagnosed with AML with e1a2 BCR::ABL1 fusion (Rev Bras Hematol Hemoter 2017;39:379)
  • 68 year old man with a history of dementia presented with leukocytosis and was diagnosed with AML with BCR::ABL1 (Hemasphere 2020;4:e484)
  • 77 year old woman who presented with pancytopenia underwent a bone marrow biopsy and was diagnosed with AML with BCR::ABL1 (Leuk Res Rep 2020;15:100233)
  • 78 year old man with no prior history of CML presented with leukocytosis and was diagnosed with AML with BCR::ABL1 (BMC Cancer 2019;19:50)
Treatment
  • Standard AML induction chemotherapy (anthracycline / cytosine arabinoside containing regimen) response is poor (Leukemia 1998;12:1881)
  • In addition, single agent tyrosine kinase inhibitors (TKI) alone are also not effective; however, a single case report indicated achieving complete molecular remission in a patient treated with dasatinib and chemotherapy (Ann Hematol 2016;95:1211, Medicine (Baltimore) 2018;97:e12949)
  • Venetoclax and TKI combination regimens have shown good response in some studies (Acta Haematol 2020;143:567)
  • Outcome of patients < 50 years of age with TKI pretreated BCR::ABL1 positive AML receiving allogeneic stem cell transplant is relatively favorable (Am J Hematol 2018;93:31)
  • Survival status postallogeneic transplantation appears similar to intermediate risk AML, with one report demonstrating 3 year overall survival of 73% (Bone Marrow Transplant 2021;56:232)
Microscopic (histologic) description
  • AML with BCR::ABL1 has a broad morphologic spectrum, which includes AML with minimal differentiation (FAB M0), without maturation (FAB M1) or with monocytic differentiation (FAB M5) (Haematologica 1996;81:423, Leuk Lymphoma 2013;54:138)
  • AML with BCR::ABL1 demonstrates a marrow cellularity of 80%, which is lower than the 98% median marrow of CML MBP (Am J Clin Pathol 2007;127:642)
  • In comparison to CML MBP, the bone marrow in AML with BCR::ABL1 shows lower myeloid/erythroid ratio at diagnosis (median: 2.0 versus 4.8) and a lower bone marrow aspirate basophil percentage (median: 0% versus 4.5%) (Am J Clin Pathol 2007;127:642)
  • Dwarf megakaryocytes are rarely seen (Leuk Lymphoma 2013;54:138)
Microscopic (histologic) images

Contributed by Barina Aqil, M.D.
Increased blasts Increased blasts

Increased blasts

Atypical mononuclear cells

Atypical mononuclear cells

Blast morphology

Blast morphology

CD34 positive blasts

CD34 positive blasts

Peripheral smear description
  • Leukocytosis
  • Variable anemia, with normocytic or macrocytic red blood cells
  • Variable thrombocytopenia
  • Left shifted granulocytes may be seen with circulating blasts, which may be ≥ 20% by definition
  • Blasts may be myeloblasts or monoblasts / promonocytes
  • CML MBP cases have a higher peripheral blood basophil percentage (median: 2.5%) than AML with BCR::ABL1 (median: 0%) and a higher peripheral blood absolute basophil count (Am J Clin Pathol 2007;127:642)
Peripheral smear images

Contributed by Barina Aqil, M.D.
Circulating blasts

Circulating blasts

Positive stains
Negative stains
  • Myeloperoxidase may be positive or negative depending on the type of blasts
Flow cytometry description
Flow cytometry images

Contributed by Barina Aqil, M.D.
Blast phenotype Blast phenotype

Dim CD45+ myeloid blasts

Blast phenotype

Partial CD34+ and CD117+

Blast phenotype

Partial CD13+


Blast phenotype

Dim CD33+

Blast phenotype

Partial HLA-DR+

Blast phenotype

Predominantly MPO-

Molecular / cytogenetics description
Molecular / cytogenetics images

Contributed by Barina Aqil, M.D.
<i>BCR::ABL1</i> fusion by FISH

BCR::ABL1 fusion by FISH

Sample pathology report
  • Peripheral blood, bone marrow aspirate and core biopsy:
    • Acute myeloid leukemia with BCR::ABL1 (see comment)
    • Comment: The cytogenetic and FISH studies showed t(3;9;22) resulting in rearrangement of BCR::ABL1. The overall findings are consistent with acute myeloid leukemia with BCR::ABL1. The peripheral blood neutrophils show no significant shift to immaturity and the marrow myeloid to erythroid ratio is not elevated. Given these findings, a de novo acute myeloid leukemia with BCR::ABL1 is favored over blast phase of chronic myeloid leukemia with BCR::ABL1.
    • Peripheral blood: The peripheral blood smear shows pancytopenia. There is macrocytic anemia with anisopoikilocytosis including ovalocytes, teardrop forms and rare circulating nucleated red blood cells. Occasional circulating blasts are noted. The blasts are intermediate to large in size with high nuclear to cytoplasmic ratio and dispersed chromatin. Neutrophils show toxic changes but without significant shift to immaturity. Eosinophils are present but not increased. Platelets are decreased in number with unremarkable morphology.
    • Bone marrow aspirate: The bone marrow aspirate smears show increased blasts showing similar morphology to those seen in the peripheral blood smear. The erythroid precursors are decreased but show progressive mature with unremarkable morphology. Megakaryocytes are present with unremarkable morphology.
    • Bone marrow core biopsy: The bone marrow core biopsy shows hypercellular marrow (~80% cellular). Myeloid precursors are increased with a shift to immaturity including frequent blasts. The blasts are intermediate to large in size with dispersed chromatin, present scattered and in aggregates throughout the biopsy. Erythroid precursors are relatively decreased but show progressive maturation. Megakaryocytes are decreased with unremarkable morphology.
    • Cytogenetic analysis reported an abnormal male karyotype: 46,XY,t(3;9;22)(p21; q34.1; q11.2)[3]/48, idem,+8,+12[17]. The t(3;9;22) is a 3 way variant translocation resulting in BCR::ABL1 fusion. Gains of chromosomes 8 and 12 in related abnormal clone 2 represent clonal evolution.
    • FISH analysis was positive for the BCR::ABL1 fusion with a signal pattern consistent with a 3 way translocation (1F2R2G, 66%).
Differential diagnosis
Board review style question #1

Which of the following is true of acute myeloid leukemia (AML) with BCR::ABL1?

  1. Detection of BCR::ABL1 can occur posttherapy
  2. It has no co-occurrence with t(8;21) / RUNX1::RUNX1T1
  3. It is a myeloid neoplasm with < 20% myeloid blasts
  4. There can be a prior history of myelodysplastic neoplasm or myeloproliferative / myelodysplastic neoplasm
Board review style answer #1
B. It has no co-occurrence with t(8;21) / RUNX1::RUNX1T1. The presence of another cytogenetic abnormality that is defining for another de novo AML subtype in addition to t(9;22)(q34.1;q11.2) takes precedence over the diagnosis of AML with BCR::ABL1. Answer A is incorrect because AML with BCR::ABL1 is a de novo AML in which BCR::ABL1 is detected at the time of initial diagnosis. Answer C is incorrect because, in the category of AML with defined genetic abnormality, AML with BCR::ABL1 and AML with CEBPA mutation are the only two subtypes that require > 20% blasts for the diagnosis per 2022, WHO 5 th edition. According to International Consensus Classification (Virchows Arch 2023;482:27), AML with BCR::ABL1 is the only subtype which require > 20% blasts. Answer D is incorrect because BCR::ABL1 can be acquired secondarily in AML arising from prior myelodysplastic neoplasm or myeloproliferative / myelodysplastic neoplasm. So, these cases are excluded from this category.

Comment Here

Reference: AML with BCR::ABL1
Board review style question #2
In comparison with chronic myeloid leukemia (CML) myeloid blast phase, AML with BCR::ABL1 shows which of the following findings?

  1. Cryptic deletions within the immunoglobulin and T cell receptor genes
  2. High frequency of splenomegaly
  3. Increased peripheral basophil percentage
  4. Presence of additional chromosomal aberrations (trisomy 8, trisomy 19 and isochromosome 17q)
Board review style answer #2
A. Cryptic deletions within the immunoglobulin and T cell receptor genes. Cryptic deletions within the immunoglobulin and T cell receptor genes, along with losses of the IKZF1 or CDNK2A / B genes are seen in AML with BCR::ABL1. Answers B and C are incorrect because splenomegaly, high myeloid to erythroid ratio in the bone marrow and increased basophils are seen in the myeloid blast phase of chronic myeloid leukemia rather than AML with BCR::ABL1. Answer D is incorrect because cytogenetic abnormalities, such as an isochromosome of the long arm of chromosome 17, gain of an extra copy of Ph, trisomy 19 and trisomy 8, which are known to occur frequently in the blast phase of chronic myeloid leukemia, are not usually seen in AML with BCR::ABL1.

Comment Here

Reference: AML with BCR::ABL1

AML with FLT3
Definition / general
  • Not a specific WHO designated entity but has specific prognostic and treatment ramifications
  • FLT3 (FMS-like tyrosine kinase 3) gene encodes a membrane bound receptor tyrosine kinase and is located on chromosome 13q12
  • 2 mutation types in leukemia: internal tandem duplication (ITD), usually within the juxtamembrane domain, and a missense point mutation on the tyrosine kinase domain (TKD)
Essential features
  • Can be found in ~30% of cytogenetically normal acute myeloid leukemia (AML) (Br J Haematol 2017;179:530)
  • Up to 40% of acute promyelocytic leukemia (APL) patients have FLT3 mutations
  • ~20% of AML cases with t(9;11)(p21.3;q23.3) have point mutations in FLT3
  • High percentage (~70%) of AML with t(6;9)(p23;q34.1) have FLT3 mutations (Leukemia 2006;20:1295)
  • Higher prevalence in younger patients with AML (< 60 years old) (Ann Hematol 2017;96:1993)
  • There are several new FLT3 targeted drugs that show promising results, which necessitate rapid analysis of the FLT3 status in patients with AML
Terminology
  • FMS-like tyrosine kinase 3
ICD coding
  • ICD-10:
    • C92.00 - acute myeloblastic leukemia, not having achieved remission
    • C92.01 - acute myeloblastic leukemia, in remission
    • C92.02 - acute myeloblastic leukemia, in relapse
Epidemiology
  • More prevalent in patients < 60 years old
  • Found in ~31% of patients with acute promyelocytic leukemia (22% FLT3-ITD mutation and 9% FLT3 D835) (Haematologica 2011;96:1470)
  • Can be found in about 30% (28 - 34% FLT3-ITD and 11 - 14% FLT3-TKD) of cytogenetically normal AML (Br J Haematol 2017;179:530)
Clinical features
Diagnosis
  • FLT3 mutations are generally detected in the clinical laboratory by PCR and electrophoresis based product sizing as well as using next generation sequencing (NGS) platforms
Prognostic factors
  • FLT3-ITD is an independent adverse prognostic indicator in AML
  • Favorable outcome if mutated nucleophosmin (NPM1) without FLT3-ITD or with FLT3-ITDlow (allelic ratio < 0.5)
  • Intermediate outcome if mutated NPM1 and FLT3-ITDhigh (allelic ratio ≥ 0.5) or wild type NPM1 without FLT3-ITD or with FLT3-ITDlow (without adverse risk genetic lesions)
  • Adverse outcome if wild type NPM1 and FLT3-ITDhigh
  • Prognostic impact of FLT3-TKD mutations is uncertain (Leukemia 2019;33:299)
  • Acute promyelocytic leukemia with FLT3 mutations appear to represent a subset of APL patients who have a higher risk of relapse (Am J Hematol 2010;85:956)
Case reports
Treatment
Microscopic (histologic) description
  • No specific microscopic findings associated with FLT3 mutation
  • No correlation with a single, specific French-American-British (FAB) subtype was found (Blood 2002;100:59)
  • When FLT3 occurs with other mutations or cytogenetic abnormalities, the characteristic microscopic features seen with those abnormalities are usually present
Microscopic (histologic) images

Contributed by Etan Marks, D.O.

Bone marrow core biopsy

CD34 bone marrow

Virtual slides

Images hosted on other servers:

Peripheral smear of AML

Peripheral smear description
  • Similar to other acute myeloid leukemias with ≥ 20% myeloblasts or ≥ 20% promyelocytes
Peripheral smear images

Contributed by Etan Marks, D.O.

Large irregular myeloblasts

Back to back myeloblasts

Positive stains
Molecular / cytogenetics description
  • FLT3-ITD mutations consist of a duplicated coding sequence usually derived from the juxtamembrane domain inserted in tandem (ITD) (Haematologica 2011;96:1470, Cytometry B Clin Cytom 2013;84:390)
    • These in frame insertion mutations range from 3 to > 200 bp in length and result in disruption of the autoinhibitory function
  • FLT3-TKD point mutations occur in the activation loop of the kinase domain, most commonly at residue aspartate 835 (D835)
Sample pathology report
  • FLT3 is usually reported as an addendum following the initial diagnosis of AML

FLT3 report

FLT3 report

Differential diagnosis
Board review style question #1
Which mutational combination carries with it the worst overall prognosis in AML?

  1. 5q deleted AML
  2. Mutated FLT3 and wild type NPM1
  3. Wild type FLT3 and mutated NPM1
  4. Wild type NPM1 and biallelic mutated CEBPA
Board review style answer #1
B. Mutated FLT3 and wild type NPM1

Comment Here

Reference: AML with FLT3
Board review style question #2

Which AML with cytogenetic abnormalities has the highest frequency of FLT3 mutations?

  1. Acute promyelocytic leukemia
  2. AML with 5q deletion
  3. AML with t(6;9)(p23;q34.1)
  4. AML with t(9;11)(p21.3;q23.3)
Board review style answer #2
C. AML with t(6;9)(p23;q34.1)

Comment Here

Reference: AML with FLT3

AML with FUS::ERG (pending)
[Pending]

AML with KAT6A::CREBBP
Definition / general
  • Acute myeloid leukemia (AML) with t(8;16)(p11.2;p13.3) / KAT6A::CREBBP is a rare AML with recurrent cytogenetic abnormality
Essential features
  • Detection of KAT6A::CREBBP or t(8;16)(p11.2;p13.3) is required by cytogenetic or molecular studies for diagnosis
  • Should not fulfill the diagnostic criteria for AML with defining genetic abnormalities, AML myelodysplasia related (AML MR), AML postcytotoxic therapy (AML pCT) or mixed phenotype acute leukemia (MPAL)
  • Morphologically, blasts often show myelomonocytic or monocytic differentiation with some demonstrating erythrophagocytosis
  • Blasts have characteristic immunophenotype as detected by flow cytometry
Terminology
  • Acute myeloid leukemia with t(8;16)(p11;p13)
ICD coding
  • ICD-O: 9861/3 - acute myeloid leukemia, NOS
  • ICD-10: C92.0 - acute myeloblastic leukemia
  • ICD-11: 2A60.0 - acute myeloid leukemia with recurrent genetic abnormalities
Epidemiology
  • Incidence is ~0.2% of AMLs
  • AML with t(8;16)(p11;p13) is a rare entity that may arise as a congenital, de novo or therapy related neoplasm, status postchemotherapy or radiation therapy (RT) (Leukemia 2008;22:1567, Leukemia 2009;23:934)
  • Presents in all the age groups ranging from neonatal period in children to adults, with a median age of 60 years
Sites
Pathophysiology
  • t(8;16) results in a fusion of the KAT6A gene located on chromosome 8p11 with the CREBBP gene located on chromosome 16p13 (Nat Genet 1996;14:33)
Etiology
Clinical features
Diagnosis
  • Presence of ≥ 20% myeloid blasts in the bone marrow or peripheral blood per WHO 2022 but the blast requirement for the diagnosis is > 10% according to the 2022 International Consensus Classification (Virchows Arch 2023;482:27)
  • Detection of KAT6A::CREBBP / t(8;16)(p11.2;p13.3)
  • Not meeting the diagnostic criteria for AML with other defining genetic abnormalities, AML MR, AML pCT or MPAL
Laboratory
Prognostic factors
  • Overall poor prognosis in adults
  • Spontaneous remission has been reported in children diagnosed in the early neonatal period; some cases with spontaneous remission though subsequently relapse (Blood 2013;122:2704)
  • Allogeneic stem cell transplantation has shown a significantly better outcome in adults with KAT6A::CREBBP without prior cytotoxic therapy or myeloid neoplasm (Br J Haematol 2021;192:832)
Case reports
  • Full term female infant was noted at birth to have numerous blue maculopapular skin lesions over the body and found to have occasional circulating blasts (Pediatr Blood Cancer 2017;64:e26450)
  • 23 year old man presented with pain and night sweats; circulating blasts were noted along with thrombocytopenia (Blood 2016;128:314)
  • Woman in mid 40s with a history of celiac disease presented with shortness of breath and was found to have 16% circulating blasts (BMJ Case Rep 2023;16:e253812)
Treatment
  • Initial studies had reported median overall survival (OS) of 4.7 - 8.5 months in AML with t(8;16) on chemotherapy (Leuk Res 2013;37:32)
  • With the introduction of stem cell transplant (SCT), OS relatively improved to 18.2 months and was described even better in patients with de novo AML or noncomplex karyotype (Ann Hematol 2019;98:1149, Br J Haematol 2021;192:832)
  • Cases with de novo AML and t(8;16) who received allo-SCT in first clinical remission (CR) achieved improved rates, suggesting that an early transplant in first CR is likely beneficial in these patients (Ann Hematol 2019;98:1149, Br J Haematol 2021;192:832)
  • Venetoclax in combination with hypomethyling agents has shown a good response (Blood 2019;133:7)
Microscopic (histologic) description
Microscopic (histologic) images

Contributed by Barina Aqil, M.D.
Increased blasts Increased blasts

Increased blasts

Increased blasts

Increased blasts

Atypical mononuclear cells

Atypical mononuclear cells

Blast morphology

Blast morphology

Cytology description
  • Blasts are medium to large in size with irregular / folded nuclei, fine chromatin, prominent nucleoli and moderate to abundant cytoplasm
    • Some blasts show cytoplasmic granules or vacuolization
Peripheral smear description
  • Blasts often show myelomonocytic or monocytic differentiation with prominent cytoplasmic granulation with or without Auer rods
Peripheral smear images

Contributed by Barina Aqil, M.D.
Circulating blasts

Circulating blasts

Circulating blasts, MPO negative

Circulating blasts, MPO negative

Circulating blasts, ANB positive

Circulating blasts, ANB positive

Positive stains
Flow cytometry description
Flow cytometry images

Contributed by Barina Aqil, M.D.
CAPTIONTIME

Blast phenotype

Molecular / cytogenetics description
Sample pathology report
  • Peripheral blood, bone marrow aspirate and left posterior iliac crest bone marrow core biopsy:
    • Acute myeloid leukemia with KAT6A::CREBBP / t(8;16)(p11;p13) extensively involving a hypercellular bone marrow (see comment)
    • Comment: The blasts are positive for MPO and CD68 indicating a myelomonocytic lineage. Some of the blasts show erythrophagocytosis.
    • Peripheral blood smear:
      • Peripheral blood smear shows normochromic normocytic anemia with mild anisopoikilocytosis. Neutrophils are decreased with unremarkable morphology. There are occasional medium to large blasts with irregular / folded nuclei, fine chromatin, prominent nucleoli and moderate to abundant granular cytoplasm. Rare Auer rods are seen. The platelets are markedly decreased.
    • Bone marrow aspirate smear / touch preparation:
      • Bone marrow aspirate smears and touch preparations are cellular and adequate for interpretation. The myeloid series show left shifted maturation with increased blasts with similar morphology as described in the peripheral blood smear. The erythroid precursors show progressive maturation. Megakaryocytes are decreased.
    • Bone marrow core biopsy (decalcified) / particle clot:
      • Unilateral bone marrow core biopsy is hypercellular for age (80 - 90% cellular) with extensive replacement by blasts that are medium to large in size with fine chromatin and pinpoint nucleoli. The erythroid precursors show progressive maturation. Megakaryocytes are decreased but show unremarkable morphology.
Differential diagnosis
  • AML with monocytic differentiation:
  • Acute promyelocytic leukemia:
    • Presentation with coagulopathy (DIC) and blasts with prominent cytoplasmic granulation can be similar to AML with KAT6A::CREBBP but phenotype and molecular studies are helpful in resolving this differential
  • AML with KMT2A rearrangement:
    • Extramedullary involvement is seen in 33% of adult patients
    • Morphology is similar to AML with KAT6A::CREBPP with myelomonocytic or monocytic blasts
    • Erythrophagocytosis is not seen
    • Blasts show monocytic differentiation with expression of CD33, CD65, CD4, CD15, HLA-DR, lysozyme and variable CD34 and CD117
    • Presence of KMT2A rearrangement with any one of the described > 80 fusion partners
  • AML with NPM1 mutation:
    • Morphology may be similar to AML with KAT6A:CREBPP with myelomonocytic or monocytic blasts
    • Blast with cup shaped nuclear morphology is considered to be highly specific for this entity
    • Multilineage dysplasia is commonly seen
    • Immunophenotype can be quite variable; however, the majority of blasts predominantly are negative for CD34 but with expression of CD117 and CD33
    • NPM1 mutation detection is an essential diagnostic criterion
    • Concomitant FLT3 internal tandem duplication (ITD) mutation evaluation is important for risk assessment
    • Patients with AML with NPM1 mutation along with FLT3 ITD mutation have poorer prognosis than AML with NPM1 mutation alone
Board review style question #1

What is the characteristic blast phenotype in acute leukemia with KAT6A::CREBBP, which is shown in the image above?

  1. CD13+, CD33+, CD11b+, CD14-, CD64-, CD34+
  2. CD34-, CD117-, CD13+, CD33+, CD11b+, CD64+
  3. CD34+, CD117-, CD13+, CD33+, CD11b-, CD64+
  4. CD34+, CD117+, CD13+, CD33+, CD64+
Board review style answer #1
B. CD34-, CD117-, CD13+, CD33+, CD11b+, CD64+. Acute myeloid leukemia with t(8;16)(p11.2;p13.3) / KAT6A::CREBBP shows blasts that are negative for CD34 and CD117 and positive for CD13, CD33, CD11b and CD64, among others. Answer C is incorrect because blasts are CD34- with myelomonocytic markers expression. Answer A is incorrect because blasts are usually CD34- and express monocytic markers (CD14, CD11b and CD64). Answer D is incorrect because blasts are usually CD34- and CD117-.

Comment Here

Reference: AML with KAT6A::CREBBP
Board review style question #2

A 42 year old man with no significant past medical history presented with fatigue and weight loss for 2 months. Laboratory workup showed leukocytosis (WBC 23.0 K/uL), anemia (8.0 g/dL) and occasional circulating blasts in peripheral blood. The bone marrow biopsy showed > 20% myeloid blasts with coexpression of CD64 and partial CD11b. This finding (see image) is characteristically seen in ~75% cases of which disease?

  1. AML with CEBPA
  2. AML with DEK::NUP214
  3. AML with KAT6A::CREBBP
  4. AML with NPM1
Board review style answer #2
C. AML with KAT6A::CREBBP. AML with KAT6A::CREBBP has myelomonocytic / monocytic differentiation with ~75% cases demonstrating erythrophagocytosis. Answers A, B and D are incorrect because they do not show significant erythrophagocytosis; however, AML with t(6;9) / DEK::NUP214 is associated with basophilia and multilineage dysplasia.

Comment Here

Reference: AML with KAT6A::CREBBP

AML with RUNX1T3(CBFA2T3)::GLIS2
Definition / general
  • Acute myeloid leukemia (AML) with CBFA2T3(RUNX1T3)::GLIS2 is a rare subtype of pediatric AML
Essential features
  • Pediatric acute myeloid leukemia with dismal prognosis
  • CBFA2T3::GLIS2 is the most common alteration seen in non-Down syndrome acute megakaryoblastic leukemia (non-DS AMKL)
  • Often expressing the unique RAM phenotype by flow cytometry with bright CD56 and lack of HLA-DR and CD38 expression
Terminology
None
ICD coding
  • ICD-O: 9861/3 - acute myeloid leukemia, NOS
  • ICD-10: C92.0 - acute myeloid leukemia
  • ICD-11: 2A60.0 - acute myeloid leukemia with recurrent genetic abnormalities
Epidemiology
Sites
Pathophysiology
  • Cryptic inversion on chromosome 16 [inv(16)(p13.3q24.3)] results in the joining of CBFA2T3, a member of the ETO family of nuclear corepressors, to GLIS2, a member of the GLI family of transcription factors (Cancer Cell 2012;22:683)
  • CBFA2T3 and GLIS2 genes are both localized, in inverted orientation, on each arm of chromosome 16 close to the telomeres, in 16q24.3 and 16p13.3, respectively (Cancer Cell 2012;22:683)
  • CBFA2T3::GLIS2 fusion transcript leads to upregulation of BMP2 (bone morphogenetic proteins) and GATA3, a downstream target of Hedgehog signaling as well as JAK-STAT pathway (J Exp Med 2012;209:2017, Development 2004;131:1165, Cancer Cell 2012;22:683)
Etiology
  • Not known
Diagrams / tables
None
Clinical features
Diagnosis
  • Diagnostic criteria per 2022, World Health Organization (WHO) Classification of Tumors requires ≥ 20% myeloid blasts in the peripheral blood or bone marrow and detection of CBFA2T3::GLIS2 (Leukemia 2022;36:1703)
  • International Consensus Classification (ICC), 2022 requires ≥ 10% myeloid blasts in the bone marrow or peripheral blood (Blood 2022;140:1200)
  • Must exclude the diagnostic criteria for AML with other defining genetic abnormalities, AML MR (AML myelodysplasia related), AML pCT (postcytotoxic therapy) or MPAL (mixed phenotype acute leukemia)
  • This entity is recognize by the WHO 5th edition under AML with other defined genetic alterations but is not a specific entity within the ICC
Laboratory
Radiology description
Not relevant to this topic
Radiology images
None
Prognostic factors
Case reports
Treatment
  • Patients may benefit from intensive chemotherapy followed by allogeneic stem cell transplant as postremission intensification strategy (Pediatr Transplant 2018;22:e13291)
  • Overexpression of the folate receptor 1 (FOLR1) gene has shown great result with targeted therapy (J Clin Invest 2022;132:e157101)
  • Navitoclax or DT2216 treatment in combination with low dose cytarabine has been found effective in murine models (Blood Adv 2024;8:112)
Clinical images
Not relevant to this topic
Gross description
Not relevant to this topic
Gross images
Not relevant to this topic
Frozen section description
Not relevant to this topic
Frozen section images
Not relevant to this topic
Microscopic (histologic) description
  • No specific morphological features have been found associated with this fusion gene (Blood 2013;121:3469)
  • In AMKL CBFA2T3::GLIS2 positive patients, megakaryoblasts are reported as the predominant component of the blast population in bone marrow or peripheral blood (Br J Haematol 2019;184:337)
  • Megakaryoblasts are large cells with abundant cytoplasm and prominent nucleoli; they often display cytoplasmic blebs or pseudopods
  • In one of the pediatric studies, 50% (N = 10) of the fusion positive patients had a non-FAB M7 subtype, others were distributed as follows: M5 (15%), M0 (15%), M1 (10%), M2 (5%), M4 (5%) (Blood 2013;121:3469)
Microscopic (histologic) images

Contributed by Barina Aqil, M.D.
Atypical mononuclear cells

Atypical mononuclear cells

Blast morphology

Blast morphology

Increased blasts

Increased blasts

Blasts

Blasts

Virtual slides
Not relevant to this topic
Cytology description
Not relevant to this topic
Cytology images
Not relevant to this topic
Peripheral smear description
Peripheral smear images

Contributed by Barina Aqil, M.D.
Circulating blasts

Circulating blasts

Positive stains
Flow cytometry description
Flow cytometry images
None
Electron microscopy description
Not relevant to this topic
Electron microscopy images
Not relevant to this topic
Molecular / cytogenetics description
Molecular / cytogenetics images
Not available
Videos
None
Sample pathology report
  • Peripheral blood, bone marrow aspirate and left posterior iliac crest bone marrow core biopsy:
    • Acute myeloid leukemia with CBFA2T3::GLIS2 extensively involving a hypercellular (virtually 100%) bone marrow (see comment)
    • Flow cytometric immunophenotyping of the bone marrow aspirate reveals a blast population (~90% of cellular events) with a dim to negative CD45 expression that is positive for CD34, bright positive for CD56 and negative for CD38 and HLA-DR.
    • Comment: The overall morphologic, immunophenotypic and genetic findings are consistent with a diagnosis of acute myeloid leukemia with CBFA2T3::GLIS2.
    • Peripheral blood smear: The peripheral blood smear shows predominantly circulating blasts (90%) that are large in size with abundant cytoplasm and frequent prominent nucleoli with occasional cytoplasmic blebs. Normocytic normochromic anemia is present with rare nucleated red blood cells. There is absolute neutropenia and lymphopenia. Thrombocytopenia with rare large platelets.
    • Bone marrow aspirate smear: The majority of the cells present are blasts with similar morphology as seen in the peripheral blood. Background hematopoiesis is markedly decreased. Rare megakaryocytes are seen.
    • Bone marrow core biopsy (decalcified) / particle clot: The bone marrow is markedly hypercellular for age (~100%) and is composed almost entirely of blasts. Very rare myeloid and erythroid precursors are present. The particle clot shows multiple bone marrow particles composed predominantly of blasts.
Differential diagnosis
Additional references
None
Board review style question #1

What characteristic phenotype panel is helpful in diagnosing acute myeloid leukemia with CBFA2T3::GLIS2?

  1. CD34, CD56, CD38, HLA-DR
  2. CD34, CD117, CD13, CD33
  3. CD34, CD117, CD41, CD61
  4. CD34, TdT, CD19, CD79a
Board review style answer #1
A. CD34, CD56, CD38, HLA-DR. The blasts have characteristic phenotype CD56+, CD38 and CD45 (negative to dim) and HLA-DR-. Answer B is incorrect because it is the antibody panel for acute myeloid leukemia. Answer C is incorrect because it is the panel for acute megakaryoblastic leukemia. Answer D is incorrect because it is the panel for B lymphoblastic leukemia.

Comment Here

Reference: AML with RUNX1T3(CBFA2T3)::GLIS2

AML with RUNX1T3(CBFA2T3)::GLIS2 (pending)
[Pending]

AML with germline CEBPA mutation (pending)
[Pending]

AML with inv(16)(p13.1;q22) or t(16;16)(p13.1;q22); CBFB::MYH11
Definition / general
  • Acute myeloid leukemia (AML) with inv(16)(p13.1;q22) or t(16;16)(p13.1;q22); CBFB-MYH11 is a subtype of AML with recurrent genetic abnormalities characterized by myelomonocytic differentiation and presence of abnormal eosinophils
Essential features
  • Inv(16)(p13.1;q22) or t(16;16)(p13.1;q22) results in the formation of an abnormal core binding factor beta subunit / myosin heavy chain 11 (CBFB-MYH11) fusion gene
  • Most cases show myelomonocytic differentiation with presence of abnormal eosinophils
  • Diagnosis of AML with inv(16)(p13.1;q22) or t(16;16)(p13.1;q22) can be made even when the blast percentage is less than 20%
  • Prognosis is good compared with other AMLs
Terminology
  • Acute myelomonocytic leukemia with abnormal eosinophils
  • French American British (FAB) M4Eo
  • Acute myeloid leukemia with CBFB-MYH11
ICD coding
  • ICD-O: 9871/3 - acute myeloid leukemia with inv(16)(p13.1q22) or t(16;16)(p13.1;q22); CBFB-MYH11
  • ICD-10: C92.0 - acute myeloid leukemia
Epidemiology
Sites
Pathophysiology
  • Both inv(16)(p13.1;q22) and t(16;16)(p13.1;q22) result in a core binding factor beta subunit / myosin heavy chain 11 (CBFB-MYH11) fusion gene encoding an abnormal CBFB-MYH11 fusion protein (Semin Hematol 2015;52:215)
  • CBFB-MYH11 fusion protein causes aberrant self renewal and suppresses differentiation
  • Additional mutations promote leukemogenesis
Clinical features
  • Fatigue resulting from anemia
  • Easy bruising from thrombocytopenia
  • Mass effects of myeloid sarcoma, such as intestinal obstruction
  • Higher white blood cell count at diagnosis of AML with inv(16)(p.13.1q22) and t(16;16)(p.13.1;q22) compared with other AMLs (J Clin Oncol 2005;23:5705)
Diagnosis
  • Diagnosis is based on a combination of morphology, flow cytometry, conventional cytogenetics, real time PCR and FISH analysis
Laboratory
  • Anemia and thrombocytopenia
Prognostic factors
  • Overall prognosis is good compared to other AMLs (Blood 2003;102:462)
  • KIT and FLT3 mutations are associated with worse prognosis
  • Trisomy 22 is associated with a more favorable outcome
  • High levels of minimal residual disease after induction are associated with worse outcomes (Am Soc Clin Oncol Educ Book 2018;38:555)
Case reports
Treatment
  • Induction chemotherapy with 3 days of anthracycline and 7 days of cytarabine (7+3) (Am Soc Clin Oncol Educ Book 2018;38:555)
  • Cytarabine or HDAC inhibitors are used for consolidation therapy
  • Gemtuzumab ozogamicin (toxin conjugated monoclonal antibody to CD33) may improve outcomes when combined with standard chemotherapy
Microscopic (histologic) description
  • Hypercellular bone marrow core biopsy and aspirate clot sections with presence of immature cells; increased eosinophilic cells can be seen
Microscopic (histologic) images

Contributed by Ameet R. Kini, M.D., Ph.D. and Maryam F. Raouf, M.D.
Increased blasts and promonocytes

Increased blasts and promonocytes

Abnormal eosinophilic cell

Abnormal eosinophilic cell

Hypercellular bone marrow

Hypercellular bone marrow



Images hosted on other servers:

AML with abnormal eosinophils (arrow)

Abnormal eosinophils with prominent purple granules

Abnormal eosinophils cluster in patient with AMML

Abnormal immature eosinophil found in bone marrow

Abnormal eosinophil with basophilic granules

Virtual slides

Images hosted on other servers:

AML with inv(16)

Cytology description
  • Increased blasts showing nuclei with fine chromatin and high nuclear to cytoplasmic ratio
  • Some blasts display Auer rods
  • Blasts are usually > 20% but the diagnosis is made even when blasts are < 20%, provided inv(16)(p13.1;q22) or t(16;16)(p13.1;q22) is seen
  • Increased eosinophils at all stages of maturation, with the earlier stages displaying dual granulation (Haematologica 2002;87:886)
  • Increased immature monocytic cells (promonocytes)
Peripheral smear description
  • Presence of blasts and immature monocytic cells
  • Eosinophils are not usually increased (Blood 1986;68:1242)
Positive stains
Negative stains
  • Negative for nonhematopoietic markers and most lymphoid markers
Flow cytometry description
Flow cytometry images

Contributed by Ameet R. Kini, M.D., Ph.D. and Maryam F. Raouf, M.D.
Blasts with dim CD45

Blasts with dim CD45

Blasts with CD34, CD15

Blasts with CD34, CD15

Increased blasts with HLA-DR, CD33

Increased blasts with HLA-DR, CD33

Blasts with CD13, CD117

Blasts with CD13, CD117

Molecular / cytogenetics description
  • Conventional cytogenetics shows presence of inv(16)(p13.1;q22) or t(16;16)(p13.1;q22)
  • FISH analysis may be required to detect cryptic cases and shows the CBFB-MYH11 fusion gene (Genes Chromosomes Cancer 1998;22:87)
  • In rare cases, real time PCR can detect the CBFB-MYH11 transcript when cytogenetics and FISH analyses are negative
  • Real time PCR is used to assess minimal / measurable residual disease (MRD)
Molecular / cytogenetics images

Contributed by Ameet R. Kini, M.D., Ph.D. and Maryam F. Raouf, M.D.
Inversion of chromosome 16

Inversion of chromosome 16

<i>CBFB</i> FISH

CBFB FISH

Videos

AML diagnosis

Targeted therapy for inv(16)


AML diagnosis and treatment

MRD in AML

Sample pathology report
  • Bone marrow, right posterior iliac crest, core biopsy, clot section, aspirate smears and touch imprint hypercellular bone marrow (95%) with involvement by acute myeloid leukemia with inv(16)(p13.1;q22) (see comment)
    • Comment: Bone marrow core biopsy is hypercellular for age (95%) Immature cells are increased. Erythroid precursors are decreased. Mature myeloid cells are decreased. Megakaryocytes are adequate and appear normal in morphology.
    • Aspirate clot section is adequate and shows features similar to the core biopsy.
    • Bone marrow aspirate smear is adequate. Blasts are increased (30%) and show a high nuclear to cytoplasmic ratio with fine chromatin. Immature monocytic cells (promonocytes) are also increased. Scattered eosinophils and eosinophil precursors are seen throughout the aspirate smear. Some of the eosinophilic cells show dual granulation. Megakaryocytes are adequate in number and show normal morphology. Cytochemical stains show that many of the blasts are positive for myeloperoxidase. Alpha naphthyl butyrate esterase (nonspecific esterase) is positive in the immature monocytic cells.
    • Touch preparation findings are similar to the core biopsy.
    • Flow cytometry shows a large abnormal myeloid blast population with expression of CD34, CD15 (subset), CD33, CD13, HLA-DR and CD117. There is also an immature monocytic population showing characteristic expression of CD64, CD11b and CD11c, and possible absence of CD14. Gating on the lymphocytes shows no evidence of a monoclonal B cell population or T cell abnormality based on the markers assayed.
    • Cytogenetic analysis shows presence of a pericentric inversion of chromosome 16. FISH analysis using a dual color break apart probe shows rearrangement of CBFB, consistent with inv(16)(p13.1;q22).
Differential diagnosis
Additional references
Board review style question #1

An otherwise healthy 15 year old child presents to clinic with fatigue. The patient’s mother states her child has decreased energy and seems to bruise easily. CBC shows anemia and thrombocytopenia. The bone marrow biopsy is hypercellular. The bone marrow aspirate smear (figure) shows the presence of blasts and increased abnormal eosinophils. Cytochemical stains show a dual population of blasts with a subset expressing myeloperoxidase and another subset nonspecific esterase. What is the most likely diagnosis?

  1. Acute lymphoblastic leukemia
  2. Acute megakaryoblastic leukemia
  3. Acute myeloid leukemia with inv(16)
  4. Acute promyelocytic leukemia with t(15;17)
  5. Hypereosinophilic syndrome
Board review style answer #1
C. Acute myeloid leukemia with inv(16)

Comment Here

Reference: AML with inv(16)(p13.1;q22) or t(16;16)(p13.1;q22)
Board review style question #2
The inv(16)(p13.1;q22) or t(16;16)(p13.1;q22) results in the formation of which abnormal fusion gene?

  1. BCR-ABL1
  2. CBFB-MYH11
  3. KMT2A-MLLT3
  4. PML-RARA
  5. RUNX1-RUNX1T1
Board review style answer #2

AML with inv(3)(q21.3;q26.2) or t(3;3)(q21.3;q26.2); GATA2, MECOM
Definition / general
Essential features
  • Aggressive form of AML with very poor prognosis and poor response to conventional induction chemotherapy (Cancer Genet 2019;230:28)
  • Diagnosed by presence of the pathognomonic cytogenetic abnormality inv(3)(q21q26.2) or t(3;3)(q21;q26.2) [inv(3) / t(3;3)] (Blood 2016;127:2391)
  • End result is juxtaposition of the distal GATA2 enhancer with the MECOM (EVI1) proto-oncogene (Blood 2016;127:2391)
  • May present as de novo disease or evolve from preexisting myelodysplastic syndrome (Cancer Genet 2019;230:28)
Terminology
  • AML with inv(3)
  • Acute myeloid leukemia with GATA2, MECOM
  • AML with RPN1 / EVI1
  • 3q21q26 syndrome (Hematology 2015;20:435)
  • AML with RPN1-MECOM
ICD coding
Epidemiology
Sites
  • Bone marrow
  • Peripheral blood
Pathophysiology
Etiology
Clinical features
  • Presence of at least 20% blasts in peripheral blood or bone marrow
  • Often presents with anemia
  • Elevated white blood cell count (sometimes low)
  • Normal or elevated platelet count
  • Association with central diabetes insipidus (Hematology 2015;20:435)
  • Elevated LDH
Diagnosis
  • Requires enumeration of 20% blasts on peripheral blood smear or marrow aspirate differential count(s)
  • Flow cytometry or immunohistochemical stains may assist in blast phenotyping
  • Juxtaposition of GATA2 and MECOM, inv(3)(q21q26.2) or t(3;3)(q21;q26.2) [inv(3) / t(3;3)] can be detected by classic cytogenetics, FISH and other molecular / genetic studies including chromosome microarray, next generation sequencing, RT-PCR, etc.
Laboratory
  • CBC with anemia
  • Leukocytosis (sometimes leukopenia) with 20% blasts
  • Normal or elevated platelet count
  • Elevated LDH
  • Flow cytometry analysis reveals increased dim CD45 events (blasts)
Prognostic factors
Case reports
Treatment
Microscopic (histologic) description
Microscopic (histologic) images

Contributed by Alexa J. Siddon, M.D.
Hypercellular marrow with increased blasts

Hypercellular marrow with increased blasts

Aspirate with hypolobated mega

Aspirate with hypolobated megakaryocyte

Aspirate with blasts

Aspirate with blasts

Peripheral smear description
Positive stains
Negative stains
Flow cytometry description
Molecular / cytogenetics description
  • inv(3)(q21q26.2) or t(3;3)(q21;q26.2) [inv(3) / t(3;3)]
  • Increased expression of EVI1 (MECOM)
  • Relative haploinsufficiency of GATA2
  • Associated monosomy 7, complex karyotype and 5q deletion
  • Gene mutations identified include those of NRAS, GATA2, SF3B1, FLT3-ITD, KIT D816, PTPN11, CBL, KRAS and BCR-ABL1 and CEBPA (Blood 2015;125:133, Int J Lab Hematol 2019;41:380)
Molecular / cytogenetics images

Images hosted on other servers:

Double inv(3)(q21q26) and monosomy 7

FISH with EVI1 breakapart probe

Sample pathology report
  • Right posterior iliac crest, core biopsy, aspirate smear, touch imprint and clot particle:
    • Acute myeloid leukemia with inv(3)(q21.3;q26.2); GATA2, MECOM (see comment)
    • Comment: The bone core biopsy demonstrates an increase in blasts and prominent dysmegakaryocytopoiesis. The presence of inv(3)(q21.3;q26.2) confirms above diagnosis.
Differential diagnosis
  • Other AML with recurrent cytogenetic abnormalities:
    • Presence of other subtype defining cytogenetic abnormalities different from inv(3) / t(3;3)
  • MDS with inv(3):
    • Blast count < 20%
Board review style question #1
Which of the following is true about AML with inv(3) / t(3;3)?

  1. CD41 positivity on immunophenotypic studies is a key part of the diagnostic criteria
  2. Deletion of chromosome 5q is the most frequently associated cytogenetic abnormality
  3. Expression of CD7 is not consistent with the entity
  4. Expression of MECOM (EVI1) is commonly upregulated
  5. GATA2 fusion with MECOM (EVI1) is the characteristic molecular feature
Board review style answer #1
D. Expression of MECOM (EVI1) is commonly upregulated

Comment Here

Reference: AML with inv(3)(q21.3;q26.2) or t(3;3)(q21.3;q26.2)
Board review style question #2
Which of the following is true about AML with inv(3) / t(3;3)?

  1. Anemia and thrombocytopenia are hallmark findings on the peripheral smear
  2. Bone marrow is often hypocellular with extensive fibrosis
  3. Erythroid dysplasia is uncommon unlike in other subtypes of AML
  4. Finding of pseudo-Pelger-Hüet neutrophils suggest an alternative diagnosis
  5. Increased megakaryocytic dysplasia is commonly observed on bone marrow aspiration / biopsy
Board review style answer #2
E. Increased megakaryocytic dysplasia is commonly observed on bone marrow aspiration / biopsy

Comment Here

Reference: AML with inv(3)(q21.3;q26.2) or t(3;3)(q21.3;q26.2)

AML with maturation (FAB AML M2)
Definition / general
  • 10% of AML cases; 5% of childhood leukemias
  • Any age, 20% are < 25 years and 40% are 60 years+
  • Anemia, thrombocytopenia, neutropenia; variable number of blasts in peripheral blood
  • Variable prognosis
  • Criteria for diagnosis: 20%+ nonerythroid cells in peripheral blood or bone marrow are myeloblasts; monocytic precursors are < 20% in bone marrow and granulocytes are 10%+ of cells
  • Enzyme cytochemistry: most blasts are positive for myeloperoxidase or Sudan Black B, and chloroacetate esterase
Case reports
Microscopic (histologic) description
  • Usually hypercellular marrow
  • Full range of myeloid maturation through maturing neutrophils, often with abnormal segmentation and 10%+ bone marrow cells with variable degree of dysplasia
  • Auer rods in 70% of blasts; myeloblasts with or without azurophilic granules
  • Erythroid and megakaryocyte precursors may have dysplastic changes
  • Often increased eosinophilic precursors without cytological and cytochemical abnormalities of inv(16)(p13.1q22)
  • Basophils may be increased, rarely mast cell hyperplasia (Indian J Pathol Microbiol 2007;50:655)
Microscopic (histologic) images

AFIP images
Missing Image

Bone marrow
biopsy: markedly
hypercellular
marrow

Missing Image Missing Image Missing Image

Diagnosed as AML with maturation because no t(15;17) and no DIC but FISH not done, so may actually be acute promyelocytic leukemia

Missing Image

Myeloperoxidase positive blasts

Missing Image

Erythroid cells are negative



Bone marrow smears (Wright-Giemsa):
Missing Image

Type III myeloblasts

Missing Image

Myeloblasts,
promyelocytes,
myelocytes and
neutrophils

Missing Image

Several myeloblasts and maturing forms

Missing Image

Pseudopods, cytoplasm and prominent Auer rods

Missing Image

Several blasts have prominent nucleoli and Auer rods

Positive stains
Negative stains
Electron microscopy images

AFIP images
Missing Image

Numerous primary granules and fusion of Auer rods

Molecular / cytogenetics description

AML with minimal differentiation (FAB AML M0)
Definition / general
  • No definitive evidence of myeloid differentiation by morphology and light microscopy cytochemistry; need immunohistochemistry, flow cytometry, or EM cytochemistry to characterize as myeloid
  • Criteria for diagnosis: nongranular blasts; less than 3% of blasts are positive for myeloperoxidase (MPO), Sudan Black B (SBB) or naphthol-ASD-chloroacetate esterase (CAE) by enzyme cytochemistry, although blasts may express myeloperoxidase by EM or immunohistochemistry; blasts do not express classic lymphocyte antigens, but may aberrantly express some lymphocyte antigens
Clinical features
  • 5% or less of AML cases; any age, mostly infants or older adults
  • Typically presents with anemia, thrombocytopenia, neutropenia, marrow failure, but may have leukocytosis with markedly increased blasts
  • Children (Blood 2007;109:2314) and adults (Br J Haematol 2001;113:737) may have poorer outcome than other AML subtypes
  • Enzyme cytochemistry: negative for nonspecific esterase, although may have nonspecific weak or focal reaction distinct from monocytic cells
Case reports
Microscopic (histologic) description
  • Medium sized blasts, round or slightly indented nuclei with dispersed chromatin
  • One or two nucleoli, agranular cytoplasm with varying degree of basophilia and no Auer rods
  • Rarely small blasts with condensed chromatin and scant cytoplasm that may resemble lymphoblasts
  • Occasionally, residual normal population of maturing neutrophils may be present
  • Bone marrow: markedly hypercellular with poorly differentiated blasts
Microscopic (histologic) images

AFIP images
Missing Image

Bone marrow smear
(Wright-Giemsa):
no differentiated
features

Missing Image

Bone marrow biopsy:
complete replacement
of marrow by blasts
without differentiation

Missing Image

Myeloblasts are negative for myeloperoxidase



Images hosted on other servers:
Missing Image

Bone marrow smear
(Wright-Giemsa):
no differentiated
features

Missing Image

Blasts are positive for myeloperoxidase by IHC

Flow cytometry description
Flow cytometry images

Images hosted on other servers:
Missing Image

Various gating plots

Positive stains
Negative stains
Molecular / cytogenetics description
  • Often complex chromosomal abnormalities; tend to have more -5/del(5q), -7/del(7q), +8 and del(11q), but these should be reclassified as AML-MRC
  • AML1 / CBFA / RUNX1 (27%) and strong association with trisomy 13 and FLT3 mutation (16 - 22%, Haematologica 2007;92:1123)
  • More often trisomy 21 and hypodiploidy than other AML, although outcome is similar (Blood 2007;109:2314)
Electron microscopy description
  • Resembles myeloblasts; may show focal myeloperoxidase+ granules
Electron microscopy images

AFIP images
Missing Image

Granules are myeloperoxidase positive



Images hosted on other servers:
Missing Image

Granules are myeloperoxidase positive

Differential diagnosis

AML with mutated NPM1
Definition / general
  • Acute myeloid leukemia (AML) with mutations in the NPM1 gene, a common recurrent genetic abnormality found in a subset of patients with AML
Essential features
  • One of the most common recurrent genetic abnormalities in AML
  • Associated with better prognosis
  • Associated with monocytic and myelomonocytic morphology
  • NPM1 immunohistochemistry showing cytoplasmic localization may be used as a surrogate for PCR
Terminology
  • Acute myeloid leukemia with cytoplasmic nucleophosmin
ICD coding
  • ICD-11: 2A60.0 - acute myeloid leukemia with recurrent genetic abnormalities
Epidemiology
  • Typically presents de novo in adults
  • F > M
Sites
  • Bone marrow
  • Extramedullary involvement may occur
    • Most common sites are gingiva, lymph nodes and skin
Pathophysiology
  • Exon 12 mutation leading to aberrant localization of nucleophosmin and nucleophosmin associated proteins in the cytoplasm; while not completely understood, this may cause several hypothetical leukemogenic events (Leukemia 2017;31:798)
    • NPM1 mutation causes HOX gene upregulation which supports the leukemic state in NPM1 mutant AML (Cancer Cell 2018;34:499)
Etiology
  • Unknown at this time
Clinical features
  • NPM1 mutations are one of the most common recurrent genetic lesions in AML
    • Occurs in approximately 35% of AML cases overall and 50 - 60% of cases with normal karyotype (Blood Rev 2018;32:167
  • Patients often present with anemia and thrombocytopenia, although platelet counts are usually higher than other AML subtypes (Blood 2005;106:3740
Diagnosis
  • Requisite blast percentage for a diagnosis of AML is ≥ 20% myeloblasts, monoblasts / promonocytes or megakaryoblasts in the peripheral blood or bone marrow
  • NPM1 mutation
    • Detection via molecular techniques
    • Aberrant cytoplasmic expression detected via immunohistochemistry
Prognostic factors
  • AML with mutated NPM1, normal karyotype and absence of FLT3 internal tandem duplication mutation has characteristically favorable prognosis
    • In younger patients, the prognosis is similar to AML with t(8;21) or AML with inv(16)
  • NPM1 and FLT3 tyrosine kinase domain paired mutations offer higher overall and event free survival (Blood 2008;111:2527)
  • Myeloid immunophenotype is associated with poorer relapse free survival and overall survival and is associated with more CD34 expression (Blood Adv 2019;3:3322)
Case reports
Treatment
  • Most patients receive conventional induction and consolidation chemotherapy with cytarabine and an anthracycline
  • Patients do not receive allogeneic stem cell transplant
  • Possible role of all trans retinoic acid (ATRA) as an adjuvant to induction therapy
    • ATRA and arsenic work synergistically to degrade cytoplasmic NPM1, inducing apoptosis, growth arrest and differentiation
      • Also shown to sensitize NPM1 mutant leukemic cells to daunorubicin
  • Addition of gemtuzumab ozogamicin (GO), a CD33 antigen targeting immunoconjugate, has shown an increase in the likelihood of NPM1 mutation negative minimal residual disease (Leukemia 2017;31:798)
Microscopic (histologic) description
  • Wide range of morphologies are seen in AML with NPM1 mutation (N Engl J Med 2005;352:254)
  • There is a strong association between both acute myelomonocytic and acute monocytic leukemia and NPM1 mutation (Blood 2007;109:874)
  • NPM1 mutations are also detected in AML with and without maturation and in pure erythroid leukemia
  • Cup-like nuclei are highly associated with the presence of NPM1 and FLT3 internal tandem duplication mutations (Cancer 2009;115:5481)
Microscopic (histologic) images

Contributed by @DrVasudevPrabhu on Twitter
AML with mutated NPM1

AML with mutated NPM1

Peripheral smear images

Contributed by Kamran M. Mirza, M.D., Ph.D.
Blasts with cup-like indentations Blasts with cup-like indentations

Blasts with cup-like indentations

Positive stains
Negative stains
  • CD34 is negative in most cases
Flow cytometry description
Molecular / cytogenetics description
  • 4 base pair insertion in exon 12 of NPM1; detected via reverse transcription PCR
Sample pathology report
  • Bone marrow, left posterior iliac crest, bone marrow core biopsy, aspirate, clot and peripheral blood:
    • Acute myeloid leukemia with mutated NPM1
    • Pancytopenia with rare circulating blasts (see comment)
    • Comment: Peripheral smear shows pancytopenia with macrocytic anemia and mild anisopoikilocytosis with microcytes, macrocytes and occasional elliptocytes. Rare circulating blasts are present.
    • The bone marrow core biopsy is of sufficient length for evaluation. There are multiple patchy hypercellular areas showing shows increased numbers of blasts in clusters with markedly decreased maturing / mature myeloid forms. Few scattered megakaryocytes with normal morphology are apparent. Erythropoiesis is decreased, although maturing precursors are still present in clusters.
    • The aspirate smear shows that many of the blasts have a folded nuclei with often bilobed nuclei with prominent nucleoli in a few blasts reminiscent of cup-like nuclear invaginations in a proportion of blasts (8 - 9%) and moderate amount of gray-blue cytoplasm. Auer rods are not present. A differential count yields 83% blasts. Few residual erythroid precursors and occasional megakaryocytes are present without significant dysplasia in either of these 2 cell lines. Very few residual maturing myeloid forms are present. Iron stain shows decreased sideroblastic iron and adequate storage iron. Cytochemistry stains (alpha naphthyl butyrate esterase and myeloperoxidase) are performed and show the following: the blasts are myeloperoxidase positive (5 - 10% of blasts) and alpha naphthyl butyrate (monocyte) esterase is negative in all the blasts.
    • Flow cytometry analysis shows an abnormal myeloid blast population expressing dim CD45, dim CD56 (subset), dim CD38, CD34, dim CD33, CD117 and dim CD13. The blasts are negative for HLA-DR.
    • NPM1 mutation analysis confirms the presence of NPM1 mutation
Differential diagnosis
Board review style question #1
A 50 year old woman presented with generalized gingival swelling, pain and bleeding. The patient's initial laboratory workup showed leukocytosis, anemia and thrombocytosis. A bone marrow biopsy is performed which showed increased blasts with cup-like morphology and cytoplasmic NPM1 staining. Which of the following is true about this entity?

  1. Blasts are frequently CD34 positive
  2. It has poor prognosis
  3. It is most frequently associated with normal karyotypes
  4. It is rare
Board review style answer #1
C. AML with mutated NPM1 frequently has normal karyotypes.

Comment Here

Reference: AML with mutated NPM1
Board review style question #2
Which of the following histologic features is acute myeloid leukemia with mutated NPM1 most associated with?

  1. Coin on coin nuclear morphology
  2. Dwarf megakaryocytes
  3. Immature eosinophils with large, violet granules
  4. Monocytic and myelomonocytic morphology
Board review style answer #2
D. Monocytic and myelomonocytic morphology is commonly associated with NPM1 mutation.

Comment Here

Reference: AML with mutated NPM1

AML with mutated RUNX1
Definition / general
  • Due to the new classifications, this topic is now covered here
  • Prior to WHO 2022 and ICC 2022:
    • Provisional entity in the 2016 WHO classification
    • De novo acute leukemia diagnosed with ≥ 20% blasts in blood or bone marrow
    • All morphological subtypes of AML, NOS may harbor RUNX1 mutation, with the highest frequency being AML with minimal differentiation
    • Diagnosis of AML with mutated RUNX1 excludes cases that fulfill criteria for other specific AML with recurrent genetic abnormalities, therapy related myeloid neoplasms or AML with myelodysplasia related changes (Blood 2016;128:462, Leukemia 2016;30:2282)
    • Unfavorable prognostic implication, with mutated RUNX1 being associated with inferior survival than wildtype (Leukemia 2016;30:2282, J Clin Oncol 2012;30:3109)
Essential features
  • Provisional WHO entity describing a de novo acute myeloid leukemia
  • Cases possess monoallelic RUNX1 mutations (typically frameshift or missense)
  • Blast morphology most compatible with M0
  • Cases do not meet diagnostic criteria for other AML with genetic abnormality
  • Highest incidence in men and patients over 60 years of age
  • Associated gene mutations include those of SRSF2 (39%), ASXL1 (36%), DNMT3A (19%), IDH2 (17%) and SF3B1 (17%) (Leukemia 2018;32:295)
  • Other coexisting gene mutations include those of FLT3-ITD, TET2, RAS, PTPN11, p53, JAK2 and EZH2 genes (Leukemia 2018;32:295)
ICD coding
Epidemiology
Sites
  • Peripheral blood
  • Bone marrow
Etiology
  • Runt related transcription factor 1 (RUNX1) is known to play a key role in hematopoiesis as a key regulator of genes involved in the differentiation of myeloid cells (Int J Mol Sci 2017;18:1618)
  • RUNX1 mutation leads to upregulation of those genes normally expressed in primitive hematopoietic cells and B cell progenitors and downregulation of promoters of myelopoiesis (J Clin Oncol 2012;30:3109)
  • Mutation of RUNX1 therefore predisposes to the development of AML with more immature / undifferentiated morphology (M0 in the French-American-British [FAB] classification) (Leukemia 2016;30:2282, Int J Mol Sci 2017;18:1618)
  • Associated gene mutations include those of SRSF2 (39%), ASXL1 (36%), DNMT3A (19%), IDH2 (17%) and SF3B1 (17%) (Leukemia 2018;32:295)
  • Other coexisting gene mutations include those of FLT3-ITD, TET2, RAS, PTPN11, p53, JAK2 and EZH2 genes (Leukemia 2018;32:295)
Clinical features
  • Anemia
  • Leukopenia
  • Thrombocytopenia
  • Low LDH
Diagnosis
  • Bone marrow aspiration / biopsy to enumerate ≥ 20% blasts
  • Flow cytometric evaluation of peripheral blood and bone marrow aspirate to confirm blast phenotype
  • Next generation sequencing (NGS) or mutational analysis to evaluate for any RUNX1 alterations
Prognostic factors
Case reports
Treatment
  • AML with mutated RUNX1 is currently treated with standard induction chemotherapy, including regimens consisting of cytarabine and an anthracycline
  • Use of hypomethylating agent may negate the effect of RUNX1 in elderly patients (Int J Mol Sci 2017;18:1618)
  • Stem cell transplantation is a therapeutic option for suitable patients
  • Use of BET protein inhibitors in combination with agents such as venetoclax, decitabine or cytarabine, has shown promise in vitro (Leukemia 2018;32:295)
Microscopic (histologic) description
  • Majority of cases (~60%) show the M0 (FAB classification) morphology, with very immature / undifferentiated cells (Int J Mol Sci 2017;18:1618, Blood 2011;117:2348)
  • Blasts have variable size, high nucleus to cytoplasmic ratio and variably prominent nucleoli with no evidence of differentiation
  • M2 and M1 morphology are the next most common (Blood 2011;117:2348)
  • RUNX1 mutated AMLs typically demonstrate high bone marrow blast counts (Blood 2011;117:2348)
Microscopic (histologic) images

Contributed by Genevieve M. Crane, M.D., Ph.D.
Bone marrow core biopsy Bone marrow core biopsy Bone marrow core biopsy

Bone marrow core biopsy

Bone marrow clot, AML mutated RUNX1

Bone marrow clot

Bone marrow clot, CD34

Bone marrow clot, CD34

Bone marrow clot, CD117

Bone marrow clot, CD117

Cytology images

Contributed by Genevieve M. Crane, M.D., Ph.D.
Bone marrow aspirate with increased blasts

Bone marrow aspirate with increased blasts

Peripheral smear description
  • Lower hemoglobin, white blood cell and blast counts compared with wild type RUNX1
  • Thrombocytopenia
Peripheral smear images

Contributed by Genevieve M. Crane, M.D., Ph.D.
Peripheral blood smear with increased blasts

Peripheral blood smear with increased blasts

Positive stains
Negative stains
  • Stains for B and T cell differentiation are negative
Molecular / cytogenetics description
  • Mutation of RUNX1 (located on chromosome 21q22.12) is disease defining (Blood 2016;128:462)
    • Most RUNX1 mutations are monoallelic, within the runt homology domain (RHO) and the transactivation domain (TAD)
    • RUNX1 mutations are typically frameshift or missense
  • Known associated gene mutations include those of SRSF2, ASXL1, DNMT3A, IDH2, SF3B1, FLT3-ITD, TET2, RAS, PTPN11, p53, JAK2 and EZH2 genes (Leukemia 2018;32:295)
Sample pathology report
  • Right posterior iliac crest, bone core biopsy and aspirate smear:
    • Acute myeloid leukemia with mutated RUNX1 (see comment)
    • Comment: The peripheral blood smear reveals pancytopenia with circulating blasts. The blasts are intermediate in size with high N:C ratio and variably prominent nucleoli. The marrow core biopsy and aspirate smear are packed with sheets of blasts which demonstrate minimal to no myeloid differentiation. Flow cytometry analysis of the marrow aspirate reveals the blast phenotype to be dim CD45, with expression of CD34 and CD13 and absence of B and T lineage markers. Next generation sequencing demonstrates the presence of a missense pathogenic RUNX1 mutation with a variant allele frequency of 65%. FISH panel and cytogenetic analysis are normal.
Differential diagnosis
Board review style question #1
Which of the following is the most common morphologic correlation (FAB) seen in AML with mutated RUNX1?

  1. M0 (AML with minimal maturation)
  2. M1 (AML without maturation)
  3. M3 (AML with PML-RARA)
  4. M4 (acute myelomonocytic leukemia)
  5. M7 (acute megakaryocytic leukemia)
Board review style answer #1
A. M0 (AML with minimal maturation)

Comment Here

Reference: AML with mutated RUNX1
Board review style question #2
Which of the following statements is true about AML with mutated RUNX1?

  1. ASXL1 and SRSF2 are the most commonly comutated genes
  2. Comutation with FLT3-ITD negates the poor prognostic impact of RUNX1
  3. Incidence is higher in people younger than 50 years
  4. It is associated with a better prognosis than AML with RUNX1-RUNX1T1 fusion
  5. It has lower frequency of CD34 positivity in blast cells than other subtypes of AML
Board review style answer #2
A. ASXL1 and SRSF2 are the most commonly comutated genes

Comment Here

Reference: AML with mutated RUNX1

AML with mutated TP53 (pending)
[Pending]

AML with myelodysplasia defining abnormalities
Definition / general
  • ≥ 20% myeloid blasts in the blood or bone marrow with 1 or more of the following: features of myelodysplasia, prior history of myelodysplastic syndrome (MDS) or MDS / myeloproliferative neoplasm (MPN) or MDS related cytogenetic abnormalities
  • Requires absence of prior cytotoxic / radiation therapy and absence of a recurrent cytogenetic abnormality
  • In de novo acute myeloid leukemia (AML) with mutated NPM1 or biallelic mutation of CEBPA, multilineage dysplasia alone or del(9q) alone are insufficient
  • Includes refractory anemia with excess blasts in transformation defined by French-American-British (FAB) classification
Essential features
  • AML with morphological features of myelodysplasia in > 50% of the cells of 2 or more lineages or a prior history of MDS or MDS / MPN or MDS related cytogenetic abnormalities
  • Generally poorer prognosis with lower remission rates and shorter overall survival than other AML subtypes (Am J Clin Pathol 2020;154:731)
  • Commonly presents with severe pancytopenia
Terminology
  • Acute myeloid leukemia with multilineage dysplasia
  • Acute myeloid leukemia with prior myelodysplastic syndrome
  • Acute myeloid leukemia with myelodysplasia related changes (AML-MRC)
ICD coding
  • ICD-O: 9895/3 - acute myeloid leukemia with myelodysplasia related changes
Epidemiology
Sites
  • Blood and bone marrow
Clinical features
  • Frequently presents with severe pancytopenia
  • Cases with 20 - 29% blasts may have a slowly progressive course with less marked cytopenias (Am J Hematol 2014;89:E193)
Diagnosis
  • Made based on the following (Surg Pathol Clin 2010;3:1153):
    • Microscopic features, including bone marrow blast percentage and the presence and degree of dysplasia
    • Immunophenotyping to confirm myeloid lineage
    • Results of cytogenetic studies
    • Clinical correlation to evaluate for prior history of MDS or MDS / MPN and exclude prior cytotoxic or radiation therapy for an unrelated disease
Prognostic factors
  • Generally poorer prognosis with lower remission rates and shorter overall survival than other AML subtypes (Am J Clin Pathol 2020;154:731)
  • Diagnosis of AML-MRC solely based on multilineage dysplasia or previously untreated MDS or MDS / MPN have significantly better outcomes than those with MDS defining cytogenetic abnormalities or disease secondary to a previously treated MDS or MDS / MPN (Am J Hematol 2020;95:612)
  • In de novo AML cases lacking myelodysplasia related cytogenetic abnormalities, select dysplastic features may carry more prognostic significance than others with micromegakaryocytes and hypogranulated myeloid cells identified as significantly associated with shorter event free survival (Mod Pathol 2015;28:965)
Case reports
  • 47 year old man with a history of Crohn's disease and new onset acute myeloid leukemia with myelodysplasia related changes (Case Rep Oncol 2018;11:573)
  • 54 year old man with a prior history of acute myeloid leukemia with myelodysplasia related changes presenting with new onset pleural effusion with leukemic blasts (Diagn Cytopathol 2011;39:451)
  • 77 year old man with acute myeloid leukemia with myelodysplasia related changes (Intern Med 2011;50:3037)
  • 77 year old man with acute myeloid leukemia with myelodysplasia related changes demonstrating mixed lineage phenotype (Blood 2016;128:1663)
  • 84 year old woman with acute myeloid leukemia with myelodysplasia related changes showing basophilic differentiation (Am J Hematol 2014;89:1082)
Treatment
Microscopic (histologic) description
  • Blasts 20% or more
  • Dysplasia in > 50% of the cells in 2 or more lineages
  • Dyserythropoiesis: nuclear budding, megaloblastosis, multinucleation, nuclear budding, irregular nuclear contours, cytoplasmic vacuoles, karyorrhexis, ring sideroblasts
  • Dysgranulopoiesis: hypogranular cytoplasm, hyposegmented nucleus, abnormal nuclear segmentation
  • Dysmegakaryopoiesis: micromegakaryocytes, nuclear hyposegmentation, nuclear hypersegmentation, separated nuclear lobes
  • References: Surg Pathol Clin 2010;3:1153, Am J Clin Pathol 2020;154:731
Microscopic (histologic) images

Contributed by Srishti Gupta, M.B.B.S., M.D. and Elizabeth Courville, M.D.

Hypercellular bone marrow

Dysmegakaryopoiesis

Dysmegakaryopoiesis

CD34 positive blasts


Hypercellular bone marrow

Dysmyelopoiesis and blasts

Myeloid dysplasia with blasts

Bilineage dysplasia

Peripheral smear description
  • Nucleated red cells with dysplastic features in the form of nuclear budding and nuclear irregularities
  • Increased red blood cell anisopoikilocytosis
  • Large, hypogranular platelets
  • Circulating blasts may be present
  • Hypogranular or hypolobated myeloid elements
  • Cytopenias
  • Reference: Jaffe: Hematopathology, 2nd Edition, 2016
Peripheral smear images

Contributed by Srishti Gupta, M.B.B.S., M.D. and Elizabeth Courville, M.D.

Hypogranular neutrophils, blast

Hypogranular, hypolobate neutrophil, blast

Hypogranular, hypolobate neutrophil

Positive stains
Flow cytometry description
  • Variable immunophenotypic features
  • Immunophenotypic features are those reported in myelodysplastic syndromes, including reduced side scatter reflecting hypogranularity of neutrophils, aberrant differentiation patterns, loss of hematogones and aberrant expression of CD56 or CD7 (Acta Haematol 2019;141:232)
Molecular / cytogenetics description
  • Complex karyotypes, loss of chromosome 7 / del(7q), del(5q) and unbalanced translocations involving 5q are the most common cytogenetic abnormalities
  • Balanced translocations are less common but frequently involve 5q32-33
  • Presence of a specific genetic abnormality characteristic of AML with recurrent genetic abnormalities as defined in the WHO excludes a diagnosis of AML-MRC
  • NPM1 mutation and biallelic mutation of CEBPA are fairly uncommon and are rarely associated with MDS related cytogenetic abnormalities
  • MDS related mutations (including in U2AF1, ASXL1, TP53) are more frequent in this entity than in AML, NOS (Mod Pathol 2015;28:706)


Table 1: MDS related cytogenetic abnormalities (Am J Clin Pathol 2020;154:731)
Balanced translocation
  • t(11;16)(q23.3;p13.3)
  • t(3;21)(q26.2;q22.1)
  • t(1;3)(p36.3;q21.2)
  • t(2;11)(p21;q23.3)
  • t(5;12)(q32;p13.2)
  • t(5;7)(q32;q11.2)
  • t(5;17)(q32;p13.2)
  • t(5;10)(q32;q21.2)
  • t(3;5)(q25.3;q35.1)
Unbalanced translocation
  • -7 / del(7q)
  • del(5q) / t(5q)
  • i(17q) / t(17p)
  • -13 / del(13q)
  • del(11q)
  • del(12p) / t(12p)
  • idic(X)(q13)
Complex cytogenetic abnormality 3 or more unrelated cytogenetic abnormalities, excluding
the recurrent cytogenetic abnormalities seen in AML
Molecular / cytogenetics images

Contributed by Eli Williams, Ph.D.

Deletion of 7(q)

Sample pathology report
  • Posterior iliac crest, left / right, bone marrow aspirate, clot section and biopsy:
    • Acute myeloid leukemia with myelodysplasia related changes (see comment)
    • Comment: The patient's specimen demonstrates a hypercellular marrow with myeloid predominant trilineage hematopoiesis and increased blasts (> 20% by aspirate count and immunohistochemical stains). The peripheral blood shows leukopenia, thrombocytopenia and normocytic normochromic anemia. Flow cytometry analysis identified a large population of abnormal myeloid blasts. Overall, the findings in the current specimen are consistent with acute myeloid leukemia. Given the patient's history of myelodysplastic syndrome, acute myeloid leukemia with myelodysplastic related changes is a compatible diagnosis if a therapy related AML is clinically excluded and a defined recurrent cytogenetic abnormality is not identified (see WHO). Correlation with cytogenetic studies is recommended.
Differential diagnosis
  • Therapy related acute myeloid leukemia:
    • Characterized by history of cytotoxic chemotherapy or relevant radiation therapy for an unrelated disease
  • AML with recurrent cytogenetic abnormalities:
    • AML with t(8;21), inv(3) or t(6;9) may show evidence of multilineage dysplasia
    • These entities are defined by the presence of defined recurrent cytogenetic abnormalities
  • Acute panmyelosis with myelofibrosis:
    • Marrow is hypercellular and shows panmyelosis within a diffusely fibrotic stroma
    • No prior history of MDS or MDS / MPN
    • No AML-MRC defining cytogenetic abnormalities
    • Distinction from AML-MRC with myelofibrosis may be difficult
  • Acute megakaryoblastic leukemia:
    • No prior history of MDS or MDS / MPN
    • No AML-MRC defining cytogenetic abnormalities
    • No accompanying granulocytic or erythroid dysplasia
  • Pure erythroid leukemia:
    • Characterized by erythroid precursors representing > 80% of the bone marrow cells, with ≥ 30% proerythroblasts
    • Some of the cases previously classified as erythroleukemia (erythroid / myeloid type) may be categorized as AML-MRC according to the current classification system (Blood 2016;127:2391)
    • Pure erythroid leukemia should not be diagnosed as AML with myelodysplasia related changes, even if there is a history of a prior myeloid neoplasm, as AML-MRC by definition requires myeloblasts whereas the neoplastic cells in pure erythroid leukemia are erythroid blasts
Board review style question #1

A patient is diagnosed with de novo myeloid neoplasm. There is multilineage dysplasia (> 50% in 2 of the lineages) and > 20% myeloid blasts, as you can see in the above image from the bone marrow aspirate. The karyotype is normal and an NPM1 mutation is identified. What is the most appropriate diagnosis?

  1. Acute erythroid leukemia
  2. Acute myeloid leukemia with myelodysplasia related changes
  3. Acute myeloid leukemia, NOS
  4. Therapy related myeloid neoplasm (acute myeloid leukemia)
Board review style answer #1
C. Acute myeloid leukemia, NOS

Comment Here

Reference: AML with myelodysplasia related changes
Board review style question #2
A 69 year old woman is diagnosed with acute myeloid leukemia. Which of the following supports a diagnosis of acute myeloid leukemia with myelodysplasia related changes?

  1. Biallelic mutation of CEBPA
  2. History of below the knee amputation for diabetic complications
  3. History of cytotoxic chemotherapy for breast carcinoma
  4. History of myelodysplastic syndrome (MDS)
Board review style answer #2
D. History of myelodysplastic syndrome (MDS)

Comment Here

Reference: AML with myelodysplasia related changes

AML with other KMT2A rearrangements
Definition / general
  • Acute myeloid leukemia (AML) with other KMT2A rearrangements: subclassification of AML defined by the International Consensus Classification (ICC) as AML with rearrangement of the KMT2A gene other than t(9;11) / MLLT3::KMT2A
  • Recurrent translocations recognized in this category include AMLs with t(4;11) AFF1::KMT2A; t(6;11) / AFDN::KMT2A; t(10;11) / MLLT10::KMT2A; t(10;11) / TET1::KMT2A; t(11;19) / KMT2A::ELL; t(11;19) / KMT2A::MLLT1
  • In the 5th edition of WHO, the category of AML with KMT2A rearrangement includes any KMT2A gene rearrangement, regardless of the partner gene
Essential features
  • KMT2A rearrangement is a recurrent genetic finding identified in both acute lymphoblastic leukemias / lymphomas (ALL) and AML
  • KMT2A rearrangement has an inverse association with patient age, with a large subset of infantile cases of AML presenting with KMT2A rearrangement
  • AML with KMT2A rearrangement most commonly presents with a monocytic phenotype, with myelomonocytic and myeloid phenotypes less frequent
  • KMT2A has numerous partner genes; by International Consensus Classification (ICC) 2022 definition, AML with other KMT2A rearrangements include t(4;11) AFF1::KMT2A; t(6;11) / AFDN::KMT2A; t(10;11) / MLLT10::KMT2A; t(10;11) / TET1::KMT2A; t(11;19) / KMT2A::ELL; t(11;19) / KMT2A::MLLT1
  • Overall prognosis is poor but prognosis varies somewhat with fusion gene
  • KMT2A rearrangement can be identified by various testing methods including conventional karyotype, FISH, PCR and NGS
Terminology
  • KMT2A gene was previously named mixed lineage leukemia (MLL)
  • WHO 5th edition classification: acute myeloid leukemia with KMT2A rearrangement
  • ICC 2022 classification: acute myeloid leukemia with other KMT2A rearrangements
ICD coding
  • ICD-O: 9897/3 - acute myeloid leukemia with KMT2A rearrangement
  • ICD-10
    • C92 - acute myeloid leukemia
    • C92.6 - acute myeloid leukemia with 11q23 abnormality
  • ICD-11: 2A60.0 & XH1E41 - acute myeloid leukemia with recurrent genetic abnormalities & acute myeloid leukemia, 11q23 abnormalities
Epidemiology
  • KMT2A rearrangement
    • Incidence has an inverse relationship with age (Blood 2003;102:2395)
    • Most frequently encountered genetic abnormality in pediatric population, accounting for 20% of pediatric AML cases
    • Less common among adult population, accounting for 2 - 3% of adult AML cases
    • Has increased prevalence among patients with AML post-cytotoxic therapy (AML pCT) (Cytometry B Clin Cytom 2022;102:123)
Sites
  • By definition, AML with other KMT2A rearrangement involves bone marrow and peripheral blood
  • Prevalence of extramedullary disease is higher than that of non-KMT2A rearranged leukemia in both adults and pediatric populations with a high frequency of fusion partners (Proc Natl Acad Sci U S A 2020;117:26340)
  • Extramedullary disease most frequently manifests in the liver, spleen or CNS; less commonly as myeloid sarcoma in other locations
  • Data is limited for low frequency fusion partners
Pathophysiology
  • Myeloid precursor cell undergoes genetic event resulting in rearrangement of KMT2A gene on chromosome 11
    • KMT2A rearrangement in AML usually manifests as a translocation
  • Rearrangement of KMT2A disrupts its usual role in regulating gene expression during early development and hematopoiesis; this disruption will lead to clonal expansion of the myeloid cell lineage harboring the KMT2A (Cancer Lett 2019;458:56)
  • Clonal expansion will result in increased myeloid lineage blasts, further developing into bone marrow replacement and subsequent pancytopenias
Etiology
  • Definitive etiology is not well understood
  • Risk factors in adult patients include cytotoxic chemotherapy and other nonspecific DNA damaging events
Diagrams / tables

Images hosted on other servers:
Frequent <i>KMT2A</i> partner genes

Frequent KMT2A partner genes

Diagnosis
  • WHO criteria for AML with KMT2A rearrangement
    • Essential
      • Myeloid neoplasm with increased peripheral blood or bone marrow blasts (may be < 20%)
      • Blasts express a myeloid immunophenotype, not fulfilling immunophenotypic criteria for mixed phenotype AML
      • Presence of KMT2A rearrangement
      • Not fulfilling diagnostic criteria for myeloid neoplasm post-cytotoxic therapy
    • Desirable
      • Identification of the KMT2A fusion partner
    • ICC 2022 criteria for AML with other KMT2A rearrangement (Blood 2022;140:1200)
      • Overall criteria similar to the WHO, except importantly t(9;11) / MLLT3::KMT2A is categorized as a separate entity
      • Recognized recurrent fusion partners are described in other sections
    • Identification of KMT2A rearrangement is critical for this diagnosis (see Molecular / cytogenetics description for further description)
    • At least 10% myeloblasts in the peripheral blood or bone marrow sampling are required for diagnosis by ICC classification
Laboratory
  • Anemia and thrombocytopenia likely present at diagnosis
  • WBC will vary case to case; ranges from leukopenia to leukocytosis and correlates with number of circulating blasts
Prognostic factors
  • In general, prognosis of KMT2A rearranged AML is poor with high risk of relapse; however, prognosis varies upon fusion partner to KMT2A gene (Blood Cancer J 2021;11:162)
    • Few partner genes may give intermediate to good prognosis
  • Overall median survival in patients with KMT2A rearrangement is 0.9 years
  • Additional cytogenetic abnormalities and mutations, especially in myelodysplasia associated genes, worsens prognosis
Case reports
Treatment
  • Intensive chemotherapy as induction followed by additional consolidation therapy
  • Trials utilizing novel agents such as MEK inhibitors, demethylating agents and other targeted therapies are ongoing (Front Pharmacol 2022;13:749472)
  • Hematopoietic stem cell transplant
Microscopic (histologic) description
  • Increased myeloid lineage blasts (percentage will vary case to case)
    • WHO 5th edition does not have a blast requirement and the ICC requires at least 10% blasts in peripheral blood or bone marrow
  • On the aspirate smears, crush prep or touch prep, blast morphology will vary by phenotype
    • Myeloblasts will have open chromatin, high N:C ratio, prominent nucleoli and possible Auer rods in the cytoplasm
    • Monoblasts will have open chromatin, lower N:C ratio, possible cytoplasmic vacuoles and larger cell size
      • AML with monocytic differentiation tends to have a spectrum of monocyte differentiation, with mature monocytes, promonocytes and monoblasts (Haematologica 2009;94:994)
    • Unless there are coinciding myelodysplasia related gene mutations, significant dysplasia is not expected
  • On core biopsies, varying degrees of bone marrow involvement by blasts will be seen
    • Blasts will appear as immature cells with intermediate to large cell size, open chromatin and usually with prominent nucleoli
    • Pattern of involvement may be interstitial, aggregate or sheets
    • Degree of bone marrow effacement by leukemia will impact background trilineage hematopoiesis
    • Trilineage hematopoiesis will be affected by treatment in follow up bone marrow biopsies
Microscopic (histologic) images

Contributed by William Morrow, M.D.
Aspirate with increased blasts

Aspirate with increased blasts

Aspirate with increased monoblasts

Aspirate with increased monoblasts

Increased monoblasts

Increased monoblasts

Clot section blast aggregate

Clot section blast aggregate


Core biopsy blast sheets

Core biopsy blast sheets

Muramidase (lysozyme) IHC

Muramidase (lysozyme) IHC

CD4 IHC

CD4 IHC

Virtual slides

Images hosted on other servers:
Acute myeloid leukemia

Acute myeloid leukemia

Peripheral smear description
  • Increased circulating blasts, frequently with monocytic features, such as increased cytoplasm, vacuoles, cleaved or grooved nuclei
  • Variable degree of cytopenias depending on level of bone marrow involvement
    • Typically a normocytic, normochromic anemia with otherwise unremarkable red blood cell morphology
    • Typically a thrombocytopenia with unremarkable platelet morphology
  • No features of dysplasia in granulocytes unless there is a coinciding myelodysplasia related gene mutation(s)
  • Reference: Haematologica 2009;94:994
Peripheral smear images

Contributed by William Morrow, M.D.
Circulating blasts

Circulating blasts

Circulating monoblasts Circulating monoblasts

Circulating monoblasts

Positive stains
Negative stains
Flow cytometry description
  • Immunophenotype by flow cytometry varies but most commonly represents monocytic differentiation, with a smaller subset of cases having myelomonocytic or myeloid immunophenotype
  • Cases with monocytic differentiation will be positive for CD33, CD13, CD64, CD4, HLA-DR, CD14 (variable) and negative for CD34, CD117 and MPO
  • Cases with myeloid differentiation will be positive for CD34, MPO, CD33, HLA-DR and negative for CD64, CD4 and CD14
  • CD123 is positive in 90% of cases with KMT2A rearrangement (Cytometry B Clin Cytom 2022;102:123)
  • Markers of lymphoblastic differentiation, such as TdT, CD3, CD19, CD10 are almost universally negative
Flow cytometry images

Contributed by William Morrow, M.D.
Flow CD33 versus CD64

Flow CD33 versus CD64

Flow CD11b versus CD64

Flow CD11b versus CD64

Flow CD33 versus CD34

Flow CD33 versus CD34

Molecular / cytogenetics description
  • KMT2A gene (lysine methyltransferase 2A) is located on chromosome 11q23.3
  • More than 130 different partners have been recognized in association with KMT2A; AFF1, MLLT1, MLLT3, MLLT10, MLLT4 and ELL are the most frequently recognized partners in order of decreasing frequency (BMC Med Genomics 2020;13:106)
    • These genes represent over 90% of KMT2A rearrangement cases and are specifically recognized by the ICC 2022 classification for KMT2A with other rearrangement
  • Due to the various different breakpoints in the KMT2A gene, comprehensive testing for KMT2A rearrangement can be challenging
    • Different types of structural rearrangements can be the underlying cause of KMT2A fusions, including insertions, deletions, inversions and translocations
  • Several different molecular and cytogenetic testing modalities can identify KMT2A rearrangement
    • FISH is routinely performed using the aspirate material collected during the bone marrow biopsy procedure; KMT2A break apart probes (with a probe labeling 3’KMT2A and a probe labeling 5’KMT2A) are routinely ordered as part of a FISH panel for a diagnostic leukemia marrow
      • Some rearrangements may be cryptic by FISH, such as KMT2A::USP2
    • Routine karyotype can detect many translocations involved in KMT2A rearrangement (such as t(9;11)); however, some translocations may be cryptic (such as t(10;11) MLLT10::KMT2A)
      • Additional cytogenetic alterations can coincide with KMT2A rearrangement
    • PCR can be used to detect specific fusion transcripts
      • Some available panels include up to 20 fusion genes, including all the recurrent fusions identified
    • Myeloid next generation sequencing (NGS) panels may have KMT2A gene incorporated into a fusion panel and can detect KMT2A
  • Of note, partial tandem duplication (PTD) of KMT2A may be detected by NGS; PTD of the KMT2A gene does not qualify as KMT2A rearrangement for either classification scheme
Molecular / cytogenetics images

Contributed by William Morrow, M.D.
KMT2A break apart FISH

KMT2A break apart FISH



Images hosted on other servers:

Cryptic KMT2A::MLLT4 fusion

FISH of KMT2A::SEPT5 fusion

Sample pathology report
  • Bone marrow, biopsy:
    • Acute myeloid leukemia with other KMT2A rearrangement (see comment)
    • Comment: Increased blasts (50%) in a hypercellular marrow (90%). The blasts are morphologically and immunophenotypically consistent with myeloblasts. Concurrent FISH studies identified KMT2A rearrangement (see FISH report for full details). Concurrent karyotype studies identified t(10;11) (see karyotype report for full details). A myeloid NGS panel recognized a KMT2A::MLLT10. Overall, these findings are consistent with acute myeloid leukemia with KMT2A rearrangement (WHO 5th edition) or acute myeloid leukemia with other KMT2A rearrangement (ICC 2022).
Differential diagnosis
  • Acute myeloid leukemia with t(9;11) / MLLT3::KMT2A:
    • Differentiated by defining KMT2A rearrangement
    • Classified separately by the ICC
    • Shares classification by the WHO 5th edition
  • Acute myeloid leukemia with other defining genetic abnormalities:
    • Lacks KMT2A rearrangement
    • Presence of other defining genetic feature
  • Acute lymphoblastic leukemia with KMT2A rearrangement:
    • Differentiated by immunophenotype and cell lineage
  • Acute monocytic leukemia:
    • Myeloid leukemia with monocytic features lacking KMT2A rearrangement
  • Chronic myelomonocytic leukemia with KMT2A rearrangement:
    • No increase in blasts
    • Dysplastic features
    • Rare and difficult to diagnose
    • Increased blasts and KMT2A rearrangement better classified as AML
Board review style question #1

A patient with newly diagnosed acute myeloid leukemia had cytogenetics, FISH and myeloid NGS testing performed. FISH identified findings seen in the above image and myeloid NGS identified a KMT2A::AFDN fusion. Per the ICC 2022 guidelines, which of the following is the best classification of this patient's disease?

  1. Acute myeloid leukemia, NOS
  2. Acute myeloid leukemia with myelodysplasia related cytogenetics
  3. Acute myeloid leukemia with other KMT2A translocation
  4. Acute myeloid leukemia with t(9;11) / MLLT3::KMT2A
  5. B lymphoblastic leukemia with t(v;11q23.3) / KMT2A rearrangement
Board review style answer #1
C. Acute myeloid leukemia with other KMT2A translocation. The t(6;11) translocation (KMT2A::AFDN) is recognized by the ICC 2022 classification as one of the recurring KMT2A rearrangements other than MLLT3::KMT2A. Answer A is incorrect because this acute myeloid leukemia has a defining genetic alteration. Answer B is incorrect because the t(6;11) translocation (KMT2A::AFDN) is not recognized as a myelodysplasia related cytogenetic change or gene mutation. Answer D is incorrect because that is not the identified molecular alteration in this case. Answer E is incorrect because the question stem described this case as a myeloid leukemia, as well as a different molecular alteration.

Comment Here

Reference: AML with other KMT2A rearrangements
Board review style question #2
Acute myeloid leukemia with KMT2A rearrangement most commonly presents with what differentiation?

  1. Eosinophilic
  2. Lymphoblastic
  3. Monocytic
  4. Myeloid
  5. Myelomonocytic
Board review style answer #2
C. Monocytic. Acute myeloid leukemias with KMT2A rearrangement most commonly present with a monocytic immunophenotype. Answer A is incorrect because acute myeloid leukemia with KMT2A rearrangement does not present with eosinophilic differentiation. Answer B is incorrect because while lymphoblastic leukemias can present with KMT2A rearrangements, the question was asking for differentiation of myeloid leukemias. Answers D and E are incorrect because although acute myeloid leukemia with KMT2A rearrangement can present with myeloid or myelomonocytic differentiation, it is far more common to have pure monocytic differentiation.

Comment Here

Reference: AML with other KMT2A rearrangements

AML with other MECOM rearrangements (pending)
[Pending]

AML with pseudo-Chediak-Higashi anomaly
Definition / general
Case reports
Microscopic (histologic) description
  • Giant cytoplasmic granules
Microscopic (histologic) images

AFIP images
Missing Image

Specific granules
resemble
Chediak-Higashi
anomaly

Molecular / cytogenetics description
Electron microscopy description

AML with t(1;3)(p36;q21); RDM16::RPN1
Definition / general
  • Rare type of acute myeloid leukemia (AML)
  • AML with myelodysplastic syndrome (MDS) related changes (WHO, 2017)
    • Balanced translocation meeting criteria for AML with MDS related changes
    • Blasts > 20% and no prior therapy
  • Described as an entity under the category of AML with other rare recurring translocations in International Consensus Classification (ICC) (Blood 2022;140:1200)
  • AML with fusion of PRDM16 and RPN1
Essential features
Terminology
  • Acute myeloid leukemia with t(1;3)(p36;q21) (PRDM16::RPN1)
  • AML with t(1;3)(p36;q21) (PRDM16::RPN1)
ICD coding
  • ICD-O: 9895/3 - acute myeloid leukemia with myelodysplasia related changes
Epidemiology
Sites
  • Bone marrow, spleen and liver
  • Lymph node involvement can also occur
Pathophysiology
  • Genes at break point of translocation include RPN1 at 3q21.3 and group of genes at PRDM16 at 1p36.3 (Br J Haematol 2012;156:76)
  • Related to chromosome 3 abnormalities, such as inv(3)(q21q26), t(3;3)(q21;q26), ins(3;3)(q26;q21q26) and t(1;3)
    • t(1;3) show dysmegakaryopoietic features similar to the 3q21q26 syndrome
  • PRDM16 gene (MEL1, MDS1::EVI1-like gene) is activated as a result of translocation of the gene next to RPN1 gene at 3q21 (Genes Chromosomes Cancer 2003;36:313)
    • PRDM16 gene at 1p36.3 (the breakpoints are located within a 90 kb region in the 5' region of PRDM16)
    • RPN1 gene at 3q21 (the breakpoints are located within a 60 kb region centromeric to the breakpoint cluster region of the 3q21q26 / inv(3) syndrome)
    • Aberrant expression of PRDM16 as a result of RPN1 translocation is thought to be associated with pathogenesis of this AML subtype
Etiology
  • Aberrant expression of PRDM16 as a result of RPN1 translocation
Clinical features
  • Majority of cases present
    • Anemia (common)
    • Thrombocytosis and leukocytosis
    • Pancytopenia
  • Some cases present as MDS and then progress to AML (WHO, 2017)
  • Translocation (1;3) can be the sole chromosomal abnormality at initial diagnosis or can be associated with other chromosomal abnormalities
  • Peripheral thrombocytosis is a distinct feature (Cancer Genet Cytogenet 2010;203:187)
  • t(1;3) was first reported in 1984 (Blood 1984;64:553)
  • Aberrant expression of PRDM16 as a result of RPN1 translocation is associated with various other hematological conditions (Cancer Genet Cytogenet 2010;203:187)
    • MDS
    • Myelodysplastic / myeloproliferative neoplasms (MDS / MPN)
    • Myeloproliferative neoplasms (MPN)
    • AML
    • Acute lymphoblastic leukemia (ALL)
  • Prognosis is poor; median survival is ~6 months
  • WHO diagnostic criteria (WHO, 2017)
    • > 20% peripheral blood or bone marrow blasts
    • Any of the following:
      • History of MDS or MDS / MPN
      • MDS related cytogenetic abnormality
      • Multilineage dysplasia
    • Absence of both of the following:
      • Prior cytotoxic or radiation therapy
      • Recurrent cytogenetic abnormalities of AML

WHO revised 4th edition WHO 5th edition ICC classification
  • AML with myelodysplasia related changes (AML-MRC)
  • Blasts > 20% and no prior therapy
  • N/A
  • Only inv(3)/t(3;3) described under acute myeloid leukemia with MECOM rearrangement (3q26 abnormalities)
  • AML with other rare recurring translocations
  • Blasts > 10%
  • AML with myelodysplasia related changes eliminated
Diagnosis
  • Peripheral blood
  • Bone marrow biopsy
  • Cytogenetics
    • Karyotype analysis showing (1;3)(p36;q21) (PRDM16::RPN1)
  • Molecular genetics
  • Immunophenotyping
Laboratory
Prognostic factors
Case reports
Treatment
  • No specific treatments for this specific translocation have been reported
  • Chemotherapy
  • Stem cell transplantation
Microscopic (histologic) description
  • Presentation as AML (M1 - M5) has been reported; however, presentation as AML M4 (French American British [FAB] classification) is common (Cancer Genet Cytogenet 2010;203:187)
  • Prominent monocytic component has been reported in some cases
  • Multilineage dysplasia especially dysmegakaryopoiesis (Cancer Genet Cytogenet 2010;203:187, Genes Chromosomes Cancer 2003;36:313)
    • At least 50% dysplasia across 2 different lineages (WHO, 2017)
  • Megakaryocytes
    • Megakaryocytic hyperplasia and dysplasia
  • Peripheral thrombocytosis is a distinct feature
Microscopic (histologic) images

Contributed by Hatem Kaseb, M.D., Ph.D., M.P.H.
Numerous blasts

Numerous blasts

Numerous blasts and myelodysplasia related changes

Numerous blasts and myelodysplasia related changes

Peripheral smear images

Contributed by Hatem Kaseb, M.D., Ph.D., M.P.H.
Scattered blasts

Scattered blasts

Positive stains
Negative stains
Molecular / cytogenetics description
Sample pathology report
  • Right posterior iliac crest, core biopsy, aspirate smear, touch imprint and clot particle:
    • Acute myeloid leukemia with t(1;3)(p36;q21) (see comment)
    • Comment: The bone core biopsy demonstrates extensive replacement of marrow by blast cells in areas arranged into clusters and aggregates surrounded by fibrotic stroma. The marrow aspirate smears reveal an increased population of blasts. Blast phenotyping with immunohistochemical stains and flow cytometry analysis reveal positivity for myeloid markers. Cytogenetics evaluation reveals t(1;3)(p36;q21). Overall, the results support the diagnosis of AML with t(1;3)(p36;q21).
Differential diagnosis
  • MDS with excess blasts:
    • Blasts < 20%
    • May show other MDS related cytogenetic or molecular abnormalities
    • May show t(1;3)(p36;q21); these cases may progress to AML
  • Myeloproliferative neoplasms:
    • Blasts < 20%
    • May show t(1;3)(p36;q21); these cases may progress to AML
Board review style question #1

A 60 year old woman presented with fatigue and weight loss. The patient's initial laboratory workup showed leukocytosis, anemia and thrombocytosis. A bone marrow biopsy was performed, which showed increased blasts and multilineage dysplasia. A karyotype showed with t(1;3)(p36;q21). Which of the following is true about this entity?

  1. Blasts show a distinct phenotype characteristic of this entity
  2. Case shows overexpression of the PRDM16 gene
  3. Frequently associated with NPM1 mutation
  4. Has an intermediate prognosis
Board review style answer #1
B. Case shows overexpression of the PRDM16 gene. Overexpression of the PRDM16 gene as a result of RPN1 translocation is an important finding in in AML t(1;3)(p36;q21) patients. Answer A is incorrect because flow cytometry is usually nonspecific in AML t(1;3)(p36;q21). Answer C is incorrect because NPM1 mutation has not shown to be associated with AML t(1;3)(p36;q21). Answer D is incorrect because these cases typically show a poor prognosis.

Comment Here

Reference: AML with t(1;3)(p36;q21)

AML with t(3;5)(q25.3;q35.1); NPM1::MLF1 (pending)
[Pending]

AML with t(5;11)(q35.2;p15.4); NUP98::NSD1 (pending)
[Pending]

AML with t(6;9)(p23;q34); DEK::NUP214
Definition / general
  • Included in WHO 2008 classification under AML with recurrent genetic abnormalities
  • First identified by Rowley and Potter in 1976
  • 0.7 - 1.8% of AML
  • Occurs in both children and adults (2 - 66 years old), both sexes equally; median age children 13 years, adults 35 years
  • Associated with multilineage dysplasia and basophilia (> 2% in marrow and peripheral blood in 44 - 62% of cases)
  • In adults, median white blood cell count is 12x109/L, lower than other AML
  • In children, lower hemoglobin values and higher WBC counts than adults
  • Associated with dysplasia and a high frequency of FLT3 gene mutations (Am J Clin Pathol 2004;122:348)
Clinical features
  • Anemia, thrombocytopenia, blasts in peripheral blood
  • Often pancytopenia, fatigue, bleeding, DIC and increased incidence of infections
Prognostic features
  • Poor prognosis
  • < 50% remission after chemotherapy; often dead within 1 year after diagnosis (Leukemia 2006;20:1295)
Treatment
  • Allogeneic hematopoietic stem cell transplantation
  • Minimal residual disease: monitor by real time RT-PCR (Leukemia 2005;19:1338)
  • Current clinical trials include anti-CD33 and FLT3 inhibitors
Microscopic (histologic) description
Positive stains
Molecular / cytogenetics description
  • t(6;9)(p23;q34) is often sole clonal abnormality; some have complex karyotype
  • t(6;9) produces chimeric fusion gene between DEK (6p23) and NUP214 (9q34; formerly known as CAN)
  • DEK-NUP214 fusion gene encodes a messenger RNA involved in leukemogenesis
  • Also FLT3-ITD (69% pediatric and 78% adult cases), often additional chromosomal abnormalities; FLT3-TDK uncommon
  • Patients with FLT3-ITD have higher WBC, higher bone marrow blasts and lower rates of complete remission

AML with t(7;12)(q36.3;p13.2) / ETV6::MNX1 (pending)
[Pending]

AML with t(8;21)(q22;q22); RUNX1::RUNX1T1
Definition / general
  • AML with t(8;21)(q22;q22.1) is defined by a genetic rearrangement that results in the fusion of RUNX1 and RUNX1T1, has characteristic morphologic and immunophenotypic features and is associated with a generally favorable prognosis
Essential features
  • AML with t(8;21)(q22;q22.1) is classified in the category of AML with recurrent genetic abnormalities and is diagnostic of AML regardless of blast count
  • t(8;21)(q22;q22.1) is a balanced translocation that results in the fusion of RUNX1 and RUNX1T1
  • Blasts have distinct morphologic and immunophenotypic features, including abundant basophilic cytoplasm containing azurophilic granules with occasional Auer rods and frequent aberrant expression of B cell markers, such as CD19
  • Diagnosis is established by detection of t(8;21) by real time PCR or cytogenetics
  • Prognosis is generally favorable, while coexisting KIT mutations adversely affect outcomes
Terminology
  • AML with RUNX1-RUNX1T1
  • AML with AML1-ETO
  • Usually classified as AML M2 subtype in the previous French American British (FAB) classification system
ICD coding
  • ICD-10: C92.00 - Acute myeloblastic leukemia, not having achieved remission
Epidemiology
  • Typically occurs in younger patients and comprises approximately 1 - 5% of all AML cases
Sites
  • Peripheral blood
  • Bone marrow
  • Myeloid sarcoma soft tissue / organs
Pathophysiology
  • t(8;21)(q22;q22.1) results in fusion of the RUNX1 gene on chromosome 21q22.1 with RUNX1T1 gene on chromosome 8q22
  • RUNX1 is a member of the core binding factor family of transcription factors, which is critical for the establishment of normal hematopoiesis (Blood 2017;129:2070)
  • t(8;21), RUNX1-RUNX1T1 fusion gene expresses a chimeric protein that disrupts the normal function of the core binding factor and predisposes to acute myeloid leukemia (Cell Rep 2013;4:1131)
  • Most cases show additional chromosomal abnormalities and cooperating driver mutations that promote transformation to acute myeloid leukemia (Blood 2012;119:e67)
Diagnosis
  • Established by detection of t(8;21) by real time PCR or cytogenetics
Prognostic factors
  • AML with t(8;21)(q22;q22.1) has a relatively favorable outcome and is associated with high rate of complete remission
  • Presence of KIT mutations adversely affects prognosis (J Clin Oncol 2006;24:3904)
Case reports
Treatment
  • Typically chemotherapy with induction, followed by intensive cytarabine consolidation
Microscopic (histologic) description
  • Blast cells characteristically have slightly basophilic cytoplasm containing abundant azurophilic granules and perinuclear hofs
  • Occasionally, large pseudo Chediak-Higashi granules and thin Auer rods may be seen; Auer rods may be seen in neutrophils
  • Mature eosinophils that are morphologically normal are often increased in number in the bone marrow
  • Mature granulocytes may show dysplastic changes with pseudo Pelger-Huet anomaly
  • Generally no dysplasia in the erythroid and megakarocytic lineages
Microscopic (histologic) images

Contributed by Sean Gu, M.D., Ph.D., Alexa J. Siddon, M.D. and AFIP images
Left shift

Left shift

Blasts

Blasts

Aspirate morphology

Aspirate morphology

Bone marrow smear (Wright-Giemsa) Bone marrow smear (Wright-Giemsa)

Bone marrow smear (Wright-Giemsa)



Images hosted on other servers:

Myeloblast in AML with t(8;21)

Positive stains
Negative stains
Flow cytometry description
Flow cytometry images

Contributed by Genevieve M. Crane, M.D., Ph.D.
Partial CD19 expression in AML with t(8;21)

Partial CD19 expression in AML with t(8;21)

Molecular / cytogenetics description
  • Demonstration of t(8;21) by real time PCR or cytogenetics
Molecular / cytogenetics images

AFIP images
Missing Image

t(8;21) with abnormal chromosomes



Images hosted on other servers:
Missing Image

FISH probes showing
2 abnormal yellow
fusions made of red
and green probes

Sample pathology report
  • Left posterior iliac crest, core biopsy and aspirate smear:
    • Acute myeloid leukemia with t(8;21)(q22;q22.1); RUNX1-RUNX1T1 (see comment)
    • Comment: The bone core biopsy demonstrates an increase in blasts, which are noted to have prominent eosinophilic granules on the aspirate. The presence of t(8;21) by PCR confirms the diagnosis.
Differential diagnosis
Board review style question #1
Which of the following is true about AML with t(8;21)?

  1. AML with t(8;21) generally has a poor prognosis
  2. AML with t(8;21) frequently aberrantly expresses the lymphoid markers CD19 and PAX5
  3. Auer rods are rare in AML with t(8;21)
  4. Presence of a concomitant KIT mutation is a good prognostic finding
  5. Translocation is best identified by microarray
Board review style answer #1
B. AML with t(8;21) frequently aberrantly expresses the lymphoid markers CD19 and PAX5

Comment Here

Reference: AML with t(8;21)(q22;q22)
Board review style question #2


A 26 year old woman is referred to the emergency department when she is noted to have 32% blasts in peripheral blood. Her smear shows large basophilic blasts with prominent eosinophilic granules and rare Auer rods. What will the flow cytometry immunophenotype likely show?

  1. CD34-, CD117+, HLADR-, MPO++, CD33,
  2. CD34+, CD33dim+, MPO+, CD71-, CD19dim+
  3. CD34-, CD13+, CD11b+, CD14+, CD64+, HLA-DR+
  4. CD34+, CD117+, CD33+, CD19-, CD3-, MPO-
  5. CD34+, CD19+, CD20-, CD10+, TdT+
Board review style answer #2
B. CD34+, CD33dim+, MPO+, CD71-, CD19dim+

Comment Here

Reference: AML with t(8;21)(q22;q22)

AML with t(9;11)(p22;q23); KMT2A::MLLT3
Definition / general
Clinical features
  • Symptoms: DIC, extramedullary myeloid sarcoma and tissue infiltration (gingiva, skin)
  • Translocations occur in AML (intermediate prognosis) and ALL (poor prognosis) (Blood Cells Mol Dis 2008;40:192)
  • Prognosis may be superior to other AML with 11q23 translocations
  • MLL partial tandem duplication AML: prognosis varies from poor to similar to other AML (Br J Haematol 2006;135:438, Blood 2007;109:5164)
    • Also present in 93% of normal cord blood samples at low levels (Leuk Res 2006;30:1091)
    • Associated with normal karyotype or trisomy 11
Case reports
Microscopic (histologic) description
  • 20%+ blasts / blast equivalents (monoblasts / monocytes) in peripheral blood or bone marrow, usually myelomonocytic or monocytic (AML M4, M5) and occasionally AML with (M2) or without (M1) maturation
  • Monoblasts are large cells with abundant, moderate to intensively basophilic cytoplasm, pseudopods, azurophilic granules, vacuoles; round nuclei, lacy chromatin and one or more prominent nucleoli
  • Promonocytes have basophilic cytoplasm with granules and occasional large azurophilic granules, vacuoles; irregular and delicately convoluted nuclei
Positive stains
Negative stains
Molecular / cytogenetics description
  • Involves MLL mixed lineage or myeloid / lymphoid leukemia gene, present in both AML and ALL; also called HTRX1, HRX and ALL1
  • FISH is more sensitive than conventional cytogenetics in detecting MLL; may also detect 11q22-25 rearrangements that are MLL negative (Am J Clin Pathol 2004;122:298)

    Variant MLL translocations in AML:
  • > 80 translocations described, most commonly MLLT2(AF4) causing ALL; and MLLT3(AF9) causing AML
  • Mixed phenotype acute leukemia has t(v;11q23)
  • AML-MRC if t(2;11)(p21;q23) or t(11;16)(q23;p13.3)
  • Diagnose as t-AML if history of cytotoxic therapy
  • Other common translocations: 6q27 (MLLT4), 10p12 (MLLT10), 19p13.1 (ELL) and 19p13.3 (MLLT1)

AML without maturation (FAB AML M1)
Definition / general
  • 10 - 20% of AML cases, 44% in one Brazil hospital (Sao Paulo Med J 2006;124:45)
  • Usually adults (median age 46 years), present with anemia, thrombocytopenia and neutropenia; may have leukocytosis with markedly increased blasts
  • 4% of childhood AML
Clinical features
  • Criteria for diagnosis: 90%+ of nonerythroid cells in marrow are myeloblasts; < 10% granulocytic elements; 3%+ of blasts must be positive for myeloperoxidase or Sudan Black B and / or Auer rods by enzyme cytochemistry
  • Enzyme cytochemistry: 3%+ of blasts are positive for myeloperoxidase or Sudan Black B (confirm by immunohistochemistry if only 3 - 10% positive for MPO by enzyme cytochemistry); chloroacetate esterase positive
  • Poor prognosis
Case reports
Microscopic (histologic) description
  • Typically markedly hypercellular marrow, but normocellular and hypocellular cases occur
  • Very immature cells, usually round with few azurophilic cytoplasmic granules or Auer rods
  • Nuclei are round or indented; little maturation beyond myeloblast stage
  • Cells may may resemble lymphoblasts and not appear myeloid
Microscopic (histologic) images

AFIP images
Missing Image

Blasts show mild size variation

Missing Image

Blasts show more variation in size and number of nucleoli

Missing Image

Myeloblasts have irregular nuclei

Missing Image

Myeloblasts have marked size variation

Missing Image

Some variation in size


Missing Image

Large myeloblasts
with abundant
eosinophilic
cytoplasm

Missing Image

Agranular myeloblasts

Missing Image

Bone marrow biopsy

Missing Image

Myeloblasts are myeloperoxidase+

Missing Image

Numerous granules are Sudan black B+

Peripheral smear images

AFIP images
Missing Image

Peripheral blood smear

Positive stains
Negative stains
Electron microscopy description
Electron microscopy images

AFIP images
Missing Image

Numerous electron dense granules in Golgi region

Molecular / cytogenetics description
Differential diagnosis

APL with PML::RARA
Definition / general
  • Acute myeloid leukemia (AML) with predominance of abnormal promyelocytes and fusion of the promyelocytic leukemia (PML) gene with the retinoic acid receptor alpha (RARA) gene
Essential features
  • Acute myeloid leukemia with predominance of abnormal promyelocytes, characterized by t(15;17)(q24.1;q21.2) and leading to PML::RARA fusion
  • Young to middle aged adults are most commonly affected
  • 2 morphologic variants are hypergranular and microgranular
  • Early diagnosis and treatment are imperative due to the high risk of coagulopathy and disseminated intravascular coagulation (DIC)
  • Standard treatment includes all trans retinoic acid (ATRA) plus arsenic trioxide (ATO), with or without chemotherapy; excellent prognosis and high cure rates
  • Reference: Arch Pathol Lab Med 2015;139:1308
Terminology
  • Acute promyelocytic leukemia with PML::RARA fusion (World Health Organization Classification, 5th Edition)
  • Acute promyelocytic leukemia (APL) with t(15;17)(q24.1;q21.2) / PML::RARA (2022 International Consensus Classification)
  • Formerly called AML M3 by the French American British classification
ICD coding
  • ICD-O: 9866/3 - acute promyelocytic leukemia with PML::RARA
  • ICD-11: 2A60.0 & XH1A50 - acute myeloid leukemia with recurrent genetic abnormalities & acute promyelocytic leukemia, t(15;17)(q22;q11-12)
Epidemiology
  • Relatively uncommon acute myeloid leukemia subtype
  • Young and middle aged adults are most commonly affected (20 - 59 years)
  • Accounts for 5 - 8% of acute myeloid leukemia cases in younger patients
  • Higher incidence in Latin American population (Best Pract Res Clin Haematol 2003;16:357, Cancer 2012;118:5811)
  • M ≈ F
Sites
  • Peripheral blood and bone marrow
  • Extramedullary involvement is rarely seen
Pathophysiology
  • Characterized by balanced translocation t(15;17)(q24.1;q21.2) leading to PML::RARA fusion and a PML::RARA oncoprotein
  • PML::RARA oncoprotein suppresses gene expression by recruitment of a number of transcriptional repressors, which leads to block in differentiation and malignant transformation of myeloid cells (PLoS One 2014;9:e104906)
  • Secondary chromosomal and genetic abnormalities contribute to the phenotype
Etiology
Clinical features
  • Leukopenia is a common presenting sign
  • Leukocytosis in the microgranular variant
  • Symptoms of pancytopenia are fatigue, weakness, infection, bleeding
  • High risk of coagulopathy and disseminated intravascular coagulation, especially in microgranular acute promyelocytic leukemia (Arch Pathol Lab Med 2015;139:1308, Blood Cancer J 2021;11:123)
  • Early diagnosis and treatment are imperative
Diagnosis
  • Established by detection of t(15;17) / PML::RARA fusion by RT-PCR, conventional cytogenetics or FISH
  • RT-PCR is the most sensitive method of detection, as the translocation can be cryptic by karyotype and FISH
  • Blast count can be < 20%
  • Reference: Arch Pathol Lab Med 2015;139:1308
Laboratory
  • Complete blood count showing pancytopenia
  • Leukocytosis in microgranular acute promyelocytic leukemia
  • Prolonged prothrombin time (PT) or activated partial thromboplastin clotting time (aPTT)
  • Elevated D dimer and fibrin degradation products
  • Decreased fibrinogen (Blood Cancer J 2021;11:123)
Prognostic factors
Case reports
Treatment
  • All trans retinoic acid plus arsenic trioxide is currently the standard of care for low and intermediate risk acute promyelocytic leukemia patients
  • In high risk patients, chemotherapy is added to all trans retinoic acid and arsenic trioxide
  • If arsenic trioxide is not available, combination of all trans retinoic acid and chemotherapy is used for all risk categories (Blood Cancer J 2021;11:123)
Microscopic (histologic) description
  • Bone marrow hypercellular
  • 2 cytomorphologic variants of acute promyelocytic leukemia are hypergranular (classic) and microgranular (hypogranular)
  • Hypergranular acute promyelocytic leukemia
    • Abnormal promyelocytes vary in size and shape, frequently with kidney shaped or bilobed nuclei
    • Cytoplasm packed with large granules, staining bright pink, red or purple
    • Auer rods, frequently in bundles
    • Myeloblasts with Auer rods can be seen
  • Microgranular acute promyelocytic leukemia
    • Predominantly bilobed nuclei
    • Hypogranular appearing cytoplasm due to submicroscopic size of azurophilic granules
    • Most cases have some abnormal promyelocytes with prominent granules or Auer rods (Arch Pathol Lab Med 2015;139:1308)
  • ZBTB16::RARA t(11;17) acute promyelocytic leukemia has some characteristic morphologic features (Blood 2000;96:1287)
    • More regular nucleus, not bilobed; occasionally more condensed nuclear chromatin compared with classic acute promyelocytic leukemia
    • Coarse cytoplasmic granules or less often, fine or no granules
    • No bundles of Auer rods
    • Increased number of hypogranular pelgeroid neutrophils
Microscopic (histologic) images

Contributed by Anamarija M. Perry, M.D., @sanamloghavi on Twitter and AFIP
Hypercellular bone marrow

Hypercellular bone marrow

Abnormal promyelocytes

Abnormal promyelocytes

Contributed by @sanamloghavi on Twitter (see original post here)"> APL with PML::RARA Contributed by @sanamloghavi on Twitter (see original post here)"> APL with PML::RARA

APL with PML::RARA

Contributed by @sanamloghavi on Twitter (see original post here)"> APL with <i>PML::RARA</i>Contributed by @sanamloghavi on Twitter (see original post here)"> APL with <i>PML::RARA</i>

APL with PML::RARA


Bone marrow biopsy Bone marrow biopsy

Bone marrow biopsy

Bone marrow smear (Wright-Giemsa)

Bone marrow smear (Wright-Giemsa)

Treatment related

Treatment related

Microgranular variant Microgranular variant

Microgranular variant

Virtual slides

Images hosted on other servers:
APL, classic variant

APL, classic variant

APL, microgranular variant

APL, microgranular variant

Peripheral smear description
  • In classic variant, pancytopenia is frequently seen with occasional circulating abnormal promyelocytes
  • In microgranular acute promyelocytic leukemia, leukocytosis with numerous abnormal promyelocytes
Peripheral smear images

Contributed by Anamarija M. Perry, M.D.
Microgranular variant

Microgranular variant

Positive stains
Negative stains
Flow cytometry description
Flow cytometry images

Contributed by Anamarija M. Perry, M.D.
Classic variant, CD13+ and CD33+

Classic variant, CD13+ and CD33+

Classic variant, HLA-DR- and CD34-

Classic variant, HLA-DR- and CD34-

Classic variant, CD34- and CD64+

Classic variant, CD34- and CD64+

Classic variant, CD34- and cyMPO+

Classic variant, CD34- and cyMPO+

Microgranular variant, moderate CD45

Microgranular variant, moderate CD45

Microgranular variant, CD2+ and CD34+

Microgranular variant, CD2+ and CD34+

Electron microscopy images

AFIP images
Missing Image

Cytoplasm and nucleus

Missing Image

Cross section of Auer rod

Missing Image

Microgranular variant

Missing Image

Small granules and stellate array

Molecular / cytogenetics description
  • t(15;17)(q24.1;q21.2) leading to PML::RARA fusion
  • PML::RARA can be detected in 90 - 95% of cases (Nature 1990;347:558)
  • 3 PML::RARA transcript isoforms (depending on the PML breakpoint location on chromosome 15): long (in intron 6, Bcr1), variant (in exon 6, Bcr2) and short (in intron 3, Bcr3) (Cancers (Basel) 2020;12:624)
  • Occasional cases have cryptic t(15;17)(q24.1;q21.2) or complex rearrangements involving an additional chromosome (Leukemia 2016;30:1672)
  • Deletion 7q and trisomy 8 are the most common frequent additional cytogenetic alterations (Cancers (Basel) 2020;12:624)
  • Variant RARA translocations (Hematol Oncol Stem Cell Ther 2020;13:189, Cancers (Basel) 2020;12:967, Arch Pathol Lab Med 2015;139:1308, Front Oncol 2022;12:871590)
    • Detected in ~5% of cases, involve RARA gene at 17q21.2 and different partners
    • Should be diagnosed as acute promyelocytic leukemia with variant RARA translocations
    • Partner genes
      • ZBTB16 at 11q23 has a poor response to all trans retinoic acid and arsenic trioxide, characteristic morphology (see Microscopic (histologic) description)
      • NUMA1 at 11q13
      • NPM1 at 5q35
      • STAT5B at 17q21 has a poor response to all trans retinoic acid and arsenic trioxide
      • PRKAR1A at 17q24
      • FIP1L1 at 4q12
      • BCOR at Xp11
      • OBFC2A at 2q32
      • TBLR1 at 3q26
      • GTF2I at 7q11
      • IRF2BP2 at 1q42
      • FNDC3B at 3q26
      • STAT3 at 17q21
      • HNRNPC at 14q11
      • NUP98
      • TNRC18
  • Most common somatic mutations include FLT3 (including FLT3 ITD and FLT3 p.D835), WT1, NRAS, KRAS, USP9X, ARID1A and EED (Leukemia 2016;30:1672, Leukemia 2019;33:1387)
Molecular / cytogenetics images

Contributed by Anamarija M. Perry, M.D. and Lina Shao, Ph.D.
Karyotype

Karyotype

FISH study

FISH study

Sample pathology report
  • Bone marrow, core biopsy, touch imprints, aspirate clot, aspirate smears and peripheral blood smears:
    • Acute promyelocytic leukemia with PML::RARA
    • Adequacy: The core biopsy represents ~7 mm of evaluable bone marrow. The aspirate materials are adequate.
    • Cellularity: Hypercellular, ~90% on average.
    • Erythroid elements: Decreased quantity. Full spectrum of maturation. No dysplasia.
    • Myeloid elements: Predominance of atypical promyelocytes in bone marrow. The neoplastic promyelocytes are large cells and most of them are heavily granulated. Many feature a lobulated nucleus and some have overlapping nuclear lobes. Rare cells with numerous (10 - 15) Auer rods are found in the bone marrow and blood smears. There is little granulocytic maturation beyond the promyelocyte stage.
    • Megakaryocytes: Adequate number. No definite dysplasia.
    • Ancillary studies: Fluorescence in situ hybridization analysis of the bone marrow aspirate is positive for PML::RARA gene fusion.
    • Complete blood count:
      • White blood cell (K/uL): 0.9
      • Red blood cell (M/uL): 3.06
      • Hemoglobin (g/dL): 10.1
      • Hematocrit (%): 27.1
      • Mean corpuscular volume (fl): 88.6
      • Red cell distribution width (%): 14.5
      • Platelet (K/uL): 106
    • Peripheral blood: Absolute leukopenia with occasional leukemic promyelocytes noted. Slight red cell anisocytosis and moderate poikilocytosis with some ovalocytes but no schistocytes. A few nucleated red cells noted on scan. Slight to moderate thrombocytopenia with some variation in platelet size and occasional hypogranular forms.
Differential diagnosis
Board review style question #1

A 27 year old woman presents with fatigue of ~2 weeks' duration and pancytopenia with marked leukopenia. Bone marrow biopsy and aspiration were performed and representative morphology is shown in the image above. Which of the following studies should be done to confirm the diagnosis suspected on morphology?

  1. FISH for MYC rearrangement
  2. FISH for PML::RARA fusion
  3. Mutation analysis for FLT3
  4. Mutation analysis for NPM1
  5. RT-PCR for BCR::ABL1
Board review style answer #1
B. FISH for PML::RARA fusion. Bone marrow aspirate shows abnormal promyelocytes. These findings are suspicious for acute promyelocytic leukemia, however, a definitive diagnosis requires detection of PML::RARA fusion, which can be done with a rapid FISH study.

Comment Here

Reference: APL with PML::RARA
Board review style question #2
Which of the following immunophenotypes is characteristic for hypergranular acute promyelocytic leukemia?

  1. CD34+, CD13+, CD33+, MPO+, CD2-
  2. CD34+, CD13+, CD33+, MPO-, CD2-
  3. CD34+, HLA-DR-, CD13+, CD33+, CD64-
  4. CD34-, HLA-DR-, CD13+, CD33+, CD64+
  5. CD34-, HLA-DR+, CD13+, CD33+, CD64-
Board review style answer #2
D. CD34-, HLA-DR-, CD13+, CD33+, CD64+. Hypergranular variant of APL is characterized by low or absent expression of CD34 and HLA-DR, is usually brightly positive for CD33 and is positive for CD13 and cytoplasmic myeloperoxidase (MPO). CD64 is also frequently expressed. The other answers (A, B and C) can be eliminated since they show the phenotype of blasts that are CD34 positive or HLA-DR positive (answer E).

Comment Here

Reference: APL with PML::RARA

Acute basophilic leukemia
Definition / general
  • First described (basophilic leukemia) in 1906 by Joachim (Dtsch Arch Klin Med 1906;87:437)
  • Acute myeloid leukemia (AML) with primary differentiation to basophils
  • To qualify as a type of AML, not otherwise specificed (NOS), other specifically described groups (i.e. AML with myelodysplasia related changes, therapy related AML or AML with recurrent genetic abnormalities) must be excluded
  • Wide age range, 1 day - 82 years (Leuk Lymphoma 1999;32:269)
  • Rapid clinical progression
Essential features
  • First step in diagnosis is the recognition of the presence of coarse basophilic granules
  • Immunophenotyping and electron microscopy features are essential to identify basophilic lineage
  • Crucial to differentiate basophilic cells from mast cells
  • Diagnosis of acute basophilic leukemia (J Clin Oncol 2011;29:e623):
    • Circulating peripheral blood and bone marrow immature basophils / blasts
      • Positive with metachromatic staining (toluidine blue), acid phosphatase (diffuse pattern), some cases with periodic acid Schiff (PAS) positivity in large blocks
      • Negative for myeloperoxidase (MPO), Sudan black B (SBB) or naphthol ASD chloroacetate esterase (CAE)
Terminology
  • Basophilic leukemia (no longer used)
ICD coding
  • ICD-O: 9870/3 - acute basophilic leukemia
  • ICD-10: C94.80 - other specified leukemias not having achieved remission
Epidemiology
  • Rare disease
  • ~ 4 - 5% of all cases of acute nonlymphocytic leukemia and < 2% of all hematopoietic malignancies (Leuk Lymphoma 1999;32:269)
Sites
  • Peripheral blood, bone marrow, skin, organomegaly
Diagrams / tables
Immunophenotypic profile of basophils, plasmacytoid dendritic cells (pDC) and mast cells
Antigen
Basophils
Other cells
Normal
Neoplastic
pDC
Mast cell
CD203c Positive Positive Negative Positive
CD123 Positive Positive / Negative Positive Negative
HLA-DR Negative Positive / Negative Positive (high) Positive (low)
CD34 Negative Negative Negative Negative
CD117 Negative Negative Negative Positive
CD38 Positive Positive / Negative Negative Positive (low)
CD33 Positive Positive / Negative Negative Positive (moderate)
CD64 Negative Positive / Negative Negative Negative
Clinical features
  • Wide age range: 1 day - 82 years (Leuk Lymphoma 1999;32:269)
  • Typically presents with bone marrow failure
  • Variable circulating blasts
  • Cutaneous involvement (due to high histamine levels): pruritus, edema, urticarial rashes, focal hyperpigmentation
  • GI symptoms: nausea, vomiting, diarrhea, dyspepsia, abdominal swelling or ulcers
  • Organomegaly and lytic lesions
Diagnosis
  • Prerequisite for the diagnosis of acute basophilic leukemia (ABL) and chronic basophilic leukemia (CBL) (Leukemia 2017;31:788)
    • Presence of hyperbasophilia (HB) (absolute basophil count exceeding 1,000 per microliter of peripheral blood (PB), present for over at least 8 weeks)
    • Percentage of basophils must be ≥ 40% of total leukocytes
    • Basophils must belong to the malignant clone as evidenced by
      • (Immature) morphology of basophils
      • Type of underlying neoplasm (myeloid), if present
  • Presence of cytogenetic or molecular alterations is helpful for establishing clonality
  • It is important to distinguish ABL from CBL:
    • ABL: HB criteria + basophils ≥ 40% of nucleated bone marrow or PB cells + myeloblasts + metachromatic blasts ≥ 20%
      • Primary ABL: no preceding / underlying bone marrow neoplasm
      • Secondary ABL: known preceding / underlying bone marrow neoplasm
    • CBL: HB criteria + basophils ≥ 40% of nucleated bone marrow or PB cells + myeloblasts + metachromatic blasts < 20%
      • Primary CBL: no preceding / underlying bone marrow neoplasm
      • Secondary CBL: known preceding / underlying bone marrow neoplasm
Laboratory
  • CT scan for spleen size and lymph node status
  • Complete blood count - white blood cell, hemoglobin, platelets, differential leukocyte count (basophil)
  • Serum chemistry, including serum tryptase level
  • Allergy diagnostics, including total IgE
  • Inflammation markers (C reactive protein (CRP), erythrocyte sedimentation rate (ESR))
  • Bone marrow histology (biopsy, clot and smears) and immunohistochemistry
  • Chromosome analysis and extended FISH panel for myelodysplastic syndromes (MDS) or myeloproliferative neoplasms (MPN)
  • Molecular studies / next generation sequencing panel:
    • BCR-ABL1 to be excluded by real time PCR or FISH
    • JAK2 V617F by real time PCR
    • JAK2 exon 12
    • MPL mutation
    • CALR mutation
    • FIP1L1-PDGFRA by real time PCR or FISH
  • Reference: Leukemia 2017;31:788
Prognostic factors
  • Poor prognosis
  • Survival time ranges from 2 to > 36 months in chronic basophilic leukemia (CBL); 2 to 16 months in acute basophilic leukemia (ABL) (Leukemia 2017;31:788)
Case reports
Treatment
  • Stem cell transplant (SCT)
  • If SCT is not possible, polychemotherapy, targeted drugs, experimental drugs or palliative cytoreductive treatment
  • Histamine related symptoms (if HB is present): prophylactic histamine receptor (HR) antagonists (HR1 and HR2 blocker), proton pump inhibitors and steroids
  • Combination of daunorubicin / idarubicin and cytarabine (Oncol Lett 2014;8:2513)
Microscopic (histologic) description
  • Bone marrow biopsy / aspirate
    • Hypercellular bone marrow with medium sized blasts with high nuclear to cytoplasmic ratio, an oval, round or bilobed nucleus, dispersed chromatin with 1 - 3 prominent nucleoli
    • Moderate amounts of basophilic cytoplasm that contains a variable number of coarse basophilic granules
    • Cytoplasmic vacuolation may be present
    • Mature basophils are usually sparse
    • Dysplastic features in the erythroid precursors may be present
Microscopic (histologic) images

Contributed by Pallavi Khattar, M.D.
Bone marrow biopsy Bone marrow biopsy Bone marrow biopsy Bone marrow biopsy

Bone marrow biopsy

Bone marrow biopsy - reticulin stain

Bone marrow biopsy - reticulin stain

Positive stains
Negative stains
Flow cytometry description
Electron microscopy description
  • Granules contain an electron dense particulate substance and are internally bisected, e.g. have a theta character (θ) or contain crystalline material arranged in a pattern of scrolls or lamellae
  • Coexistence of basophil and mast cell granules may be identified in the same immature cells
Molecular / cytogenetics description
Sample pathology report
  • Bone marrow, right posterior iliac crest, core biopsy, clot section, aspirate smears and touch imprint:
    • Consistent with acute myeloid leukemia, NOS favor acute basophilic leukemia (see comment)
    • Comment: Hypercellular marrow (100%) is essentially replaced by sheets of immature myeloid cells (approximately 70% of total cellularity) with increased basophils. Bone marrow aspirate smears show blasts, 25%; atypical immature basophils, 38%; mature basophils, 7%. The blasts often had basophil-like dark, coarse granules. Special stains demonstrate that the blasts were positive for metachromatic toluidine blue but negative for myeloperoxidase (MPO). Flow cytometric analysis demonstrates immature myeloid cells (comprises 30% of total) expressing CD13, CD33, CD34, CD38 and HLA-DR and heterogeneous population of basophils (approximately 35% of total) express CD11b, CD13, CD33, CD22, CD123 and CD203. No monotypic B cells or immunophenotypically abnormal T cells are detected. Additional cytogenetic and molecular studies are in progress and will be reported separately.
    • Peripheral blood: Anemia, thrombocytopenia and leukocytosis. Hemoglobin was 8.9 g/dl, platelet 61/ 109/L and total leukocyte count 15 /109/L. The smear revealed a large population of basophils with a variable degree of maturation ranging from blasts with coarse basophilic granules to mature basophil granulocytes, some of which had signs of degranulation.
    • Bone marrow biopsy: Quality adequate. Hypercellular marrow (100%), large population of basophils with a variable degree of maturation ranging from blasts with coarse basophilic granules to mature basophil granulocytes, some of which had signs of degranulation. Bone marrow residual myeloid as well as erythroid lineages were markedly reduced to absent. Rare unremarkable megakaryocytes were seen. There are no granulomas. The bony trabeculae show evidence of remodeling. Stainable iron is present (iron stain). There is a diffuse increase in reticulin fibers with focal collagen fibrosis; (reticulin stain, trichrome stain).
    • Bone marrow clot section: Quality adequate. Cellularity - 100% morphologic features are similar to those observed in the core biopsy.
    • Bone marrow aspirate: Quality adequate. The smears show blast cells, 25%; atypical immature basophils, 38%; mature basophils, 7%; normal mature neutrophils, 11%; lymphocytes, 7%; monocytes, 2% and erythroid cells, 10%. The blasts often had basophil-like dark, coarse granules and basophils had characteristic confluent vacuoles merged with the cell membrane suggestive of degranulation.
    • Special Stains: Blasts were toluidine blue metachromatic but negative for myeloperoxidase.
    • Flow Cytometry Analysis: Gating - mononuclear cells by forward and side light scatter.
    • Interpretation: Immature myeloid blasts (comprises 30% of total) were positive for CD13, CD33, CD34, CD38 and HLA-DR. In addition, heterogeneous population of basophils (approximately 35% of total) express CD11b, CD22, CD13, CD33, CD123 and CD203. The immature myeloid blasts were negative for TdT, CD10, CD14, CD61, CD64, CD2, sCD3, cCD3, CD4, CD5, CD8, CD16, CD56, CD19, CD20, CD22, CD23, cCD79a, CD103, CD117, FMC7, kappa and lambda.
Differential diagnosis
Board review style question #1
What is the most specific marker for basophils in peripheral blood?

  1. CD25
  2. CD117
  3. CD123
  4. CD203c
Board review style answer #1
D. CD203c is a relatively specific marker for basophils in peripheral blood. Other markers such as CD13 or CD33, CD123, CD11b, CD34 are also positive, while CD25, CD117 and tryptase are negative. This immunophenotype helps to differentiate acute basophilic leukemia from mast cell leukemia.

Comment Here

Reference: Acute basophilic leukemia
Board review style question #2

Which of the following is positive for CD34 and CD117 immunostain?

  1. Basophil
  2. Eosinophil
  3. Mast cell
  4. Myeloblast
Board review style answer #2
D. Myeloblast. Recommended markers to delineate between basophils, eosinophils and mast cells are CD34, CD117, tryptase, 2D7 and BB1 (basogranulin). Myeloblasts are often CD34 or CD117 positive; Mast cells express CD117 and tryptase and are negative for CD34. Eosinophils and basophils are negative for both CD34 and CD117 and express BB1 and 2D7.

Comment Here

Reference: Acute basophilic leukemia

Acute lymphoblastic leukemia / lymphoma
Definition / general
  • Highly aggressive neoplasm of precursor lymphoid (blast) cells
  • 2 main subtypes based on lymphoid lineage
    • B lymphoblastic leukemia / lymphoma (B-ALL / LBL)
    • T lymphoblastic leukemia / lymphoma (T-ALL / LBL)
Essential features
  • Characterized by neoplastic proliferation of clonal precursor B cells or T cells that typically have blastic cytomorphology
  • Lymphoblastic lymphoma generally refers to tissue mass lesion, while leukemia refers to bone marrow or blood involvement
  • B-ALL / LBL has a better prognosis in children than in adults
  • Childhood B lineage ALL / LBL has a better prognosis than childhood T lineage ALL / LBL
  • Immature cell markers, such as CD34 and TdT, can help to differentiate lymphoblasts from Burkitt lymphoma which, is considered a mature high grade B cell lymphoma that mimics lymphoblastic lymphoma / leukemia
Terminology
  • Lymphoblastic leukemia / lymphoma
  • Acute lymphoblastic leukemia (ALL)
  • Precursor B cell lymphoblastic leukemia / lymphoma or precursor T lymphoblastic leukemia / lymphoma
  • Separation of lymphoblastic leukemias and lymphomas based on a measure of disease burden / stage
    • Cases with tissue involvement + replacement of < 25% marrow involvement by lymphoid blasts = lymphoblastic lymphoma (LBL)
    • Cases with ≥ 25% marrow involvement = lymphoblastic leukemia
ICD coding
  • ICD-O:
    • 9811/3 - B lymphoblastic leukemia / lymphoma, NOS
    • 9837/3 - T lymphoblastic leukemia / lymphoma
  • ICD-10:
    • C83.5 - Lymphoblastic lymphoma, B cell or T cell
Epidemiology
Sites
  • Bone marrow, peripheral blood or nodal involvement
  • Anterior mediastinum
    • Particularly T-LBL
    • Compromised cardiac function and superior vena cava syndrome may result
  • Less frequent sites involved
    • Skin
    • Soft tissues
    • Central nervous system
Pathophysiology
  • Clonal proliferation of precursor B cells in the bone marrow
  • Clonal proliferation of precursor T cells in the thymus (thymocytes)
  • Specific sequential pathologic mechanisms that follow appear to be variable, based on the varying genetic aberrations identified
Etiology
  • Idiopathic and likely multifactorial
  • Chromosomal aberrations with gene mutations and rearrangements commonly cited
  • Specific risk factors seen in a minority of patients:
    • Prenatal radiation exposure (Xradiation)
    • Postnatal exposure to high doses of radiation (e.g. therapeutic radiation was previously used for tinea capitis and thymus enlargement)
    • Genetic conditions, such as Down syndrome, neurofibromatosis and ataxia telangiectasia
    • Inherited genetic polymorphisms
  • Reference: Swerdlow: WHO Classification of Tumors of Haematopoietic and Lymphoid Tissues (Medicine), 4th Edition, 2017
Clinical features
  • B-ALL / LBL
    • Common presentation of fever, fatigue, bone or joint pain, bleeding or anorexia (signs of bone marrow infiltration)
    • May present with widespread lymphadenopathy, hepatosplenomegaly, involvement of skin, soft tissue and testes, with predilection for the central nervous system
    • Mediastinal mass uncommon
  • T-ALL / LBL
    • Most commonly presents with anterior mediastinal mass with or without superior vena cava syndrome
    • Lymph node involvement on Xray / CT / MRI showing thymic enlargement or pleural effusion, resulting in symptoms of respiratory distress
    • Generalized lymphadenopathy and hepatosplenomegaly also frequent
  • Reference: Swerdlow: WHO Classification of Tumors of Haematopoietic and Lymphoid Tissues (Medicine), 4th Edition, 2017
Diagnosis
  • Multimodal pathologic evaluation with some combination of morphology, flow cytometry, immunohistochemistry, cytogenetics, FISH, PCR, NGS and basic clinical laboratory tests
  • Nodal excisional biopsy may be the initial diagnostic procedure, although marrow involvement is frequently present simultaneously
  • If marrow involvement suspected, initial approach may consist of complete blood count and peripheral blood smear, followed by bone marrow aspiration and biopsy
Laboratory
  • Complete blood count
    • Almost universally shows anemia and thrombocytopenia
      • ~25% have severely decreased platelet count (< 20,000/uL)
    • Total white blood cell (WBC) count variable but most commonly presents with neutropenia
      • 50% show WBC count < 10,000/uL
      • 30% show WBC count 5,000 - 50,000/uL
      • 20% show WBC count > 50,000/uL
      • 20% show preleukemic state with cytopenia < 1 year before overt leukemia
    • Rare reports of asymptomatic hypereosinophilia (Cancer 1984;54:1058), most commonly seen in the setting of t(5;14) (q31;q32) or relatively less common recurrent abnormalities, such as translocation t(7;12)(q22;p13); also reported in the setting of hyperdiploidy (Blood 2020;135:974, Swerdlow: WHO Classification of Tumors of Haematopoietic and Lymphoid Tissues (Medicine), 4th Edition, 2017)
Radiology description
  • Lymphadenopathy
  • T-LBL may show an anterior mediastinal mass due to thymic enlargement or presence of pleural effusion on Xray
Radiology images

Images hosted on other servers:

T-LBL mediastinal involvement

Prognostic factors
  • Prognosis generally determined by lineage (B cell or T cell phenotype), plus cytogenetic or molecular abnormalities detected
  • B-ALL / LBL
    • Unfavorable prognostic factors:
      • Infancy and adult age of diagnosis
      • High white blood cell count
      • Slow response to initial therapy
      • Central nervous system involvement at the time of diagnosis
      • Minimal residual disease after therapy
  • T-ALL / LBL
    • Worse prognosis than B-ALL / LBL among pediatric population
    • Better prognosis than B-ALL / LBL among adult population
  • Reference: Swerdlow: WHO Classification of Tumors of Haematopoietic and Lymphoid Tissues (Medicine), 4th Edition, 2017
Case reports
Treatment
  • Multiagent chemotherapeutic regimen stratified based on epidemiology, presentation and other patient specific associated characteristics of the disease, as well as presence of specific cytogenetic abnormalities and therapeutic response to initial induction (Pediatr Int 2018;60:4)
  • Intrathecal prophylactic therapy and radiation for those presenting with central nervous system involvement (Biol Blood Marrow Transplant 2016;22:575)
  • Allogeneic stem cell / bone marrow transplantation in those demonstrating high risk findings (Am J Hematol 2017;92:18)
  • Targeted tyrosine kinase inhibitors or JAK inhibitors may be used in patients with Ph-like ALL (Best Pract Res Clin Haematol 2018;31:351)
  • CAR-T cell immunotherapy in relapsed patients (Curr Hematol Malig Rep 2018;13:396)
  • Supportive therapy for cytopenias
Microscopic (histologic) description
  • In tissue masses or marrow, effacement of architecture by round blue cells raises the differential diagnoses of ALL / LBL versus neuroblastoma, Ewing sarcoma and other mimickers
  • Proliferation of typically small to medium sized primitive cells but some tumor cells may appear as larger blasts containing cytoplasmic vacuoles
  • B and T cell precursors are morphologically indistinguishable
  • In nodal involvement, blasts usually display a diffuse growth pattern
    • Partial nodal involvement of T-LBL may have a predominantly interfollicular distribution
    • Lymphoblasts may be oval in shape and display an indented nucleus with finely dispersed chromatin and inconspicuous nucleoli
  • Revised criteria to the original French American British (FAB) classification of ALL cytologic subtypes
    • L1 and L2 subtype segregation not shown to correlate with clinical or biologic behavior; thus, most commonly used for descriptive purpose at this time
      • L1 subtype: most common, with intermediate size, uniform features, scant basophilic cytoplasm, slightly condensed chromatin with inconspicuous / absent nucleoli
      • L2 subtype: more heterogeneous appearance, with slightly more cytoplasm and more prominent nucleoli, resembling myeloblasts
      • L3 subtype: now corresponds to leukemic phase of Burkitt lymphoma, a high grade mature B cell lymphoma (Blood Cancer J 2017;7:e577)
Microscopic (histologic) images

Contributed by Patricia Tsang, M.D.
B-LBL retroperitoneal fibroadipose

B-LBL retroperitoneal fibroadipose

B-LBL tongue, 20x

B-LBL tongue

B-LBL tongue, 60x

B-LBL tongue

B-LBL tongue TdT

B-LBL tongue TdT

B-LBL tongue PAX5

B-LBL tongue PAX5


B-LBL tongue CD20, 40x

B-LBL tongue CD20

T-LBL starry sky, 20x

T-LBL starry sky

T-LBL LN, 60x

T-LBL lymph node

T-LBL CD1a, 40x

T-LBL CD1a

Cytology description
Peripheral smear description
  • Predominance of lymphoblasts
Peripheral smear images

Contributed by Patricia Tsang, M.D.
T-lymphoblast on blood smear T-lymphoblast on blood smear

T lymphoblast on blood smear

Positive stains
Negative stains
Flow cytometry images

Contributed by Patricia Tsang, M.D.
Flow cytometry of B-lymphoblastic lymphoma

B lymphoblastic lymphoma

Molecular / cytogenetics description
  • Most B-ALL show clonal IgH (immunoglobulin heavy locus) gene rearrangement; simultaneous T cell receptor (TCR) gene rearrangement present in up to 70%
  • Recurrent genetic abnormalities in B-ALL often have prognostic significance:
    • t(12;21) TEL-AML: very favorable prognosis; most common rearrangement in childhood B-ALL / LBL (25%), with cures of > 90%
    • Hyperdiploidy: favorable prognosis; trisomies 4, 10 and 17 have the best prognosis
    • t(9;22) BCR-ABL1 (Philadelphia chromosome): very poor prognosis; incidence increases with age, 3% of childhood cases and 25% of adult
    • t(v;11q23) MLL gene rearrangement: poor prognosis
    • Hypodiploidy: poor prognosis
    • BCR-ABL1-like IGH-CRLF2 translocation (and less commonly EPOR): CRLF2 translocation frequent in Down syndrome; standard risk in children and high risk in adolescents and adults; more frequent in Hispanics and Native Americans
    • t(5;14) IL3-IGH: uncertain prognostic significance; reactive eosinophilia is characteristic
  • Most T-ALL / LBL harbor T cell receptor gene rearrangement; simultaneous IGH gene rearrangement present in about 20%
  • In T-ALL / LBL, recurrent cytogenetic abnormalities can also be seen:
    • TCR loci at 14q11.2, 7p35 and 7p14-15, with various partner genes
    • Del (9p) with loss of tumor suppressor gene CDKN2A occurs in at least 30%
    • NOTCH1 gene mutations can lead to shorter survival in adult T-ALL / LBL
  • Reference: Swerdlow: WHO Classification of Tumors of Haematopoietic and Lymphoid Tissues (Medicine), 4th Edition, 2017
Sample pathology report
  • Base of tongue, biopsy:
    • B lymphoblastic lymphoma, not otherwise specified (see microscopic description and FISH)
    • Microscopic description: Biopsy of the tongue shows benign squamous mucosa with an extensive and diffuse subepithelial infiltrate of medium sized to large neoplastic cells, some containing a prominent nucleolus. Mitoses and apoptotic bodies are readily observed. Immunohistochemistry demonstrates that the neoplastic cells are of precursor B cell origin (CD45+, PAX5+, CD79a+, TdT+, CD10+, CD43+, CD34 subset +, CD20 few scattered cells +). Not expressed are CD117, CD3, CD30, myeloperoxidase, lysozyme, EBV (EBER) and MYC. Proliferative fraction is > 95% based on Ki67.
    • FISH: Interphase FISH analysis performed on formalin fixed paraffin embedded tissue shows the following results
      • Positive for tetraploidy, consistent with a favorable prognostic implication in B-ALL / LBL.
      • Negative for MYC-IGH t(8;14), BCR-ABL t(9;22), BCL2 rearrangement and BCL6 rearrangement.
      • No specific genetic abnormalities were identified in the FISH ALL panel to fulfill the diagnostic criteria of B-ALL/LBL with recurrent genetic abnormalities.
Differential diagnosis
  • Burkitt lymphoma, a high grade B cell lymphoma:
    • In BL, there is typically monotypic surface immunoglobulin light chain expression
    • Starry sky histologic pattern and cytoplasmic vacuoles on smear or touch preparation
    • Unlike B-ALL / LBL, its phenotype is mature showing consistent CD20 but without CD34 or TdT expression
  • Hematogones, non-clonal B cell precursors in bone marrow, can be confused with B-ALL, especially in minimal residual disease:
    • Typically smaller than lymphoblasts
    • Have characteristic immunophenotypic expression pattern by flow cytometry
  • Thymoma with reactive thymic T cells exhibiting CD4+ CD8+ immature T cell phenotype, resembling T-LBL in anterior mediastinal mass:
    • Immunohistochemistry highlights the neoplastic epithelial cell component
    • Lobulated architecture
    • May be associated with myasthenia gravis in adults
  • Incidental indolent T lymphoproliferative proliferations (Cytometry B Clin Cytom 2020;98:282)
Board review style question #1

A 4 year old boy is brought to the physician by his mother due to a 5 week history of lethargy, a progressively enlarging left sided neck mass and a 5 day onset of unexplainable bilateral diffuse lower leg petechial hemorrhage. On exam, the left posterior cervical lymph node is enlarged and nontender to palpation. A lymph node biopsy shows a predominance of interfollicular infiltrate containing numerous blast cells and focal necrosis. Immunohistochemistry is positive for PAX5, CD10 and TdT. Upon further workup, which of the following translocations would be associated with a good prognosis?

  1. t(12;21)
  2. t(14;18)
  3. t(8;14)
  4. t(9;22)
Board review style answer #1
Board review style question #2
A 58 year old Hispanic woman comes to the physician because of a 2 month history of bilateral axillary lymphadenopathy and a new onset right breast mass at 12 o'clock. The patient has a previous history significant for Down syndrome and a diagnosis of acute lymphoblastic leukemia 20 years ago for which she received treatment and has since been free of disease. Needle core biopsy of the breast mass reveals extensive involvement by monotonous medium sized lymphoid cells with a high N/C ratio and scant cytoplasm, consistent with a diagnosis of B lymphoblastic lymphoma. Which of the following genetic mutations is most likely detected in blast cells?

  1. BCR-ABL1
  2. CRLF2
  3. ETV6-RUNX1
  4. FLT3
Board review style answer #2

Acute myelomonocytic leukemia (AMML)
Definition / general
  • As per the current 5th edition of WHO classification (2022), acute myelomonocytic leukemia (AMML) is a distinct entity within acute myeloid leukemia (AML), defined by differentiation category (Leukemia 2022;36:1703)
  • Acute myeloid leukemia, not otherwise specified (AML, NOS) according to the International Consensus Classification (ICC)
Essential features
  • Acute myelomonocytic leukemia shows evidence of both granulocytic and monocytic differentiation
  • Does not meet the criteria for inclusion in any of the other AML groups
  • Blasts / blast equivalents include myeloblasts, monoblasts and promonocytes and must comprise at least 20% of blood or bone marrow cells
  • Granulocytic differentiation is present but cells of monocytic lineage must be at least 20% of cells
Terminology
  • AML M4 (FAB)
  • AMMoL
  • AMML
ICD coding
  • ICD-10: C92.5 - acute myelomonocytic leukemia
  • ICD-11: 2A60.33 - acute myelomonocytic leukemia
Epidemiology
Sites
Pathophysiology
  • Proliferation of a malignant hematopoietic stem cell which retains the capacity to differentiate into both granulocytic and monocytic cells
  • Genetic drivers of AML include both chromosomal alterations and gene mutations
Etiology
  • Exact causes are unknown
Clinical features
  • Signs and symptoms relate to anemia, thrombocytopenia and increased number of infections
    • Early symptoms include fever, fatigue, weakness, weight loss, shortness of breath
    • Also can present as organomegaly, lymphadenopathy and other tissue infiltration (monocytic infiltrate)
  • Reference: Curr Oncol 2022;29:6245
Diagnosis
  • Diagnosis is based on integration of morphology, immunohistochemistry, cytochemistry, flow cytometry studies, cytogenetics and molecular findings
  • Immunophenotyping is useful to confirm that both myeloid and monocytic populations are present
  • Genetic testing excludes AML with recurrent genetic abnormalities which have myelomonocytic morphology
  • Criteria for diagnosis:
    • Blasts / blast equivalents comprise ≥ 20% of bone marrow cells
      • Blasts / blast equivalents include myeloblasts, monoblasts and promonocytes
      • Cells of monocytic lineage (monoblasts, promonocytes, monocytes) overall comprise at least 20% of cells
    • AMML is similar to but distinct from acute monoblastic / monocytic leukemia, which requires ≥ 80% of blasts / blast equivalents to be of monocytic lineage
    • Additional criteria: ≥ 3% of blasts should be positive for MPO by cytochemical stain (indicates myeloid lineage)
      • Monocytic lineage may be ascertained by nonspecific esterase stain (not always present) or monocytic immunophenotype (e.g., expression of CD14, CD11c, CD64, CD163, lysozyme)
    • No recurrent genetic abnormalities identified (Leukemia 2022;36:1703)
      • AMML of AML, defined by differentiation, morphologically resembles AML with CBFB::MYH11 fusion (formerly known as FAB M4Eo), AML with KMT2A rearrangement and some cases of AML with NPM1 mutation
Laboratory
  • Leukocytosis is common
  • Anemia, thrombocytopenia
Prognostic factors
  • Poor prognostic factors:
    • Adverse mutation profile, including wild type NPM1 and FLT3 ITDhigh, mutated RUNX1 and mutated TP53 (Am J Hematol 2018;93:1267)
    • Complex karyotype
Case reports
Treatment
  • Standard induction chemotherapy with cytarabine given twice daily for 10 days combined with an anthracycline drug for 3 days
  • Consolidation therapy often with higher doses of same drugs used for induction or an allogenic stem cell transplant
  • Targeted therapy if respective mutations are present (e.g., FLT3 or IDH1 / IDH2 inhibitors)
  • Reference: Curr Oncol 2022;29:6245
Microscopic (histologic) description
  • By definition, blasts and blast equivalents comprise at least 20% of cells in marrow or blood, while cells of monocytic lineage (monoblasts, promonocytes and monocytes) are also at least 20% of cells
  • Myeloblasts have high N:C ratios, round nuclei and variably granular cytoplasm; Auer rods occasionally seen
  • Monoblasts are large cells with abundant, moderately to intensely basophilic cytoplasm
    • May have pseudopod formation, scattered fine azurophilic granules and vacuoles
    • Often with round nuclei with lacy chromatin and one or more large nucleoli
  • Promonocytes have more irregular nuclei, typically with delicate folds and abundant, less basophilic cytoplasm which is more obviously granulated with occasional large azurophilic granules and vacuoles
  • Reference: Haematologica 2009;94:994
Microscopic (histologic) images

Contributed by Ramya Gadde, M.D., Rose Beck, M.D., Ph.D. and AFIP

Increased myelomonocytic population

Lysozyme immunostain

Missing Image

Bone marrow biopsy

Increased blasts and blast equivalents


Missing Image

Mixture of monocytes and neutrophils

Missing Image

Promonocytes

Missing Image

Promonocytes and dysplastic neutrophil

Missing Image

Promonocytes and myelocytes

Missing Image

Chloroacetate and nonspecific esterase stains

Peripheral smear description
  • Peripheral blood often has leukocytosis with blasts, may show increased monocytes and is often more mature than the monocytic cells observed in bone marrow
Peripheral smear images

AFIP images
Missing Image

Myeloblast, neutrophilic myelocyte and promonocyte

Positive stains
Negative stains
Flow cytometry description
Flow cytometry images

Contributed by Ramya Gadde, M.D. and Rose Beck, M.D., Ph.D.

Blast and monocytic population

Molecular / cytogenetics description
  • Nonspecific cytogenetic abnormalities in most cases, such as +8, monosomy 7 or del (7q), monosomy 5 or del(5q)
Sample pathology report
  • Bone marrow, core biopsy with touch imprint, clot section, aspirate smears, left iliac crest:
    • Acute myelomonocytic leukemia (see comment)
    • Comment: Hypercellular bone marrow for age (80 - 90%) with involvement by acute myeloid leukemia, defined by differentiation. Immature myelomonocytic cells are increased. Erythroid precursors are decreased. Mature myeloid cells are decreased. Megakaryocytes are adequate and appear normal in morphology.
    • Bone marrow aspirate smear is adequate. Blasts / blast equivalents including myeloblasts, monoblasts and promonocytes are increased (50%). Megakaryocytes are adequate in number and show normal morphology.
    • Flow cytometry studies demonstrate an abnormal myeloid blast population with expression of dim CD45, CD34, CD15 (subset), CD33, CD13, HLA-DR and CD117. There is also an immature monocytic population with CD64, CD11b, CD11c, CD13, CD14, CD33 and CD64 expression.
Differential diagnosis
Board review style question #1

What is the most likely overall morphology seen in this bone marrow?

  1. Acute myelomonocytic leukemia
  2. Acute promyelocytic leukemia
  3. AML with maturation
  4. AML without maturation
Board review style answer #1
A. Acute myelomonocytic leukemia

Comment Here

Reference: AMML
Board review style question #2
In acute myelomonocytic leukemia (AMML), flow cytometry would most likely show a blast population that has which of the following expression profiles?

  1. CD34+, CD117+, CD33+, HLA-DR+
  2. CD34-, CD117+, CD33+, HLA-DR-
  3. CD34-, CD117-, CD14+, CD33+, HLA-DR+
  4. CD34+, CD117+, CD33+, HLA-DR+ (myeloblast population) and CD34-, CD117-, CD14+, CD33+, HLA-DR+ (monocytic population)
Board review style answer #2
D. CD34+, CD117+, CD33+, HLA-DR+ (myeloblast population) and CD34-, CD117-, CD14+, CD33+, HLA-DR+ (monocytic population)

Comment Here

Reference: AMML

Acute panmyelosis with myelofibrosis
Definition / general
  • Rare subtype of acute myeloid leukemia (AML), NOS (Blood 2016;127:2391)
  • Accounts for 2% of all acute leukemias (Acta Med Port 2013;26:613)
  • Does not fulfill criteria for AML with recurrent genetic abnormalities, myelodysplastic related changes or other AML entity
  • Median survival is 2 - 9 months
Essential features
  • Acute panmyeloid proliferation with increased blasts (≥ 20% of cells in the bone marrow or peripheral blood) and fibrosis of the bone marrow
  • Marrow is diffusely fibrotic, showing panmyelosis, with immature granulocytic, megakaryocytic and erythroid cells
  • Weakness, fatigue, fever, bone pain, pancytopenia; usually no marked splenomegaly
Terminology
  • Also called acute (malignant) myelofibrosis, acute (malignant) myelosclerosis, acute myelodysplasia with myelofibrosis
ICD coding
  • ICD-10: C94.4 - acute panmyelosis with myelofibrosis
Epidemiology
Sites
  • Bone marrow
Pathophysiology
  • Involves granulocytic, megakaryocytic and erythroid cells, resulting in panmyelosis; marrow becomes fibrotic but with a high blast count of > 20%
Clinical features
  • Acute onset of disease, pancytopenia and bone marrow fibrosis in the absence of splenomegaly
Diagnosis
  • Peripheral blood shows pancytopenia, which is usually marked
  • Rare erythroblast can be seen, without presence of teardrop shaped cells
  • Dysplastic changes in myeloid cells are frequent
  • Abnormal platelets may be seen
  • Bone marrow biopsy is often suboptimal or unsuccessful due to degree of fibrosis; on histology the bone marrow biopsy shows diffuse stromal fibrosis, with increased erythrocyte precursors, megakaryocyte precursors and granulocyte precursors (panmyelosis), which show dysplastic features
  • Degree of presence of blasts in acute panmyelosis with myelofibrosis (APMF) is uncertain but recent studies have reported 20 - 25%
  • Degree of myelofibrosis is also variable but most patients have 3/3 reticulin fibrosis (Swerdlow: WHO Classification of Tumours of Haematopoietic and Lymphoid Tissues, Revised Edition, 2017)
Laboratory
  • Complete blood count with differential (CBC w/ diff)
    • Leukocytosis, lymphocytosis and atypical lymphocytes (if leukemic involvement)
  • Comprehensive blood chemistry (CMP)
    • Including liver and kidney analysis
  • Lactate dehydrogenase (LDH)
    • Elevated (indicator of tumor lysis)
  • Serum beta 2 microglobulin
    • Variable (indicator of tumor mass)
  • Reference: Clin Adv Hematol Oncol 2018;16:619
Prognostic factors
  • Overall rapidly progressive clinical course, with a median survival of < 1 year
Case reports
  • 16 year old boy admitted with severe pancytopenia and rapidly progressive bone marrow fibrosis (Acta Med Port 2013;26:613)
  • 45 year old man with abrupt onset of rapidly progressing low backache, weakness, pancytopenia without organomegaly (Mediterr J Hematol Infect Dis 2013;5:e2013042)
  • 66 year old man with suspicion of acute myeloid leukemia was admitted and treated for acute myeloid leukemia, NOS and later found to have APMF with EVI1 amplification (Cancer Genet 2012;205:255)
Treatment
Microscopic (histologic) description
Microscopic (histologic) images

Contributed by Julie Feldstein, M.D. and AFIP
Bone marrow trephine biopsy Bone marrow trephine biopsy

Bone marrow trephine biopsy

Reticulin Reticulin

Reticulin


Bone marrow biopsy Bone marrow biopsy

Bone marrow biopsy

Lymph node biopsy Lymph node biopsy

Lymph node biopsy


Bone marrow smear (Wright-Giemsa)

Bone marrow smear (Wright-Giemsa)

Stains Stains

Stains

Peripheral smear images

AFIP images
Peripheral smear (Wright-Giemsa)

Peripheral smear (Wright-Giemsa)

Positive stains
Negative stains
Flow cytometry description
  • Percentage of blasts varies between 3 and 30% (Mod Pathol 2005;18:603)
  • Distinct population of myeloid blasts positive for CD34, CD33 and CD117
  • Some cases have shown an aberrant coexpression of CD7 and CD13 antigens
  • Myeloid blasts in acute panmyelosis with myelofibrosis typically lack megakaryocytic associated antigens, such as CD41 and CD61 (Mod Pathol 2005;18:603)
Molecular / cytogenetics description
Sample pathology report
  • A. Posterior iliac crest, bone marrow biopsy and aspirate:
    • Acute panmyelosis (see comment)
    • Reticulin fibrosis
    • Comment: This is a hypercellular bone marrow for age featuring left shifted myeloid maturation with increased blasts. Numerous megakaryocytes with severe dysplastic features and focal clustering are seen. Flow cytometry shows left shifted myeloid maturation with increased blasts (~18%). The findings raise the differential diagnosis of acute panmyelosis with fibrosis versus MDS with excess blasts - 2. Clinical correlation regarding extramedullary hematopoiesis as well as molecular and cytogenetic results is recommended.
    • Bone marrow biopsy:
      • Iron: present (biopsy and smear)
      • Reticulin: increased (2 - 3 of 3)
    • Microscopic description:
      • The biopsy is adequate and consists of trabecular bone and bone marrow elements. The cellularity ranges from 70 to 90% and consists of left shifted myeloids admixed with erythroid precursors. Numerous megakaryocytes are present and show marked dysplastic features, such as hyperchromatic and monolobated forms.
      • CD34 immunostain is performed on formalin fixed paraffin embedded (FFPE) block A1, which highlights some of numerous dysplastic megakaryocytes as well as blasts, estimated around 15 - 20%.
    • Bone marrow aspirate:
      • Differential count:
        • M:E = 4.6:1
        • Blasts: 15%, promyelocytes: 6%, myelocytes / metamyelocytes: 15%, bands / segs: 34%, erythroids: 18%, lymphocytes: 14%, eosinophils: 1%, plasma cells: 1%, monocytes: 3%
      • The aspirate smear is hypoparticulate and hypocellular. There is a moderate increase in blasts (13%). The blasts are small to intermediate in size, with scant cytoplasm and high N/C ratio. Mild dysplastic changes are noted in the myeloid precursors in the form of abnormal segmentation and few pseudo Pelger-Huët cells as well as occasional binucleated erythroids.
Differential diagnosis
Board review style question #1


A 67 year old man previously in good state of health presented with fatigue, fever, bone pain and shortness of breath without splenomegaly. A complete blood count demonstrated anemia and thrombocytosis. A bone marrow biopsy shows a hypercellular marrow with panmyelosis, increased immature cells, dysplastic megakaryocytes and 2+ reticulin fibrosis. No other genetic abnormalities are found. What is the most likely diagnosis?

  1. Acute myeloid leukemia
  2. Acute panmyelosis with myelofibrosis
  3. Chronic myelogenous leukemia
  4. Myelodysplastic syndrome
Board review style answer #1
B. Acute panmyelosis with myelofibrosis. The symptoms and bone marrow biopsy support the diagnosis of acute panmyelosis with myelofibrosis. The biopsy above shows hypercellular bone marrow with panmyelosis, increased immature cells, dysplastic megakaryocytes and 2+ reticulin fibrosis. Additionally, the patient's rapid progression of symptoms and lack of splenomegaly further support this diagnosis.

Comment Here

Reference: Acute panmyelosis with myelofibrosis
Board review style question #2
What is the most common genetic mutation in acute panmyelosis with myelofibrosis (APMF)?

  1. ABL
  2. CALR
  3. Del5;t5
  4. JAK2
Board review style answer #2
C. Del5;t5. The most common but nonspecific chromosomal aberration seen in acute panmyelosis with myelofibrosis (APMF) is chromosome 5 abnormalities, including del5;t5. Primary myelofibrosis cases harbor a phenotypic mutation in JAK2 (most commonly) and CALR mutations. ABL is seen in chronic myelogenous leukemia.

Comment Here

Reference: Acute panmyelosis with myelofibrosis

Acute undifferentiated leukemia (pending)
[Pending]

Ambiguous lineage NOS
Definition / general
  • Acute leukemias in which the morphologic, cytochemical and immuno-phenotypic features of the blasts:
    • Lack sufficient evidence to classify as myeloid or lymphoid origin
    • Or, have morphologic and / or immunophenotypic characteristics of both myeloid and lymphoid cells
    • Or, have both B and T lineages (acute bilineal leukemia and acute biphenotypic leukemia)

    Acute leukemias of ambiguous lineage include the following:
    • Acute undifferentiated acute leukemia
    • Mixed phenotype acute leukemia with t(9;22)(q34;q112.); BCR-ABL1
    • Mixed phenotype acute leukemia with t(v;11q23); MLL rearranged
    • Mixed phenotype acute leukemia, B/myeloid, NOS
    • Mixed phenotype acute leukemia, T/myeloid, NOS
    • Mixed phenotype acute leukemia, NOS rare types
    • Other acute leukemias of ambiguous lineage
Epidemiology
  • < 4% of all acute leukemias, more frequent in adults
Etiology
  • Unknown
Clinical features
  • Related to bone marrow failure: fatigue, infections, bleeding
Morphology
  • Acute undifferentiated leukemia
    • Leukemic cells lack any differentiating features
  • Acute biphenotypic and acute bilineal leukemias
    • May present as one subtype of AML with features of ALL (B, T or B and T)
Immunophenotype
Undifferentiated acute leukemia:
  • Leukemias lack specific lineage markers
  • cCD79a, cCD22, CD3 and MPO
  • Generally don’t express more than one lineage associated marker
  • Often express HLA-DR, CD34, CD38, may express TdT and CD7

Bilineal acute leukemia
Biphenotypic acute leukemia
  • Blasts co-express myeloid and T or B lineage markers
  • Or, concurrent B and T lineage markers
  • Rarely co-express markers for myeloid, T and B lineages

  • Co-expression of one or two cross-lineage (nonspecific) markers is not sufficient for biphenotypic leukemia; e.g. myeloid antigen positive ALL or Lymphoid antigen positive AML
  • Lineage switch after therapeutic intervention
    • Possible expansion of pre-existing minor population of blasts of different lineage following therapeutic suppression of the major population
    • Possible lineage instability
Genetics
  • High degree of cytogenetic abnormalities
  • 1/3 of cases have Ph chromosome
  • t(4;11)(q21;q23) or 11q23, typically have CD10(–) precursor B lymphoid component
  • T/myeloid biphenotypic or bilineal leukemia can have complex karyotype
Cell of origin
  • Multipotent progenitor stem cell
Prognostic factors
  • Unfavorable, particularly in adults
  • t(4;11) or Ph particularly unfavorable
Treatment
  • Usually aggressive chemotherapy or bone marrow transplant
Case reports
Microscopic (histologic) images

AFIP images
Missing Image

Child with t(4;11)(q21;q23)

Missing Image

Lymphoblasts and monoblasts

Missing Image

Monoblasts, promonocytes and lymphoblasts

Electron microscopy images

AFIP images
Missing Image

Monocytoid blast

Molecular / cytogenetics description
  • Many cases have IgH and TCR rearrangements or deletions
Differential diagnosis
  • For biphenotypic acute leukemia
    • Myeloid antigen positive ALL
    • Lymphoid antigen positive AML
  • For undifferentiated acute leukemia

Atypical chronic myeloid leukemia; BCR::ABL1 negative
Definition / general
  • Leukemic disorder with myelodysplastic / myeloproliferative features present at initial diagnosis
  • Leukocytosis characterized by increased morphologically dysplastic neutrophils and their precursors
Essential features
  • Clonal hematopoietic stem cell disorder that manifests with leukocytosis (always > 13 × 109/L, frequently > 25 × 109/L)
  • Features are distinct from other myelodysplastic / myeloproliferative neoplasms (chronic myelomonocytic leukemia (CMML), juvenile myelomonocytic leukemia (JMML), myelodysplastic / myeloproliferative neoplasm with ring sideroblasts and thrombocytosis (MDS / MPN-RS-T))
  • Does not meet WHO diagnostic criteria for BCR-ABL1 positive chronic myeloid leukemia, primary myelofibrosis, polycythemia vera or essential thrombocythemia
  • No Philadelphia chromosome / BCR-ABL1 fusion gene
  • No PDGFRA, PDGFRB or FGFR1 rearrangements
  • No PCM1-JAK2 translocation
Terminology
  • Philadelphia chromosome negative (Ph1-) atypical CML
ICD coding
  • ICD-O: 9876/3 - Atypical chronic myeloid leukemia, BCR-ABL1 negative
Epidemiology
Sites
  • Peripheral blood and bone marrow always involved
  • Splenic and hepatic involvement common
Pathophysiology
Etiology
  • Unknown currently
Clinical features
Diagnosis
  • WHO diagnostic criteria:
    • Peripheral blood leukocytosis ~13 × 109/L, due to increased numbers of neutrophils and their precursors (i.e. promyelocytes, myelocytes and metamyelocytes), with neutrophil precursors constituting ≥ 10% of the leukocytes
    • Dysgranulopoiesis, which may include abnormal chromatin clumping or projection, abnormal segmentation or hypogranularity
    • No or minimal absolute basophilia; basophils constitute < 2% of the peripheral blood leukocytes
    • No or minimal absolute monocytosis; monocytes constitute < 10% of the peripheral blood leukocytes
    • Hypercellular bone marrow with granulocytic proliferation and granulocytic dysplasia, with or without dysplasia in the erythroid and megakaryocytic lineages
    • < 20% blasts in the blood and bone marrow
    • No evidence of PDGFRA, PDGFRB or FGFR1 rearrangement or of PCM1-JAK2 translocation
    • WHO criteria for BCR-ABL positive chronic myeloid leukemia, primary myelofibrosis, polycythemia vera or essential thrombocythemia are not met
    • History of MPN, the presence of MPN features in the bone marrow or MPN associated mutations (e.g. JAK2, CALR or MPL) should be excluded
  • References: PLoS One 2014;9:e98258, Haematologica 2006;91:1566, Ann Oncol 2000;11:441, Blood 1991;78:205, Blood 2014;123:2645
Laboratory
  • Increased LDH
Radiology description
  • Splenomegaly
Prognostic factors
Case reports
Treatment
Microscopic (histologic) description
  • Bone marrow (Jaffe: Hematopathology, 2nd Edition, 2016, Blood 2014;123:2645):
    • Hypercellular marrow with increased granulocytic proliferation
    • Increased myeloid:erythroid ratio > 10:1
    • May have increased blasts but < 20%
    • Granulocytic dysplasia often marked
    • Dyserythropoiesis can be present in up to half of cases
    • Dysmegakaryopoiesis common, including small or micromegakaryocytes and hypolobated forms
    • Reticulin fibrosis increased in some patients
  • Syndrome of abnormal chromatin clumping (Haematologica 1990;75:532, Nouv Rev Fr Hematol 1995;37:245):
    • Considered a variant of aCML
    • Characterized by high percentage of granulocytes and precursors with exaggerated chromatin clumping identified in peripheral blood or bone marrow
Microscopic (histologic) images

Contributed by Ling Zhang, M.D.

Bone marrow biopsy

Bone marrow biopsy


Bone marrow aspirate smear

Peripheral smear description
  • Leukocytosis with white blood cells (WBC) ≥ 13 × 109/L with median values of 24 - 96 × 109/L
  • < 20% blasts in peripheral blood and bone marrow; usually < 5%
  • Neutrophil precursors (promyelocytes, myelocytes and metamyelocytes) comprising > 10% of WBC
  • Prominent dysgranulopoiesis
    • Acquired Pelger-Huët abnormality
    • Clumped chromatin
    • Hypersegmentation of nuclei
    • Abnormal nuclear projections
    • Cytoplasmic hypogranularity
  • Absent or minimal monocytosis; < 10% of leukocytes
  • Absent or minimal basophilia; < 2% of leukocytes
  • Anemia and thrombocytopenia are common
  • References: PLoS One 2014;9:e98258, Haematologica 2006;91:1566, Ann Oncol 2000;11:441, Blood 1991;78:205, Blood 2014;123:2645
Peripheral smear images

Contributed by Ling Zhang, M.D.

Peripheral blood smear

Positive stains
Flow cytometry description
  • No specific flow cytometry findings
Molecular / cytogenetics description
Sample pathology report
  • Peripheral blood:
    • Mild normocytic anemia
    • Marked leukocytosis with neutrophilia (70%) and circulating immature granulocytic precursors (22%)
    • Dysplastic neutrophils
    • No circulating blasts or mature monocytosis identified
    • Mild thrombocytopenia
  • Bone marrow, right posterior iliac crest, core biopsy, clot section and aspirate smear:
    • Hypercellular bone marrow with marked granulocytic hyperplasia and dysgranulopoiesis consistent with atypical chronic myeloid leukemia (see comment)
    • Comment: Of note, there is a marked leukocytosis with overt dysmyelopoiesis. FISH study is reportedly negative for t(9;22) / BCR-ABL1. Gene mutation profile showed a mutation involving the SETBP1 gene (VAF of 66%). Karyotyping was reportedly normal female karyotype, 46,XX[20]. The overall findings are consistent with atypical chronic myeloid leukemia.
    • Peripheral smear: Manual review of the peripheral blood shows normochromic, normocytic anemia with mild thrombocytopenia. RBCs: mild normochromic, normocytic with minimal anisopoikilocytosis. WBCs: marked leukocytosis with markedly increased dysplastic neutrophils and left shifted myeloid precursors. Platelets: thrombocytopenia with rare large forms.
    • Bone marrow biopsy: Quality: adequate. Cellularity: 90%. The bone marrow shows hypercellularity (90%) with marked granulocytic hyperplasia and diminished erythroid precursors. Megakaryocytes are increased and many of them are smaller than normal in size. The M:E ratio is increased. Reticulin stain highlights mild reituculin fibrosis (MF1). Iron stores are decreased to absent.
    • Bone marrow clot section: Quality: adequate. Cellularity: 90% morphologic features are similar to those observed in the core biopsy.
    • Bone marrow aspirate: Quality: adequate. Granulocytes: markedly increased; left shifted maturation without overt increase in blasts; marked dysplasia (abnormal chromatin pattern, hyposegmented nuclei or hypogranular cytoplasm). Erythrocytes: markedly decreased with progressive maturation, mild dysplasia (slight nuclear irregularities, basophilic stippling of cytoplasm or N:C maturation asynchrony). Megakaryocytes: adequate in number; dysplastic (variable in size, increased hypolobated forms, frequently with nuclear hyperchromasia). Blasts: overall ~2% of nucleated cells. No ring sideroblasts are present.
Differential diagnosis
  • Myeloproliferative neoplasm (MPN):
    • No dysplasia (except for abnormal megakaryocytes)
    • Cytosis (see CML and CNL below for differential findings)
    • BCR-ABL fusion
    • JAK2, CALR, MPL, CSF3R mutations
  • Chronic myeloid leukemia (CML):
    • Presence of Philadelphia chromosome / BCR-ABL fusion gene
    • Dysplasia is minimal in CML
    • Basophilia > 2% in CML
  • Chronic neutrophilic leukemia (CNL):
    • Leukocytosis comprised mostly of neutrophils
    • No significant left shift in proliferating neutrophils
    • CSF3R mutation more common in CNL and favor this entity
  • Myelodysplastic syndrome (MDS):
    • Dysplasia in ≥ 1 lineage
    • No leukocytosis
    • Cytopenias predominate
  • Myeloid / lymphoid neoplasms with eosinophilia and gene rearrangement:
  • Chronic myelomonocytic leukemia (CMML):
    • Monocytosis, > 10% of WBCs
    • More severe dysplasia in aCML
    • Higher survival
  • Myelodysplastic / myeloproliferative neoplasm, unclassifiable (MDS / MPN, U):
    • Leukocytosis and dysplastic changes in MDS / MPN, U raises differential diagnosis of aCML
    • Presence of SETBP1 mutations favor the diagnosis of aCML
    • Often lower WBC count when compared to aCML
    • Distinction cannot be made in a subset of cases
    • Clinically they are indistinguishable
    • Significant blood and bone marrow morphological overlapping features
    • aCML carries a worse prognosis (overall survival of 12.4 months versus 21.8 months for MDS / MPN, U)
  • Reference: Jaffe: Hematopathology, 2nd Edition, 2016
Board review style question #1

A 71 year old man presented to his primary care physician with complaints of fatigue and nausea. Upon further workup, he was found to have normocytic anemia, thrombocytopenia and marked leukocytosis (70 × 109/L) with no monocytosis. Imaging studies demonstrated a mildly enlarged spleen with no discernible lesions. A bone marrow biopsy was performed showing no increase in blasts with representative images shown above. FISH studies for BCR-ABL were negative. Gene mutation profile demonstrated a mutation involving the ETNK1 gene. Which of the following is the most likely diagnosis?

  1. Chronic myeloid leukemia (CML)
  2. Chronic neutrophilic leukemia (CNL)
  3. Acute myeloid leukemia (AML)
  4. Chronic myelomonocytic leukemia (CMML)
  5. Atypical chronic myeloid leukemia (aCML)
Board review style answer #1
E. aCML is a myeloproliferative / myelodysplastic (MPN / MDS) neoplasm associated with leukocytosis with no associated monocytosis. Clinically, patients often present with anemia, thrombocytopenia and splenomegaly. Bone marrow biopsy shows marked granulocytic hyperplasia and dysplastic neutrophils and left shifted myeloid precursors. FISH studies are characteristically negative for BCR-ABL with molecular studies often showing presence of SETBP1 and ETNK1 mutations. Answer choices A, B and C all represent additional MPN / MDS overlap neoplasms and can be all be ruled out with the findings above: CML is characterized by the presence of a BCR-ABL translocation. CNL is characterized by markedly increased neutrophils with no significant dysplasia or left shifted myeloid precursors. CMML shows a persistent monocytosis, not seen in the above case. AML is characterized by increased blasts > 20% of the total cellularity, wherein the blast count is reportedly normal above.

Comment Here

Reference: Atypical chronic myeloid leukemia
Board review style question #2
Which of the following cytogenetic or molecular alterations is most frequently identified in aCML?

  1. CSF3R
  2. JAK2
  3. PCM-JAK2
  4. PDGFR
  5. SETBP1
Board review style answer #2
E. SETBP1. PCM-JAK2 and PDGFRA / B gene rearrangements are independent entities listed in 2017 WHO that should be excluded before a diagnosis of aCML is established. JAK2 gene mutation is extremely rarely identified in aCML and when positive, aCML should be excluded. Although CSF3R mutation was reported in a subset of aCML, the overall incidence rate (< 10%) is much lower than SETBP1 (24.3%). The presence of CSF3R mutations, in conjunction with neutrophilia (> 80% of differential count) without dysplasia, favors a diagnosis of chronic neutrophilic leukemia (CNL) over aCML (Nat Genet 2013;45:18).

Comment Here

Reference: Atypical chronic myeloid leukemia

BCR::ABL1-like
Definition / general
Essential features
ICD coding
  • ICD-10: C91.00 - acute lymphoblastic leukemia not having achieved remission
Epidemiology
Pathophysiology
  • Harbors a large number of kinase activating gene rearrangements primarily involving the ABL class, JAK / STAT or RAS pathway associated signaling pathways
  • Common genes involved include ABL1, ABL2, CRLF2, CSF1R, EPOR, NTRK3, PDGFRB, JAK1, JAK2, JAK3, FLT3, IL7R, SH2B3 and IKZF1
    • These genes encode proteins involved in B cell development, proliferation and differentiation, cell cycle regulation and cell signaling
  • Overexpression of cytokine receptors such as cytokine receptor-like factor 2 (CRLF2) occurs in both ALL with BCR-ABL1-like / Philadelphia-like and other B ALL categories (Eur J Cancer 2017;82:203)
Clinical features
  • B ALL in general usually presents with symptoms related to bone marrow suppression by lymphoblasts
    • Patients can present with anemia, leucopenia or thrombocytopenia or a combination of these
    • Symptoms include bruising or bleeding due to thrombocytopenia, pallor or fatigue due to anemia and recurrent infections caused by neutropenia / leucopenia or bone pains
    • May also present with lymphadenopathy (> 10 mm in single dimension of the lymph node), hepatomegaly or splenomegaly
  • B ALL with BCR-ABL1-like and other B ALL with recurrent genetic abnormalities show no unique clinical presentation or microscopic findings
    • Further, similar to some other B ALL with recurrent genetic abnormalities, B ALL with BCR-ABL1-like shows no unique immunophenotypic profile (Pediatr Blood Cancer 2017;64:10.1002)
  • Essentially a diagnosis of exclusion
  • Identifying patients with B ALL with BCR-ABL1-like can influence the management of the patient
    • Those with PDGFRB translocation can benefit from tyrosine kinase inhibitors, while patients with JAK translocations may benefit from JAK inhibitors (Pediatr Blood Cancer 2017;64:10.1002)
  • Adults tend to have poor outcome even with high intensity chemotherapy regimens
Laboratory
  • B ALL with BCR-ABL1-like patients do not show a BCR-ABL1 fusion protein expressed from t(9;22)(q34;q11.2); however, they have a gene expression profile similar to ALL with BCR-ABL1
  • Almost none of the genetic alterations are detected by standard genetic diagnostic methods such as conventional karyotyping and FISH because these genetic alterations are diverse and often cryptic
  • To date, there are over 60 different identified rearrangements and approximately 16 different targetable genes
  • Patients can only be diagnosed by gene expression panels (genome wide Affymetrix gene expression arrays):
    • Currently 2 gene expression signatures have been verified for diagnosis of B ALL with BCR-ABL1-like / Ph-like
  • BCR-ABL1-like gene expression array consists of 110 expression probe sets and was originally developed to classify B ALL in a population based Dutch / German cohort
    • Gene expression of a group of cases were similar to ALL with BCR-ABL1 and therefore named the category ALL with BCR-ABL1-like
    • These had an overall unfavorable outcome (Lancet Oncol 2009;10:125)
  • Philadelphia-like signature was first demonstrated in cases of B ALL with IKZF1 deletions in a high risk United States cohort (N Engl J Med 2009;360:470)
  • Differences between these 2 gene expression signatures can be explained by their different origins and discovery cohorts
  • Few reference laboratories are currently offering low density microarray classifiers (8 or 15 gene) that may help screen patients
  • One of these reference laboratories is Tricore reference laboratories (BCR-ABL1 like ALL (Ph-like ALL): TriCore Reference Laboratories [Accessed 8 May 2019])
  • Some institutions and reference laboratories have developed multiplex FISH testing that may help screen cases
  • Some institutions such as St. Jude utilize transcriptome and whole exome sequencing
  • Flowcytometry:
    • There is evidence that B-ALL with BCR-ABL1-like present as common-ALL on IPT, however this maturation stage is not distinct of the entity and is common in other B-ALL subt
    • ypes B-ALL with BCR-ABL1-like patients with CRLF translocations show high levels of surface expression of the protein on flow cytometry, which can be used as a screening tool
Prognosis
  • Prognostic factors include age, white blood cell count, immunophenotype, genetics and detection of measurable residual disease
    • Numerous stratification schemes to assess risk in B ALL
    • NCI risk group:
      • NCI standard risk group: WBC < 50,000/microL and age 1 to < 10 years
      • NCI high risk group: WBC ≥ 50,000/microL or age ≥ 10 years (up to 13 years if treated on a COG protocol)
    • One scheme stratifies patients into standard risk and high risk based on age (< 10 years = standard, > 10 years = high) and WBC count (< 50,000/mm3 = standard, > 50,000/mm3 = high)
  • Recurrent genetic abnormalities are identified in the majority of B ALL cases
  • Prognosis of B ALL with BCR-ABL1-like is generally unfavorable
Case reports
Treatment
  • B ALL with BCR-ABL1-like is managed by standard combination chemotherapy
    • Effective treatment modality for B ALL since the 1950s
    • Usually administered in 3 distinct phases (induction, consolidation and maintenance) and should include intrathecal treatment, which is directed to the central nervous system
  • Addition of tyrosine kinase inhibitors in B ALL with BCR-ABL1-like depends on the specific genetic alteration and has been shown to improve patient's outcome
  • Both imatinib and dasatinib specifically inhibit ABL1, ABL2, PDGFRB and CSF1R kinases; ruxolitinib inhibits JAK / CRLF2 / EPOR alterations (Eur J Cancer 2017;82:203)
  • Sensitivity to ruxolitinib in patients with JAK class aberrations is promising and has been successful in 2 JAK class fusion positive patients and 1 JAK2 mutated ALL (Eur J Cancer 2017;82:203)
  • Adults tend to have poor outcome even with high intensity chemotherapy regimens; therefore, bone marrow transplantation becomes an important treatment option
Microscopic (histologic) description
  • Blasts have scant agranular cytoplasm, coarse to fine chromatin, often with indistinct nucleoli
  • No Auer rods and no dysplastic myeloid cells
Microscopic (histologic) images

Contributed by Hatem Kaseb, M.D., Ph.D., M.P.H.

Lymphoblasts

Peripheral smear images

Contributed by Hatem Kaseb, M.D., Ph.D., M.P.H.

Lymphoblasts

Positive stains
Negative stains
Molecular / cytogenetics description
  • Gene expression profiling is the gold standard for diagnosis of B ALL with BCR-ABL1-like / Ph-like (Lancet Oncol 2009;10:125, N Engl J Med 2009;360:470)
  • Genetic alterations in B ALL with BCR-ABL1-like are harder to detect by standard genetic diagnostic approaches (Eur J Cancer 2017;82:203)
  • Common gene rearrangements include CRLF2, EPOR and IGH
  • Down syndrome patients uniquely show CRLF2 translocation
  • ABL1 oncogene translocation with ABL2, PDGFRB, NTRK3, TYK2, CSF1R and JAK2 have all been reported
  • Many cases of B ALL with BCR-ABL1-like may additionally show other deletions or mutations that have a clear role in leukemogenesis such as IKZFA and CDKN2A / B
  • FISH and karyotyping are helpful in ruling out other B ALL entities, especially ALL with recurrent genetic abnormalities
  • Multiplex FISH testing can be useful in ruling out B ALL with recurrent genetic abnormalities
  • Identifying patients with B ALL with BCR-ABL1-like can influence patients' prognosis and management of the patient
    • PDGFRB translocation: may benefit from tyrosine kinase inhibitors
    • JAK translocations: may benefit from JAK inhibitors
Differential diagnosis
Board review style question #1
What is the most sensitive test in diagnosing B lymphoblastic leukemia / lymphoma with BCR-ABL1-like / Ph-like?

  1. Flow cytometry
  2. Gene expression profiling
  3. Karyotyping
  4. Multiplex FISH
Board review style answer #1
B. Only gene expression profiling can diagnose the entity B ALL with BCR-ABL1-like / Ph-like

Comment Here

Reference: BCR-ABL1-like
Board review style question #2
Which of the following is a characteristic of B lymphoblastic leukemia / lymphoma with BCR-ABL1-like / Ph-like?

  1. NCI high risk features
  2. NCI intermediate risk features
  3. NCI low risk features
  4. NCI risk features is not applicable
Board review style answer #2
A. ALL with BCR-ABL1-like shows the following characteristics: NCI high risk features, persistent postinduction minimal residual disease (MRD), genetic alterations that activate kinase signaling (Pediatr Blood Cancer 2017;64:10.1002). NCI risk stratifications include NCI standard risk group: WBC < 50,000/microL and age 1 to < 10 years; NCI high risk group: WBC ≥ 50,000/microL or age ≥ 10 years (up to 13 years if treated on a COG protocol).

Comment Here

Reference: BCR-ABL1-like

Blastic plasmacytoid dendritic cell neoplasm
Definition / general
  • Blastic plasmacytoid dendritic cell neoplasm (BPDCN) is a highly aggressive hematologic malignancy derived from plasmacytoid dendritic cells (pDC) (Curr Hematol Malig Rep 2018;13:477)
Essential features
  • Nonneoplastic counterpart: plasmacytoid dendritic cells (Adv Anat Pathol 2009;16:392)
  • Over 80% of patients present with skin lesions; other common sites include the bone marrow and lymph nodes
  • Expression of CD123 and TCF4 is required for diagnosis; in addition, most cases are positive for CD4, CD56 and TCL1
  • Distinction from acute myeloid leukemia (AML) in particular is critical for appropriate therapy selection
Terminology
  • Blastic plasmacytoid dendritic cell neoplasm
  • Agranular CD4+ / CD56+ hematodermic neoplasm / tumor
  • Agranular CD4+ NK cell leukemia
  • CD56+ TdT+ blastic NK cell lymphoma
ICD coding
  • ICD-O: 9727/3 - blastic plasmacytoid dendritic cell neoplasm
Epidemiology
  • Rare, representing < 1% of hematologic malignancies (Leuk Res 2018;73:21)
  • Incidence is 0.04 cases per 100,000 in U.S.
  • M:F = 3:1
  • Most patients are in fifth or sixth decades of life but the disease can affect any age group, including pediatric
Sites
  • Skin, bone marrow, lymph nodes and spleen are most common but any anatomic site can be involved
  • CSF involvement at presentation in up to 10% of patients, more at relapse (Oncotarget 2016;7:10174)
Pathophysiology
  • Postulated cell of origin is pDC, which plays an important role in bridging the innate and adaptive immune systems (Curr Hematol Malig Rep 2018;13:477)
  • Monoallelic and biallelic 12p / ETV6 deletions seem to represent an early pathogenic event (Leuk Res 2018;73:86)
  • BPDCN specific transcriptional network regulated by the E-box transcription factor TCF4 (also known as E2-2) that plays a master regulatory role in BPDCN cells and appears to be in turn regulated by the bromodomain and extraterminal domain (BET) protein BRD4 (Cancer Cell 2016;30:764)
  • Epigenetic dysregulation is very common and results from mutations in genes involved in DNA methylation, histone methylation and chromatin remodeling (Haematologica 2019;104:729)
  • Frequent NR3C1 haploinsufficiency linked to aberrant overexpression of a novel long noncoding RNA (lncRNA) gene, lincRNA-3q, whose product is involved in G1/S cell cycle transition through E2F (Blood 2016;127:3040)
  • BPDCN cells are highly dependent on BCL2 for survival (Cancer Discov 2017;7:156)
  • Neoplastic cells in BPDCN have features of nonactivation state and seem to arise in a background of immunodeficiency (Blood Cancer J 2019;9:99)
Etiology
  • No known etiology
  • Frequent association with other myeloid malignancies, including chronic myelomonocytic leukemia and myelodysplastic syndrome
  • No bacterial or viral pathogen (Blood Adv 2020;4:1006)
Diagrams / tables

Contributed by Joseph Khoury, M.D.
Markers expressed by pDCs

Markers expressed by pDCs

Clinical features
  • Skin lesions, with heterogeneous manifestations of bruise-like, violaceous maculopapular lesions, nodules, plaques or exanthema
  • ~10% of patients have an overt leukemic presentation (Am J Hematol 2013;88:1055)
  • B symptoms
  • Lymphadenopathy
  • Splenomegaly
Diagnosis
  • Malignant infiltrate of blastoid cells; need to distinguish from mature pDC proliferations (MPDCP)
  • Confirmation of pDC immunophenotype by flow cytometry or immunohistochemistry
  • Cytogenetic and molecular studies usually not required for diagnosis
Laboratory
  • Cytopenias are common but some patients have normal CBC findings
Prognostic factors
  • No known pathologic or molecular prognostic factors
Case reports
  • 62 year old woman with BPDCN exhibiting unusual lymphoid features and macrovacuoles (Ann Hematol 2019;98:2221)
  • 63 year old man with disseminated blastic plasmacytoid dendritic cell neoplasm (Blood 2015;126:558)
Treatment
  • Tagraxofusp, a CD123 targeted immunotoxin, is the first FDA approved treatment for BPDCN (N Engl J Med 2019;380:1628)
  • ALL type induction chemotherapy regimens are associated with better outcomes compared with AML type regimens
  • Allogeneic stem cell transplant is recommended for patients who achieve first complete remission and are fit (Hematol Oncol Clin North Am 2020;34:621)
Microscopic (histologic) description
  • Diffuse infiltrate seen on bone marrow biopsy (trephine) and clot preparation with replacement of hematopoietic elements
  • On Wright-Giemsa preparations, the neoplastic cells are elongated with a tapered lightly basophilic agranular cytoplasm that ends with a tail shaped structure; the nucleus has blastoid features with open chromatin and a prominent nucleolus
  • In some cases, neoplastic cells have cytoplasmic microvacuoles with a peculiar arrangement along the cytosolic aspect of the cell membrane resembling a pearl necklace
  • Skin involvement is characterized by infiltrates of immature neoplastic cells with blastoid morphology involving the superficial and deep dermis, often with extension into the subcutis; the epidermis is typically spared; mitotic figures are usually identified readily
Microscopic (histologic) images

Contributed by Joseph Khoury, M.D.
Cytomorphology of BPDCN cells

Cytomorphology of BPDCN cells

Bone marrow involvement by BPDCN

Skin involvement by BPDCN Skin involvement by BPDCN

Skin involvement by BPDCN


TCF4 / CD123 dual color stain in bone marrow

TCF4 / CD123 dual color stain in bone marrow

 TCF4/CD123 dual color stain in skin

TCF4/CD123 dual color stain in skin

CD123

CD123

CD4

CD4

CD56

Positive stains
Negative stains
Flow cytometry description
  • 3 primary diagnostic aims (Haematologica 2020 Apr 2 [Epub ahead of print]):
    • Distinguish between BPDCN and other types of acute leukemia, mainly AML
    • Detect mature pDC proliferations in the context of other myeloid neoplasms, primarily chronic myelomonocytic leukemia
    • Detection of measurable / minimal residual disease (MRD)
  • Neoplastic cells fall into the blast gate on CD45 / SSC
  • Detection of MRD requires careful distinction between BPDCN cells and nonneoplastic pDCs
Flow cytometry images

Contributed by Joseph Khoury, M.D.
Flow cytometry findings in BPDCN

Flow cytometry findings in BPDCN

Molecular / cytogenetics description
  • Complex karyotype is common
  • Many adult patients have MDS associated chromosomal abnormalities and gene mutational profile
  • Gene rearrangements detected commonly: ETV6, MYC
  • Structural chromosomal losses, commonly involving 13q14 (RB1), 9p21.3 (CDKN2A/CDKN2B), 12p13.2-p13.1 (CDKN1B, ETV6), 7p12.2 (IKZF1)
  • Mutations detected commonly: TET2, ASXL1, IDH1, IDH2, DNMT3A, EZH2, SF3B1, SRSF2, ZRSR2, FLT3, KIT, KRAS, NRAS
Sample pathology report
  • Bone marrow, aspiration, clot and core biopsy:
    • Diagnosis: blastic plasmacytoid dendritic cell neoplasm (see comment)
    • Comment:
      • Immunohistochemistry: positive for TCF4 / CD123 coexpression
      • Flow cytometry: large population of neoplastic cells, positive for CD123, CD4, CD56, CD303 (decreased), HLA-DR and TdT; negative for CD3 (surface and cytoplasmic), CD19 and myeloperoxidase
      • Karyotype: 48, XY, t(6;8)(p12;q24.2), add(7)(p11.2), del(13)(q12q22), +16, add(20)(q11.2), +21[10]/46, XY[10]
      • Mutation profile: ASXL1, TET2, NRAS
Differential diagnosis
Board review style question #1
Which of the following immunophenotypic markers is typically negative in blastic plasmacytoid dendritic cell neoplasm?

  1. CD56
  2. CD123
  3. CD3
  4. CD4
  5. CD45
Board review style answer #1
C. CD3. All others are typically positive in BPDCN but can also be positive in other neoplasms including acute myeloid leukemia.

Comment Here

Reference: Blastic plasmacytoid dendritic cell neoplasm
Board review style question #2
Which of the following dual stains are specific and sensitive for differentiating blastic plasmacytoid dendritic cell neoplasm from acute leukemia, both of which may show overlapping features?

  1. CD56 and CD4
  2. CD123 and TCF4
  3. CD123 and CD19
  4. TCF4 and MPO
  5. TCL1 and CD3
Board review style answer #2
B. CD123 and TCF4. Highly sensitive and specific tool for differentiating blastic plasmacytoid dendritic cell neoplasm from acute leukemia.

Comment Here

Reference: Blastic plasmacytoid dendritic cell neoplasm

Chronic eosinophilic leukemia
Definition / general
  • Chronic eosinophilic leukemia (CEL) is an autonomous, clonal proliferation of eosinophils that leads to a persistent increase in eosinophils in peripheral blood, bone marrow and sometimes tissue
    • Absolute eosinophil count is ≥ 1.5 x 109/L on at least 2 occasions for 4 weeks (notable change from 6 months in the 4th edition of WHO)
    • Bone marrow shows abnormal morphology (e.g., dysmegakaryopoiesis, eosinophilic infiltrate associated overt fibrosis)
    • Clonal
  • Clonal myeloid or lymphoid neoplasms and reactive causes of eosinophilia should be excluded
Essential features
  • Rare myeloproliferative neoplasm characterized by a gradual increase in circulating eosinophils with cytologic atypia, left shifted granulocytic maturation and often hepatomegaly or splenomegaly
  • Organ / tissue infiltration by eosinophils and eosinophilic microabscesses are frequently present, leading to tissue damage and organ dysfunction
  • Clinical course is variable from indolent to life threatening; transformation to acute myeloid leukemia can occur and prognosis is generally poor
    • Fatal cardiac damage has been documented secondary to the release of cytokines produced by eosinophils
    • Diagnosis should be made after exclusion of reactive causes of eosinophilia and myeloid and lymphoid neoplasms associated with PDGFRA, PDGFRB, FGFR1, FLT3 or JAK2 rearrangements with evidence of clonality (see updated full WHO criteria) (Am J Hematol 2017;92:1243)
  • Cell of origin: pluripotent hematopoietic stem cell
Epidemiology
  • Due to difficulty in distinguishing CEL from nonclonal hypereosinophilic syndrome, the true incidence is unknown
  • A recent study indicates that only 1.2% of cases with peripheral eosinophilia met criteria for CEL (Am J Hematol 2020;95:E172)
  • The Surveillance, Epidemiology and End Results (SEER) database from 2004 - 2015 estimates the incidence of hypereosinophilic syndrome (HES) including CEL, not otherwise specified (NOS) is 0.4 cases per 1,000,000 (Am J Hematol 2022;97:129)
  • Some reports show a predilection for men in the seventh decade (Mod Pathol 2016;29:854, Am J Hematol 2012;87:643)
Sites
  • Peripheral blood and bone marrow are always involved
  • Splenic and hepatic involvement are also common
  • Other frequent sites of involvement include the heart, lungs, central nervous system (CNS), skin and gastrointestinal (GI) tract
Clinical and laboratory features
  • Natural history varies considerably between individuals
  • Patients may be asymptomatic with longstanding eosinophilia (2 episodes over a 4 week interval), without specific etiology (i.e., no allergy, asthma, drug reaction, parasitic infection or connective disease)
    • Absolute eosinophil count can vary from 1.5 to 400 x 109/L
    • Eosinophils account for 10% of white blood cells (WBCs)
  • Can have leukocytosis (20 - 30 x 109/L), anemia, thrombocytopenia or thrombocytosis
  • Symptoms: weight loss, night sweats, fever (12%), fatigue (26%), myalgias or angioedema (14%), cough (24%), dyspnea (16%), rhinitis (10%), pruritus and diarrhea
  • Skin manifestations are the most common, followed by lung (44%) and GI tract (38%)
  • Cardiovascular
    • Endomyocarditis, endomyocardial fibrosis followed by restrictive cardiomegaly
    • Hypertension, atherosclerosis and heart failure
    • Scarring of mitral and tricuspid valves leading to regurgitation, thrombi formation and embolization in end organs
  • Peripheral neuropathy, CNS dysfunction and rheumatologic findings (Am J Hematol 2012;87:643)
Diagnosis
  • World Health Organization (WHO) classification (5th edition)
    • Essential
      • Persistent eosinophilia (≥ 1.5 x 109/L) on at least 2 occasions over a minimum of 4 weeks in which reactive and genetically defined causes have been excluded
      • Evidence of clonality (see note 2)
      • Abnormal bone marrow morphology
      • Does not meet WHO diagnostic criteria for other myeloid or lymphoid neoplasms (see note 3)
        • Blasts of < 20% in peripheral blood and bone marrow cells and does not have the following cytogenetic aberrations
          • Tyrosine kinase gene fusions including PDGFRA, PDGFRB or FGFR1 rearrangements
          • FLT3 rearrangements (Leukemia 2022;36:1703)
          • ETV6::ABL1 fusion (Leukemia 2022;36:1703)
          • JAK2 rearrangements: t(8;9)(p22;p24.1) / PCM1::JAK2, t(9;12)(p24.1;p13.2) / ETV6::JAK2 or t(9;22)(p24.1;q11.2) / BCR::JAK2 fusions
          • AML with inv(16)(p13.1;q22) or t(16;16)(p13.1;q22) / CBFB::MYH11; t(15;17)(q22;q11-12) / PML::RARA or t(8;21)(q22;q22.1) / RUNX1::RUNX1T1
          • CML with t(9;22)(q24;q31) / BCR::ABL1
    • Notes
      • Note 1
        • Interval of eosinophilia has been reduced from 6 months (4th edition of WHO) to 4 weeks
        • Both clonality and abnormal bone marrow morphology are required for diagnosis
        • A blast count ≥ 2% in the peripheral blood or ≥ 5% in the bone marrow are no longer considered alternatives to clonality requirement
      • Note 2: possibility of clonal hematopoiesis of indeterminate potential (CHIP) should be considered
        • Certain gene mutations (e.g., DNMT3A, TET2, ASXL1 with variant allele frequency ≥ 2%) can be seen in a subpopulation of elderly patients who do not have hematologic disorders, otherwise known as CHIP
        • Diagnostic interpretation should be done with caution (N Engl J Med 2014;371:2477, Nat Med 2014;20:1472)
        • Mutations in other genes with variant allele frequency (VAF) > 10% in combination with other mutations is a better indicator of clonality
      • Note 3: including myeloproliferative neoplasms (MPN), myelodysplastic syndrome (MDS), MDS / MPN, myeloid / lymphoid neoplasms with eosinophilia (MLN-eo), mastocytosis and acute myeloid leukemia (AML)

  • International Consensus Classification (ICC) criteria; requires all 6 criteria
    • Hypereosinophilia (≥ 1.5 x 109/L) with eosinophils comprising ≥ 10% of white blood cells
    • Does not meet criteria for AML; blasts < 20% in the peripheral blood and bone marrow, excluding AML with recurrent genetic abnormalities
      • AML with recurrent genetic aberrations with < 20% is also excluded
    • No tyrosine kinase fusion genes including BCR::ABL1, other ABL, PDGFRA, PDGFRB, FGFR1, JAK2 or FLT3 fusions
    • Does not meet criteria for MPN, chronic myelomonocytic leukemia (CMML) or systemic mastocytosis (SM)
      • Of note, although eosinophilia can be seen with SM, true CEL, NOS may also occur as SM with an associated myeloid neoplasm (Blood 2022;140:1200)
    • Marrow shows increased cellularity with dysplastic megakaryocytes with or without dysplasia in other cell lineages; often with significant myelofibrosis, associated eosinophilic infiltrate or increased blasts ≥ 5% in bone marrow or ≥ 2% in peripheral blood
    • Clonal cytogenetic abnormality or somatic mutation identified
    • If no clonality / somatic mutation demonstrated or no increased blasts, persistent eosinophilia may suffice provided the bone marrow findings are supportive and other causes of eosinophilia have been ruled out
Prognostic factors
  • WHO classification (5th edition)
    • Variable survival; median survival is 22.2 months secondary to infection, bleeding and age related comorbidity
    • Transformation to acute leukemia can occur
    • Response to imatinib therapy is uncommon except for some CEL with PDGFRA mutations (Am J Hematol 2012;87:643, Blood 2011;117:2935)
    • STAT5B p. N642H with or without SF3B1 mutations demonstrated better overall survival (Leukemia 2019;33:415)
    • 4 cases showed somatic activating KIT M541L mutation that was responsive to low dose imatinib therapy (Oncotarget 2014;5:4665)
    • Unfavorable prognostic findings include marked splenomegaly, bone marrow fibrosis, megakaryocytic atypia and thrombocytopenia (Am J Hematol 2020;95:E172)
Case reports
  • 52 year old man with t(5;12)(q31;p13) / ETV6::ACSL6 gene fusion, a novel variant of myeloid proliferative neoplasm with eosinophilia (Hum Pathol (N Y) 2016;5:6)
  • 68 year old woman with liver infiltration resembling Budd-Chiari syndrome (Rinsho Ketsueki 2007;48:505)
  • 72 year old man with autoimmune hemolytic anemia and erythrophagocytosis by eosinophils (Am J Hematol 2006;81:458)
Microscopic (histologic) description
  • Bone marrow
    • Hypercellular due to eosinophilic proliferation; however, maturation is orderly without disproportionate increase in myeloblasts
    • Charcot-Leyden crystals often present
    • Usually normal erythropoiesis and megakaryocytopoiesis; however, abnormal dysplastic megakaryocytes can be seen with or without dysplasia in other lineage (Blood 2022;140:1200)
    • Increased myeloblasts (commonly 5 - 19% in bone marrow)
    • ~33% of cases show myelofibrosis; often significant fibrosis associated with eosinophilic infiltrate (Blood 2022;140:1200)
  • Tissue
    • Eosinophilic infiltration or microabscesses
    • Charcot-Leyden crystals
    • Fibrosis (caused by the degranulation and release of eosinophilic basic and cationic proteins)
Microscopic (histologic) images

Contributed by Lynh Nguyen, M.D. and AFIP

Numerous eosinophilic precursors

Myeloid preponderance

Marked hypercellularity

Granulocytic hyperplasia


Reticulin fibrosis

Markedly fibrotic marrow

Markedly hypercellular

Peripheral smear description
  • Peripheral blood
    • Striking eosinophilia (≥ 1.5 x 109/L) mainly composed of mature eosinophils and occasional immature eosinophilic precursors
      • Circulating blasts can be seen but comprise < 20%
    • Spectrum of nonspecific eosinophil abnormalities: sparse granulation, cytoplasmic vacuolation, nuclear hyper / hyposegmentation or increased size
    • Often accompanied by neutrophilia, some with mild monocytosis (> 1 x 109/L) and a few with mild basophilia
Peripheral smear images

Contributed by Lynh Nguyen, M.D.
Moderately degranulated eosinophils

Moderately degranulated eosinophils

Patient with longstanding hypereosinophilia

Patient with longstanding hypereosinophilia

Circulating abnormal eosinophils

Circulating abnormal eosinophils

Markedly increased eosinophils

Markedly increased eosinophils

Immunophenotype
  • No specific immunophenotype has been associated with CEL, NOS; however, immunophenotyping is important to exclude T lymphocyte driven eosinophilia or acute leukemia
Electron microscopy images

AFIP images

Abnormal eosinophil with decreased granules

Molecular / cytogenetics description
  • No specific cytogenetic or molecular genetic abnormality identified
  • Must exclude neoplasms with rearrangements of BCR::ABL1, PDGFRA, PDGFRB, FGFR1, FLT3 or PCM1::JAK2, ETV6::JAK2 or BCR::JAK2 fusions
  • Presence of recurrent translocations and karyotypic abnormalities that are seen in myeloid disorders [-8, -7, i(17q)] also support the diagnosis (Br J Haematol 1996;95:2, Br J Haematol 1986;62:659)
  • Next generation sequencing (NGS) is helpful in establishing clonality
  • Common mutations in ASXL1, TET2 and EZH2; occasional JAK2 mutation (Blood 2005;106:2162, Mod Pathol 2016;29:854)
Sample pathology report
  • Peripheral blood, peripheral blood smear:
    • Mild normocytic anemia
    • Leukocytosis with marked eosinophilia
    • No circulating blasts identified
  • Bone marrow, aspirate, clot section and biopsy:
    • Markedly hypercellular marrow with myeloid preponderance, marked eosinophilic hyperplasia and scattered blasts (2%) (see comment)
    • Moderate myelofibrosis (MF2)
    • Comment: Examination of the peripheral blood and bone marrow aspirate smears show a marked increase in eosinophils with abnormal morphology including sparse granulation and nuclear hyper / hyposegmentation. There is no peripheral monocytosis or basophilia. Manual differential counts performed on the submitted bone marrow aspirate demonstrate approximately 2% myeloblasts. Megakaryocytes are present and slightly increased in number. Erythroid precursors show essentially normoblastic maturation. The M:E ratio is increased, estimated 5:1. The bone marrow core biopsy demonstrates normal bony trabeculae surrounded marrow space with overt myeloid predominance, increased eosinophils and their precursors. Occasional blasts are identified. Megakaryocytes are evenly distributed without syncytial clustering. Erythroid precursors are present and relatively low in proportion. Occasional small loose lymphoid aggregates composed of small mature forms are focally noted. No granulomata are evident. Flow cytometry performed on the bone marrow aspirate with adequate cells showed no immunophenotypic evidence of clonal B cell or aberrant T cell population. Rare CD34 positive blasts (1 - 2%) are identified.
    • Karyotyping showed normal karyotype, 46,XX[20]. Fluorescence in situ hybridization is negative for BCR::ABL1, PDGFRA, PDGFRB or FGFR1 rearrangements. Next generation sequencing identified multiple mutations involving ASXL1, TET2, EZH2, SETBP1, CBL and NOTCH genes. According to WHO classification, the patient showed typical clinical presentation, along with morphologic and molecular findings supporting a diagnosis of chronic eosinophilic leukemia. Clinically, all other neoplastic and reactive causes of eosinophilia have been excluded.
Differential diagnosis
Board review style question #1
Which of the following is useful in definitively diagnosing the lymphocytic variant of hypereosinophilic syndrome?

  1. Abnormal T cell population (e.g., CD3- / CD4+)
  2. Elevated IL5 level
  3. Eosinophilia ≥ 1.5 k/L
  4. Eosinophilic tissue infiltrate
  5. Skin rush and pruritus
Board review style answer #1
A. Abnormal T cell population (e.g., CD3- / CD4+). Lymphocytic variant of hypereosinophilic syndrome (L-HES) is characterized by abnormal cytokine release by clonal T cells. CD3- / CD4+ T cells are often found in circulating blood but CD3+ / CD4+ / CD7- and CD3+ / CD4- / CD8- T cells have also been described. TCRγδ rearrangements are frequently identified in these patients; however, TCRαβ+ L-HES has also been reported. Answers B - D are incorrect because elevated IL5 levels, eosinophilia and eosinophilic infiltration can be seen in a number of reactive, neoplastic and iatrogenic conditions and are not diagnostic or specific for L-HES. Answer E is incorrect because pruritus and skin rash can occur as a result of eosinophilia but this finding is not specific.

Comment Here

Reference: Chronic eosinophilic leukemia
Board review style question #2

Which of following morphologic or molecular genetic findings is frequently identified in chronic eosinophilic leukemia, not otherwise specified?

  1. Dysplastic noneosinophilic granulocytes
  2. ETV6::JAK2 fusion
  3. Hypercellular marrow with markedly increased eosinophils and left shifted myelopoiesis
  4. KIT D816V gene mutation
  5. Marked monocytosis
Board review style answer #2
C. Hypercellular marrow with markedly increased eosinophils and left shifted myelopoiesis. Abnormal bone marrow findings are 1 of 3 key diagnostic features of CEL (see above). Hypercellular marrow along with eosinophilic proliferation and left shifted myelopoiesis, though nonspecific, are the most common features of CEL. Answers A and E are incorrect. Dysplastic megakaryocytes with or without dysplasia in other cell lineages can be seen in CEL. Spectrum of nonspecific eosinophil abnormalities can be seen (sparse granulation, cytoplasmic vacuolation, nuclear hyper / hyposegmentation or increased size). CEL often shows neutrophilia, some with mild monocytosis (> 1 x 109/L) and a few with mild basophilia. Answers B and D are incorrect. ETV6::JAK2 fusion can seen in myeloid / lymphoid neoplasms with eosinophilia (MLN-eo) and tyrosine kinase (TK) gene fusions while KIT D816V gene mutation is frequently seen in systemic mastocytosis.

Comment Here

Reference: Chronic eosinophilic leukemia

Chronic myeloid leukemia (CML), BCR::ABL1 positive
Definition / general
Essential features
  • Occurs in 3 different phases (Am J Hematol 2018;93:442)
    • Chronic phase (CP)
    • Accelerated phase (AP)
    • Blast phase (BP)
  • Natural progress: most cases progress from chronic phase to blast phase in 3 - 5 years
  • Relatively rare in pediatric patients
  • Children and young adults tend to have a more aggressive clinical presentation than older adults, more frequent in accelerated or blast phase (Blood 2012;119:1821)
Terminology
  • Chronic myelogenous leukemia, BCR-ABL1 positive
  • Chronic granulocytic leukemia, BCR-ABL1 positive
  • Chronic myelogenous leukemia, Philadelphia chromosome positive (Ph+)
  • Chronic myelogenous leukemia, t(9;22)(q34;q11)
  • Chronic granulocytic leukemia, Philadelphia chromosome positive (Ph+)
  • Chronic granulocytic leukemia, (9;22)(q34;q11)
  • Chronic granulocytic leukemia, BCR-ABL1
ICD coding
  • ICD-10: C92.1 - Chronic myeloid leukemia, BCR-ABL1 positive
Epidemiology
  • Incidence: 10 - 20 cases per million/year
  • M:F = 1.2 ~ 1.7:1 (Ann Hematol 2015;94:S241)
  • Median age: about 65 years; pediatric cases rare
Sites
  • Chronic phase: blood, bone marrow, spleen and liver
  • Blast phase: spleen, liver, lymph nodes, skin and soft tissue are the most common extramedullary sites of involvement
Pathophysiology
  • Reciprocal translocation t(9;22)(q34;q11) leads to the formation of the chimeric fusion gene BCR-ABL1, which is responsible for production of the oncoprotein BCR-ABL
  • BCR-ABL protein has uncontrolled tyrosine kinase activity, making abnormal pluripotent hematopoietic progenitor cell initiate excessive production of all myeloid lineage cells (Am J Hematol 2020;95:691)
Etiology
Clinical features
  • Most patients are diagnosed in the chronic phase
  • Almost 50% of patients diagnosed in the United States are asymptomatic and diagnosed during a routine examination (Mayo Clin Proc 2015;90:1440)
  • Insidious onset with leukocytosis, anemia, fatigue, weight loss, night sweats
  • 50% of patients have splenomegaly with dragging sensation in left upper quadrant
  • Rare patients present with marked thrombocytosis without leukocytosis, mimicking essential thrombocythemia
  • 5% of patients present with accelerated or blast phase
  • Accelerated or blast phase: fever, bone pain, bleeding
Diagnosis
  • Chronic phase (CP) (Mayo Clin Proc 2015;90:1440):
    • Leukocytosis (usually 12 - 1,000 x 10⁹/L, median: ~ 80 x 10⁹/L)
    • < 2% blasts in blood
    • < 5% blasts in bone marrow
    • BCR-ABL1 positive by cytogenetics or molecular study
    • Does not meet any of the following diagnostic criteria for accelerated or blast phase
  • Accelerated phase (AP) is defined by the presence of ≥ 1 of the following criteria:
    • 10 - 19% blasts in blood or marrow (Drug Des Devel Ther 2019;13:825)
    • Persistent or increasing WBC > 10 x 10⁹/L, unresponsive to therapy
    • Persistent platelet count > 1,000 x 10⁹/L, unresponsive to therapy
    • Persistent platelet count < 100 x 10⁹/L, unrelated to therapy
    • ≥ 20% basophils in blood
    • Persistent or increasing splenomegaly, unresponsive to therapy
    • Additional clonal chromosomal abnormalities in Philadelphia chromosome positive cells at diagnosis, including so called major route abnormalities (a second Ph chromosome, trisomy 8, isochromosome 17q, trisomy 19), complex karyotype and abnormalities of 3q26.2
    • Large clusters or sheets of small, abnormal megakaryocytes associated with marked reticulin or collagen fibrosis may be considered presumptive evidence of accelerated phase, although these findings are usually associated with one or more of the criteria listed above
  • Blast phase (BP):
    • ≥ 20% blasts in marrow or blood or the presence of an extramedullary proliferation of blasts (Front Oncol 2019;9:1132)
    • Approximately 70% have myeloid blasts (may include neutrophilic, monocytic, megakaryocytic, basophilic, eosinophilic or erythroid blasts), approximately 30% have lymphoid blasts (usually B lymphoblasts but T and NK blasts have been reported) (J Clin Invest 2010;120:2254)
Laboratory
  • Chronic phase:
    • Leukocytosis with predominantly neutrophils and myelocytes
    • Basophilia and eosinophilia
    • t(9;22)(q34;q11.2) by FISH, conventional cytogenetics or PCR
    • Alkaline phosphatase decreased
  • Reference: Mayo Clin Proc 2015;90:1440
Radiology description
  • Splenomegaly
Radiology images

Contributed by Yang Shi, M.D., Ph.D.

Splenomegaly

Prognostic factors
  • Most important prognostic indicator is response to tyrosine kinase inhibitors at the hematological, cytogenetic and molecular level (Cancer 2017;123:4391)
  • 5 year progression free and overall survival rates: 80 - 95%
  • Long term overall survival approaching that of aged matched controls
Case reports
  • 11 year old boy with chronic myeloid leukemia possessing the unique combination of T lymphoblastic transformation and a subclone harboring inv(3)(q21q26.2) at diagnosis (Biomark Res 2016;4:14)
  • 39 year old asymptomatic man with hematologically typical chronic phase chronic myeloid leukemia showed an unusual and complex translocation involving chromosomes 9, 11 and 22 (Arch Pathol Lab Med 2001;125:437)
  • 52 year old woman with occurrence of chronic myeloid leukemia followed by CLL (Int J Hematol Oncol Stem Cell Res 2014;8:49)
  • 68 year old woman with p190 BCR-ABL+ chronic myeloid leukemia and persistent moderate monocytosis (J Hematol 2018;7:120)
  • 68 year old man with JAK2 V617F positive PV, evolving to BCR-ABL positive chronic myeloid leukemia (AJHO 2017;13:24)
Treatment
  • Chronic phase: tyrosine kinase inhibitor (TKI), acting by competitive inhibition at the ATP binding site of BCR-ABL1 oncoprotein, resulting in inhibition of phosphorylation of proteins involved in cell signal transduction (Ulster Med J 2007;76:8)
  • Tyrosine kinase inhibitor agents help achieve long term control of chronic myeloid leukemia in majority of patients (Leukemia 2020;34:966)
  • Imatinib is the prototype drug and first approved tyrosine kinase inhibitor (N Engl J Med 2017;376:917)
  • Before tyrosine kinase inhibitor era, interferon alpha and hydroxyurea were used to treat chronic myeloid leukemia (Leukemia 2013;27:803)
  • Chronic myeloid leukemia resistant to imatinib can be treated by second generation tyrosine kinase inhibitor agents such as nilotinib, dasatinib and bosutinib (Clin Lymphoma Myeloma Leuk 2015;15:323)
  • Complete hematological response (Blood Rev 2011;25:139):
    • WBC < 10 x 10⁹/L
    • Platelet count < 450 x 10⁹/L
    • No immature granulocytes in the differential
    • Spleen not palpable
  • Treatment failure occurs most commonly due to compliance issues followed by BCR-ABL mutations altering drug binding
  • Hematopoietic stem cell transplant is reserved for those rare patients who (Semin Hematol 2010;47:354):
    • Present with blast phase
    • Are resistant / intolerant to all tyrosine kinase inhibitors
    • Progress to accelerated or blast phase while on tyrosine kinase inhibitor therapy
Microscopic (histologic) description
  • Bone marrow (chronic phase):
    • Hypercellular marrow with absolute myeloid hyperplasia and myelocyte bulge
    • Myeloid to erythroid ratio ≥ 10:1
    • Layer of immature granulocytes (5 - 10 cells in thickness) around the bone trabeculae (2 - 3 cells layer in normal marrow)
    • No significant dysplasia
    • Increased basophils and eosinophils (Intern Med 2011;50:501)
    • Blasts < 5%
    • Normal / slightly decreased megakaryocytes; 40 - 50% show moderate to marked megakaryocytes proliferation
    • Megakaryocytes often smaller and hyposegmented with dwarf morphology
    • Pseudo-Gaucher cells are common (Leukemia 1998;12:233)
  • Bone marrow (accelerated phase):
    • 10 - 19% blasts (Am Soc Clin Oncol Educ Book 2015;e381)
    • Large clusters or sheets of small, abnormal megakaryocytes associated with marked reticulin or collagen fibrosis may be considered presumptive evidence of accelerated phase
  • Bone marrow (blast phase):
    • ≥ 20% blasts in marrow or blood or the presence of an extramedullary proliferation of blasts (Leuk Lymphoma 2014;55:1451)
    • Approximately 70% have myeloid blasts (may include neutrophilic, monocytic, megakaryocytic, basophilic, eosinophilic or erythroid blasts) and approximately 30% have lymphoid blasts (usually B lymphoblasts but T and NK blasts have been reported) (J Clin Invest 2010;120:2254)
Microscopic (histologic) images

Contributed by Yang Shi, M.D., Ph.D.

Chronic phase marrow

Chronic phase marrow smear

Accelerated phase marrow

Blast phase B-ALL

Bone marrow biopsy

Bone marrow aspirate smear

Peripheral smear description
  • Chronic phase:
    • Absolute leukocytosis (12 - 1,000 x 10⁹/L, median: ~80 x 10⁹/L)
    • Left shifted with 2 peaks: myelocytes and segmented neutrophils
    • Blasts < 2%
    • No significant granulocytic dysplasia
    • Absolute monocytosis may be present but are < 3%
    • Absolute basophilia (100%) and eosinophilia (80 - 90%)
    • Platelet counts are normal or thrombocytosis ≥ 1,000 x 10⁹/L
  • Reference: Mayo Clin Proc 2015;90:1440
Peripheral smear images

Contributed by Yang Shi, M.D., Ph.D.

Chronic phase blood smear

Positive stains
Flow cytometry description
  • Peripheral blood (chronic phase) (Mayo Clin Proc 2015;90:1440):
    • Left shifted leukocytosis
    • No significant increase in blasts: < 2%
    • Relative lymphopenia
    • Myeloid lineage shows 2 peaks: correspond to myelocytes and segmented neutrophils respectively
  • Accelerated phase: 10 - 19% blasts in blood or marrow
  • Blast phase: ≥ 20% blasts in marrow or blood or the presence of an extramedullary proliferation of blasts (Front Oncol 2019;9:1132)
  • Bona fide lymphoblasts in the peripheral blood or bone marrow (even if < 10%) should prompt immediate investigation for lymphoblastic transformation
Flow cytometry images

Contributed by Yang Shi, M.D., Ph.D.

Peripheral blood chronic phase

Molecular / cytogenetics description
  • 90 - 95% of chronic myeloid leukemia is characterized by t(9;22)(q34.1;q11.2) reciprocal translocation that results in the Ph chromosome, der(22)t(9;22) (Haematologica 2016;101:541)
  • Translocation fuses the BCR gene on chromosome 22 with regions of ABL1 on chromosome 9 and generates the BCR-ABL1 fusion gene that characterizes leukemic cells (Am J Hematol 2018;93:442)
  • 5 - 10% cases have either variant translocations that involve a third or even a fourth chromosome in addition to chromosomes 9 and 22 or a cryptic translocation of 9q34.1 and 22q11.2 that cannot be identified by routine cytogenetic analysis (Oncotarget 2017;8:29906)
  • In these cases, the presence of BCR-ABL1 fusion gene can be detected by FISH or RT-PCR
  • In most chronic myeloid leukemia cases the breakpoint cluster region is almost always in the major BCR (M-BCR), spanning exons 12 - 16 and form an abnormal fusion protein p210
  • Rarely, the breakpoint in the BCR gene occurs in the p-BCR region, spanning exons 17 - 20 and form a larger fusion protein p230 (Haematologica 2001;86:320)
  • Small amounts of P190 transcript with breaks in the minor breakpoint region, m-BCR (exons 1 - 2) can be detected in > 90% of patients with chronic myeloid leukemia due to alternative splicing of the BCR gene
  • Very rare chronic myeloid leukemia cases are associated with p190 only and frequently show monocytosis, mimicking chronic myelomonocytic leukemia (J Hematol 2018;7:120)
Molecular / cytogenetics images

Contributed by K.H. Ramesh, Ph.D.

Conventional cytogenetics chronic myeloid leukemia

BCR-ABL+ chronic myeloid leukemia FISH

Videos

Brief introduction of chronic myeloid leukemia

Chronic myeloid leukemia: pathogenesis, symptoms and treatment

Blast crisis in chronic myeloid leukemia


Chronic myeloid leukemia from Khan Academy

Treatment of chronic myeloid leukemia

Peripheral blood smear of chronic myeloid leukemia

Sample pathology report
  • Bone marrow, left posterior iliac crest, core biopsy, clot section, aspirate smears and touch imprint:
    • Hypercellular (90%) bone marrow with absolute myeloid hyperplasia and < 5% blasts (see comment)
    • Comment: Flow cytometric analysis demonstrates 2.5% blasts. There are < 5% blasts by CD34 and CD117 immunohistochemical stains on the core biopsy. A CBC obtained on 03/29/2020 revealed leukocytosis (30.2 x 10⁹/L) and thrombocytosis (625 x 10⁹/L). FISH analysis from peripheral blood (03/29/2020): BCR-ABL1 dual color, dual fusion translocation DNA probe revealed a fusion of 9q34.1 (ABL1) and 22q11.2 (BCR) regions in 62 out of 100 cells (62%) interphase nuclei.
    • Peripheral smear: Manual review of the peripheral blood shows left shifted leukocytosis with two bulges: myelocytes and segmented neutrophils. No definite morphologic dysplasia is seen. Rare (~1%) circulating blasts are noted. RBCs: mild normochromic, normocytic anemia. Platelets: moderate thrombocytosis with rare large platelets.
    • Bone marrow biopsy: Quality: adequate. Cellularity: > 90%. Hematopoiesis: trilineage maturation with absolute myeloid hyperplasia and relative erythroid hypoplasia. Megakaryocytes are mildly to moderately increased. Most megakaryocytes are small and hyposegmented. Infiltrate: no apparent increase in blasts. Special stains: reticulin: mild diffuse increase in reticulin fibrosis. Trichrome: negative for collagen deposition.
    • Bone marrow clot section: Quality: adequate. Cellularity: > 90% morphologic features are similar to those observed in the core biopsy.
    • Bone marrow aspirate: Quality: adequate. Granulocytes: markedly increased with significant expansion of myelocytes. No definite morphologic dysplasia. Erythrocytes: relative erythroid hypoplasia with progressive maturation and no definite dysplasia. Megakaryocytes: mildly to moderately increased; most of them are small and hyposegmented. No sheets of megakaryocytes are present. Blasts: < 5% of nucleated cells. Iron: storage iron is adequate (2+ on a scale of 0 - 4). No ring sideroblasts are present.
Differential diagnosis
  • Leukemoid reaction (Am J Hematol 2018;93:442):
    • Underlying infection / inflammation
    • Myelocyte bulge is absent
    • Basophilia is absent
    • Toxic granulation and cytoplasmic vacuoles might be present
    • Alkaline phosphatase increased
  • Chronic neutrophilic leukemia:
    • Peripheral blood:
      • Segmented neutrophils + band neutrophils ≥ 80%
      • Neutrophil precursors (promyelocytes, myelocytes and metamyelocytes) < 10%
    • Neutrophils might show toxic granulation and Dohle bodies
    • No dysplasia
    • Strong association with CSF3R mutation (Blood 2017;129:715)
  • Atypical chronic myeloid leukemia, BCR-ABL1 negative (Hematology Am Soc Hematol Educ Program 2015;2015:264):
    • Dysplastic neutrophils and their precursors
    • Multilineage dysplasia is common
    • No or minimal absolute basophilia, basophils < 2% of WBC
    • No or minimal monocytosis, monocytes < 10% of WBC
    • SETBP1 and ETNK1 mutations are common
  • Chronic myelomonocytic leukemia (Am J Hematol 2018;93:824):
    • Monocytes ≥ 10% of WBC
    • Dysplasia involving ≥ 1 myeloid lineages
    • Common mutations: TET2, SRSF2, ASXL1, SETBP1
  • Chronic eosinophilic leukemia (Blood Cancer J 2018;8:15):
    • Eosinophil count ≥ 1.5 x 10⁹/L
    • Blasts < 20%
    • A clonal cytogenetics or molecular genetic abnormality is present or blasts ≥ 2% in blood or blasts ≥ 5% in marrow
    • Absence of BCR-ABL1 translocation
    • Absence of rearrangement of PDGFRA, PDGFRB or FGFR1 and no PCM1-JAK2, ETV6-JAK2 or BCR-JAK2 fusion
    • Other myeloproliferative neoplasms such as polycythemia vera, essential thrombocythemia, primary myelofibrosis, chronic neutrophilic leukemia, chronic myelomonocytic leukemia and BCR-ABL1 negative atypical chronic myeloid leukemia are excluded
    • AML with inv(16)(p13.1q22), t(16;16)(p13.1;q22), t(8;21)(q22;q22.1) are excluded
  • Essential thrombocythemia (Mayo Clin Proc 2015;90:1440):
    • Platelet ≥ 450 x 10⁹/L
    • Bone marrow biopsy shows significant megakaryocytic proliferation
    • Megakaryocytes are large, hyperlobated
    • No significant increase or left shifted granulopoiesis or erythropoiesis
    • No basophilia
    • JAK2, CALR or MPL mutation (+)
Board review style question #1
Which of the following is a criterion for the diagnosis of accelerated phase of chronic myeloid leukemia according to WHO 2017 criteria?

  1. 5 - 10% blasts in bone marrow or peripheral blood
  2. Persistent platelet count < 100,000 related to therapy
  3. ≥ 10% basophils in blood
  4. Persistent or increasing splenomegaly
  5. Additional clonal chromosomal abnormalities in Philadelphia chromosome positive cells at diagnosis, including so called major route abnormalities (a second Ph chromosome, trisomy 8, isochromosome 17q, trisomy 19), complex karyotype and abnormalities of 3q26.2
Board review style answer #1
E. Additional clonal chromosomal abnormalities in Philadelphia chromosome positive cells at diagnosis, including so called major route abnormalities (a second Ph chromosome, trisomy 8, isochromosome 17q, trisomy 19), complex karyotype and abnormalities of 3q26.2

Comment Here

Reference: Chronic myeloid leukemia (CML)
Board review style question #2

A 15 year old boy with no significant medical history presented with worsening knee pain for 5 days. CBC: WBC: 229.4 x 10⁹/L, hemoglobin 12.1 g/dl, platelet 161 x 10⁹/L. Bone marrow biopsy showed > 90% cellular marrow with absolute myeloid hyperplasia without significant increase in blasts. FISH study showed BCR-ABL1 translocation. Which of the following statements is right?

  1. CML is very common in pediatric patients
  2. Children and young adults with CML tend to have a less aggressive clinical presentation than older adults
  3. Children and young adults with CML present with accelerated or blast phase more frequently
  4. Children and young adults tend to have higher complete cytogenetic response and molecular response rates compared with older adults
Board review style answer #2
C. Children and young adults with CML present with accelerated or blast phase more frequently

Comment Here

Reference: Chronic myeloid leukemia (CML)
Board review style question #3
Which of the following statements about chronic myeloid leukemia is correct?

  1. Monocytosis rules out the possibility of chronic myeloid leukemia
  2. Thrombocytosis rules out the possibility of chronic myeloid leukemia
  3. Eosinophilia with basophilia is common in chronic myeloid leukemia
  4. Toxic granulation and Dohle bodies are common in chronic myeloid leukemia
  5. Alkaline phosphate is increased in chronic myeloid leukemia
Board review style answer #3
C. Eosinophilia with basophilia is common in chronic myeloid leukemia

Comment Here

Reference: Chronic myeloid leukemia (CML)

Chronic myelomonocytic leukemia
Definition / general
  • Clonal myeloid neoplasm defined by persistent relative (> 10%) and absolute monocytosis (≥ 0.5 x 109/L) in peripheral blood
  • Features of both a myeloproliferative neoplasm and a myelodysplastic syndrome
  • Inherent risk for transformation to acute myeloid leukemia
Essential features
  • No evidence of BCR::ABL1, PDGFRA, PDGFRB, FGFR1 or JAK2 rearrangement
  • Persistent relative (> 10%) and absolute monocytosis (≥ 0.5 x 109/L) in peripheral blood
  • < 20% blasts (including myeloblasts, monoblasts and promonocytes) in peripheral blood and bone marrow
  • Morphologic dysplasia involving at least 1 myeloid lineage
  • Acquired clonal cytogenetic or molecular abnormality
Terminology
  • Chronic myelomonocytic leukemia type I: ≤ 5% blasts in the blood and ≤ 10% blasts in the bone marrow and no Auer rods
  • Chronic myelomonocytic leukemia type II: 6 - 19% blasts in the blood, 10 - 19% blasts in the bone marrow or Auer rods are present
  • Myelodysplastic CMML (MD-CMML): WBC count < 13 × 109/L
  • Myeloproliferative CMML (MP-CMML): WBC count ≥ 13 × 109/L
ICD coding
  • ICD-10: C93.1 - chronic myelomonocytic leukemia
Epidemiology
  • Annual incidence rate of ~0.4 cases per 100,000 persons with the highest incidence in patients older than 80 (Leuk Res 2015;39:177, Blood 1984;63:634)
  • M:F = ~2.5:1
  • Reported median age at diagnosis ranges from 65 to 75 years
Sites
  • Blood and bone marrow are invariably involved
  • Spleen, liver, skin and lymph nodes are the most common extramedullary sites of involvement
Pathophysiology
  • Similar to other myelodysplastic syndromes, chronic myelomonocytic leukemia (CMML) often results in ineffective hematopoiesis and cytopenia
Etiology
  • Acquired cytogenetic and molecular changes involving hematopoietic stem cells
Clinical features
Diagnosis
  • Persistent absolute (≥ 0.5 x 109/L) and relative (> 10%) peripheral blood monocytosis
  • Does not meet WHO criteria for chronic myeloid leukemia, primary myelofibrosis, polycythemia vera or essential thrombocythemia
  • No evidence of BCR::ABL1, PDGFRA, PDGFRB, FGFR1 or JAK2 rearrangement
  • < 20% blasts (including myeloblasts, monoblasts and promonocytes) in peripheral blood and bone marrow
  • Morphologic dysplasia involving at least 1 myeloid lineage
    • Myelodysplasia may be absent if all of the above are present and there is evidence of an acquired clonal cytogenetic or molecular abnormality or if monocytosis has persisted ≥ 3 months and all reactive causes of monocytosis have been excluded
  • Acquired clonal cytogenetic or molecular abnormality
  • Reference: Leukemia 2022;36:1703
Laboratory
  • Leukocytosis or leukopenia with relative and absolute monocytosis and neutrophilia or neutropenia
  • Anemia
  • Thrombocytopenia or rarely thrombocytosis
  • Increased serum lactate dehydrogenase (Am J Hematol 2016;91:631)
Radiology description
Radiology images

Contributed by Sanam Loghavi, M.D.

FDG PET / CT

Prognostic factors

Chronic myelomonocytic leukemia prognostic models
MD Anderson Prognostic System (MDAPS)
  • Hemoglobin < 12 gm/dL
  • Circulating immature myeloid cells
  • Absolute lymphocyte count > 2.5 x 109/L
  • ≥ 10% bone marrow blasts
Global MD Anderson Prognostic System (Global MDAPS)
  • Older age
  • Poor performance status
  • Thrombocytopenia
  • Anemia
  • Increased bone marrow blasts
  • Leukocytosis (> 20 x 109/L)
  • -7/del7q, complex karyotype
  • Prior transfusion
CMML Specific Prognostic Scoring System (CPSS)
  • WHO subgroups (CMML 1 and CMML 2)
  • MD-CMML versus MP-CMML
  • RBC transfusion dependency or hemoglobin level
  • Karyotype
    • Diploid karyotype or -Y: low risk
    • +8 and -7/del7q and complex karyotype: high risk
    • All other karyotypes: intermediate risk
CMML Specific Prognostic Scoring System-Mol (CPSS-Mol)
  • WBC count
  • Transfusion dependency
  • Bone marrow blast count
  • CPSS cytogenetic risk groups
  • ASXL1, NRAS, SETP1 and RUNX1 status
Groupe Francophone des Myelodysplasies
  • ASXL1 mutation
  • Age > 65 years
  • WBC > 15 x 109/L
  • Thrombocytopenia < 100 x 109/L
  • Hemoglobin < 10 gm/dL in women and < 11 gm/dL in men
Mayo Prognostic Model
  • Hemoglobin < 10 gm/dL
  • Thrombocytopenia < 100 x 109/L
  • Absolute monocyte count > 10 x 109/L
  • Circulating immature myeloid cells
Mayo Molecular Model
  • ASXL1 mutation
  • Circulating immature myeloid cells
  • Thrombocytopenia < 100 x 109/L
  • Hemoglobin < 10 gm/dL
  • Absolute monocyte count > 10 x 109/L
Case reports
Treatment
  • Allogeneic hematopoietic stem cell transplantation remains the only curative treatment option
  • Other therapeutic means are also used to alleviate the symptoms related to the associated cytopenia and cytoses; these include the use of erythropoiesis stimulating agents, cytoreductive chemotherapy and hypomethylating agents (Blood 2017;130:126)
  • Wait and watch approach may be used in patients with low risk and clinically stable disease
  • Ruxolitinib may be considered in order to abrogate the effects of aberrant RAS signaling pathway activation (Clin Cancer Res 2016;22:3707)
Clinical images

Contributed by Michael Tetzlaff, M.D., Ph.D.

Hemorrhagic papules

Microscopic (histologic) description
  • Bone marrow is often hypercellular and shows a myelomonocytic proliferation resulting in increased myeloid:erythroid (M:E) ratio but an increase in erythroid precursors may also be seen in some cases
  • Monocytic proliferation is always present but may be difficult to appreciate on routine H&E stained sections
  • Dysgranulopoiesis is almost always present
    • Dyserythropoiesis is also commonly observed but is less striking
    • Dysmegakaryopoiesis is seen in up to 80% of cases
  • Mild to moderate reticulin fibrosis may be seen
  • Neoplastic proliferation of mature plasmacytoid dendritic cells (CD123+) is seen in a subset of cases (Am J Hematol 2010;85:893)
  • Splenic red pulp is usually infiltrated by chronic myelomonocytic leukemia
Microscopic (histologic) images

Contributed by Sanam Loghavi, M.D.

Deep dermis

Bone marrow

Myelomonocytic hyperplasia in bone marrow

Monocytosis and morphologic dysplasia

Morphologic dysplasia

Cytology description
  • Monocytes may be comprised of mature or immature forms (monoblasts, promonocytes)
  • Immature forms in this context are regarded as blast equivalents
  • There is typically background dysplasia in one or more of the myeloid lineages including myeloid, erythroid or megakaryocytic lineages
Peripheral smear description
  • Relative and absolute monocytosis, variable increased immature monocytes (promonocytes, monoblasts)
  • May show neutrophilia or neutropenia with variable dysplastic neutrophils (hypogranular, hypolobated)
  • Left shifted granulocytes may be seen but usually account for < 10% of leukocytes and blasts are < 20% by definition
  • Variable anemia, with normocytic or macrocytic red blood cells
  • Variable thrombocytopenia
  • Reference: Leukemia 2022;36:1703
Positive stains
Negative stains
Flow cytometry description
  • Flow cytometry can differentiate reactive monocytosis from chronic myelomonocytic leukemia but findings are not specific and may be seen in other myeloid neoplasms (Eur J Haematol 2015;95:168)
  • CD34+ myeloid blasts show antigenic aberrancies (median of 6 aberrancies per case) including increased intensity of CD117, CD123, CD13 and CD34 expression and decreased intensity of CD38 expression as well as altered pattern of CD45 / side scatter and aberrant expression of lineage infidelity markers such as CD2, CD5, CD7, CD19 and CD56, among others
  • Monocytic and granulocytic cells show frequent immunophenotypic aberrancies (96% and 83%, respectively)
  • Granulocytes tend to show altered patterns of maturation markers CD11b / CD13 / CD16
  • Monocytes show decreased expression of CD45 and HLA-DR, altered (increased or decreased) expression of CD64 and CD14 and aberrantly increased expression of CD56 (positive in ≥ 25% of monocytes) and CD2
  • Hematogones are completely absent in ~93% of cases
  • Classical monocyte (CD14+ / CD16-) population is expanded (Blood 2015;125:3618)
  • Expansion of classical monocyte subsets (≥ 94%) has also been shown to be able to discriminate from other myelodysplastic syndrome subtypes with relatively good sensitivity (72%) and specificity (86%)
  • Flow cytometry immunophenotyping can distinguish from other myeloproliferative with monocytosis using a classical monocyte fraction cutoff of 92% with a sensitivity of 93% and specificity of 100% (Blood Cancer J 2017;7:e584)
Molecular / cytogenetics description
  • Clonal cytogenetic abnormalities are seen in 20 - 30%
  • Most frequent chromosomal alterations include +8 (23%), -Y (20%), -7/del 7q (14%), +21 (8%) and del20q (8%) and der(3q) (8%) (Haematologica 2011;96:375)
  • Cytogenetic changes have important prognostic implications and are included in all major risk stratification models (Am J Hematol 2014;89:813)
  • Clones have up to 15 mutations per kilobase of coding DNA
  • Most commonly mutated genes including epigenetic and transcriptional regulation
    • TET2 (60%)
    • ASXL1 (40%)
    • EZH2 (< 5%; more common in myeloproliferative chronic myelomonocytic leukemia), DNMT3A (~5%), IDH1 (< 2%) and IDH2 (~5%), mutations in the spliceosome machinery including SRSF2 (50%), U2AF1 (7%), SF3B1 (5 - 10%), ZRSR2 (3%), PRPF8 (Nat Commun 2016;7:10767)
  • Epigenetic and splicing gene mutations frequently co-occur with mutations involving genes in the cell signaling pathways (~30%) including JAK2 (5%), KRAS (5 - 10%), NRAS (10 - 20%), CBL (13%), BRAF (7%), PTPN11 (< 5%), FLT3 (< 5%), NPM1 (< 3%) (J Clin Oncol 2013;31:2428, Am J Hematol 2014;89:499)
  • Signaling pathway mutations are more frequent in myeloproliferative chronic myelomonocytic leukemia
  • Mutations in the BRAF kinase domain in ~7%
  • Co-occurrence of SRSF2 and TET2 mutations is highly specific (~98%) to myeloid neoplasms with a CMML-like phenotype (Blood 2014;124:1513)
  • Worse survival
  • Favorable outcome
  • RUNX1 mutations
    • Associated with more severe cytopenia and progression to acute myeloid leukemia
    • No impact on survival (Leukemia 2009;23:1426)
  • TP53 mutations
    • Uncommon (~4%, more common in myeloproliferative chronic myelomonocytic leukemia)
    • May be clonal or subclonal
    • Mostly detected at initial diagnosis
    • In contrast to other cases with TP53 mutations, usually have a noncomplex karyotype
    • Prognostic significance is unclear (Leuk Res 2018;70:97)
Sample pathology report
  • Bone marrow, right posterior iliac crest, core biopsy, clot section, aspirate smears and touch imprint:
    • Chronic myelomonocytic leukemia (CMML 1) (see comment)
    • Hypercellular (90%) bone marrow showing myelomonocytic and megakaryocytic hyperplasia, trilineage dysplasia, monocytosis and 5% blasts
    • Comment: Flow cytometric analysis demonstrates changes consistent with myelodysplastic syndrome or related stem cell disorder. CD34+ myeloblasts are ~1.2% (note that hemodilution and other technical factors may limit the precision of blast counts by flow cytometry). They have an altered immunophenotype (CD34+, CD117+, CD13+, CD33+, CD123+, CD38 decreased). Hematogones are absent. Granulocytes show decreased side scatter, in keeping with aberrant hypogranularity. Monocytes are increased (~12%) show increased expression of CD2. No monotypic B cells or immunophenotypically abnormal T cells are detected. Fluorescence in situ hybridization is negative for BCR::ABL1. The monocyte count as indicated by the accompanying CBC / differential is noted; however, a manual 500 cell differential count of the peripheral blood smear (10/2/2020) reveals 11% monocytes resulting in an absolute monocyte count of (1.02 K/uL). A CBC obtained on 04/29/2022 also revealed relative (19%) and absolute (1.4 K/uL) monocytosis. Additional cytogenetic and molecular studies are in progress and will be reported separately.
    • Peripheral smear: Manual review of the peripheral blood shows normochromic, normocytic anemia, thrombocytopenia, relative (11%) and absolute (1.02 K/uL) monocytosis and rare circulating blasts. RBCs: mild normochromic, normocytic anemia with minimal anisopoikilocytosis. WBCs: overall adequate in number with relative neutrophilia and relative and absolute monocytosis. A manual 500 cell differential count reveals 11% monocytes, resulting in an absolute monocyte count of 1.02 K/uL). Granulocytes show left shifted maturation and are dysplastic (increased hypogranular or hyposegmented forms). Monocytes include atypical forms. Rare (~1%) circulating blasts are noted. Platelets: thrombocytopenia with occasional large, hypogranular platelets.
    • Bone marrow biopsy: Quality: adequate. Cellularity: 90%. Hematopoiesis: trilineage maturation with myelomonocytic hyperplasia and relative erythroid hypoplasia. Megakaryocytes are increased (15/high power field on average) and are dysplastic (pleomorphic increased small or hypolobated forms, many with hyperchromatic nuclei, rarely with separation of nuclear lobes. Also identified are large forms with abnormal nuclear lobulation). Infiltrate: no apparent increase in blasts. Special stains: loose network of reticulin without significant intersections (minimal reticulin fibrosis); trichrome is negative for collagen deposition.
    • Bone marrow clot section: Quality: adequate. Cellularity: 90% morphologic features are similar to those observed in the core biopsy.
    • Bone marrow aspirate: Quality: adequate. Granulocytes: increased; left shifted maturation; dysplastic (hyposegmented nuclei or hypogranular cytoplasm). Erythrocytes: progressive maturation, mild dysplasia (slight nuclear irregularities, basophilic stippling of cytoplasm or N:C maturation asynchrony). Megakaryocytes: increased; dysplastic (variable in size, increased hypolobated forms, frequently with nuclear hyperchromasia). Blasts: overall ~5% of nucleated cells, including monocytic precursors. Iron: storage iron is adequate (2+ on a scale of 0 - 4). Occasional (9%) ring sideroblasts are present.
Differential diagnosis
  • Myeloid neoplasms with PDGFRA, PDGFRB, FGFR1 or JAK2 rearrangement:
    • Typically show eosinophilia
    • Specifically exclude these entities using FISH in cases of monocytosis with prominent eosinophilia
  • Chronic myeloid leukemia:
    • May present with monocytosis
    • Presence of BCR::ABL1
  • Reactive monocytosis:
    • Lacks clonal cytogenetic and molecular alterations
    • Bone marrow morphology is unremarkable, dysplasia is absent
    • Monocyte partitioning studies may be helpful
  • Primary myelofibrosis with monocytosis:
    • Megakaryocytes are large, atypical and clustered in primary myelofibrosis
    • Primary myelofibrosis with monocytosis is reported to have a higher JAK2 V617F allelic burden (median: 43%; range: 20 - 62%) compared with CMML cases in which the reported JAK2 V617F allelic burden is lower (median: 17%; range: 5 - 36%) (Hum Pathol 2019;85:290)
    • SRSF2 and RAS mutations
    • Rare gray zone cases exist with hybrid features (Am J Surg Pathol 2018;42:799, Hum Pathol 2019;85:290)
Board review style question #1
Which of the following is a required criterion for the diagnosis of chronic myelomonocytic leukemia according to WHO 5th criteria?

  1. < 10% blasts (including monoblasts and promonocytes) in the peripheral blood and bone marrow
  2. Presence of ASXL1 mutation of BCR::ABL1, PDGFRA, PDGFRB or FGFR1 rearrangement or PCM1::JAK2
  3. Persistent absolute (≥ 0.5 x 109/L) and relative (> 10%) peripheral blood monocytosis
  4. Persistent absolute (≥ 1 x 109/L) and relative (> 10%) peripheral blood monocytosis
  5. Presence of SRSF2 mutation
Board review style answer #1
C. The prerequisite criteria for a diagnosis of CMML per WHO 5th are:

1. Persistent absolute (≥ 0.5 × 109/L) and relative (≥ 10%) peripheral blood monocytosis.
2. Blasts constitute < 20% of the cells in the peripheral blood and bone marrow.
3. Not meeting diagnostic criteria of chronic myeloid leukemia or other myeloproliferative neoplasms.
4. Not meeting diagnostic criteria of myeloid/lymphoid neoplasms with eosinophilia and defining gene rearrangements (e.g. PDGFRA, PDGFRB, FGFR1, or JAK2).

Answer A is incorrect because up to 19% blasts are permitted for the diagnosis. Answer B is incorrect because ASXL1 mutation is not a required criterion and BCR::ABL1, PDGFRA, PDGFRB or FGFR1 rearrangement or PCM1::JAK2. Exclude the diagnosis of CMML. Answer D is incorrect because the minimum absolute monocyte count has been lowered to (≥ 0.5 × 109/L) in the WHO 5th edition. Answer E is incorrect because SRSF2 mutation is not a required criterion.

Comment here

Reference: Chronic myelomonocytic leukemia
Board review style question #2

A bone marrow aspirate shows the above image. A diagnosis of CMML is established. What is the most likely mutated gene in this case?

  1. DNMT3A
  2. EZH2
  3. IDH2
  4. TET2
Board review style answer #2
D. TET2 is the most commonly mutated gene in CMML. Answers A - C are incorrect, because while these mutations may be seen in CMML, they occur with substantially less frequency compared to TET2 mutations.

Comment here

Reference: Chronic myelomonocytic leukemia

Chronic neutrophilic leukemia
Definition / general
  • Rare myeloproliferative neoplasm characterized by persistent peripheral blood neutrophilia, bone marrow hypercellularity and hepatosplenomegaly
  • Requires exclusion of reactive neutrophilia as well as other myeloproliferative neoplasms such as chronic myeloid leukemia (including typical, atypical and neutrophilic variants), polycythemia vera, primary myelofibrosis or myelodysplastic / myeloproliferative neoplasm
Essential features
  • Persistent blood neutrophilia (≥ 25 x 109/L); with neutrophil ≥ 80% and myeloid precursors (promyelocytes, myelocytes and metamyelocytes) comprising < 10% of the white blood cells in peripheral blood
  • Hypercellular bone marrow with predominance of neutrophilic granulocytes and blasts < 5%
  • No dysgranulopoiesis
  • CSF3R activating mutations are common
  • Lacks BCR-ABL1 gene rearrangement, which distinguishes CNL from the neutrophilic variant of chronic myeloid leukemia
  • Must be distinguished from reactive leukemoid reactions that can be seen in association with infectious or inflammatory processes or as a paraneoplastic phenomenon secondary to plasma cell neoplasms, lymphomas and some carcinomas
ICD coding
  • ICD-10: D47.1 - Chronic myeloproliferative disease; also applicable to myeloproliferative disease, unspecified
Epidemiology
Sites
  • Peripheral blood
  • Bone marrow
  • Spleen
  • Liver
Pathophysiology
Etiology
  • Unknown
  • Some cases are associated with cytotoxic chemotherapy (Br J Haematol 2005;128:275)
  • Substantial number of reported cases are associated with plasma cell neoplasm; these cases likely represent a neutrophilic leukemoid reaction secondary to production of G-CSF by neoplastic plasma cells (Br J Haematol 2015;171:400)
Clinical features
Diagnosis
  • World Health Organization (WHO) 2016 revised diagnostic criteria:
    1. Peripheral blood white blood cell count ≥ 25 x 109/L
      • Segmented neutrophils plus bands neutrophils constitute ≥ 80% of the white blood cells
      • Neutrophil precursors (promyelocytes, myelocytes and metamyelocytes) constitute < 10% of the white blood cells
      • Myeloblasts rarely if ever observed
      • Monocyte count < 1 x 109/L
      • No dysgranulopoiesis
    2. Hypercellular bone marrow
      • Neutrophilic granulocytes increased in percentage and number
      • Neutrophil maturation appears normal
      • Myeloblasts constitute < 5% of the nucleated cells
    3. Not meeting WHO criteria for BCR-ABL1 positive chronic myeloid leukemia, polycythemia vera, essential thrombocythemia or primary myelofibrosis
    4. No rearrangement of PDGFRA, PDGFRB or FGFR1 and no PCM1-JAK2 fusion
    5. CSF3R T618I or another activating CSF3R mutation or persistent neutrophilia (≥ 3 months), splenomegaly and no identifiable cause of reactive neutrophilia including absence of a plasma cell neoplasm or if a plasma cell neoplasm is present, demonstration of clonality of myeloid cells by cytogenetic or molecular studies (Pathology 2021;53:339)
Laboratory
  • Persistent mature neutrophilia
  • Absence of monocytosis, basophilia, eosinophilia or increased immature myeloid cells
  • Majority of patients present with mild anemia (Leukemia 2005;19:313)
  • Elevated lactate dehydrogenase (LDH) and leukocyte alkaline phosphatase (LAP)
  • Elevated vitamin B12 and uric acid
Prognostic factors
Case reports
Treatment
Microscopic (histologic) description
  • Hypercellular bone marrow with increased myeloid to erythroid ratio; up to 20:1
  • Increase in neutrophilic granulocytes
  • Myeloblasts and promyelocytes are not increased
  • No dysplasia in erythroid and myeloid series or megakaryocytes
  • Spleen with leukemic neutrophilic infiltrate in the red pulp
  • Liver with mature leukemic granulocytes in sinusoids or portal spaces
Microscopic (histologic) images

Contributed by Jingwei Li, M.D., Ph.D., Vignesh Shanmugam, M.D. and Roberto N. Miranda, M.D.

Hypercellular bone marrow with myeloid predominance

Increased neutrophilic granulocytes

Toxic granulation and Döhle bodies

Myeloid cells

Peripheral smear description
  • Peripheral blood with neutrophilia, including mature neutrophils and bands
  • Toxic granulation and Döhle bodies in neutrophils
Peripheral smear images

Contributed by Jingwei Li, M.D., Ph.D., Vignesh Shanmugam, M.D. and Roberto N. Miranda, M.D.

Neutrophilic leukocytosis

Toxic granulation and Döhle bodies

Leukocytosis

Molecular / cytogenetics description
  • CSF3R mutations, up to 89% in a case series (N Engl J Med 2013;368:1781)
  • Can associate with ASXL1 or SETBP1 mutations
  • JAK2 V617F can also be observed
  • Cytogenetic studies are normal in 90% of cases
  • Remaining cases show:
    • Gains of chromosomes 8, 9 and 21
    • del(7q), del(11q), del(12p) and del(20q)
    • Nullisomy 17
  • Absence of BCR-ABL1 rearrangement
  • Absence of rearrangements in PDGFRA, PDGFRB, FGRF1 or PCM1-JAK2
Sample pathology report
  • Bone marrow core biopsy and clot specimen, aspirate smears and peripheral blood:
    • Chronic neutrophilic leukemia, 1% blasts (see comment)
    • Comment:
      • The bone marrow core biopsy is adequate in size and quality. The marrow is markedly cellular (> 95% cellular). Erythroid precursors are decreased and exhibit adequate maturation. Myeloid cells are markedly increased and exhibit maturation. Megakaryocytes are present in normal number and morphologically are unremarkable. Scattered mast cells are seen and do not form aggregates. Scattered lymphocytes and plasma cells are seen. Lymphoid aggregates are not identified. Bone trabeculae exhibit focal osteoblastic activity with no osteosclerosis.
      • Reticulin stain shows no significant reticulin fibrosis. Trichrome stain shows no abnormal collagen fibrosis.
      • Immunohistochemistry for CD34 reveals that less than 5% of marrow cells are positive. In situ hybridization for kappa and lambda immunoglobulin light chains demonstrate polytypic expression in the plasma cells.
      • Bone marrow aspirate smears show a cell differential with blasts 1%, promyelocytes 2%, myelocytes 2%, metamyelocytes 2%, and granulocytes and bands aggregate to 82%; lymphocytes 3%, plasma cells 2% and erythroid precursors 4%. The myeloid to erythroid ratio is 23 (91:4).
      • FISH assays did not show rearrangements of PDGFRA, PDGFRB, FGFR1 or BCR-ABL1.
      • Next generation sequencing showed the following findings: Pathogenic single nucleotide variants and small insertions / deletions:
          • ASXL1 NM_015338 c.1534C>T p.Q512* - in 46.6% of 1104 reads
          • CSF3R NM_156039 c.1853C>T p.T618I - in 41.2% of 699 reads
        • Read count analysis shows gain of JAK2 (on 9p).
        • FLT3 ITD is not detected.
      • Comment: The findings are of a hypercellular marrow with myeloid hyperplasia with no significant fibrosis or increase in blasts. No significant dysplasia is noted on the aspirate smear. Next generation sequencing using probes for 81 gene mutations associated with myeloid neoplasms demonstrated a canonical CSF3R (T618I) mutation along with pathogenic ASXL1 mutations. FISH analysis did not show rearrangements involving PDGFRA, PDGFRB, FGFR1 or BCR-ABL1.
      • The diagnosis of chronic neutrophilic leukemia is rendered based on the presence of persistent neutrophilia (69.78 x 109/L neutrophils, immature myeloid cells < 10%) and CSF3R mutation.
Differential diagnosis
  • Chronic myeloid leukemia (CML):
    • Associated with BCR-ABL1 fusion gene, frequent basophilia, thrombocytosis or eosinophilia
  • Neutrophilic chronic myeloid leukemia (N-CML):
    • Uncommon BCR-ABL1 fusion that results in a 230 kD fusion protein (p230)
  • Atypical CML:
    • Prominent granulocytic dysplasia and left shift
  • Chronic myelomonocytic leukemia (CMML):
    • Chronic monocytosis (> 1 x 109/L) and dysplasia
  • Leukemoid reaction / infection:
    • Rule out underlying infection or other malignancies (plasma cell neoplasms and others)
    • Lacks CSF3R mutations or other cytogenetic abnormalities seen in CNL listed above in the molecular / cytogenetics section
Board review style question #1

A 53 year old man presents with abdominal pain. Abdominal imaging shows splenomegaly and a CBC shows leukocytosis (70 x 109/L) with 95% neutrophils. A representative image of the bone marrow biopsy is shown below. An extensive laboratory work up is negative for infection or other causes that could explain these findings. Molecular studies do not reveal a BCR-ABL rearrangement. What is the gene that is most commonly mutated in this condition?

  1. CSF3R
  2. JAK2
  3. NOTCH2
  4. RUNX1
  5. TET2
Board review style answer #1
A. CSF3R, which is the gene that is most frequently mutated in chronic neutrophilic leukemia. JAK2 mutations are most frequent in polycythemia vera. NOTCH2 mutations are more frequent in B cell lymphomas (splenic marginal zone lymphoma, diffuse large B cell lymphoma). RUNX1 mutations are seen in other myeloid neoplasms (acute myeloid leukemia, myelodysplastic syndrome) but are uncommon in myeloproliferative neoplasms. TET2 mutations are frequent in a variety of myeloid neoplasms and are not specific to any particular class.

Comment Here

Reference: Chronic neutrophilic leukemia
Board review style question #2
Which of the following features distinguishes chronic neutrophilic leukemia (CNL) from atypical chronic myeloid leukemia, BCR-ABL1 negative?

  1. Lack of basophilia in CNL
  2. Lack of dysgranulopoiesis in CNL
  3. Lack of monocytosis in CNL
  4. Presence of ASXL1 mutation in CNL
  5. WBC count > 50 x 109/L
Board review style answer #2
B. Lack of dysgranulopoiesis. Granulocytic dysplasia is not seen in chronic neutrophilic leukemia, while it is a typical feature of atypical CML.

Comment Here

Reference: Chronic neutrophilic leukemia

Clonal cytopenia of undetermined significance (CCUS) (pending)
[Pending]

Clonal hematopoiesis of indeterminate potential (CHIP) (pending)
[Pending]

Cryoglobulinemia
Definition / general
  • See also topic in Kidney nontumor chapter
  • Cryoglobulinemia is a clinical disorder defined as the presence of immunoglobulins that precipitate below 37°C but redissolve upon warming
Essential features
  • Cryoglobulinemia is a clinical entity characterized by the presence of cryoglobulins in the serum
  • Cryoglobulins precipitate below normal body temperature (37°C) and dissolve on rewarming
  • Underlying causes of cryoglobulinemia include B cell / plasma cell malignances, infections such as hepatitis C virus and collagen vascular disorders such as Sjögren syndrome
  • Clinical features of cryoglobulinemia include the triad of purpura, arthralgia and weakness
ICD coding
  • ICD-10: D89.1 - cryoglobulinemia
Epidemiology
  • Overall prevalence is unknown (Lancet 2012;379:348)
  • Cryoglobulinemia is associated with infection (most frequently hepatitis C), autoimmune disease (e.g. Sjögren syndrome) and malignancy (B cell / plasma cell hematopoietic neoplasms)
  • 12 - 56% of hepatitis C virus patients have circulating cryoglobulins
  • Prevalence in patients coinfected with HIV and hepatitis C virus is 35 - 64%
Sites
  • Cold exposure leads to precipitation of cryoglobulins (typically in distal extremities) (Lancet 2012;379:348)
  • Internal organ involvement may occur
Pathophysiology
  • 3 types exist: type I (monoclonal IgM or monoclonal IgG), type II (mixture of monoclonal IgM and polyclonal IgG) and type III (polyclonal IgM and polyclonal IgG) (Blood 2017;129:289)
  • Type I cryoglobulinemia is characterized by hyperviscosity and occlusion of small blood vessels
  • In type II and III cryoglobulinemia, immune complexes are formed between monoclonal / polyclonal IgM and polyclonal IgG with RF formation and complement fixation
  • In hepatitis C virus (HCV) associated cryoglobulinemia, HCV drives a clonal B cell expansion producing monoclonal IgM that binds to anti-HCV IgG, forming immune complexes that promote vasculitis
Etiology
Clinical features
  • Purpura, arthralgia and weakness are noted in 80% of patients (Lancet 2012;379:348)
  • Type I cryoglobulinemia patients may present with hyperviscosity syndrome
  • Flares of immune complex mediated vasculitis can be seen
  • Joint pain may be present without inflammation
  • 20% of patients present with nephropathy
  • 17 - 60% present with peripheral neuropathy
  • Less common manifestations include gastrointestinal involvement, pulmonary involvement and myocardial vasculitis
Diagnosis
  • Cryoglobulins are identified by collecting blood (transported at 37°C) in a red top tube and dividing the serum into 3 tubes (Am J Hematol 2011;86:500)
    • First tube is used to quantify the amount of cryoprecipitate (cryocrit) by cooling to 4°C for up to 72 hours
    • Second tube is used to identify and analyze the cryoprecipitate
    • Third tube is used to demonstrate solubility of the cryoprecipitate on rewarming to 37°C
  • Diagnosis of underlying conditions is made on clinical, laboratory and histopathologic findings
Laboratory
  • Monoclonal gammopathy in types I and II (Lancet 2012;379:348)
  • Hyperviscosity
  • Unexplained low complement (C4)
  • Unexplained high rheumatoid factor
  • Raised serum creatinine
  • Microscopic hematuria and red blood cell casts
Prognostic factors
  • Worse outcome with older age, renal insufficiency, pulmonary involvement and gastrointestinal involvement
Case reports
  • 47 year old man with persistent mixed cryoglobulinemia despite successful treatment of hepatitis C, aggressive B cell directed therapies and long term plasma exchanges (Kidney Int Rep 2019;4:1194)
  • 55 year old woman with IgG4 related sialadenitis complicated with type III mixed cryoglobulinemia (Medicine (Baltimore) 2019;98:e16571)
  • 56 year old man with type I immunoglobulin M cryoglobulinemic vasculitis with chronic lymphocytic leukemia and a history of hepatitis C virus (Cureus 2019;11:e4729)
  • 58 year old woman with severe cutaneous necrosis associated with type I cryoglobulinemia (JAAD Case Rep 2019;5:736)
Treatment
  • Treatment is based on underlying pathogenesis (Lancet 2012;379:348)
  • Conventional immunosuppression: similar to treatment of other vasculitides
  • Apheresis: removes cryoglobulin from circulation
  • Antiviral therapy: for hepatitis C virus related cryoglobulinemia
  • Biologics: rituximab driven B cell depletion
Microscopic (histologic) description
  • Precipitated cryoglobulin appear as hyaline thrombi that occlude blood vessels (Lancet 2012;379:348)
  • In the kidney, glomeruli may show a membranoproliferative pattern of injury with PAS positive intracapillary deposits and double contour basement membrane
  • Vasculitis may involve small and medium sized blood vessels; skin, kidney and nerves are most commonly affected
Microscopic (histologic) images

Contributed by Maria M. Picken, M.D., Ph.D.

Cryoglobulinemic nephropathy

Peripheral smear description
  • Cryoglobulins (seen as pale staining material), rouleaux formation, pseudothrombocytosis
Peripheral smear images

Contributed by Anas Nasir, M.Sc.

Circulating cryoglobulins

Videos

Managing HCV cryoglobulinemia

Cryoglobulinemia and vasculitis

Cryoglobulinemia, lymphoproliferation and hepatitis C

Differential diagnosis
Board review style question #1

A 62 year old woman has increasing weight loss, fatigue and lack of appetite over the past 7 months. She is now complaining of intermittent headaches, dizziness and decreased vision bilaterally. Her hands get cold easily and she gets frequent nosebleeds. On physical exam, she has generalized lymphadenopathy. Laboratory studies show presence of cryoglobulin. Serum electrophoresis / immunofixation is pictured above (ELP = serum electrophoresis, G = IgG, A = IgA, M = IgM, K = kappa, L = lambda). Which underlying condition is most likely in this patient?

  1. Henoch-Schönlein purpura
  2. Hepatitis C infection
  3. Sjögren syndrome
  4. Systemic lupus erythematosus
  5. Waldenström macroglobulinemia
Board review style answer #1
E. Waldenström macroglobulinemia

Comment Here

Reference: Cryoglobulinemia
Board review style question #2
A 58 year old man with a medical history significant for hypertension, hyperlipidemia and diabetes presents to the clinic for a 5 month history of oliguria, arthralgia and fatigue. Physical exam shows needle track marks in his left antecubital fossa. Urinalysis shows microscopic hematuria. Serum creatine is increased and complement levels are decreased. Which of the following antibodies is most likely to be detected on further laboratory evaluation?

  1. Anti centromere antibody
  2. Anti desmosome antibody
  3. Anti glomerular basement antibody
  4. Anti hemidesmosome antibody
  5. Anti hepatitis C antibody
Board review style answer #2
E. Anti hepatitis C antibody

Comment Here

Reference: Cryoglobulinemia
Board review style question #3
A 54 year old African American man with hepatitis C and hypertension presents to his physician for recurrent joint pains, decreased urination and fatigue, which has been ongoing for the past 3 months. Physical examination shows several nonblanching purpuric papules on the patient's lower extremities and trunk. Urinalysis is positive for red blood cell casts. Renal biopsy is most likely to show which of the following?

  1. Crescent moon shape on light microscopy
  2. Mesangial expansion with eosinophilic nodular glomerulosclerosis on light microscopy
  3. Normal renal biopsy
  4. PAS positive intracapillary deposits with double contour basement membrane
  5. Podocyte hypertrophy, capillary wall thickening and diffuse glomerular sclerosis
Board review style answer #3
D. PAS positive intracapillary deposits with double contour basement membrane

Comment Here

Reference: Cryoglobulinemia

Cutaneous mastocytosis

ETV6::ABL1 fusion with other tyrosine kinase gene fusions (pending)
[Pending]

Early T cell precursor lymphoblastic leukemia
Definition / general
  • Aggressive T cell neoplasm with immunophenotypic markers of early T cell differentiation, including stem cell markers or myeloid differentiation markers
Essential features
  • Histologic morphology of lymphoblastic leukemia / lymphoma
  • Early T cell precursor (ETP) blasts have the following immunophenotype: positive for cytoplasmic CD3, absent (< 5% positive blasts) CD1a and CD8 expression, absent or dim (< 75% positive blasts) CD5 expression, positive (≥ 25% positive blasts) for ≥ 1 myeloid (CD11b, CD13, CD33, CD65, CD117) or stem cell (CD34, HLA-DR) marker and negative (< 3%) myeloperoxidase (Lancet Oncol 2009;10:147)
Epidemiology
Sites
  • Can involve the bone marrow, thymus, lymph nodes or extranodal sites
Pathophysiology
  • Physiologically, early T cell precursors characteristically migrate from bone marrow to thymus
  • They express lymphoid, myeloid and stem cell markers and are characterized by multilineage differentiation potential (Nature 2008;452:764, Nature 2008;452:768)
  • Single cell RNA sequencing in ETP ALL demonstrates leukemic cell plasticity, with both stem cell and T cell signatures and can differentiate to stimulate an immunosuppressive microenvironment through expression of the LGALS9 checkpoint molecule and support of dysfunctional CD8 positive T cells (Blood 2021;137:2463)
  • ETP ALL shows increased expression of MEF2C and decreased expression of BCL11B and GATA3 (Cancer Cell 2011;19:484, Blood 2021;138:773, Br J Haematol 2021;194:1034, J Exp Med 2009;206:2987, PLoS One 2013;8:e53190)
  • Concurrent EZH2 and RUNX1 inactivating mutations in a recent murine molecular pathogenesis model were hypothesized to be the initiating events that lead to expansion of early thymic progenitors with an ETP ALL gene expression signature
    • Leukemic potential is further stimulated by acquisition of mutations (e.g., FLT3), resulting in constitutive activation of the RAS signaling pathway (Cancer Cell 2018;33:274)
  • Pathogenesis in this model is similar to that of myeloid neoplasms, by harboring activating mutations in genes regulating cytokine receptor and RAS signaling (FLT3, NRAS, KRAS, IL7R, JAK3, JAK1, SH2B3 and BRAF), hematopoietic development (GATA3, ETV6, RUNX1, IKZF1) and histone modifications (EZH2, EED, SUZ12, SETD2, EP300) (Nature 2012;481:157, Blood 2013;121:4749, PLoS One 2013;8:e53190)
  • NOTCH1 mutations are uncommon in ETP ALL, in contrast to other T lymphoblastic lymphoma / leukemia (T ALL) (PLoS One 2013;8:e53190)
  • Frequency of CDKN2A / CDKN2B deletion is low in patients with ETP ALL (Haematologica 2020;105:e294, Am J Hematol 2021;96:312, Blood Adv 2021;5:2890)
  • Nonrearranged TRG gene rearrangement is present in most ETP ALL patients; they may lack more frequent biallelic deletion of the TRG locus than non-ETP ALL, confirming early maturation stage (PLoS One 2013;8:e53190, Haematologica 2014;99:94)
Etiology
  • Unknown etiology in most cases
  • A study has demonstrated an association between germline RUNX1 variants and ETP ALL; it has been observed that mice with RUNX1 variants and mutated JAK3 induce ETP ALL (J Clin Invest 2021;131:e147898)
Clinical features
  • Cervical, supraclavicular and axillary lymphadenopathy (50%)
  • Mediastinal mass (50 - 75%) with associated compressive superior vena cava syndrome, tracheal obstruction and pericardial effusion (Blood Cancer J 2012;2:e55)
  • CNS involvement (18%) (Blood Cancer J 2012;2:e55)
  • Features related to extramedullary hematopoiesis (such as hepatosplenomegaly)
Diagnosis
  • Morphologic features of lymphoblastic leukemia / lymphoma (Lancet Oncol 2009;10:147)
  • Blasts with expression of ETP immunophenotype: positive for cytoplasmic CD3, absent (< 5% positive blasts) CD1a and CD8 expression, absent or dim (< 75% positive blasts) CD5 expression, positive (≥ 25% positive blasts) for ≥ 1 myeloid (CD11b, CD13, CD33, CD65, CD117) or stem cell (CD34, HLA-DR) marker and negative (< 3%) myeloperoxidase (Lancet Oncol 2009;10:147, Blood 2016;127:1863)
Laboratory
  • White blood count (WBC) may or may not be elevated and in some studies, ETP ALL patients tend to have lower WBC in comparison to those with non-ETP ALL (Pediatr Blood Cancer 2021;68:e28719)
  • Anemia and thrombocytopenia are secondary to bone marrow involvement
Prognostic factors
Case reports
Treatment
  • Myeloid oriented chemotherapy (as frontline induction treatment) along with gene targeting inhibitors, followed by allogeneic hematopoietic stem cell transplant
  • Early T cell precursor acute lymphoblastic leukemia: diagnosis, updates in molecular pathogenesis, management and novel therapies (Front Oncol 2021;11:750789, Onco Targets Ther 2021;14:3795)
Microscopic (histologic) description
  • As seen in patients with other ALL, blasts from patients with ETP ALL are small to medium sized with scant cytoplasm and inconspicuous nucleoli
Microscopic (histologic) images

Contributed by Kamran M. Mirza, M.D., Ph.D. and Yazan Alhalaseh, M.D.
Hypercellular bone marrow Hypercellular bone marrow core biopsy

Hypercellular bone marrow core biopsy

Hypercellular bone marrow clot section

Hypercellular bone marrow clot section

Hypercellular bone marrow

Hypercellular bone marrow

CD3 IHC

CD3 IHC

CD34 IHC

CD34 IHC

Peripheral smear description
  • Peripheral blood smear reveals increase in circulating blasts with high N:C ratio
Peripheral smear images

Contributed by Kamran M. Mirza, M.D., Ph.D. and Yazan Alhalaseh, M.D.
Atypical circulating blasts Atypical circulating blasts Atypical circulating blasts

Atypical circulating blasts

Positive stains
Negative stains
Flow cytometry description
Flow cytometry images

Contributed by Kamran M. Mirza, M.D., Ph.D. and Yazan Alhalaseh, M.D.
Side scatter versus CD45 panel

Side scatter versus CD45 panel

CD7 versus CD2 panel

CD7 versus CD2 panel

CD56 versus CD3 panel

CD56 versus CD3 panel

Molecular / cytogenetics description
  • Not relevant for diagnostic evaluation
Videos

ETP ALL case

Sample pathology report
  • Right posterior iliac crest, core biopsy, clot section, aspirate smears and touch imprints:
    • T lymphoblastic leukemia / lymphoma (T ALL phenotypically compatible with ETP ALL) (see comment)
    • Hypercellular marrow with 87% blasts and markedly diminished trilineage hematopoiesis
    • Correlation with cytogenetics is recommended for complete assessment
    • Eosinophils (2%), lymphocytes (2%), plasma cells (1%) (500 cell count on aspirate smear)
    • Comment: The peripheral blood film reveals 76% circulating blasts with high N:C ratio. The most recent hemogram shows leukocytosis, anemia and thrombocytopenia.
    • The bone marrow core is fragmented and reveals sheets of blasts replacing normal bone marrow elements. The clot shows similar findings. Immunohistochemical stains (CD34, TdT, CD3) on the core and clot reveal blast positivity for CD34 and CD3. Stains for MPO and TdT are negative. Cytochemical analysis for MPO and alpha naphthyl acetate esterase (ANAE) stains are performed. The blasts are negative for MPO and ANAE by cytochemical as well.
    • The marrow aspirate smears reveal markedly diminished normal trilineage hematopoiesis and blasts for 87% of a 500 cell differential count. The blasts have high N:C ratio, smudged chromatin with irregular nuclear membranes and occasional cytoplasmic vacuoles. Flow cytometry phenotyping of the blasts (dim CD45 gate) reveals positivity for cytoplasmic CD3, CD34, dim CD4, CD56, CD38, CD33, CD13 and CD117. The blasts are negative for MPO, surface CD3, CD5, CD8, CD22, CD10, CD20, HLA-DR, CD11c, CD64, CD14, TdT and cytoplasmic CD22. Gating on the mature lymphocytes reveals no monotypic B cell population or T cell abnormality based on markers assayed.
    • Overall, the presence of cytoplasmic CD3 expression by flow cytometry and CD3 positivity by immunohistochemistry with concurrent lack of MPO (flow cytometry, immunohistochemistry and cytochemistry) monocytic markers and B cell antigens is diagnostic of T lymphoblastic leukemia / lymphoma. Please correlate with corresponding cytogenetics and FISH findings for complete assessment. The phenotype (CD3, CD4, CD7, CD33, CD13 positive, with negativity for MPO, CD8 and CD5) is compatible with ETP ALL (Nature 2012;481:157).
Differential diagnosis
Board review style question #1
A 57 year old man presents to his primary care physician with a 2 month history of persistent fatigue. Vital signs are within normal limits. Physical examination is remarkable for pallor and hepatosplenomegaly. Laboratory workup reveals white blood cell (WBC) count of 17,000, hemoglobin of 9.0 and platelet count of 78,000. A peripheral blood smear shows 30% circulating blasts. A bone marrow core biopsy shows hypercellular bone marrow with 80% blasts. Immunohistochemical stains on the bone marrow core biopsy highlight the blast population that are positive for CD34 and CD3. Flow cytometry shows blasts that are positive for cytoplasmic CD3, CD34, dim CD4, CD56, CD33, CD13 and CD117. Genetic testing for BCR::ABL1 by cytogenetics was negative. Which of the following is the most likely diagnosis?

  1. Chronic myeloid leukemia
  2. Early T cell precursor lymphoblastic leukemia
  3. Myelodysplastic neoplasm
  4. Natural killer cell (NK) large granular lymphocytic leukemia
Board review style answer #1
B. Early T cell precursor lymphoblastic leukemia. The patient presents with symptoms with concern for hematologic malignancy. Workup is notable for leukocytosis. The bone marrow biopsy shows hypercellular bone marrow with increased blasts. The immunohistochemical features are specific for early T cell precursor (ETP) given the CD3, CD56 and CD34 expression but absent CD1a, CD8 and myeloperoxidase.

Answer C (myelodysplastic neoplasm) is incorrect because the patient's blasts are significantly increased in the peripheral blood and bone marrow biopsy. Answer A (chronic myeloid leukemia) is incorrect because it can have increased blasts in the blast phase; however, the absence of BCR::ABL1 translocation rules out this diagnosis. Answer D (natural killer cell [NK] large granular lymphocytic leukemia) is incorrect because it is not characterized by increased CD34+ blasts.

Comment Here

Reference: Early T cell precursor lymphoblastic leukemia
Board review style question #2


A 48 year old woman presents for evaluation of shortness of breath and fatigue over a 4 month period. She notes swelling in her neck and axillary region. Physical examination shows generalized lymphadenopathy and hepatosplenomegaly. Her complete blood count shows a white blood cell (WBC) count of 22,000, hemoglobin of 8.0 and platelet count of 100,000. A peripheral blood smear and flow cytometry on the peripheral blood are shown above. A bone marrow biopsy is performed as shown above and there is a neoplastic lymphoid population that is positive for cytoplasmic CD3, CD5 and CD34 and negative for CD1a, CD8 and myeloperoxidase. Which of the following is the most likely diagnosis?

  1. Adult T cell leukemia
  2. Early T cell precursor lymphoblastic leukemia
  3. Mixed phenotype acute leukemia (MPAL), T / myeloid
  4. Natural killer cell (NK) large granular lymphocytic leukemia
Board review style answer #2
B. Early T cell precursor lymphoblastic leukemia. The patient presents with history and exam with concern for hematologic malignancy. The peripheral blood smear demonstrates atypical lymphocytes. The third figure above depicts flow cytometry revealing blasts (red) with low side scatter and dim CD45. The fourth figure shows blasts are positive for CD7 and negative for CD2. The fifth figure demonstrates blasts are positive for CD56 and dim positive for CD3. In the first figure, bone marrow biopsy is notable for hypercellularity. Given these findings in the setting of positive cytoplasmic CD3, CD5 and CD34 and negative CD1a, CD8 and myeloperoxidase, early T cell precursor lymphoblastic leukemia is the most likely diagnosis.

Answer C (mixed phenotype acute leukemia [MPAL], T / myeloid) is incorrect because the tumor cells in this case lack the expression of CD8 and myeloperoxidase and the immunoprofile meets criteria for early T cell precursor acute lymphoblastic leukemia. Answer D (natural killer cell [NK] large granular lymphocytic leukemia) is incorrect because it is not characterized by increased CD34 positive blasts. Answer A (adult T cell leukemia) is incorrect because the tumor cells of this entity do not express stem cell markers such as CD34.

Comment Here

Reference: Early T cell precursor lymphoblastic leukemia

Essential thrombocythemia
Definition / general
  • Essential thrombocythemia is a chronic myeloproliferative neoplasm that primarily involves the megakaryocytic lineage
Essential features
  • Characterized by sustained thrombocytosis (platelet count > 450 x 109/L) in the peripheral blood and increased numbers of large, atypical megakaryocytes in the bone marrow; clinically by the occurrence of thrombosis or hemorrhage
Terminology
  • Idiopathic thrombocythemia / thrombocytosis; essential hemorrhagic thrombocythemia; idiopathic hemorrhagic thrombocythemia; idiopathic thrombocythemia
  • Has also been called essential thrombocytosis and primary thrombocytosis
ICD coding
  • ICD-10: D47.3 - essential (hemorrhagic) thrombocythemia
Epidemiology
  • In the U.S. and Europe the incidence is 0.2 - 2.3 cases/100,000
  • Median age of 60 years at diagnosis but may occur at any age
  • Higher incidence among African Americans
  • More common in women (F:M = 2:1) (Br J Haematol 2016;174:382)
  • Rare in children (must rule out hereditary thrombocytosis)
Pathophysiology
  • Clonal stem cell disorder characterized by increased platelet production
  • Serum thrombopoietin levels can be normal or elevated
  • Harbor a phenotypic driver mutation in JAK2 (present in 50 - 60% of cases), CALR (30%) or MPL (3%); ~12% of cases are triple negative for these mutations (Leuk Lymphoma 2017;58:2786)
    • JAK2, CALR or MPL mutations result in the upregulation of JAK-STAT pathway
      • JAK2 is a nonreceptor tyrosine kinase and has been found to play a crucial role in hematopoiesis
      • JAK2 mutations leads to a gain of function leading to the activation of intracellular signaling pathways associated with the receptors of hematopoietic cytokines
      • CALR is involved in cellular proliferation, differentiation and apoptosis
Etiology
  • Etiology unknown
  • Most cases are sporadic
  • Some mutations of JAK2 and GSN genes have been found in families with hereditary thrombocytosis
Clinical features
  • Patient can either be asymptomatic or symptomatic
  • Asymptomatic
    • More than half of cases
    • Detected on routine peripheral blood count
  • Less than half present with symptoms related to thrombosis or hemorrhage
    • Long asymptomatic periods alternating with thrombotic or hemorrhagic crises
    • May rarely progress to myelofibrosis
  • Symptoms are due to qualitative and quantitative abnormalities in platelets and include
    • Bleeding tendency
    • Microvascular occlusion
    • Thrombosis of major arteries leading to symptoms such as stroke and transient ischemic attacks
    • Vasomotor manifestations such as headache, transient visual disturbances and syncope
    • Mild splenomegaly
    • Hepatomegaly and lymphadenopathy are uncommon
    • Unexplained abortion (Blood 2007;110:485)
  • Closely related to polycythemia vera but no increase in red blood cell mass
  • Increased proliferation is usually confined to megakaryocytes with platelet count > 450 x 109/L
Diagnosis
  • Diagnostic criteria for early / prefibrotic essential thrombocythemia: requires either all major criteria or the first 3 major criteria plus the minor criterion (Blood 2022;140:1200, Leukemia 2022;36:1703) (WHO 2022 and ICC 2022)
    • Major
      • Criterion #1: sustained platelet count ≥ 450 x 109/L
      • Criterion #2: megakaryocyte proliferation, large and mature with hyperlobulated nuclei (atypical), little to no granulocyte or erythroid proliferation; very rarely a minor (grade P) increase in reticulin fibers
      • Criterion #3: does not meet WHO criteria for chronic myeloid leukemia, polycythemia vera, primary myelofibrosis, myelodysplastic syndrome or other myeloid neoplasm
      • Criterion #4: demonstration of JAK2 or CALR or MPL mutation
    • Minor
      • Presence of a clonal marker
      • Absence of evidence of reactive thrombocytosis
  • Diagnostic criteria for post essential thrombocythemia myelofibrosis (Blood 2022;140:1200, Leukemia 2022;36:1703)
    • 2 major criteria and at least 2 minor criteria (WHO 2022 and ICC 2022)
      • Required criteria
        • Established diagnosis of essential thrombocythemia
        • Bone marrow fibrosis grade 2 - 3 (on 0 - 3 scale)
      • Additional criteria (2 required)
        • Anemia or decrease in hemoglobin by 2 gm/dL from the baseline level
        • Leukoerythroblastic peripheral blood
        • Splenomegaly
        • Rise in lactate dehydrogenase (LDH) levels
        • Constitutional symptoms (at least 2 or all 3): unexplained fever (> 37.5 °C), night sweats, > 10% weight loss in 6 months
    • Bone marrow examination is necessary to exclude other myeloproliferative neoplasms, myelodysplastic syndromes associated with isolated del(5q) and myelodysplastic / myeloproliferative neoplasm
Prognostic factors
  • Essential thrombocythemia is generally an indolent disease with a good prognosis
  • Initial management is largely dictated by the risk of thrombotic complication based on age, JAK2 mutation status and the history of thrombosis, and is stratified as follows (Blood Cancer J 2015;5:e369)
    • High risk: history of thrombosis at any age or age > 60 with a JAK2 V617F mutation
    • Intermediate risk: age > 60, no JAK2 mutation detected and no history of thrombosis
    • Low risk: age ≤ 60 with JAK2 mutation and no history of thrombosis
    • Very low risk: age ≤ 60, no JAK2 mutation detected and no history of thrombosis
  • Variable rate of transformation to acute myeloid leukemia or myelodysplastic syndrome; generally these patients have a poor prognosis
  • Can undergo delayed disease progression into a fibrotic state called post essential thrombocythemia myelofibrosis; < 5% conversion rate after 10 - 20 years
  • Median survival is 10 - 15 years, hence life expectancy is almost normal (Am J Med 2004;117:755)
  • CALR mutations are associated with a favorable prognosis in essential thrombocythemia compared to other genetic changes (Semin Hematol 2018;55:215)
Case reports
Treatment
  • Goal of management is to control symptoms arising from thrombotic and hemorrhagic complications
  • Aspirin is given to the majority of patients, to reduce the odds of vascular events and vasomotor symptoms
  • Low risk and very low risk patients (Br J Haematol 2000;110:577)
    • Should undergo an annual or semi annual visits with aspirin treatment
    • No randomized trials comparing low risk and very low risk treatment options
  • Intermediate risk and high risk patients (Br J Haematol 2000;110:577)
    • Should be treated with cytoreductive therapy (preferably hydroxyurea followed by anagrelide) or systemic anticoagulation (such as low dose aspirin)
Microscopic (histologic) description
  • Bone marrow
    • Normal / mildly hypercellular
    • Atypical megakaryocytes (Blood 2016;127:2391)
      • Increased number and large atypical forms with deeply lobulated or hyperlobulated nuclei (staghorn nuclear appearance)
      • Dispersed throughout the marrow with occasional loose clusters; dense clusters are uncommon
      • No highly bizarre / dysplastic forms, no predominance of small megakaryocytes with monolobulated nuclei
    • Delicate reticulin fibers (myelofibrosis absent / minimal) but no overt fibrosis (usually grade 1)
    • Usually normal erythroid and granulocytic proliferation
    • Usually no dyserythropoiesis, dysgranulopoiesis, macrocytosis or monocytosis
  • Features in bone marrow aspirate / biopsy suggesting an alternative diagnosis (Blood 2016;127:2391)
    • Megakaryocytes with highly atypical morphology
    • Increased myeloblasts
    • Myelodysplastic features
    • Significant (> grade 1) reticulin fibrosis or collagen fibrosis
Microscopic (histologic) images

Contributed by Hatem Kaseb, M.D., Ph.D., M.P.H.
Missing Image Missing Image

Bone marrow biopsy with atypical megakaryocytes

Missing Image Missing Image

Bone marrow aspirate with atypical megakaryocytes

Peripheral smear description
  • Normal / mild increase in the white blood cell count
  • Moderate to marked thrombocytosis
  • Platelet anisocytosis, abnormally large platelets and increased platelets; bizarre shapes, pseudopods and agranular forms
Peripheral smear images

Contributed by Hatem Kaseb, M.D., Ph.D., M.P.H.
Missing Image

Anisocytosis and bizarre forms

Flow cytometry description
  • Normal number and phenotype myeloblasts with normal myeloid scatter by CD45 / SSC
  • Normal CD10 / CD13 / CD16 / CD11b myeloid maturation pattern and all other myeloid markers are normally expressed; hence, there is no immunophenotypic evidence of myelodysplasia
  • No evidence for monoclonal B cell lymphoproliferative disease
  • Although no unusual phenotype T cells are identified, surface markers will not routinely detect monoclonal T lymphocytes
Flow cytometry images

Contributed by Hatem Kaseb, M.D., Ph.D., M.P.H.
Missing Image

No myelodysplasia

Molecular / cytogenetics description
  • Phenotypic driver mutation
    • JAK2 (present in 50 - 60% of cases), CALR (30%) or MPL (3%), ~12% of cases are triple negative for these mutations
    • Mutations are specific for essential thrombocythemia but their presence excludes reactive thrombocytosis
  • Abnormal karyotype is found in 5 - 10% of cases
    • No consistent abnormality
    • Reported abnormalities include gain of chromosome 8, abnormalities in 9q and del(20q) and unbalanced translocations between 1q and 7p
  • JAK2 mutant allele burden contributes to clinical phenotype (Haematologica 2009;94:144)
  • Subset of triple negative cases have been found to have
    • Gain of function mutations (e.g., MPL S204P and MPL Y591N) through whole exome sequencing or other molecular techniques
    • W515L and W515K mutations in thrombopoietin receptor c-Mpl (J Transl Med 2006;4:41)
  • Must rule out BCR::ABL1 fusion gene status
  • Children usually do not have JAK2 mutation
Sample pathology report
  • Bone marrow, biopsy and aspirate:
    • Myeloproliferative neoplasm (see comment)
    • Comment: The patient's positive JAK2 mutation is noted. The morphologic findings are compatible with a myeloproliferative neoplasm and in conjunction with the thrombocytosis, essential thrombocythemia is favored. Correlation with clinical, cytogenetic and laboratory findings is recommended.
    • CBC (4/29/2019): HGB: 12.7 g/dL; MCV: 89.7 fL; WBC: 9.7 K/uL; PLT: 735 K/uL
    • Bone marrow biopsy microscopic description:
      • Slightly hypercellular marrow for age (30% cellular). Megakaryocytes are increased in number, occasionally in clusters and show frequent atypical forms, including large and hyperlobated forms. The myeloid:erythroid (M:E) ratio is normal. Erythroid elements exhibit maturation. Myeloid elements exhibit maturation. Prominent lymphoid aggregates are not seen. Granulomas are not seen. Reticulin stain reveals mild and patchy reticulin fibrosis. Trabecular bone is unremarkable. The clot section is mainly blood and shows small fragments with findings similar to the core biopsy. CD34 highlights blasts comprising < 5% of the marrow cellularity.
    • Bone marrow aspirate microscopic description:
      • The marrow aspirate smears are spicular and cellular. Megakaryocytes are increased. The M:E ratio is approximately 1:1. Erythroid maturation is seen. Myeloid maturation is seen. Prussian blue iron stain shows decreased iron storage and no significant increase in ring sideroblasts.
      • Aspirate cell count: A 226 cell count reveals 2% blasts, 9% promyelocytes / myelocytes, 34% maturing granulocyte forms, 38% erythroid forms, 8% lymphocytes, 5% eosinophils, 2% plasma cells, 0% basophils / mast cells and 0% monocytes.
Differential diagnosis
  • Other myeloproliferative neoplasms:
    • Chronic myeloid leukemia:
      • BCR::ABL translocation is diagnostic
    • Polycythemia vera:
      • Most cases harbor a clonal JAK2 mutation
      • Increase in erythroid / granulocytic population should raise suspicion of polycythemia vera; polycythemia vera patients also present with decreased erythropoietin (EPO)
      • Polycythemia vera patients usually present with some symptoms related to hyperviscosity such as headaches or visual disturbance; some patients may present with pruritus exacerbated by hot water
    • Primary myelofibrosis and prefibrotic myelofibrosis:
      • Increased bone marrow reticulin fibrosis, markedly bizarre megakaryocytes and a leukoerythroblastic peripheral blood smear favor myelofibrosis
    • Prefibrotic myelofibrosis:
      • Will show features of primary myelofibrosis
      • The main feature that can help differentiate this entity from essential thrombocythemia is the presence of reticulin fibrosis in pre-PMF
    • Myelodysplastic syndrome:
      • Dysplastic features and genetics such as 5q loss
    • Familial essential thrombocythemia:
      • Autosomal dominant disease
      • Genetic analysis will show mutations in thrombopoietin, TPO receptor (c-MPL) or others related to thrombopoiesis
    • Reactive thrombocythemia:
      • Causes include exercise, allergic reaction, reaction to medications, inflammatory disorders, asplenism, infection, connective tissue disorders, metastatic cancer, lymphoproliferative disorders and iron deficiency
    • Spurious thrombocytosis:
      • Due to the miscount of structures that are not platelets (e.g., cryoglobulin crystals, cytoplasmic fragments of circulating leukemic cells and bacteria) as platelets leading to thrombocytosis (Thrombosis 2011;2011:536062)
Board review style question #1

What is the prevalence of genetic mutation in essential thrombocythemia?

  1. CAL > JAK2 > MPL > triple negative
  2. JAK2 > CAL > MPL > triple negative (JAK2, CAL and RET)
  3. JAK2 > CAL > triple negative > MPL
  4. JAK2 > MPL > CAL > triple negative
Board review style answer #1
C. JAK2 > CAL > triple negative > MPL. Essential thrombocythemia cases harbor a phenotypic driver mutation in JAK2 (50 - 60% of cases), CALR (30%) or MPL (3%) and ~12% of cases are triple negative for these mutations.

Comment Here

Reference: Essential thrombocythemia
Board review style question #2
Which of the following is a characteristic of essential thrombocythemia?

  1. Abnormal karyotype
  2. JAK2 mutation
  3. Leukoerythroblastic reaction
  4. Positive BCR::ABL translocation
Board review style answer #2
B. JAK2 mutation. Most cases of essential thrombocythemia show JAK2 mutation is present in up to 60% of cases and normal karyotype. Negative BCR::ABL excludes chronic myeloid leukemia and is one of the diagnostic criteria of essential thrombocythemia. Leukoerythroblastic reaction favors primary myelofibrosis.

Comment Here

Reference: Essential thrombocythemia

FGFR1 rearrangement
Definition / general
  • Pluripotent, hematopoietic, stem cell driven malignancy that may present as myeloproliferative neoplasm (MPN), acute myeloid leukemia (AML), T or B lymphoblastic lymphoma leukemia (T-ALL, B-ALL) or mixed phenotypic acute leukemia (MPAL)
Essential features
  • MPN or myelodysplastic syndrome / myeloproliferative neoplasm (MDS / MPN) with prominent eosinophilia, with or without lymphocytosis or neutrophilia
  • Alternatively, AML or T-ALL or B-ALL or MPAL with eosinophilia in peripheral blood or bone marrow
  • Presence of t(8:13)(p11.2;q12) or a variant of translocation leading to FGFR rearrangement in a myeloid progenitor, a lymphoblast or both
Terminology
  • Hematopoietic stem cell neoplasm with FGFR1 abnormalities
  • Myeloid and lymphoid neoplasm with FGFR1 abnormalities
  • 8p11 stem cell leukemia / lymphoma syndrome
  • 8p11 myeloproliferative syndrome
  • 8p11 stem cell syndrome
ICD coding
  • ICD-10: D47.7 - other specified neoplasms of uncertain or unknown behavior of lymphoid, hematopoietic and related tissue
Epidemiology
Sites
  • Bone marrow, peripheral blood, liver, spleen and lymph nodes
Pathophysiology
  • FGFR1 gene rearrangement affects the pluripotent stem cells, which could differentiate to myeloid or lymphoid neoplasm
  • Most cases present with chronic phase then may transform to acute blast phase; however, some cases may represent as de novo acute proliferation
Etiology
  • Etiologic factors have not been identified
Diagrams / tables

Cytogenetics and molecular genetics reported in myeloid / lymphoid neoplasms with FGFR1 rearrangement
Cytogenetics
Molecular genetics
t(8;13)(p11.2;q12.1) ZMYM2-FGFR1, previously ZNF198-FGFR1
t(8;9)(p11.2;q33.2) CNTRL-FGFR1, previously CEP110-FGFR1
t(6;8)(q27;p11.2) FGFR10P-FGFR1
t(8;22)(p11.2;q11.2) BCR-FGFR1
t(7;8)(q33;p11.2) TRIM24-FGFR1
t(8;17)(p11.2;q11.2) MY018A-FGFR1
t(8;19)(p11.2;q13.3) HERVK-FGFR1
ins(12;8)(p11.2;p11.2;p22) FGFR10P2-FGFR1
t(1;8)(q31.1;p11.2) TPR-FGFR1
t(2;8)(q13;p11.2) RANBP2-FGFR1
t(2;8)(q37.3;p11.2) LRRFIP1-FGFR1
t(7;8)(q22.1;p11.2) CUX1-FGFR1
t(8;12)(p11.2;q15) CPSF6-FGFR1

  • FGFR1 rearrangement has also been found in association with t(8;12)(p11.2;q15) and t(8;17)(p11.2;q25) but suspected involvement of FGFR1 in t(8;11)(p11.1-p11.2;p15) was not confirmed
  • Additional cytogenetic abnormalities have been reported, of which trisomy 21 is the most common secondary chromosomal abnormality
Clinical features
  • Lymphadenopathy, mediastinal mass or organomegaly
  • Myeloproliferative features
  • AML-like features or myeloid sarcoma
  • Systemic symptoms (e.g., fever, weight loss and night sweats)
Diagnosis
  • Cases usually present as chronic eosinophilic leukemia (CEL), atypical chronic myeloid leukemia (aCML) or chronic myelomonocytic leukemia (CMML), the latter are usually with eosinophilia
  • Overall, ~90% of chronic phase patients have peripheral blood or bone marrow clonal eosinophilia (including those with acute transformation), neutrophilia and occasionally monocytosis
  • De novo AML or de novo T-ALL with or without eosinophilia
  • Some cases show subsequent transformation to T-ALL, B-ALL, MPAL or AML (including myeloid sarcoma)
  • Basophilia is not typical but may be seen in cases with BCR-FGFR1 fusion, t(1;8)(q31.1;p11.2) and TPR-FGFR, t(1;8)(q25;p11.2)
  • Polycythemia vera has been reported in 5 cases with t(6;8) / FGFR10P-FGFR1 fusion (Hematol J 2004;5:534)
  • Some patients primarily present as B-ALL or MPAL with t(8;13)
  • T-ALL characteristically shows eosinophilic infiltration within the lymphoma, which can point toward this diagnosis
Laboratory
  • Leukocytosis and sometimes anemia or thrombocytopenia
  • Often presenting with eosinophilia but may show normal eosinophil count
Prognostic factors
  • Poor prognosis, even for patients presenting in the chronic phase due to the high incidence of transformation
Case reports
  • 34 year old woman with no subjective symptoms diagnosed with T lymphoblastic lymphoma (LBL) with ZNF198-FGFR1 or ZMYM2-FGFR1; bone marrow (BM) aspirate and biopsy showed hypercellularity with hypereosinophilia (Hematology 2018;23:470)
  • 48 year old Japanese man with peripheral leukocytosis, diagnosed with B-ALL with BCR-FGFR1 gene rearrangement (Bone Marrow Transplant 2019;54:326)
  • 66 year old man had clinical features that resembled CML; however, bone marrow cytogenetic and fluorescent in situ hybridization (FISH) studies showed t(8;22)(p11;q11) and BCR-FGFR1 fusion gene (Case Rep Hematol 2018;2018:5724960)
Treatment
  • In several cases, interferon has induced a cytogenetic response
  • Intensive chemotherapy with regimens such as hyper-CVAD for T or B cell lymphoblastic lymphoma, followed by early allogeneic transplantation, is recommended
  • Hematopoietic stem cell transplantation should be considered, even for patients who present in the chronic phase
  • Optional targeted therapy:
Microscopic (histologic) description
  • B-ALL with FGFR1 rearrangement:
    • Peripheral blood smear (PBS): shows left shifted granulocytosis with eosinophilia
    • Bone marrow biopsy:
      • Low power: hypercellular marrow (75%) with involvement by myeloid / lymphoid neoplasm
      • High power: abnormal immature B cell proliferation indicates B lymphoblastic leukemia / lymphoma
    • Aspirate smear:
      • Granulocytic hyperplasia with increased eosinophils (10%)
      • Immature lymphoid cells are increased, approximately 15% (hemodiluted aspirate)
      • Megakaryocytes are present and show normal morphology
      • Erythroid precursors show normoblastic maturation
  • AML with FGFR1 rearrangement:
    • PBS: leukocytosis with increased blast count (> 20%), with eosinophilia (20%) and normocytic anemia with thrombocytopenia
    • Bone marrow biopsy:
      • Hypercellular marrow (80%) with involvement by sheets of blasts > 40%, which stains positive for CD34 and MPO
      • Reticulin fibrosis is mildly increased (1/3)
    • Aspirate smear: hypercellularity and diffuse infiltration by myeloblasts
  • Myeloproliferative neoplasms (MPN) with FGFR1 rearrangement:
    • PBS: leukocytosis with eosinophilia and normocytic anemia with thrombocytopenia
    • Bone marrow biopsy: hypercellular marrow (80%) with left shifted maturation of the granulocyte lineage with increased eosinophils (which may be seen in clusters)
    • Aspirate smear: hypercellularity with left shifted maturation of the granulocytes and eosinophilia
Microscopic (histologic) images

Contributed by Ling Zhang, M.D.

AML with FGFR1 rearrangement

B-ALL with FGFR1 rearrangement


AML with FGFR1 rearrangement

B-ALL with FGFR1 rearrangement


Reticulin stain

TdT positive stain

CD34 positive stain

Peripheral smear description
  • B-ALL with FGFR1 rearrangement:
    • Left shifted leukocytosis with eosinophilia; normocytic normochromic red cell morphology and adequate reticulocytosis
    • Anemia or thrombocytopenia is present, occasional monocytosis
  • AML with FGFR1 rearrangement:
    • Normocytic normochromic red cell morphology with adequate reticulocytosis; left shifted leukocytosis with increased blast cells and eosinophilia
Peripheral smear images

Contributed by Ling Zhang, M.D.

Myeloblast with eosinophils

Positive stains
  • TdT: positive in lymphoblasts in cases of B-ALL or T-ALL with FGFR1 rearrangement
  • CD34: highlights the blast cells
Molecular / cytogenetics description
  • FISH test with FGFR1 break apart probe: 1 red, 1 green and 1 fusion signal pattern indicates positive FGFR1 gene rearrangement detected in 72.5% of cells
Molecular / cytogenetics images

Contributed by Ling Zhang, M.D.

FISH for FGFR1 rearrangement



Images hosted on other servers:

FISH assay of FIM (also called ZNF198) and FGFR1

Sample pathology report
  • Peripheral blood:
    • Left shifted granulocytosis with eosinophilia
  • Bone marrow, right posterior iliac crest, aspirate and biopsy:
    • Hypercellular marrow (75%) with involvement by myeloid / lymphoid neoplasm with FGFR1 rearrangement (see comment)
    • Decreased stainable storage iron
    • Comment: The concurrent flow cytometry identified immature abnormal B cell population. Immunohistochemical stains are performed on the current bone marrow core biopsy shows 30 - 40% PAX5+, TdT+, CD34+ subset immature B cells. FISH study reveals 64.5% cells with FGFR1 rearrangement. The patient's previous outside bone marrow biopsy showed abnormal karyotype with t(8:13)(p11.2;q12.1).
    • The abnormal immature B cell proliferation indicates B lymphoblastic leukemia / lymphoma. With the clinical presentation of left shifted leukocytosis, monocytosis, eosinophilia and FGFR1 rearrangement, the findings are consistent with a myeloid / lymphoid neoplasm with FGFR1 rearrangement. Please correlate with pending FoundationOne Heme analysis, reported separately.
    • Microscopic description:
      • Peripheral blood smear:
        • Cytopenias: not present
        • Red cell morphology: normocytic normochromic
        • Reticulocytosis: adequate
        • Other findings: left shifted leukocytosis with monocytosis and eosinophilia
      • Aspirate smear:
        • Adequacy: adequate
        • Significant findings: There is granulocytic hyperplasia with increased eosinophils (20%). Immature lymphoid cells are increased, approximately 15%. Megakaryocytes show normal morphology. Erythroid precursors show normal maturation.
      • Clot section and core biopsy:
        • The clot section (A) and the core biopsy (B) are flaccid in B plus fix (BBC) fixative. The core biopsy (B) is decalcified and formical (formic acid, formalin diet and EDTA). On level of the clot section (A) and core biopsy (B) are stained with PAS to evaluate myeloid maturation and more accurately estimate and M:E ratio. One level of the clot section (A) and core biopsy (B) is each stained with Prussian blue to evaluate iron stores and the average value reported. One level of the colon biopsy (B) is stained with reticulin stain to evaluate marrow fibrosis.
      • Bony trabeculae: unremarkable
      • Cellularity: 75%
      • Megakaryocytes: adequate in number with normal morphology
      • Estimated M:E ratio: 3 - 4:1
      • Iron stores: decreased
      • Reticulin fibrosis: normal
      • Significant findings: hypercellular marrow with increased eosinophils and increased immature mononuclear cells
      • Immunohistochemical stains are performed on the core biopsy with adequate controls. PAX5 and TdT highlight markedly increased to scattered and aggregates of immature B cells, overall 30 - 40%. Majority of this immature B cells are also positive for CD34. Spectrin stains mildly decreased erythroid precursor. CD61 highlights megakaryocytes.
Differential diagnosis
  • Hypereosinophilic syndrome:
    • Sustained eosinophilia (> 1.5 x 109/L) for at least 6 months with other reactive processes excluded with induced tissue damage, other reactive processes excluded
    • Exclude cytokine related lymphoid variant hypereosinophilia
    • Increased eosinophils with either no dysplastic features or no increase in blast cells in the bone marrow
    • No lymphoid or myeloid neoplasms (e.g., MDS / MPNs, MPNs, systemic mastocytosis or AML)
    • No cytogenetic or molecular alteration is identified
    • No FGFR, PDGFRA, PDGFRB rearrangement, PCM1-JAK2, BCR-ABL1, ETV6-JAK2 or BCR-JAK2 fusions
    • No inv(16)(p13.1q22), t(16;16)(p13.1q22) or t(8;21)(q22;q22.1)
  • Reactive eosinophilia:
    • Occurs secondary to allergies, drugs, infection, skin diseases, asthma, adrenal insufficiency, gastrointestinal eosinophilia
    • Peripheral blood shows increased eosinophils (may be with abnormal forms)
    • No evidence of FGFR1 rearrangement or active mutation or other cytogenetic / molecular abnormalities
  • Systemic macrocytosis with eosinophilia:
    • Major diagnostic criteria: multifocal infiltrates of mast cells (≥ 15 mast cells in aggregates) + 1 minor criterion or 4 minor criteria
    • Minor criteria:
      • > 25% of mast cells with atypical morphology (e.g., spindle shape or immature nuclear features)
      • Tryptase levels are usually more than 20 ng/dL
      • CD2 or CD25 positive on mast cells
      • Presence of KIT D816V
    • Bone marrow shows eosinophilia with variable dysplastic features
    • No FGFR1 rearrangement
  • Chronic eosinophilic leukemia, NOS:
    • Sustained eosinophilia (> 1.5 x 109/L) with other reactive processes excluded
    • < 20% blasts in peripheral blood or bone marrow
    • No inv(16)(p13.1q22), t(16;16)(p13.1q22), t(8;21)(q22;q22.1)
    • Bone marrow aspirate is hypercellular with abundant eosinophils and eosinophilic precursors with dysplastic features
    • Clonal cytogenetic (e.g., +8, -20q) or molecular genetic abnormalities (e.g., TET2, ASXL1 and DNMT3A); if no clonal abnormalities are found, then blast count must be > 2% in peripheral blood or > 5% in bone marrow aspirate
    • No PDGFRA, PDGFRB, FGFR rearrangement, BCR-ABL1, PCM1-JAK2, ETV6-JAK2 or BCR-JAK2 fusions
    • No lymphoid or myeloid neoplasms (e.g., MDS / MPN, MPNs, systemic mastocytosis or AML) are identified
  • Idiopathic hypereosinophilia:
    • Sustained eosinophilia (> 1.5 x 109/L) for at least 6 months with no tissue damages
    • No reactive causes, lymphoid or myeloid neoplasms are identified
    • Increased eosinophils with no dysplastic features or no increase in blasts in the bone marrow
    • No molecular alteration is identified and no PDGFRA, PDGFRB, FGFR rearrangement, BCR-ABL1, PCM1-JAK2, ETV6-JAK2 or BCR-JAK2 fusions
Board review style question #1
What is the most common partner for FGFR1 in myeloid / lymphoid neoplasms with FGFR1 rearrangement?

  1. BCR
  2. CNTRL
  3. FGFR10P
  4. ZMYM2
Board review style answer #1
D. ZMYM2-FGFR1 / t(8;13)

Comment Here

Reference: FGFR1 rearrangement
Board review style question #2
Which of the following molecular aberration is associated with basophilia in myeloid / lymphoid neoplasms with FGFR1 rearrangement?

  1. BCR-FGFR1 fusion, t(1;8)(q31.1;p11.2)
  2. CPSF6-FGFR1 fusion, t(8;12)(p11.2;q15)
  3. CUX1-FGFR1 fusion, t(7;8)(q22.1;p11.2)
  4. FGFR10P-FGFR1 fusion, t(6;8)
Board review style answer #2
A. BCR-FGFR1 fusion, t(1;8)(q31.1;p11.2)

Comment Here

Reference: FGFR1 rearrangement

FLT3 rearrangement (pending)
[Pending]

Gaucher disease
Definition / general
  • Gaucher disease (GD) is an autosomal recessive lysosomal storage disorder caused by mutation of the GBA1 gene that codes for glucocerebrosidase (GCase)
  • Impaired activity of GCase causes its substrate, glucocerebroside, to accumulate in lysosomes of reticuloendothelial system (liver, spleen, bone marrow)
  • Macrophages (Gaucher cells) become laden with lipid and dysfunctional
Essential features
  • One of the most common lysosomal storage disorders with a predisposition among those of Ashkenazi Jewish descent
  • Caused by mutation in GBA1 gene on chromosome 1q that codes for GCase enzyme
  • Impaired enzymatic activity leads to accumulation of Gaucher cells in reticuloendothelial system (liver, spleen, bone marrow), which leads to hepatosplenomegaly and bone marrow infiltration with cytopenia and bone pain
  • Diagnosis is made by assessing GCase activity
  • Treatment includes enzyme replacement therapy (GCase infusions) and substrate reduction therapy
Terminology
  • Gaucher disease (pronounced as GO-SHEY)
ICD coding
  • ICD-10: E75.22 - Gaucher disease
Epidemiology
  • First described in 1882 in a patient with massive splenomegaly (Int J Mol Sci 2017;18:441)
  • Prevalence is highest among the Ashkenazi Jewish population (Eastern European) with a frequency of 6% compared to 0.7 - 0.8% of all other ethnic groups (Hematology 2017;22:65)
Sites
  • Liver
    • Hepatomegaly with or without elevated liver enzymes (alkaline phosphatase, aminotransferase) due to intracellular accumulation of Gaucher cells and secondary inflammatory response
    • Concurrent chronic liver diseases such as chronic viral hepatitis, steatohepatitis and vascular diseases are often seen
    • Liver fibrosis (20 - 50% of cases) and cirrhosis with portal hypertension due to accumulation of Gaucher cells inciting an inflammatory response with fibrosis causing ischemia, infarction and progression of liver fibrosis (Mol Genet Metab Rep 2020;22:100564)
    • Focal liver lesions also known as Gaucheroma (benign cluster of Gaucher cells associated with inflammation, fibrosis and iron accumulation)
    • Increased risk of chronic liver disease and hepatocellular carcinoma due to cirrhosis
    • Hypergammaglobulinemia and hyperferritinemia with probable underlying factor include chronic inflammation, impaired macrophage function and dysregulation in hepcidin - ferroportin axis
  • Spleen
    • Splenomegaly with focal splenic lesions characterized by clusters of Gaucher cells involving mostly the splenic cords in the red pulp
    • Extramedullary hematopoiesis
  • Bone marrow
    • Infiltration of bone marrow by Gaucher cells causes reduced hematopoiesis and cytopenia
    • Reduced proportion of marrow fat
  • Bone
    • Skeletal remodeling is also affected by replacement of marrow adipocytes with Gaucher cells; this causes fractures, remodeling defects, loss of bone mineral density and bone infarction
    • Renal involvement is rare but proteinuria with or without renal insufficiency can occur
  • Reference: Am J Hematol 2015;90:S6
Pathophysiology
  • Mutation in GBA1 gene causes decrease in GCase activity with accumulation of glucocerebroside within activated macrophages (Gaucher cells) that infiltrate liver, spleen, bone marrow and other organs, leading to organomegaly
  • Gaucher cells release cytokines and chemokines (IL1 beta, IL6, TNF alpha, IL10, IL18 and CCL18)
  • Accumulated glucocerebroside is metabolized into sphingosine that can cause direct damage to hepatocytes, hematopoietic cells and the CNS
  • GGase deficiency causes reduced osteoblast differentiation and function, reduced bone mineral deposition, defective bone remodeling caused by decreased bone formation leading to bone weakening, cortical thinning and trabecular resorption
  • Infiltration of marrow by Gaucher cells leads to change in medullary microenvironment and compression of intraosseous blood vessels with potential bone infarction and necrosis
  • References: Int J Mol Sci 2017;18:441, Am J Hematol 2015;90:S6
Etiology
  • Autosomal recessive disease caused by mutation in gene GBA1 that codes for GCase causing impaired function and excessive accumulation of glucocerebroside in the reticuloendothelial system (liver, spleen, bone marrow)
  • GD patients inherit 2 copies of mutated GBA1 (homozygous) from parents who are carriers
  • Reference: Am J Hematol 2015;90:S6
Diagrams / tables

Images hosted on other servers:

Glucosylceramide metabolic pathway

Clinical features
  • Splenomegaly is often the initial presentation
  • Primary hypersplenism with anemia and thrombocytopenia, leukopenia also occurs frequently
  • Hepatomegaly occurs late in the disease course with an increase in liver enzymes
  • Bone marrow involvement / infiltration leading to anemia, thrombocytopenia, plasma cell dyscrasia
  • Skeletal involvement in the form of osteopenia, osteoporosis, defective bone remodeling, osteonecrosis
  • Pulmonary involvement in the form of interstitial lung disease
  • Central nervous system (CNS) involvement
    • Impaired cognition (type I GD)
    • Convulsions, intellectual disability, apnea (type II GD)
    • Myoclonus, seizures, dementia (type III GD)
  • Categorized into 3 main types
  • Associated with increased risk of multiple myeloma (relative risk: 5.9) (Blood 2005;105:4569)
  • References: Expert Rev Endocrinol Metab 2018;13:107, Hematology 2017;22:65, J Bone Miner Res 2019;34:996
Diagnosis
  • Diagnosis is made by assessing GCase activity in circulating lymphocytes or fibroblast culture; < 10 - 15% of mean normal activity is diagnostic
  • Disease activity or response to therapy is evaluated by measuring biochemical markers in activated macrophages (e.g., chitotriosidase, angiotensin converting enzyme, ferritin, chemokine CCL18 / PARC)
  • Genotype testing in Ashkenazi Jewish patients for the most common alleles (L444P, N370S, IVS2+1g>a, V394L and R496H)
  • References: Int J Mol Sci 2017;18:441, Am J Hematol 2015;90:S6
Laboratory
  • Anemia
  • Thrombocytopenia due to hypersplenism
  • Hyperferritinemia
  • Hypergammaglobulinemia
  • GCase enzyme activity assay
  • Serum protein electrophoresis
  • Bone marrow
    • Large (30 - 100 micron) macrophages with fibrillary pale blue to grey cytoplasm
  • Liver biopsy
    • Performed for hepatomegaly and shows clusters of Gaucher cells (macrophages with amphophilic cytoplasm resembling wrinkled tissue paper)
  • Reference: Mol Genet Metab Rep 2020;22:100564
Radiology description
  • Radiographic / Xray findings
    • Erlenmeyer flask deformity (enlargement of metaphysis and absence of the typical concave diametaphyseal curve) in distal femur
    • Osteonecrosis, proximal femur
    • Osteopenia
    • Lytic lesions
  • MRI performed to assess skeletal involvement
    • Osteonecrosis
    • Gaucher cell infiltration produce low signal on T1 - T2 weighted images
    • Degree of bone marrow infiltration by Gaucher cells
  • CT / ultrasound findings
    • Hepatosplenomegaly
  • Bone scan
    • Decreased uptake
  • EEG and swallowing studies for neuronopathic GD
  • Reference: J Bone Miner Res 2019;34:996
Radiology images

Images hosted on other servers:

Gaucher lytic lesions

Erlenmeyer flask deformity

Prognostic factors
  • Variable prognosis depending on clinical type of disease, ranging from asymptomatic to fatal outcome
    • Type I: normal life span
    • Type II: patients die within first 3 years of life
    • Type III: intermediate between type I and type II
  • Early diagnosis and treatment can have a good outcome
  • Reference: Mol Genet Metab 2021;132:49
Case reports
  • 4 year old Albanian boy diagnosed with GD type 1 through newborn screening presented with multiple osteonecrosis in femur on follow up (JIMD Rep 2022;63:414)
  • 15 year old boy who presented with fever, pulmonary interstitial fibrosis and hepatomegaly and was ultimately diagnosed with GD (J Med Case Rep 2018;12:306)
  • 18 year old man diagnosed with neuroblastoma, also found to have GD (Blood Cells Mol Dis 2018;68:106)
  • 24 year old woman diagnosed as carrier of GD when she presented with gestational thrombocytopenia and anemia that did not respond to any medications (J Med Case Rep 2022;16:203)
  • 38 year old man who presented with solitary swelling of proximal tibia, mimicking a musculoskeletal tumor (J Orthop Case Rep 2022;12:64)
Treatment
  • Enzyme replacement therapy, which includes glucocerebrosidase IV infusions
    • FDA approved Cerezyme (imiglucerase), VPRIV (velaglucerase alfa) and Elelyso (taliglucerase alfa) effective against hematologic, visceral and bone symptoms
    • Not known to be effective against neurologic symptoms
  • Substrate reduction therapy (oral medication) to reduce the accumulation of toxic substrate
    • Cerdelga (eliglustat), a glucosylceramide synthase inhibitor, is indicated only for type 1 GD
    • Zavesca (miglustat) for type 1 GD; not known to be effective against neurological symptoms (Ann Neurol 2008;64:514)
  • Symptomatic treatment
    • Bone remodeling: targeted treatments and vitamin D
    • Splenectomy for hypersplenism
    • Analgesics for bone pain and joint pain
    • Bone marrow transplant only in cases of neuronopathic GD
  • Somatic cell gene therapy under investigation
  • References: Expert Rev Endocrinol Metab 2018;13:107, Mol Genet Metab 2021;132:49, Eur J Pharmacol 2022;926:175023
Clinical images

Images hosted on other servers:

Perinatal collodion Gaucher (type 2)

Gross description
  • Bone
    • Mass-like lesions that are soft, hemorrhagic and irregular, resulting in extensive localized bone loss
    • Osteonecrosis of bone appears as pale yellow zones
  • Spleen
    • Splenomegaly with pale homogenous cut surface with reddish yellow nodules of infarction
  • References: Am J Hematol 2016;91:736, J Orthop Case Rep 2022;12:64
Gross images

Images hosted on other servers:

Splenomegaly in Gaucher disease

Microscopic (histologic) description
  • Bone marrow, liver, spleen
    • Infiltration by Gaucher cells containing abundant cytoplasm with fine, fibrillary, amphophilic characteristics resembling wrinkled tissue paper
    • May have increased reticulin fibers and reduced fat in bone marrow biopsy
  • Liver often demonstrates infiltration in the zone 3 region by Gaucher cells
  • Small focal accumulations (Gaucheroma) or diffuse replacement by large, ovoid histiocytes (30 - 100 microns) with abundant, fibrillary eosinophilic granular cytoplasm resembling wrinkled tissue paper
  • Small, bland nucleus that may be centrally or eccentrically located
  • Cytoplasm has periodic acid Schiff diastase (PASD) resistant granules
  • Reference: Mol Genet Metab Rep 2020;22:100564
Microscopic (histologic) images

Contributed by Patricia Tsang, M.D. and Mowafak Hamodat, M.B.Ch.B., M.Sc. (Case #164)
Gaucher in marrow biopsy

Gaucher in marrow biopsy

Fibrillary Gaucher cell cytoplasm

Fibrillary Gaucher cell cytoplasm

Gaucher cells marrow clot

Gaucher cells marrow clot

Liver biopsy

Liver biopsy

PASD positive Gaucher cells

PASD positive Gaucher cells

Marrow aspirate

Marrow aspirate

Virtual slides

Images hosted on other servers:

Granular, fibrillary blue gray cytoplasm

Cytology description
  • Wright-Giemsa stained bone marrow aspirate shows Gaucher cell containing abundant granular or fibrillary, blue-gray cytoplasm with wrinkled tissue paper-like appearance
Positive stains
Negative stains
Electron microscopy description
  • Elongated / rod shaped lysosomes filled with tubular structures
Electron microscopy images

Images hosted on other servers:

Gaucher cell cytoplasmic fibrils

Molecular / cytogenetics description
  • Gene encoding glucocerebrosidase (Gcase) GBA1 is located on the long arm of chromosome 1 (1q21) and genetic testing is performed using saliva or blood samples
  • DNA result is obtained by whole gene sequencing or selective sequencing within families
  • More than 400 different mutations have been described in GBA1 gene; most prevalent are N370s, L444P
  • c.1226A>G (N370S) mutation is common in the Ashkenazi Jewish population and most of these patients are asymptomatic
  • L444P mutation seen in GD is at high risk for developing CNS complications
  • Homozygous (D409H) mutation presents with cardiac complications
  • References: Int J Mol Sci 2017;18:441, Mol Genet Metab 2021;132:49
Videos

Gaucher disease involving bone

Sample pathology report
  • Liver, right lobe, needle core biopsy:
    • Gaucher disease involving liver (see comment)
    • Comment: Sections of the liver biopsy show clusters of Gaucher cells (pale staining histiocytes with striated cytoplasm, PASD positive) involving mainly the perivenular zone with atrophy of hepatocyte plates.
Differential diagnosis
  • Pseudo-Gaucher cells:
    • Mimic Gaucher cells on light microscopy but iron stain negative and lack the typical electron microscopy findings seen in GD (Turk J Haematol 2014;31:428)
    • Seen in conditions that cause rapid cell turnover (e.g., myeloproliferative and myelodysplastic neoplasms, myeloma)
    • Mycobacterial infection (AFB stain+)
  • Sea blue histiocytosis (Arch Pathol Lab Med 2008;132:851):
    • Seen in various conditions with increased marrow cell turnover (e.g., myeloproliferative or myelodysplastic neoplasms)
    • Macrophage contains ceroid (insoluble lipid pigment); inclusions are globular (not fibrillary)
    • Stains more intensely blue with Wright-Giemsa than Gaucher cells
  • Niemann-Pick disease (Humpath.com: Niemann-Pick Diseases [Accessed 6 June 2023]):
    • Lipid storage disorder caused by sphingomyelinase deficiency, which leads to accumulation of lipids in histiocytes that appear foamy
    • Electron microscopy shows lamellar inclusions in lysosomes
Board review style question #1

A 5 year old boy presents to the clinic for evaluation of peripheral pancytopenia. His parents are of Ashkenazi Jewish descent. On physical examination, he has hepatosplenomegaly. Bone marrow aspirate smears show large cells with fibrillary cytoplasm that are positive for periodic acid Schiff (PASD) stain. Which of the following lysosomal enzymes are most likely deficient?

  1. Alpha galactosidase
  2. Glucocerebrosidase
  3. Hexosaminidase A
  4. Sphingomyelinase
Board review style answer #1
B. Glucocerebrosidase. Gaucher disease is caused by deficiency of glucocerebrosidase due to mutation of the GBA1 gene. Glucocerebrosidase deficiency causes abnormal accumulation of its substrate, glucocerebroside, in the macrophages of liver, spleen, bone marrow, etc. Patients can present with pancytopenia, hepatosplenomegaly and bone pain with fractures. Diagnosis can be made on finding a low glucocerebrosidase enzyme level in peripheral blood leukocytes. Answer D is incorrect because sphingomyelinase A deficiency is seen in Niemann-Pick Disease. The clinical presentation may overlap with Gaucher disease; however, there is an accumulation of foamy vacuolated macrophages in Niemann-Pick disease. Answer A is incorrect because alpha galactosidase deficiency is seen in Fabry disease, which typically presents with neurological manifestations. Answer C is incorrect because hexosaminidase deficiency is seen in Tay-Sachs disease.

Comment Here

Reference: Gaucher disease
Board review style question #2
Which of the following clinical characteristics is associated with type I Gaucher disease?

  1. Gastric ulcers
  2. Hydrops fetalis
  3. Normal expected life span
  4. Oculomotor symptoms
Board review style answer #2
C. Normal expected life span. Type 1 Gaucher disease is nonneuronopathic and associated with a normal life span. Answer A is incorrect because Gaucher disease is not typically associated with gastric symptoms. Answers B and D are incorrect because hydrops fetalis is associated with type 2 Gaucher disease and oculomotor symptoms are associated with type 3.

Comment Here

Reference: Gaucher disease

Heavy chain disease
Definition / general
  • Rare B cell neoplasm characterized by production of monoclonal heavy chain unable to bind light chain (truncated or mutated)
  • Alpha heavy chain disease is the most common; it is also known as immunoproliferative small intestinal disease (IPSID), a variant of extranodal marginal zone lymphoma of mucosa associated lymphoid tissue (MALT lymphoma)
Essential features
  • 3 main types: alpha (IgA), gamma (IgG) and mu (IgM)
  • Resultant lymphoma resembles other lymphoma types with each heavy chain disease associated with distinct features
  • Must demonstrate free heavy chain by protein electrophoresis / immunofixation
  • Alpha heavy chain disease is the most common, presents in younger patients, marginal zone phenotype
  • Gamma is often associated with autoimmune disease, may be polymorphic
  • Mu may resemble chronic lymphocytic leukemia (CLL) with mix of lymphocytes and characteristic vacuolated plasma cells
Terminology
  • Alpha heavy chain is also known as immunoproliferative small intestinal disease (IPSID)
ICD coding
  • ICD-10: C88.2 - heavy chain disease
Epidemiology
  • Alpha: younger age, peak incidence in 20s and 30s, equal prevalence in men and women, more common in areas neighboring Mediterranean Sea, most prevalent in Africa and the Middle East, associated with factors linked to low socioeconomic status (poor hygiene, malnutrition, frequent intestinal infections)
  • Gamma: rare, ~150 cases reported, median age 60, female predominance
  • Mu: extremely rare, 30 - 40 cases reported, median age 60, equal prevalence in men and women
Sites
  • Alpha: mainly small intestine, mesenteric lymph nodes; gastric and colonic mucosa may be involved, rarely respiratory tract and thyroid; usually not bone marrow or other organs
  • Gamma: lymph nodes or extranodal, Waldeyer ring, gastrointestinal tract, bone marrow, liver, spleen, peripheral blood, skin, subcutaneous tissue
  • Mu: spleen, liver, bone marrow, blood, usually not lymphadenopathy
Pathophysiology
  • All: deletions in heavy chain gene result in defective heavy chain protein that cannot bind to light chain to form complete Ig molecule
Etiology
  • Alpha: chronic intestinal infection (e.g. Campylobacter jejuni) may underlie development of the disease
Clinical features
  • Alpha: typically presents with malabsorption, diarrhea, hypocalcemia, abdominal pain, wasting, fever; patients may die from malnutrition, intestinal obstruction, sepsis or other complications due to massive bowel involvement
  • Gamma: typically associated with systemic symptoms (anorexia, weakness, fever, weight loss, bacterial infections), concurrent autoimmune disease, generalized disease (hepatomegaly, splenomegaly, lymphadenopathy); rarely transforms to large cell lymphoma; disease patterns range from indolent to aggressive
  • Mu: may resemble chronic lymphocytic leukemia (CLL) but differs in the high frequency of hepatosplenomegaly and absence of peripheral lymphadenopathy; has mixture of plasma cells and B lymphocytes with monoclonal cytoplasmic mu heavy chain; similar slow progression


Diagnosis
  • Combination of histopathologic features and detection of serum heavy chain by immunofixation
Laboratory
  • Variably sized heavy chains may be produced, which may NOT produce characteristic monoclonal peak on serum protein electrophoresis
  • Immunofixation may be critical for detection
  • Gamma: may detect IgG without light chain in blood or urine
  • Mu: mu heavy chain not detected in urine but Bence Jones light chains may still be detected (especially kappa) (Am J Hematol 1992;40:56)
Prognostic factors
  • Alpha: early stages may respond to antibiotic therapy; more advanced disease requires multiagent chemotherapy
Case reports
Treatment
  • Alpha: early disease may respond to antibiotic therapy; for more advanced immunoproliferative small intestinal disease, anthracycline containing regimens have been reported to result in remission and longterm survival
Microscopic (histologic) description
  • Alpha: lamina propria of bowel mucosa heavily infiltrated with plasma cells and admixed lymphocytes which separate the crypts; may have marginal zone B cells, lymphoepithelial lesions and villous atrophy; progression to diffuse large B cell lymphoma may be characterized by destructive sheets of large plasmacytoid cells, immunoblasts and ulceration
  • Gamma: circulating plasma cells, may resemble splenic marginal zone or MALT lymphoma; polymorphous proliferation with lymphocytes, plasma cells, immunoblasts, histiocytes, eosinophils
  • Mu: bone marrow contains vacuolated plasma cells admixed with small lymphocytes

Microscopic (histologic) images

Contributed by Genevieve M. Crane, M.D., Ph.D.

Lamina propria is diffusely expanded by a plasma cell rich infiltrate

CD3 highlights scattered infiltrating T cells

CD20 highlights rare B cell nodules


CD79a stains the majority of the infiltrate

CD138 highlights plasma cells and epithelium

Plasma cells are predominantly positive for cytoplasmic IgA



Images hosted on other servers:

Characteristic
intracytoplasmic
vacuoles

Positive stains
  • Nonspecific B and plasma cell markers are positive
  • Alpha: CD20 on marginal zone cells, CD138 on plasma cells
  • Gamma: CD79a, cytoplasmic gamma chain; CD20 on lymphocytic component, CD138 on plasma cells
  • Mu: cells contain monoclonal heavy chain, B cell antigens

Negative stains
  • All forms are usually negative for CD5, CD10
Molecular / cytogenetics description
  • All have clonally rearranged IGH
  • Deletions / insertions in the IGHV or CH1 domain result in defective or truncated heavy chain that cannot bind light chain to form a complete immunoglobulin molecule
  • Alpha: case reports of cytogenetic abnormalities but not the t(11;18)(q21;q21) (BIRC3 / MALT1) translocation associated with gastric and pulmonary MALT lymphoma
  • Gamma: abnormal karyotype in 50% of cases, MYD88 mutation is absent
  • Mu: immunoglobulin genes show high levels of somatic mutation
Differential diagnosis
May resemble other lymphoma subtypes:
Board review style question #1
The presence of a mixed population of vacuolated plasma cells and lymphocytes in the bone marrow is most characteristic of which process?

  1. Lymphoplasmacytic lymphoma
  2. MALT variant immunoproliferative small intestinal disease
  3. Mu heavy chain disease
  4. Myeloma with a t(11;14) translocation
  5. Splenic marginal zone lymphoma
Board review style answer #1
C. It is characteristic of mu heavy chain disease, a very rare disease.

Comment Here

Reference: Heavy chain disease
Board review style question #2
Which heavy chain disease is most likely to be associated with underlying autoimmune disease?

  1. Alpha heavy chain disease
  2. Gamma heavy chain disease
  3. Mu heavy chain disease
  4. None of the above
Board review style answer #2
B. Gamma heavy chain disease.

Comment Here

Reference: Heavy chain disease

How to do a bone marrow biopsy (procedure) (pending)
[Pending]

Idiopathic hypereosinophilic syndrome
Definition / general
  • Idiopathic hypereosinophilic syndrome is a disorder defined by peripheral blood eosinophilia (absolute eosinophil count ≥ 1.5 x 109/L) for at least 6 months with organ damage / dysfunction attributable to tissue hypereosinophilic infiltrate per biopsy and no discernible underlying etiology
  • Diagnosis of exclusion
Essential features
  • Hypereosinophilia (HE) is defined by an increase in eosinophils in the peripheral blood (≥ 1.5 x 109/L); however, if eosinophilia is sustained for ≥ 6 months and there is associated tissue damage, this should be classified as hypereosinophilic syndrome (HES)
  • Clinically, if treatment is necessary to minimize tissue / organ damage, the criteria of 6 months might not be enforced and an eosinophil count ≥ 1.5 x 109/L on 2 occasions for ≥ 1 month apart may be sufficient (Pathobiology 2019;86:39)
  • Idiopathic hypereosinophilia diagnostic criteria
    • Eosinophil count ≥ 1.5 x 109/L for ≥ 6 months
    • Tissue or organ damage associated with eosinophilic infiltration
      • Cardiac and cutaneous manifestations are common but liver, CNS, muscle, pulmonary and nasal sinus involvement may occur as well
    • No discernible underlying cause
      • Exclude reactive causes of eosinophilia (allergy, infection including parasites, drug related, connective tissue disorder, etc.)
      • Exclude acute myeloid leukemia, myeloproliferative neoplasms, myelodysplastic syndromes, myelodysplastic / myeloproliferative neoplasms, systemic mastocytosis and myeloid / lymphoid neoplasms with eosinophilia and PDGFRA, PDGFRB, FGFR1 or PCM1-JAK2 rearrangement
      • Exclude cytokine producing, immunophenotypically aberrant T cell population
    • No cytogenetic or molecular abnormalities, however, must exclude clonal hematopoiesis (CHIP) mutations
Terminology
  • Idiopathic HES
ICD coding
  • ICD-10: D72.110 - idiopathic hypereosinophilic syndrome (IHES)
Epidemiology
Sites
  • Skin and cardiac involvement common
  • Minority of patients present with splenomegaly and lymphadenopathy
  • Liver, CNS, muscle, pulmonary and nasal sinus involvement occur more commonly than in eosinophilic leukemia (Haematologica 2014;99:e148)
Etiology
  • Cause is unknown
Clinical features
  • Onset of symptoms is often insidious with eosinophilia being detected incidentally; however, some patients initially present with severe and life threatening problems due to the rapid progression of cardiovascular and neurologic complications
  • Retrospective study of 188 patients with hypereosinophilia reported the frequency of symptoms at presentation (J Allergy Clin Immunol 2009;124:1319):
    • Dermatologic symptoms were most common, followed by pulmonary, gastrointestinal, cardiac and lastly neurologic
    • 6% of patients presented initially with incidentally detected asymptomatic hypereosinophilia
  • Dermatologic: angioedema, dermographism, eczema, erythroderma, pruritis, urticaria
  • Pulmonary: cough, dyspnea, wheezing
  • Gastrointestinal: weight loss, abdominal pain, vomiting, diarrhea, hepatitis, cholangitis, Budd-Chiari syndrome
  • Cardiac: cardiac damage, valvular fibrosis, thromboembolism
  • Neurologic: behavioral changes, confusion, ataxia, memory loss, peripheral neuropathy
Diagnosis
  • Unexplained eosinophilia of ≥ 1.5 x 109/L for at least 6 months that leads to tissue damage
    • Exclude reactive causes of eosinophilia (parasites, drug related, allergy, etc.)
    • Exclude acute myeloid leukemia, myeloproliferative neoplasms, myelodysplastic syndromes, myelodysplastic / myeloproliferative neoplasms and systemic mastocytosis
    • Exclude cytokine producing, immunophenotypically aberrant T cell population
    • Tissue damage present due to hypereosinophilia
  • Bone marrow with increased eosinophilic precursors; proposed criteria of ≥ 20% marrow cellularity with or without peripheral blood eosinophilia (Pathobiology 2019;86:39)
  • Affected organs may have increased eosinophils with eosinophil degranulation (Pathobiology 2019;86:39)
Laboratory
Prognostic factors
  • Slowly progressive clinical course with death occurring due to cardiac damage in some cases
Case reports
Treatment
  • Patients are typically observed during the initial stages of the disease
  • As HES progresses, initial treatment is often systemic glucocorticoid therapy with second line treatments including imatinib, interferon alfa and hydroxyurea
  • Recent study showed that mepolizumab could be effective at treating flares in HES (J Allergy Clin Immunol 2020;146:1397)
Microscopic (histologic) description
  • Bone marrow
    • Normocellular or hypercellular marrow with an increase in eosinophils and eosinophilic precursors with orderly maturation
    • Bone marrow eosinophilia proposed criteria: ≥ 20% marrow cellularity with or without peripheral blood eosinophilia (Pathobiology 2019;86:39)
    • Normal differential counts
    • No increase in blasts (< 5%)
    • No neoplastic process
  • Tissue / organ
    • Eosinophils are typically scattered or absent in normal tissue; affected organs may have increased eosinophils with eosinophil degranulation (Pathobiology 2019;86:39)
    • Possible eosinophilic microabscesses
    • Organs affected include gastrointestinal tract, lung, lymph nodes, spleen and thymus
Microscopic (histologic) images

AFIP images

Bone marrow aspirate

Peripheral smear description
Peripheral smear images

AFIP images

40% eosinophils with normal morphology

Flow cytometry description
Molecular / cytogenetics description
  • No cytogenetic or molecular genetic abnormalities identified by definition (Jaffe: Hematopathology, 2nd Edition, 2016)
  • Must exclude neoplasms with rearrangements of BCR-ABL1, PDGFRA, PDGRB or FGFR1 or PCM1-JAK2, ETV6-JAK2, BCR-JAK2 or FLT3 fusions as well as activating PDGFRA gene mutation
Sample pathology report
  • Bone marrow, posterior iliac crest, core biopsy, clot section, aspirate smears and touch imprint:
    • Hypereosinophilic syndrome (see comment)
    • Hypercellular (80 - 90%) bone marrow with myeloid hyperplasia with markedly increased eosinophils and no overt increase in blasts (< 5%)
    • No evidence of PDGFRA, PDGFRB, FGFR1 or PCM1-JAK2 gene rearrangements
    • No evidence of BCR-ABL1, CBFB-MYH11 / inv(16) gene rearrangements
    • No gene mutations identified per next generation sequencing (NGS) panel
    • Comment: There is no morphologic evidence of a myeloid neoplasm associated with eosinophilia. Clinically there are no identifiable etiologies of reactive eosinophilia such as allergic reactions, parasitic infections, autoimmune diseases and medications. The patient presents with a persistent eosinophilia for over 6 months (ranging from 1600 - 5500/µL). Electrocardiogram and echocardiograms were reportedly abnormal. Imaging studies showed subendothelial fibrosis within the ventricles. The corresponding tissue biopsy showed eosinophilic infiltration (20 - 40 eosinophils/high power field) with degranulation within the myocardium.
    • Flow cytometric analysis demonstrates occasional CD34+ myeloblasts which comprise about 1.0% of total events. Hematogones are absent. No clonal B cell population or immunophenotypically abnormal T cells (e.g. CD3- / CD4+) were noted to suggest lymphoid variant of hypereosinophilic syndrome or T cell lymphoma. No clonal plasma cell population is identified.
    • Additional immunohistochemical stains performed on the core biopsy show no increase in CD34 or CD117 positive immature myeloid precursors. CD117 also highlights occasional normal appearing mast cells which do not form aggregates. CD3 and CD20 highlight scattered T and B cells respectively. CD138 stains plasma cells, which comprise < 1% of the marrow cellularity.
    • Peripheral smear: Manual review of the peripheral blood shows normochromic, normocytic anemia, mild leukocytosis, absolute eosinophilia and essentially normal platelets. There is no absolute monocytosis or circulating blasts. Morphologically, RBCs show normochromic, normocytic anemia with mild anisopoikilocytosis. WBCs: slightly increased in number (13000/µL) without neutrophilia, blastosis or immature monocytosis. Occasional circulating immature granulocytic precursors, e.g. metamyelocytes and myelocytes, are noted. A manual 500 cell differential count reveals 42.0% of eosinophils, resulting in an absolute eosinophilic count of 5500/µL and 3.0% monocytes, resulting in an absolute monocyte count of 390/µL. No dysplastic granulocytes such as hypogranular or hyposegmented forms are identified. Platelets: normal by count with occasional large platelets.
    • Bone marrow biopsy: Quality: adequate. Cellularity: 80 - 90%. Hematopoiesis: trilineage maturation with mild myeloid hyperplasia and relatively normal erythropoiesis. Megakaryocytes are identified and normally distributed. There is no apparent increase in blasts or abnormal localization of immature myeloid precursors. Mature eosinophils are increased, making up approximately 30 - 40% of marrow cellularity. Special stains: reticulin: loose network of reticulin without significant intersections (minimal reticulin fibrosis). Small loose lymphoid aggregates composed of small mature lymphocytes are focally noted, overall < 5% of marrow cellularity.
    • Bone marrow clot section: Quality: adequate. Cellularity: 80 - 90%, morphologic features are similar to those observed in the core biopsy.
    • Bone marrow aspirate: Quality: adequate. Granulocytes: mildly increased; mild left shifted maturation without morphologic dysplasia. Eosinophilic precursors, predominantly mature forms, are markedly increased comprising approximately 40 - 50% of cellularity. Erythrocytes exhibit normal normoblastic maturation. Megakaryocytes appear normal by morphology and adequate in number. Myeloblasts: overall ~3% of nucleated cells. There is no monocytosis or increase promonocytes. Iron: storage iron is adequate (2+ on a scale of 0 - 4). No ring sideroblasts are present. Scattered mature mast cells are present (2%).
Differential diagnosis
Board review style question #1
In addition to tissue damage, what minimum level and duration of eosinophilia is required to make a diagnosis of hypereosinophilic syndrome?

  1. 1.0 x 109/L for at least 3 months
  2. 1.0 x 109/L for at least 6 months
  3. 1.0 x 109/L for at least 12 months
  4. 1.5 x 109/L for at least 6 months
  5. 1.5 x 109/L for at least 12 months
Board review style answer #1
D. 1.5 x 109/L for at least 6 months

Comment Here

Reference: Hypereosinophilic syndrome
Board review style question #2
Which of the following is true of idiopathic hypereosinophilic syndrome?

  1. Aberrant T cell population must be present
  2. Can accompany systemic mastocytosis
  3. Carries an autosomal dominant inheritance
  4. End organ damage is absent
  5. No cytogenetic or molecular abnormalities are present
Board review style answer #2
E. No cytogenetic or molecular abnormalities are present

Comment Here

Reference: Hypereosinophilic syndrome

JAK2
Definition / general
  • PCM1-JAK2 fusion is a rare genomic abnormality resulting from t(8;9)(p22;p24.1), which fuses the Janus activated kinase 2 gene (JAK2) with the human autoantigen pericentriolar material gene 1 (PCM1), resulting in a constitutively activated tyrosine kinase
  • Observed in both myeloid and lymphoid neoplasms and can present as myeloproliferative neoplasm (MPN), myelodysplastic / myeloproliferative neoplasm (MDS / MPN), acute leukemia (B or T lymphoid, myeloid or mixed phenotype), lymphoblastic lymphoma or blast stage of a chronic myeloid neoplasm
Essential features
  • Presence of t(8;9)(p22;p24.1) PCM1/JAK2
  • Chronic myeloid leukemia and rearrangements of PDGFRA, PDGFRB and FGFR1 must be excluded
  • Variable peripheral blood and bone marrow eosinophilia
  • Bone marrow morphology; usually like an MPN, MDS / MPN or acute leukemia - both myeloid and lymphoid
  • Proliferation of early erythroblasts in the bone marrow
  • Dyserythropoiesis may be present
  • Marrow reticulin fibrosis
Terminology
  • Neoplasms with t(8;9)(p22;p24.1) / PCM1-JAK2 are included as a provisional entity in the 2016 revision of the WHO classification in the category of myeloid / lymphoid neoplasm with eosinophilia and specific gene rearrangements
  • Rare cases with other fusion partners of JAK2, such as t(9;12)(p24.1;p13) / ETV6-JAK2 and t(9;22)(p24.1;q11.2) / BCR-JAK2, are considered as variants of this entity
ICD coding
  • ICD-O: 9968/3 - Myeloid / lymphoid neoplasms with PCM1-JAK2
Epidemiology
  • M:F = 27:5 showing distinct male preponderance
  • Wide age range (12 - 75), with median age of 47 years
  • Reference: Br J Haematol 2014;166:809
Sites
  • Peripheral blood and bone marrow
  • Rarely lymph nodes and extramedullary locations
Pathophysiology
  • Due to the presentation as both myeloid and lymphoid neoplasms, the cell of origin is purported to be the pluripotential hematopoietic stem cell (Cancer Res 2005;65:2662)
  • PCM1-JAK2 fusion mediates an oligomerization that brings together the linked JAK2 domains, resulting in a constitutively active tyrosine kinase domain of JAK2, which activates multiple downstream signal transducers via the JAK-STAT pathway to promote cell proliferation and differentiation
  • Underlying pathobiology does not depend on the fusion partner of JAK2
  • As far as is known, the translocation partners do not affect the clinical picture, which is determined by the activation of the JAK2 tyrosine kinase
Diagrams / tables

Images hosted on other servers:

Structure of PCM1-JAK2 translocation

Clinical features
  • Clinical features are variable depending on the presentation as a chronic myeloid neoplasm, acute leukemia or a lymphoblastic lymphoma (Leukemia 2005;19:1692)
  • Common presenting symptoms are fatigue, pallor, weight loss, abdominal discomfort, splenomegaly and lymph node enlargement (Leukemia 2005;19:1692)
  • May represent the blastic phase of an underlying chronic neoplasm
  • Reference: Ann Lab Med 2016;36:79
Diagnosis
  • Review of the CBC and peripheral smear evaluation; if eosinophilia is present, all secondary causes of eosinophilia should be excluded clinically and by appropriate investigation (of note, eosinophilia may not be a prominent feature)
  • Flow cytometry to exclude the presence of an immunophenotypically aberrant T cell population as cause of eosinophilia (also known as lymphocyte variant eosinophilia)
  • Bone marrow core biopsy and aspirate morphology
  • Conventional cytogenetics or FISH to exclude BCR-ABL1 and FISH to detect rearrangements of PDGFRA, PDGFRB and FGFR1
  • Identification of the t(8;9)(p22;p24.1) PCM1-JAK2 translocation by conventional cytogenetics and confirmation by FISH or RT-PCR for the fusion gene product
  • References: Blood 2017;129:704, Am J Hematol 2019;94:1149
Laboratory
  • Peripheral blood evaluation may show anemia or pancytopenia, normal, mildly or markedly increased WBC, leukoerythroblastosis, mild or striking eosinophilia or increased blasts if presentation is as an acute leukemia
  • Bone marrow shows myeloid proliferation with maturation; there may be dysplasia in one or more lineages, variable eosinophilia and may show increase in mast cells, usually as loose clusters
    • Frequent finding is presence of paratrabecular clusters of early erythroid precursors
    • At least some myelofibrosis is frequently present, which is highlighted by the reticulin stain (Eur J Haematol 2011;86:87)
  • Metaphase cytogenetics will reveal the characteristic translocation
    • In the variant forms of the PCM1-JAK2 translocation involving other fusion partners with JAK2, a JAK2 break apart FISH probe identifying a rearrangement as a first step may be a clue for further FISH studies to identify the specific chromosomal partner
    • Fusion is confirmed by PCM1-JAK2 dual color, dual fusion probe or by RT-PCR for the fusion gene product
    • In the variant forms of the PCM1-JAK2 translocation involving other fusion partners with JAK2, or in rare cases of crytic rearrangement, a JAK2 break apart FISH probe identifying a rearrangement as a first step may be a clue for further FISH studies to identify the specific chromosomal partner
  • Next generation sequencing usually does not reveal additional somatic mutations (Am J Clin Pathol 2021;155:160)
Prognostic factors
  • Aggressive behavior; usually does not respond to imatinib
  • Good response has been seen with JAK1/2 inhibitor ruxolinitib, although eventually allo-SCT is required
  • Prognosis is quite variable (from few weeks to > 5 years) or longer if patient undergoes allogenic bone marrow transplant
  • Reference: Blood 2012;120:1529
Case reports
  • 35 year old man presented with a history of abdominal discomfort, early satiety, low grade fever, night sweats and weight loss (Blood 2013;122:861)
  • 42 year old man with left inguinal and multiple cervical lymphadenopathy; a CT scan identified splenomegaly (Ann Lab Med 2016;36:79)
  • 51 year old woman was diagnosed as having acute myeloid leukemia at a tertiary hospital (Ann Lab Med 2018;38:492)
  • 57 year old man presented with fatigue, weight loss and dyspnea for 3 months; clinical examination only revealed a splenomegaly (Eur J Haematol 2011;86:87)
Treatment
  • Neoplasms resulting from PCM1-JAK2 mutation do not respond to imatinib; the JAK1/2 inhibitor ruxolitinib has shown good response, although eventually these patients require allogenic stem cell transplantation
Microscopic (histologic) description
  • Classic triad features of bone marrow biopsy are: hypercellularity with eosinophilic infilterate, aggregates of immature erythroblasts and frequent marrow fibrosis; this triad may not be seen in variants with fusion of ETV6-JAK2 and BCR-JAK2 (Am J Clin Pathol 2021;155:160)
  • Variable morphology: may be of a chronic myeloid neoplasm or of acute leukemia; the latter may develop as a blast crisis of the chronic myeloid neoplasm
  • Some cases may not have eosinophilia
  • Megakaryocytes may be increased or decreased but are often dysplastic
  • Variable grades of reticulin fibrosis; usually present
  • Reference: Eur J Haematol 2011;86:87
Microscopic (histologic) images

Contributed by Zeba N. Singh, M.B.B.S., M.D.

Markedly hypercellular bone marrow

Megakaryocytic atypia

Bone marrow eosinophilia

Large aggregate of early erythroid precursors

Reticulin fibrosis

CD34 immunostain

Peripheral smear description
  • Normochromic, normocytic anemia
  • Leukocytosis, leukopenia or leukoerythroblastic picture
  • Eosinophilia often but not always
  • In cases that present as acute leukemia, circulating blasts are present
  • Reference: Am J Hematol 2019;94:1149
Peripheral smear images

Contributed by Zeba N. Singh, M.B.B.S., M.D.

Peripheral blood eosinophilia

Molecular / cytogenetics description
  • PCM1-JAK2 fusion mediates an oligomerization that brings together the linked JAK2 domains resulting in a constitutively active tyrosine kinase
  • JAK2 is a cytoplasmic tyrosine kinase; it activates multiple downstream signal transducers via the JAK-STAT pathway to promote cell proliferation and differentiation
  • BCR-JAK2 fusion protein contains the coiled coil dimerization domain of BCR and the protein tyrosine kinase domain (JH1) of JAK2
    • t(8;9)(p22;p24.1) resulting in PCM1-JAK2 fusion
    • t(9;12)(p24.1;p13.2) resulting in ETV6-JAK2
    • t(9;22)(p24.1;q11.2) resulting in fusion of BCR-JAK2
  • Reference: Cancer Res 2005;65:2662
Molecular / cytogenetics images

Images hosted on other servers:

Karyogram and FISH analysis

Sample pathology report
  • Bone marrow, right posterior iliac crest, core biopsy and clot section:
    • Chronic myeloid neoplasm with t(8;9)(p22;p24.1) PCM1-JAK2 fusion (see comment)
    • Hypercellular (90%) bone marrow showing myeloid hyperplasia, dysmegakaryopoiesis, eosinophilia and reticulin fibrosis (grade 1/3); blasts less than 5%.
    • Comment: The morphological features of the bone marrow core biopsy and clot section are consistent with a myelodysplastic / myeloproliferative neoplasm, unclassifiable (MDS / MPN-U). There is peripheral blood eosinophilia and an increase in marrow eosinophils. Cytogenetic analysis identified t(8;9)(p22;p24.1) abnormality in 20 / 20 metaphases. RT-PCR for the chimeric BCR-ABL1; FISH for PDGFR alpha, PDGFR beta, FGFR1 rearrangements and JAK2 V617F mutation were negative. The patient had hepatosplenomegaly, pulmonary, cardiac and bone involvement.
    • The t(8;9)(p22;p24.1) abnormality fuses the PCM1 gene with the JAK2 gene. Based on the WHO classification (2016 revision), the neoplasm is classified under the group of myeloid / lymphoid neoplasms with eosinophilia and abnormalities of PDGFRA, PDGFRB, FGFR1 rearrangements or t(8;9) PCM1-JAK2 fusion. The neoplasm is aggressive in behavior. Response to ruxolitinib has been demonstrated (Blood 2012;120:1529).
    • Peripheral smear: Manual review of the peripheral blood shows normochromic, normocytic anemia, thrombocytopenia, relative (11%) and absolute (1.65 K/uL) eosinophilia. RBCs: Mild normochromic, normocytic anemia with minimal anisopoikilocytosis. WBCs: There is mild leukocytosis with absolute eosinophilia. The granulocytes show normal morphology. There is a mild left shift with few metamyelocytes and myelocytes. Blasts are not present. The lymphocytes are mature. There is no increase in large granular lymphocytes. Platelets are mildly reduced (100 K/uL) and include some large forms.
    • Bone marrow biopsy: Quality: adequate. Cellularity: 90%. Hematopoiesis: full spectrum trilineage hematopoiesis. There is myeloid hyperplasia. There is a prominence of eosinophils and their precursors. Large, paratrabecular aggregates of early erythroid precursors are identified. There is no overt increase in blasts. Megakaryocytes are normal in number but have dysplastic morphology including nuclear hypolobation, hyperchromasia, loose clustering and paratrabecular localization. There is no increase in lymphocytes or plasma cells. Special stains: reticulin; loose network of reticulin without significant intersections (grade 1 fibrosis on a scale of 0 - 3). Trichrome: negative for collagen deposition.
    • Bone marrow clot section: Quality: adequate. Cellularity: 90% morphologic features are similar to the core biopsy. Bone marrow aspirate: Quality: adequate. Spicules present. Myeloid hyperplasia. M:E ratio increased (5:1). Full maturation in myeloid and erythroid series. No dysgranulopoiesis or increase in blasts. Eosinophils and eosinophil precursors are increased. Clusters of early erythroid precursors present. Dysmegakaryopoiesis with hypolobated nuclei.
    • Immunohistochemistry: Immunohistochemical stains were performed on paraffin fixed and decalcified bone core biopsy sections using appropriate controls. CD34 marks mostly vascular endothelium. Occasional blasts (less than 5%) are identified. CD117 marks the blasts, promyelocytes, early erythroid precursors and mast cells (15%). E-cadherin marks the large aggregates of early erythroid precursors.
Differential diagnosis
  • Reactive eosinophilia:
    • Nonclonal, secondary cause for eosinophilia (e.g. allergic conditions, parasitic infections, drugs, etc.)
    • Detailed clinical history should be taken for exclusion of an underlying cause of eosinophilia
  • Chronic myeloid leukemia:
    • Uncommonly CML may have absolute eosinophilia
    • Absence of BCR-ABL1 fusion excludes CML
  • Chronic myelomonocytic leukemia (CMML):
    • Absolute monocytosis of 1000/uL or more and absence of genetic abnormalities defining CM, or neoplasms with rearrangements of PDGFRA, PDGFRB and FGFR1; both of these entities are excluded by absence of specific genetic abnormality
  • Myeloid neoplasms with PDGFRA, PDGFRB or FGFR1:
    • These neoplasms share many morphological features but are distinguished by the specific genetic abnormality
  • Chronic eosinophilic leukemia, NOS:
    • Persistent eosinophilia, clonal in etiology but not one of the chromosomal translocations included in the entities listed above
  • Systemic mastocytosis:
    • May be associated with peripheral blood, bone marrow and tissue eosinophilia; bone marrow shows more multifocal aggregates of mast cells with abnormal immunophenotype (CD25+, CD2 variable)
    • May be associated with non mast cell clonal hematological disorders
    • Lack the specific abnormalities (PDGFRA, PDGFRB, FGFR1 rearrangements or PCM1-JAK2 translocation)
  • Idiopathic hypereosinophilic syndrome:
    • Peripheral blood eosinophil ≥ 1,500/uL associated with tissue damage, after exclusion of secondary causes of eosinophilia and morphological, flow cytometric, cytogenetic and molecular evaluation for clonal evidence for an acute or chronic myeloid or lymphoproliferative disorder
  • Other entities to consider: atypical chronic myeloid leukemia, T and B acute lymphoblastic leukemia and acute myeloid leukemia; all of these entities can be excluded by specific genetic abnormality
  • Lymphocyte variant eosinophilia:
    • Flow cytometry shows aberrant T cell population
Board review style question #1

A 60 year old man with leukocytosis, mild thrombocytopenia and a differential leukocyte count (showing neutrophils 55%, bands 4%, lymphocytes 22%, monocytes 7%, eosinophils 12%, basophils 1%, metamyelocytes 2%, myelocytes 2% and blasts 1%) undergoes a bone marrow evaluation. Images from the bone marrow core biopsy are provided. Which set of investigations is the most appropriate?

  1. FISH for inv(16)(13.1;q22), 11q23 / MLL and t(8;21)
  2. Metaphase cytogenetics, comparative genomic hybridization and next generation sequencing for myeloid mutations
  3. Metaphase cytogenetics, FISH for PDGFRA, PDGFRB and FGFR1 rearrangement
  4. qRT-PCR for BCR-ABL1, FISH for NPM1-ALK, FLT3 ITD
Board review style answer #1
C. Metaphase cytogenetics, FISH for PDGFRA, PDGFRB and FGFR1 rearrangement

Comment Here

Reference: PCM1-JAK2

Juvenile myelomonocytic leukemia
Definition / general
  • Clonal proliferation of (predominantly) granulocytic and monocytic precursors in children, with < 20% blasts in blood and bone marrow
  • Mutations involving RAS pathway are characteristic
Essential features
  • Myelodysplastic / myeloproliferative disorder in childhood, can occur in children from 1 month of age to adolescence
  • Approximately 75% of cases occur in children less than 3 years old
  • Greater than 75% of cases have mutations in the RAS pathways: PTPN11, KRAS, NRAS, CBL and NF1
  • About 15% of cases are diagnosed in patients with Noonan syndrome (or Noonan-like syndrome) and ~10% occur in conjunction with neurofibromatosis type 1
  • Patients present with hepatosplenomegaly and sustained increased monocyte counts
  • Stem cell transplant improves survival
Terminology
  • Juvenile chronic myelomonocytic leukemia
ICD coding
  • ICD-O: 9946/3 - juvenile myelomonocytic leukemia
  • ICD-10: C93.30 - juvenile myelomonocytic leukemia, not having achieved remission
Epidemiology
  • About 1.2 per million people per year
  • 75% of cases occur in children < 3 years old, more frequent in males (~2:1)
  • Median age of 2 years
  • 10% of cases are associated with neurofibromatosis 1 and deletion of NF1 gene
  • 15% of cases in infants with Noonan syndrome (CBL gene mutation)
  • Rare association with EBV infection (Leuk Res 2008;32:181)
Sites
  • Primarily involves peripheral blood and bone marrow
  • Nearly all cases involve the liver and spleen
  • Other common sites of involvement include lymph nodes, skin and respiratory tract
  • Reference: Blood 1998;91:365
Pathophysiology
  • Due to stem cell defect causing deranged hematopoiesis (Blood 2019;133:1060)
  • In vitro hypersensitivity to GM-CSF, considered to be a hallmark of the disease (and previously used for diagnosis)
  • Multiple genes affecting the RAS / MAPK signaling pathways have been implicated
    • Gain of function mutations in more than 75% of patients
      • Somatic mutations in NRAS or KRAS identified in ~30% of cases, which can lead to accumulation of activated protein
      • ~30% of cases have PTPN11 mutations, which can be somatic or germline as seen in Noonan syndrome
    • Loss of function mutation in NF1, which can be somatic or germline
      • ~15 - 20% of cases can have NF1 mutation or deletion without neurofibromatosis
    • CBL mutation, which can be somatic or germline (Noonan-like syndrome)
      • ~10 - 15% of cases can have this loss of function mutation
  • Leads to reduced degradation of tyrosine kinase receptors and therefore increased RAS pathway signaling
Clinical features
  • Varied clinical presentation may include failure to thrive, malaise, fever, bleeding, pallor, infection, with or without lymphadenopathy; rarely CNS involvement (J Pediatr Hematol Oncol 2007;29:770)
  • Splenomegaly: found in all cases and prerequisite for diagnosis of JMML but may be initially absent at presentation
  • Skin lesions: can be seen in patients with germline mutations (e.g., café au lait spots in NF1 and CBL mutations)
  • Gastrointestinal and pulmonary symptoms can also be observed
  • Most patients die of disease, although it may wax and wane
Diagnosis
  • Multiple diagnostic criteria, according to WHO
  • Genetic criteria (at least 1):
    • Somatic mutations in PTNP11, KRAS or NRAS
      • If present, germline mutations or transient abnormal myelopoiesis of Noonan syndrome have to be excluded
    • NF1 mutation or clinical diagnosis of neurofibromatosis type 1
    • CBL mutation (germline) or loss of heterozygosity
  • Clinical and hematologic criteria (must fulfill all 4):
    • Sustained peripheral blood absolute monocyte count ≥ 1 x 109/L
    • < 20% blasts + promonocytes in blood and bone marrow
    • No Philadelphia chromosome or BCR::ABL1 fusion
    • Splenomegaly
  • If no genetic criteria are present, then the following criteria should be used (along with the clinical and hematologic criteria):
    • Monosomy 7 or other chromosomal abnormality
    • Or 2 or more of the following criteria:
      • Increased hemoglobin F (for age)
      • Myeloid / erythroid precursors on peripheral blood smear
      • In vitro hypersensitivity to granulocyte macrophage colony stimulating factor (GM-CSF or CSF2)
      • Hyperphosphorylation of STAT5
  • Typically, WBC count is 20 - 30 x 109/L with granulocytes and monocytes and occasional dysplasia (which may not be prominent) (Blood 2019;133:1060)
Laboratory
  • Peripheral blood monocytosis ≥ 1 x 109/L
  • Abnormally increased hemoglobin F for patient's age
  • Peripheral smear showing leukocytosis with myeloid left shift, in addition to thrombocytopenia and often anemia
  • Nucleated RBCs are often seen
  • Reference: Blood 1998;91:365
Prognostic factors
  • According to WHO, low platelet counts, age at diagnosis greater than 2 years and high HgF levels are clinical predictors of shorter survival
  • PTPN11 mutated JMML or in patients with NF1 is fatal without rapid treatment
  • Aggressive JMML with increased risk of progression to AML seen in patients with more than 1 RAS activating mutation (Leukemia 2020;34:1658)
  • BMP4, CALC4, CDKN2B, RARB, RASA4 isoform 2: other genes known to be hypermethylated in JMML
  • Methylation pattern in JMML divided into 3 categories with different prognosis:
    • Favorable prognosis: low methylation, underlying somatic mutations in NRAS and CBL
    • Poor prognosis: high methylation, somatic PTPN11 mutations
    • Intermediate category: somatic KRAS mutations and monosomy 7
  • Reference: Blood Adv 2021;5:5507
Case reports
Treatment
  • Stem cell transplantation
    • Recommended early in disease in patients with PTPN11, KRAS or NF1 mutations
    • Deferred in patients with CBL or NRAS (germline) mutations (mainly due to spontaneous regression of JMML)
  • Reference: Haematologica 2015;100:17
Microscopic (histologic) description
  • Bone marrow: hypercellular with granulocytic hyperplasia, 5 - 30% monocytes, decreased megakaryocytes, blasts + promonocytes < 20%, dysplasia not prominent, reticulin fibrosis is rare (Blood 1997;89:3534)
  • Skin: common presentation, with increased myelomonocytic infiltrates in the papillary and reticular dermis
  • Liver: leukemic infiltrates in the portal regions and sinusoids
  • Lung: leukemic infiltrates spread from the capillaries within the alveolar septa into alveoli
  • Spleen: infiltrates in the red pulp with a preference for trabecular and central arteries
Microscopic (histologic) images

Contributed by Alexa J. Siddon, M.D. and AFIP
Hypercellular marrow

Hypercellular marrow

Myeloid predominant marrow

Myeloid predominant marrow

Aspirate with monocytes and promonocytes

Aspirate with monocytes and promonocytes

Peripheral smear description
  • Important specimen for diagnosis
    • Leukocytosis with neutrophilia with immature forms and monocytosis, WBC count is 20 - 30 x 109/L with granulocytes and monocytes and occasional dysplasia (which may not be prominent) (Blood 2019;133:1060)
    • Anemia: most commonly normochromic and nucleated red blood cells are often identified
      • Macrocytosis seen in cases with monosomy 7
    • Thrombocytopenia
  • Blasts and blast equivalents are usually less than 5% (no more than 20%)
Peripheral smear images

AFIP images
Blood shows mature monocytes and neutrophils

Blood shows mature monocytes and neutrophils

Positive stains
Flow cytometry description
  • Can see increased monocytes
Molecular / cytogenetics description
  • PCR used for KRAS, NRAS, NF1, PTNT11 or CBL mutations
    • RAS pathway mutations can occur in about 90% cases
  • Next generation sequencing (NGS) has identified other mutations in patients with JMML
    • SETBP1: found at disease initiation and progression, also considered a potential marker for how aggressive the disease is
    • JAK3, ALK, ROS1
    • EZH2, ASXL1 and DNMT3A: in about 15% of cases
  • 35% of cases can have abnormal karyotype, most commonly monosomy 7
  • References: Am J Blood Res 2021;11:1, Pediatr Int 2016;58:681
Sample pathology report
  • Left posterior iliac crest, core biopsy and aspirate smear:
    • Hypercellular marrow with granulocytic proliferation and (< 20%) blasts, consistent with juvenile myelomonocytic leukemia (see comment)
    • Comment: The bone core biopsy demonstrates hypercellularity and myeloid predominance, with no increase in blasts. In conjunction with peripheral blood monocytosis, splenomegaly, negative BCR::ABL1 studies and a clinical diagnosis of neurofibromatosis type 1, the findings are consistent with juvenile myelomonocytic leukemia.
Differential diagnosis
Board review style question #1

A 9 year old boy with relevant prior history was noted to have enlarged tonsils and cough. His CBC showed leukocytosis with monocytosis. His physical exam was notable for lymphadenopathy and splenomegaly. Flow cytometry of blood showed no increase in blasts or evidence of acute leukemia. A bone marrow biopsy was pursued, which showed a myeloid predominant marrow with no increase in blasts and little granulocytic dysplasia. A pathogenic mutation in which gene is most supportive of a diagnosis of juvenile myelomonocytic leukemia?

  1. IDH2
  2. KRAS
  3. NPM1
  4. SF3B1
Board review style answer #1
B. KRAS. Greater than 75% of cases of JMML have RAS pathway mutation.

Comment Here

Reference: Juvenile myelomonocytic leukemia
Board review style question #2
Which of the following is true of juvenile myelomonocytic leukemia?

  1. Average age of newly diagnosed patients is 50
  2. Commonly associated with neurofibromatosis type 2
  3. There is often increased hemoglobin F for age
  4. Usually driven by Epstein-Barr virus
Board review style answer #2
C. There is often increased hemoglobin F for age. The hemoglobin F can be a hint as it is often abnormally increased for the patient's age.

Comment Here

Reference: Juvenile myelomonocytic leukemia

Juvenile myelomonocytic leukemia associated with Noonan syndrome-like disorder (CBL syndrome) (pending)
[Pending]

Juvenile myelomonocytic leukemia associated with neurofibromatosis (pending)
[Pending]

M5a
Definition / general
  • Acute monoblastic leukemia (M5a)
  • 5 - 8% of AML
  • Children and young adults
Diagnosis
  • 80%+ of monocyte lineage cells are monoblasts
Case reports
  • 66 year old man with erythropoietin dependent transformation of refractory anemia with ringed sideroblasts into acute monoblastic leukemia (Blood 2001;98:3492)
  • 73 year old woman with coexisting mantle cell lymphoma (Leuk Lymphoma 2005;46:1813)
  • 82 year old man with acute monoblastic leukemia following granular lymphocyte proliferative disorder (Rinsho Ketsueki 2011;52:1870)
Microscopic (histologic) description
  • Hypercellular marrow with large number of monoblasts
  • Monoblasts are large with moderately abundant intensely basophilic cytoplasm, variably basophilic and delicate azurophilic granules but no / rare Auer rods
  • May have pseudopods or vacuoles
  • Have round nuclei and lacy chromatin with one or more prominent nucleoli but no folds
  • Promonocytes have abundant less basophilic cytoplasm with obvious azurophilic granules and nuclei have delicate folds
Microscopic (histologic) images

AFIP images
Missing Image

Abundant cytoplasm with azurophilic granules

Missing Image

Monoblasts are large with abundant cytoplasm

Missing Image

Some monoblasts also show pseudopods

Missing Image

Monoblasts have variable cytoplasm

Missing Image

Monoblasts and promonocytes

Missing Image

Marrow completely replaced by monoblasts


Missing Image

Monoblasts are large with abundant pale cytoplasm

Missing Image

Large monoblasts with abundant cytoplasm

Missing Image

Nonspecific esterase positive

Missing Image

CD68 #1 (KP-1) positive

Missing Image

PAS positive

Positive stains
Negative stains
Electron microscopy images

AFIP images
Missing Image

Scattered electron dense deposits

Missing Image

Cytoplasm contains focal area of glycogen deposition

Molecular / cytogenetics description
  • 75% have cytogenetics abnormalities, including 11q23 in 30% (these cases should be classified as a recurrent genetic abnormality)
  • FLT3 mutations in 7%

M5b
Definition / general
  • Acute monocytic leukemia (M5b)
  • 3 - 6% of AML
  • Affects all ages
  • Mature monocytes or promonocytes predominate in peripheral blood (< 80% of monocyte lineage cells are monoblasts, usually < 20%)
  • Treatment may cause tumor lysis syndrome, DIC and falsely elevated platelet counts (Arch Pathol Lab Med 1999;123:1111)
Case reports
Microscopic (histologic) description
  • Leukemic cells are often promonocytes with less basophilic cytoplasm and more azurophilic granules than monoblasts
  • Have folded or cerebriform nuclei with fine chromatin
  • Erythrophagocytosis is common
Microscopic (histologic) images

AFIP images
Missing Image

Monocytic cells have range of differentiation

Missing Image

2 myeloblasts and 3 promonocytes

Missing Image

Moderate granular cytoplasm

Missing Image

Nonspecific esterase positive

Peripheral smear images

AFIP images
Missing Image

Promonocytes have abundant cytoplasm

Positive stains
Negative stains
Electron microscopy images

AFIP images
Missing Image

Promonocyte has cytoplasm with numerous small cisterna

Molecular / cytogenetics description
  • 30% have cytogenetics abnormalities, including 11q23 in 12% (these cases should be classified as a recurrent genetic abnormality)
  • FLT3 mutations in 30%
  • t(8;16)(p11;p13) fuses MOZ gene at 8p11 with CBP gene at 16p13 and is associated with erythrophagocytosis and coagulopathy
Differential diagnosis

MDS / MPN - unclassifiable
Definition / general
  • This is a provisional myelodysplasia / myeloproliferative disorder, with JAK2 V617F mutation present in 48 - 67% (Haematologica 2008;93:34, Blood 2006;108:2173)
  • Diagnostic criteria: features of one of the myelodysplasia / myeloproliferative categories but no history of MDS/MPN, no history of growth factor or cytotoxic therapy, no BCR-ABL1 gene fusion, no rearrangement of PDGFRA, PDGFRB or FGFR1, no del(5q), no inv(3)(q21;q26) OR de novo disease with MPN and MDS features which cannot be assigned to any specific category of MDS/MPN
  • Includes refractory anemia with ringed sideroblasts (RARS) with dyserythropoiesis and marked thrombocytosis (RARS-T) with megakaryocytes similar to those in ET, PV or primary myelofibrosis
Case reports

MDS / MPN with SF3B1 and thrombocytosis
Definition / general
  • Clonal myeloid disorder with mixed myelodysplastic and myeloproliferative features
  • Ring sideroblasts ≥ 15% of erythroblasts (Cancer Biol Med 2016;13:360)
  • Persistent thrombocytosis ≥ 450 x 109/L (Blood Cancer J 2018;8:15)
  • No increase in blast count, < 1% in blood and < 5% in bone marrow (Blood Cancer J 2018;8:15)
  • Lacking BCR-ABL1, isolated del(5q), t(3;3)(q21.3;q26.2) and inv(3)(q21.3;q26.2) (Cancer Biol Med 2016;13:360)
  • No previous history of myelodysplastic syndrome (MDS) or myeloproliferative neoplasm (MPN), except for MDS with ring sideroblasts
  • New distinct entity in 2016 WHO classification; related to 2008 WHO provisional entity, refractory anemia with ring sideroblasts associated with marked thrombocytosis (RARS-T) (Blood Cancer J 2018;8:15, Haematologica 2015;100:1117)
Essential features
  • Mixed myelodysplastic and myeloproliferative features with ring sideroblasts (≥ 15% of marrow erythroblasts), persistent thrombocytosis (≥ 450 x 109/L), anemia and normal blast count (Blood Cancer J 2018;8:15)
  • Strong association with SF3B1 mutation; if absent, there should be no history of recent cytotoxic or growth factor therapy that could explain the MDS / MPN overlap features
  • Often seen in conjunction with JAK2 V617F (and less often CALR or MPL gene mutations), which support but are not required for diagnosis (Haematologica 2015;100:1117, Blood 2016;127:2391)
  • In contrast to MDS with ring sideroblasts, at least 15% ring sideroblasts are required even if an SF3B1 mutation is present (Blood 2016;127:2391)
Terminology
  • Myelodysplastic and myeloproliferative neoplasm with ring sideroblasts and thrombocytosis (MDS / MPN-RS-T)
  • Previously, refractory anemia with ring sideroblasts associated with marked thrombocytosis (RARS-T)
ICD coding
  • ICD-10: D46.9 - myelodysplastic syndrome, unspecified
Epidemiology
Sites
  • Myeloid cells primarily in bone marrow and blood
  • Splenomegaly 40%
  • Occasional hepatomegaly
Pathophysiology
  • Probable multistep genetic and epigenetic changes, leading to signaling deregulation, impaired DNA damage response and other cellular abnormalities (Int J Hematol 2015;101:229)
Etiology
  • Unclear
Clinical features
Diagnosis
Laboratory
  • Macrocytic or normocytic anemia
  • Thrombocytosis, ≥ 450 x 109/L
  • White blood cell (WBC) absolute and differential counts usually normal but mild leukocytosis can occur
Prognostic factors
Case reports
Treatment
Microscopic (histologic) description
  • Hypercellularity of bone marrow with < 5% blasts
  • Erythroid hyperplasia with dyserythropoiesis, including megaloblastic change, common in marrow
  • Ring sideroblasts ≥ 15% of normoblasts on iron stain (Blood Cancer J 2018;8:15)
  • Multilineage dysplasia possible
  • Atypical megakaryocytic hyperplasia and cluster formation with large, hyperlobated megakaryocytes, reminiscent of BCR-ABL negative MPN (Haematologica 2015;100:1117)
  • Marrow fibrosis can occur
Microscopic (histologic) images

Contributed by Patricia Tsang, M.D., M.B.A.

Hypercellular marrow

Megakaryocyte proliferation

Iron stain with ring sideroblast


Dyserythropoiesis

Normoblast with megaloblastic change

Peripheral smear description
  • Normocytic to macrocytic, normochromic red blood cells
  • Thrombocytosis, sometimes with large, giant or atypical platelets
  • No or rare blasts, < 1% (Blood Cancer J 2018;8:15)
Peripheral smear images

Contributed by Patricia Tsang, M.D., M.B.A.

Thrombocytosis

Positive stains
  • Iron stain on aspirate smear shows ring sideroblasts
  • Immunostains generally not needed for diagnosis
Flow cytometry description
  • No increase in blasts detected
  • Aberrant CD56 on myeloid cells sometimes seen
Molecular / cytogenetics description
Sample pathology report
  • Bone marrow biopsy and aspirate smears, left posterior iliac crest:
    • Myelodysplastic / myeloproliferative neoplasm with ring sideroblasts and thrombocytosis, formerly refractory anemia with ring sideroblasts and thrombocytosis
    • Cytogenetics, next generation sequencing for myeloid disorders, reticulin and trichrome stains are pending for further evaluation
  • Peripheral blood smear:
    • Macrocytic anemia with occasional teardrop RBCs
    • Mild neutrophilia and marked thrombocytosis with some giant platelets

  • Microscopic description:
    • The peripheral blood smear shows leukocytosis with mild neutrophilia (8,400/mcL) and no circulating blasts. Anisopoikilocytosis with macrocytic RBCs and occasional teardrop cells are seen. Platelet count is elevated (610,000/mcL).
    • The bone marrow aspirate (dry tap) is aspicular and hypocellular with evidence of maturing granulopoiesis and erythropoiesis. Some of the normoblasts show megaloblastic change and nuclear lobation, consistent with dyserythropoietic changes. The iron stain on aspirate reveals adequate storage iron. Ring sideroblasts constitute about 35% of the erythroid precursors.
    • The bone marrow biopsy is hypercellular (about 80%) with trilineage hyperplasia. There is focal crush cell artifact, suggestive of marrow fibrosis. Myeloid:erythroid ratio appears unremarkable. No increase in myeloblasts is seen histologically. Proliferation of atypical megakaryocytes with cluster formation is present. Megakaryocytes are characteristically enlarged and hyperlobated. No abnormal lymphoid or plasma cell infiltrate is noted.
Differential diagnosis
Board review style question #1
Myelodysplastic / myeloproliferative neoplasm with ring sideroblasts and thrombocytosis is most frequently associated with alterations of which pair of genes?

  1. ABL1 and CALR
  2. ASXL1 and KRAS
  3. SETBP1 and TP53
  4. SF3B1 and JAK2
  5. TET2 and FLT3
Board review style answer #1
D. SF3B1 and JAK2 are the two most frequently altered genes in myelodysplastic / myeloproliferative neoplasm with ring sideroblasts and thrombocytosis

Comment Here

Reference: Myelodysplastic / myeloproliferative neoplasm with ring sideroblasts and thrombocytosis
Board review style question #2
Which of the following features is characteristic of myelodysplastic / myeloproliferative neoplasm with ring sideroblasts and thrombocytosis?



  1. Blasts < 1% in the blood and < 5% in the bone marrow
  2. Circulating ring sideroblasts > 15% in the peripheral blood
  3. Deletion 5q or monosomy 5
  4. Persistent monocytosis with no secondary etiology
  5. Proliferation of hypolobated megakaryocytes in the bone marrow
Board review style answer #2
A. Blasts < 1% in the blood and < 5% in the bone marrow. No increase in blasts is seen in the blood or bone marrow in myelodysplastic / myeloproliferative neoplasm with ring sideroblasts and thrombocytosis.

Comment Here

Reference: Myelodysplastic / myeloproliferative neoplasm with ring sideroblasts and thrombocytosis

MDS in childhood
Clinical features
  • 3 - 9% of pediatric hematologic malignancies
  • Estimated annual incidence in U.S. is 0.5 to 4 cases per million for children compared to 20 - 40 cases for adults
  • Usually diagnosed at ages 6 - 8 years, although juvenile myelomonocytic leukemia is usually diagnosed at age 2 years
  • Difficult to diagnose because:
    1. Less dysplasia at younger ages
    2. Dysplasia may be confused with treatment effects of G-CSF
    3. Bone marrow often hypocellular, resembling aplastic anemia
    4. Resembles HHV6 (Pediatr Blood Cancer 2006;47:543) or parvovirus infections (Rinsho Ketsueki 2001;42:1096, Pediatr Hematol Oncol 2000;17:475)
  • Modified WHO classification for childhood MDS
  • 69% idiopathic, 24% associated with constitutional / inherited disorders (Down syndrome, neurofibromatosis, Bloom syndrome, Fanconi anemia, Atlas of Genetics and Cytogenetics), 7% therapy related (Arch Pathol Lab Med 2007;131:1110)
  • Similar prognosis as AML M6 and M7, with poor induction success rate (Pediatr Blood Cancer 2007;49:17)
Molecular / cytogenetics description
  • Monosomy 7 in 30%

MDS prognostic scoring
Definition / general
  • Myelodysplastic neoplasms (MDS) are a heterogeneous group of diseases with variable outcomes depending on multiple parameters
  • International Prognostic Scoring System for MDS is the most commonly used tool to predict long term outcomes
Essential features
  • Outcomes of MDS are variable and depend on many different parameters including cytogenetics, blast percentage, hemoglobin levels, absolute neutophil count and platelet count; the International Prognostic Scoring System for MDS is the most commonly used tool to predict long term outcomes
  • Patients in the low risk group will often have more prolonged survival and oncological goals include improving their cytopenias (neutropenia, anemia and thrombocytopenia) and reducing iron overload
  • Patients in the high risk group with increased blasts and cytogenetic abnormalities often have shorter survival and are treated more aggressively with 5-azacitidine to delay the transformation to acute myeloid leukemia
Terminology
  • Myelodysplastic syndrome / myelodysplastic neoplasm (MDS)
  • International Prognostic Scoring System (IPSS)
  • Revised International Prognostic Scoring System (IPSS-R)
ICD coding
  • ICD-10: D46.9 - myelodysplastic syndrome, unspecified
Epidemiology
  • Median age at the diagnosis of MDS: 77 years (Br J Haematol 1992;82:358)
  • Incidence of MDS below age 40 is extremely low (0.1 cases per 100,000) (Blood Rev 2019;34:1)
  • Early age of onset (< 50 years) is associated with MDS with genetic predisposition due to inherited genetic mutations (Blood Rev 2019;34:1)
  • Younger age of onset is more common amongst the Asian population (Leuk Res 1998;22:453)
  • Early onset can also be caused by exposure to chemicals (e.g., benzene) or radiation (J Natl Cancer Inst 2012;104:1724)
  • Men are more commonly affected compared to women, except in MDS with low blasts and 5q deletion (MDS 5q), where women are the predominate population affected (Leuk Res 2011;35:1591)
  • There is a higher rate of MDS among the Caucasian population (Blood Rev 2019;34:1)
Sites
  • Bone marrow
Pathophysiology
  • Genetic defects play a major role in the pathogenesis of MDS, including cytogenetic abnormalities, genetic mutations and alterations, abnormal gene expression and epigenetic dysregulation (Int J Clin Oncol 2019;24:885)
  • Chromosomal abnormalities have been seen in at least 50% of MDS patients, including the abnormalities of chromosome 5, 7, 8 and 20; complex karyotypes are also identified (Blood 2008;111:1534)
  • Genetic abnormalities have been identified in over 50 genes in at least 80% of MDS patients, including SF3B1, SRSF2, U2AF1, ZRSR2, LUC7L2, DDX41, ASXL1, EZH2, TET2, DNMT3A, IDH1 / IDH2, CSNK1A1, RUNX1, BCOR, TP53 and CUX1 (Eur J Haematol 2015;95:3)
Etiology
  • Old age is considered a significant risk factor and the risk increases with advancing age due to an increased propensity for genetic mutations (Blood 2013;122:2943)
  • Exposure to certain environmental toxins and certain chemicals (e.g., benzene), has been linked to an increased risk of developing MDS (Leuk Lymphoma 1999;35:269)
  • Previous exposure to chemotherapy or radiation therapy for the treatment of another malignancy is also known risk factor for developing secondary MDS (Cancer Genet Cytogenet 2007;176:1, Eur J Haematol 1991;47:17)
Diagrams / tables

Contributed by Daniel Rivera, M.D.
Features of dysplasia

Features of dysplasia

Diagnostic criteria

Diagnostic criteria

Revised international prognostic score

Revised international prognostic score

Clinical features
Diagnosis
Laboratory
  • Cytopenia in at least 1 hematopoietic lineage is usually seen in most cases of MDS
    • Anemia: Hb < 13 g/dL in men and < 12 g/dL in women
    • Leukopenia: absolute neutrophil count < 1.8 x 109/L
    • Thrombocytopenia: platelets < 150 x 109/L
  • Chromosome analysis or conventional karyotyping, fluorescence in situ hybridization (FISH) and next generation sequencing (NGS) for mutation analysis are required for some diagnoses
Radiology description
Not applicable
Radiology images
Not applicable
Prognostic factors
  • Revised IPSS (IPSS-R) utilizes cytogenetics, blast percentage in the bone marrow, hemoglobin levels, platelet levels and the absolute neutrophil count to create the prognostic risk scores and classify patients under the risk categories
  • Please see IPSS-R table above (Blood 2012;120:2454)
  • Molecular IPSS (IPSS-M) utilizes molecular data for risk prognostication using a list of 31 focused genes: TP53, MLL, FLT3, ASXL1, CBL, DNMT3A, ETV6, EZH2, IDH2, KRAS, NPM1, NRAS, RUNX1, SF3B1, SRSF2, U2AF1, BCOR, BCORL1, CEBPA, ETNK1, GATA2, GNB1, IDH1, NF1, PFH6, PPM1D, PRPF8, PTPN11, SETBP1, STAG2, WT1 (NEJM Evid 2022;1:EVIDoa2200008)
Case reports
Treatment
  • Treatment approach can be divided based on the risk group; for those with low risk MDS, it depends on the transfusion needs
  • Transfusion independent patients are followed with observation only
  • Patients with severe anemia with increasing red blood cell transfusion needs can be managed with erythroid stimulating agents (ESA) (Am J Hematol 2023;98:1307)
  • Patients with severe neutropenia with increased infection risk can be managed with granulocyte colony stimulating factors (GCSF) (Am J Hematol 2023;98:1307)
  • Lenalidomide is the standard of care for low risk MDS with del(5q), anemia and increasing transfusion needs; it is contraindicated if there is thrombocytopenia
    • 5-azacitidine and decitabine are used for transfusion dependent, low risk MDS cases that have failed or are not candidates for growth factors or lenalidomide (Am J Hematol 2023;98:1307)
  • For low risk MDS that have failed multiple therapies, allogeneic stem cell transplant (allo-SCT) must be considered (Am J Hematol 2023;98:1307)
  • 5-azacitidine is the standard of care for high risk MDS; furthermore, allo-SCT should also be considered in high risk MDS (Am J Hematol 2023;98:1307)
Clinical images
Not applicable
Gross description
Not applicable
Gross images
Not applicable
Frozen section description
Not applicable
Frozen section images
Not applicable
Microscopic (histologic) description
  • Myelodysplastic neoplasms with defining genetic abnormalities
    • Myelodysplastic neoplasm with biallelic TP53 inactivation (MDS biTP53)
      • In general, MDS biTP53 has high risk morphologic findings, such as multilineage dysplasia, higher blast count and fibrosis in the bone marrow
      • Dysplastic changes overlap with those of other MDS types; please see features of dysplasia table
      • Bone marrow is typically hypercellular but can be normocellular or hypocellular
      • Blast percentage < 20% in the bone marrow and < 20% in the peripheral blood
    • Myelodysplastic neoplasm with low blasts and 5q deletion (MDS 5q)
      • Complete blood count and peripheral blood smear evaluation can show thrombocytosis in the absence of a cytopenia
      • Megakaryocytes are usually dysplastic with small and hypolobated forms (micromegakaryocytes)
      • Dysplasia in the erythroid and granulocytic lineages are less prominent; however, dysplastic changes can still overlap with those of other MDS types (please see features of dysplasia table)
      • Ring sideroblasts (or SF3B1 mutation) can be present and do not exclude the diagnosis of MDS 5q, if the other criteria are met
      • Bone marrow is typically hypercellular or normocellular and can have megakaryocytic hyperplasia and erythroid hypoplasia
      • Blast percentage < 5% in the bone marrow and < 2% in the peripheral blood
    • Myelodysplastic neoplasm with low blasts and SF3B1 mutation (MDS SF3B1)
      • Complete blood count and peripheral blood smear evaluation can show normocytic to macrocytic anemia
      • Bone marrow is typically hypercellular with left shifted erythroid predominance and dyserythropoiesis
      • Ring sideroblasts are increased; ring sideroblasts are defined as having ≥ 5 iron granules or granules encircling at least one - third or more of the nucleus
      • Dysplastic changes overlap with those of other MDS types; please see features of dysplasia table
      • Blast percentage < 5% in the bone marrow and < 2% in the peripheral blood
  • Myelodysplastic neoplasms morphologically defined
    • Myelodysplastic neoplasm with increased blasts
      • Bone marrow is usually hypercellular
      • Dysplastic changes overlap with those of other MDS types; please see features of dysplasia table
      • Subtypes
        • MDS with increased blasts 1 (MDS IB1): blast percentage 5 - 9% in the bone marrow or 2 - 4% in the peripheral blood without significant fibrosis
        • MDS with increased blasts 2 (MDS IB2): blast percentage 10 - 19% in the bone marrow or 5 - 19% in the peripheral blood or Auer rods without significant fibrosis
        • MDS with increased blasts and fibrosis (MDS-f): Blast percentage 5 - 19% in the bone marrow or 2 - 19% in the peripheral blood with fibrosis grade 2 to 3 out of 3
    • Myelodysplastic neoplasm, hypoplastic (MDS-h)
      • Bone marrow is hypocellular; the hypocellularity can be diffuse or patchy
      • Dysplastic changes overlap with those of other MDS types; please see features of dysplasia table
      • Blast percentage < 5% in the bone marrow and < 2% in the peripheral blood
    • Myelodysplastic neoplasm with low blasts (MDS LB)
      • Bone marrow is usually hypercellular
      • Dysplastic changes overlap with those of other MDS types; please see features of dysplasia table
      • Blast percentage < 5% in the bone marrow and < 2% in the peripheral blood
  • Myelodysplastic neoplasms of childhood
    • Childhood myelodysplastic neoplasm with increased blasts (cMDS IB)
      • Complete blood count and peripheral blood smear evaluation shows at least 1 cytopenia
      • Bone marrow is usually hypercellular
      • Dysplastic changes overlap with those of other MDS types; please see features of dysplasia table
      • Blast percentage 5 - 19% in the bone marrow and 2 - 19% in the peripheral blood
    • Childhood myelodysplastic neoplasm with low blasts (cMDS LB)
      • Complete blood count and peripheral blood smear evaluation shows at least 1 cytopenia
      • Bone marrow is usually hypercellular
      • Dysplastic changes overlap with those of other MDS types; please see features of dysplasia table
      • Blast percentage < 5% in the bone marrow and < 2% in the peripheral blood
Microscopic (histologic) images

Contributed by Brenda Mai, M.D.
Hypercellular marrow for age

Hypercellular marrow for age

Osteosclerosis

Osteosclerosis

Micromegakaryocytes Micromegakaryocyte

Micromegakaryocyte

Paratrabecular megakaryocyte

Paratrabecular megakaryocyte

Grade 1 reticulin fibrosis

Grade 1 reticulin fibrosis


Grade 2 reticulin fibrosis

Grade 2 reticulin fibrosis

Increased blasts

Increased blasts

Irregular erythroids

Irregular erythroids

Erythroid binucleation

Erythroid binucleation

Erythroid budding and multinucleation

Erythroid budding and multinucleation

Erythroid karryorhexis and budding / binucleation

Erythroid karryorhexis and budding / binucleation


Erythroid vacuolization

Erythroid vacuolization

Megaloblastoid change

Megaloblastoid change

Hypolobated megakaryocyte

Hypolobated megakaryocyte

Trichrome fibrosis

Trichrome fibrosis

CD34 with increased blasts

CD34 with increased blasts

CD61 with micromegakaryocytes

CD61 with micro-
megakaryocytes

Virtual slides
Not applicable
Cytology description
  • 500 cell count is recommended for morphologic evaluation in bone marrow aspirate smears and touch preparations
  • Limitations occur when there is a dry tap or the aspirate smears are aspicular and suboptimal for evaluation
  • Cytologic dysplastic changes are listed in dysplastic features table, especially the top table, features of dysplasia seen in aspirate smears
Cytology images
Images moved to Micro images
Peripheral smear description
  • Cytopenia in at least 1 hematopoietic lineage is usually seen in most cases of MDS
    • Anemia: Hb < 13 g/dL in men and < 12 g/dL in women
    • Leukopenia: absolute neutrophil count < 1.8 x 109/L
    • Thrombocytopenia: platelets < 150 x 109/L
  • Only granulocytic dysplasia and increased blasts are usually identified in the peripheral blood; erythroid dysplasia and megakaryocytic dysplasia are not seen
  • Cytologic dysplastic changes are listed in dysplastic features table, especially the top table, especially the top table, features of dysplasia seen in aspirate smears, under dysgranulopoiesis
Peripheral smear images

Contributed by Brenda Mai, M.D.
Hyposegmented neutrophil

Hyposegmented neutrophil

Hypersegmented neutrophil

Hypersegmented neutrophil

Hypogranular neutrophil

Hypogranular neutrophil

Hypogranular and hyposegmented neutrophil

Hypogranular and hyposegmented neutrophil

Neutrophil that is larger in size

Neutrophil that is larger in size

Positive stains
  • CD34 and CD117 can be used to enumerate blasts in the tissue sections
  • CD42b, CD61 and factor VIII can be used to evaluate for small megakaryocytes and atypical clustering in the tissue sections
  • Reticulin and trichrome are used to evaluate for fibrosis
  • PAS positivity in the erythroids is a feature of dysplasia
  • Prussian blue iron stain is used to visualize ring sideroblasts
Negative stains
Not applicable
Flow cytometry description
Flow cytometry images

Contributed by Brenda Mai, M.D.
Decrease side scatter

Decreased side scatter

Increased blasts

Increased blasts

Electron microscopy description
Not applicable
Electron microscopy images
Not applicable
Molecular / cytogenetics description
  • Chromosomal abnormalities have been seen in at least 50% of MDS patients, including the abnormalities of chromosome 5, 7, 8 and 20; complex karyotypes are also identified (Blood 2008;111:1534)
  • Genetic abnormalities have been identified in over 50 genes in at least 80% of MDS patients, including SF3B1, SRSF2, U2AF1, ZRSR2, LUC7L2, DDX41, ASXL1, EZH2, TET2, DNMT3A, IDH1 / IDH2, CSNK1A1, NRAS, STAG2, RUNX1, BCOR, TP53 and CUX1 (Eur J Haematol 2015;95:3)
  • Cases of childhood MDS can been associated with SETBP1, GATA2, PTPN11, RAS and SAMD mutations (Nat Commun 2017;8:1557)
Molecular / cytogenetics images

Contributed by Brenda Mai, M.D.
Complex karyotype

Complex karyotype

Karyotype with deletion 5q

Karyotype with deletion 5q

FISH with deletion 5q

FISH with deletion 5q

Videos

Updates in myelodysplastic syndromes

Sample pathology report
  • Bone marrow, biopsy:
    • Hypercellular (80%) bone marrow with dyserythropoiesis and ~12% blasts (see comment)
    • Comment: Bone marrow aspirate smears show erythroid dysplasia and a mild increase in blasts, constituting ~10% of the total nucleated cells on the aspirate smear. The bone marrow core biopsy and clot section are hypercellular for age with increased scattered blasts highlighted by CD34 immunostain. Flow cytometric analysis shows increased abnormal myeloblasts that constitute about 11.3% of total events with a normal immunophenotype (CD13+, CD33+, CD34+, CD38+, CD117+, MPO+). These findings are compatible with a myelodysplastic neoplasm with increased blasts (MDS IB2); however, correlation with karyotype, FISH and NGS studies is required for complete evaluation.
Differential diagnosis
Board review style question #1

A 72 year old man presents with fatigue and easy bruising. Complete blood count reveals

  • Hemoglobin: 8.5 g/dL
  • Absolute neutrophil count: 3,500/μL
  • Platelets: 60,000/μL

Peripheral blood smear shows dysplastic neutrophils and macrocytic red blood cells with occasional circulating nucleated RBCs; circulating blasts are not identified. The bone marrow aspirate smears show erythroid dysplasia with 4% blasts on differential. The marrow cellularity is 60% without evidence of fibrosis. A Prussian blue iron stain is shown above. Which of the following is the most likely MDS subtype given the provided information?

  1. MDS-f
  2. MDS-h
  3. MDS IB2
  4. MDS SF3B1
Board review style answer #1
D. MDS SF3B1. Answer B is incorrect because the patient is 72 years old, so his normal marrow cellularity should be around 20 - 40%; the question stem states that the marrow cellularity is 60%, which would be hypercellular for age and precludes the diagnosis of MDS-h. Answer C is incorrect because blasts are not increased, which precludes the diagnosis of MDS IB2. Answer A is incorrect because the question stem states there is no fibrosis; MDS-f requires grade 2 - 3 of fibrosis.

Comment Here

Reference: MDS prognostic scoring
Board review style question #2
A 65 year old woman with a history of chemotherapy for breast cancer presents with recurrent infections and persistent neutropenia. Blood work reveals

  • White blood cells: 1,200/μL (neutrophils 500/μL)
  • Hemoglobin: 11 g/dL
  • Platelets: 150,000/μL

The peripheral blood smear shows 3% blasts. The bone marrow aspirate smears show dysplastic erythroid precursors and 8% blasts. Reticulin shows grade 1 fibrosis. Karyotype analysis shows a normal female. Which of the following is the most likely MDS subtype given the provided information?

  1. MDS 5q
  2. MDS biTP53
  3. MDS-f
  4. MDS IB1
Board review style answer #2
D. MDS IB1. Answer A is incorrect because the patient has a normal karyotype. Answer B is incorrect because no next genome sequencing (NGS) or TP53 mutation was mentioned. Answer C is incorrect because the question stem states there is grade 1 fibrosis; MDS-f requires grade 2 - 3 of fibrosis.

Comment Here

Reference: MDS prognostic scoring

MDS prognostic scoring (pending)
[Pending]

MDS unclassifiable
Definition / general
Myelodysplastic syndrome, unclassifiable (MDS-U) diagnosis can be made in 3 instances:
  1. Unequivocal dysplasia < 10% in one or more lineage, with cytogenetic abnormalities presumptive of MDS, and < 1% blasts in peripheral blood, 5% blasts in bone marrow
  2. Refractory anemia or refractory cytopenia with multilineage dysplasia with 1% blasts in peripheral blood
  3. Unilineage dysplasia with pancytopenia

  • Cases can be reclassified into specific subtypes later if characteristic features develop
  • Some cases are associated with prior aplastic anemia and monosomy 7 (Am J Clin Pathol 2006;126:925)
  • Myelofibrosis: when present, often is difficult to obtain bone marrow aspirate; patients often have pancytopenia with dysplasia in 3 lineages; CD34 staining helps identify blast population
Case reports
Microscopic (histologic) images

AFIP images

Pancytopenia, abnormally bilobed nuclei in granulocytes

Marked increase in megakaryocytes

Erythroid hyperplasia

Large erythroid precursors


Hypercellular marrow with marked fibrosis

Cluster of large megakaryocytes

Marked reduction in megakaryocytes

Recurrence with increased marrow cellularity



Images hosted on other servers:

Megakaryocyte clustering and hyperlobulated forms (figs 2 / 3)

Differential diagnosis

MDS with biallelic TP53 inactivation
Definition / general
  • Myelodysplastic neoplasms (MDS), previously called myelodysplastic syndromes, are clonal hematopoietic stem cell neoplasms characterized by ineffective marrow hematopoiesis that results in morphologic dysplasia and peripheral cytopenia; there is increased risk of transformation to acute myeloid leukemia (AML)
  • New MDS subtype per WHO 5th edition and international consensus classification (ICC) 2022
Essential features
  • MDS biTP53 (MDS with biallelic TP53 inactivation) is a new subtype per WHO 5th edition, 2022
  • Detection of ≥ 1 TP53 mutation, meeting molecular diagnostic criteria of biallelic TP53 mutations (Nat Med 2020;26:1549)
  • 33% of TP53 mutated MDS patients had monoallelic mutations whereas 67% had multiple hits (multihit) consistent with biallelic targeting (Nat Med 2020;26:1549)
  • MDS biTP53 is characterized by increased blast counts, higher risk of leukemic transformation and higher risk of mortality
  • TP53 mutated myeloid neoplasms (MN) represent a unique entity with very poor prognosis, irrespective of whether the blast percentage indicates MDS or AML or whether the disease is therapy related or de novo in its ontogeny (Blood Adv 2022;6:2847)
Terminology
  • Per the 5th edition of WHO published in 2022, the terminology of myelodysplastic neoplasms has been introduced to replace the previous term of myelodysplastic syndromes, in parallel to myeloproliferative neoplasm and to emphasize the neoplastic course of the disease; however, the abbreviation of MDS is kept for myelodysplastic neoplasm
ICD coding
  • ICD-10: D46 - myelodysplastic syndromes
Epidemiology
  • In the U.S., the age adjusted incidence of MDS is estimated to be 4 in 100,000 individuals (Blood Rev 2019;34:1)
  • Incidence notably increases with advancing age, male gender, smoking, obesity and history of radiotherapy or chemotherapy (Blood Rev 2019;34:1)
  • Estimated incidence of MDS biTP53 is ~0.03/100,000/year
    • ~10% of MDS patients exhibit TP53 alteration, with 67% of these cases involving biallelic alterations (Leukemia 2019;33:1747)
  • TP53 mutated MDS has lower response to treatment compared to non-TP53 mutated MDS and an overall survival (OS) of 5 - 10 months with prognosis worsening in biallelic TP53 mutations (Cancer Discov 2022;12:2516, Nat Med 2020;26:1549)
  • It is important to note that TP53 has been found to play an important role in both de novo and therapy related MDS; interestingly TP53 mutations were found to be the most common genetic mutation in treatment related MDS (t-MDS) (Nature 2015;518:552)
    • History of cytotoxic treatment is a qualifier that moves the diagnosis to myeloid neoplasm postcytotoxic therapy
Sites
  • Defining characteristic of MDS is bone marrow failure, which occurs as hematopoietic stem cells acquire mutations, resulting in ineffective hematopoiesis, morphological dysplasia and peripheral blood cytopenias (Blood 2019;133:1086)
Pathophysiology
  • TP53 suppressor gene is mutated in about half of human tumors (Nat Med 2020;26:1549)
  • TP53 gene encodes for the p53 protein, which plays a crucial role in regulating various cellular processes, such as the cell cycle and cell death, which is usually triggered by oxidative stress (WHO 5); thus, the inactivation of p53 allows tumor initiation and its evasion from cell death (Nat Rev Cancer 2018;18:89)
  • It is hypothesized that monoallelic TP53 inactivation may contain biallelic TP53 inactivation subclones that can negatively affect disease outcomes, as patients with single TP53 and VAF of > 23% have similar outcomes to biallelic TP53 patients
  • Patients with TP53 mutations were found to have fewer comutated genes; however, they more commonly exhibit del(5q) chromosomal abnormality, monosomal karyotype and complex karyotype (Leukemia 2019;33:1747)
  • Neurofibromin 1 (NF1), which is a negative regulator of Ras signaling, and del(12p), which includes ETV6, were found to be prevalent in TP53 inactivation AML / MDS cases (Blood Adv 2023;7:4586)
  • TP53 mutated MN represents a unique entity with very poor prognosis, irrespective of whether the blast percentage indicates MDS or AML or whether the disease is therapy related or de novo in its ontogeny (Blood Adv 2022;6:2847)
    • Current assignment of these cases into MDS, AML with myelodysplasia related changes (AML MRC) or therapy related myeloid neoplasms (t-MN) appears to divide a biologically similar disease into different diagnostic groups
  • TP53 mutations play an important role in the pathogenesis of many cases of t-AML / t-MDS (Nature 2015;518:552)
    • Loss of TP53 provides a selective advantage for neoplastic growth; TP53 mutations are resistant to chemotherapy and expand preferentially after treatment
Etiology
  • Variable and includes
    • Idiopathic
    • Exposure to certain DNA damaging agents such as chemotherapy or radiation
      • TP53 mutations were found to be more common in t-MN, which was also a poor diagnostic factor (Leukemia 2019;33:2842)
      • Chemotherapy leads to selective expansion of the progenitor cells with TP53 mutations that are resistant to treatment (Nature 2015;518:552)
    • Inherited genetic disorders
      • E.g., Li-Fraumeni syndrome or inherited bone marrow failure syndromes
Diagrams / tables

WHO 4R WHO 5th ICC classification
  • Not an entity
    • TP53 mutation is listed as a one of the common genetic mutations in MDS
    • Prognostic impact is adverse
    • Mutation should be correlated with morphological findings
  • MDS with biallelic TP53 inactivation
    • Fulfilling diagnostic criteria of MDS
    • 2 or more TP53 mutations or 1 mutation with TP53 copy number loss or cnLOH
    • Myeloblasts < 20% bone marrow (BM) and peripheral blood (PB)
  • MDS with mutated TP53
    • Multihit TP53
    • Blasts 0 - 9% in BM / PB
  • AML / MDS with mutated TP53
    • Any somatic TP53 mutation (VAF > 10%)
    • Blasts 10 - 19% in BM / PB


ICC classification of MDS with mutated TP53
MDS with mutated TP53 AML / MDS with mutated TP53
  • Any cytopenia
  • Blasts: 0 - 9%
  • Genetics: multihit TP53, TP53 mutation (VAF > 10%) and complex karyotype often with loss 17p
  • Any cytopenia
  • Blasts: 10 - 19%
  • Genetics: any somatic TP53 mutation (VAF > 10%)


WHO 5th edition, 2022 ICC 2022
Morphologically defined
Lineage Subclassification using dysplastic lineages removed, replaced with MDS LB (MDS with low blasts) MDS, NOS SLD
MDS, NOS MLD
Excessive blast counts in bone marrow
(5 - 9%) or in peripheral blood (2 - 4%)
MDS IB1 MDS EB
(blast count 5 - 9% in bone marrow, 2 - 4% in peripheral blood)
Excessive blast counts in bone marrow
(10 - 19%) or in peripheral blood (5 - 19%)
MDS IB2 MDS AML
(blast count 10 - 19% in bone marrow and 2 - 4% in peripheral blood)
Others cMDS (MDS of childhood)
MDS-h (hypoplastic MDS)
MDS-f (MDS with fibrosis, blasts 5 - 19% in bone marrow, 2 - 19% in peripheral blood)
MDS, NOS (without dysplasia)
Defining genetic abnormalities
SF3B1 mutation MDS SF3B1 (MDS with low blasts and SF3B1 mutation or MDS with ring sideroblasts if SF3B1 wild type) MDS SF3B1 (MDS with mutated SF3B1) or MDS, NOS (with ring sideroblasts and SF3B1 wild type)
del(5q) MDS 5q
(MDS with low blasts and 5q deletion)
[5q deletion alone or with 1 other genetic aberration other than del(7q) / -7]
MDS with del(5q)
[del(5q) is isolated or with up 1 genetic aberration except del(7q) / -7 or multihit TP53]
TP53 MDS biTP53
(MDS with biallelic TP53 inactivation)
MDS with mutated TP53
Multihit TP53
Includes MDS, MDS / AML, AML
Diagnosis
  • Multihit TP53 is approximated by (Blood Adv 2022;6:2847, Nat Med 2020;26:1549)
    • Next generation sequencing analysis or Sanger sequencing covering at least exons 4 - 11 of TP53 gene is required for detection of biallelic TP53 alterations, coupled with a technique to detect copy number status, usually fluorescence in situ hybridization (FISH) with a probe set specific for the TP53 locus on 17p13.1 (Leukemia 2014;28:241, Blood 2013;122:3616)
    • Detection of ≥ 2 TP53 mutations, usually affecting both alleles that can be considered multihit status (Nat Med 2020;26:1549)
    • After exclusion of constitutional changes, a TP53 VAF > 49% may be regarded as presumptive (not definitive) of copy loss on the trans allele or copy neutral LOH
    • In the presence of one TP53 mutation, evidence of TP53 copy loss or copy neutral LOH is required as concurrent 17p LOH, suggestive of biallelic TP53 alterations (Nat Med 2021;27:1239, Nat Commun 2020;11:4980)
  • TP53 mutations are typically associated with complex karyotype; these 2 features are associated with unfavorable risk (N Engl J Med 2016;375:2023, Nat Med 2020;26:1549)
    • Up to 55% of patients with complex karyotype (CK) (> 3 cytogenetic abnormalities) harbor a TP53 mutation (Leukemia 2019;33:1747, Blood Adv 2022;6:2847)
    • Complex karotype TP53 mutated MDS has been shown to predict poor prognosis irrespective of blast percentage and other TP53 molecular characteristics (Blood Adv 2022;6:2847)
  • Presence of TP53 mutation is an independent impact on prognosis that is as significant as severe anemia and more significant than having a bone marrow blast proportion of 10 - 29% (Leukemia 2019;33:1747)
    • To assess patients with TP53 mutations, research has shown that relying on VAF estimates is not sufficient for precise assessment of TP53 allelic state and recommends other approaches for confirmation (Nat Med 2020;26:1549)
Prognostic factors
  • Factors associated with significant decrease in survival in MDS patients with TP53 mutations include (Leukemia 2019;33:1747)
    • Severe anemia (hemoglobin < 8.0 g/dL), NRAS mutation, SF3B1 mutation, TP53 mutation, elevated blast percentage (> 10%), abnormal 3q, abnormal 9 and monosomy 7 as having the greatest survival risk
  • Monosomal karyotype, high complexity and TP53 mutation are individually associated with shorter overall survival (Leukemia 2019;33:1747)
  • Type and abundance of TP53 mutation (VAF) in question may further refine its prognostic impact (Leukemia 2019;33:1747)
  • TP53 mutant CK MDS patients also had significantly higher bone marrow blast proportion and lower platelet counts, 2 factors strongly associated with elevated prognostic risk considered by clinical scoring systems like the IPSS-R (Leukemia 2019;33:1747)
  • Some studies have shown that the effect of TP53 on survival is significant in both TP53 monoallelic and TP53 multihit (Blood Adv 2022;6:2847)
    • No TP53 mutation (median: 33.9 months)
    • TP53 monoallelic (median: 12.5 months)
    • TP53 multihit (median: 9.4 months)
    • No significant influence of TP53 VAF as a continuous variable or using VAF cutoffs (Blood Adv 2022;6:2847)
  • Some studies have shown that only MDS with TP53 multihit shows poor outcomes and response to therapy compared to MDS with TP53 monoallelic mutations (Nat Med 2020;26:1549)
    • Median OS in MDS TP53 multihit state was 8.7 months
    • Median OS in MDS TP53 monoallelic patients was 2.5 years
    • Median OS wild type patients was 3.5 years
    • Patients with monoallelic TP53 mutations and VAF > 22% (n = 38) had increased risk of death compared to wild type patients; monoallelic TP53 mutations and VAF ≤ 22% (n = 87) had OS similar to wild type patients
Case reports
  • 7 year old boy, 44 year old woman and 47 year old woman with Li-Fraumeni syndrome and MDS (Arch Pathol Lab Med 2010;134:1010)
  • 65 year old woman with TP53 germline mutation developed Bowen disease and myelodysplastic syndrome (Intern Med 2005;44:490)
  • 76 year old man presented with refractory anemia with ring sideroblasts, i(17q) and TP53 mutation (Cancer Genet Cytogenet 2002;136:86)
  • 9 patients with TP53 mutant myelodysplastic syndromes managed by allogeneic hematopoietic stem cell transplantation and decitabine containing preconditioning regimen (Front Oncol 2022;12:928324)
Treatment
  • Risk stratification is essential in the clinical care of patients with myelodysplastic syndromes (Blood 2012;120:2454)
    • Types of chromosomal abnormalities present, co-occurring somatic mutations and clinical features all contribute to the actual risk in patients (Leukemia 2019;33:1747)
  • Revised International Prognostic Scoring System (IPSS-R) (Blood 2012;120:2454)
    • MDS are usually classified according to the IPSS-R into 5 prognostic groups
  • Molecular International Prognostic Scoring System (IPSS-M) (NEJM Evid 2022;1:EVIDoa2200008)
    • MDS is classified into 6 IPSS-M risk categories with prognostic differences
    • Compared with the IPSS-R, the IPSS-M shows better prognostic discrimination
  • Patients with TP53 mutations who receive conventional chemotherapy, such as anthracycline based or cytarabine based induction chemotherapy, have poor outcomes; decitabine has been shown to improve overall survival (N Engl J Med 2016;375:2023, Leukemia 2019;33:1747)
    • Possible explanation is that standard chemotherapy leads to outgrowth of preexisting TP53 mutated clones rather than creating TP53 mutations (Nature 2015;518:552)
Microscopic (histologic) description
  • Blast count must be < 19% in bone marrow
  • Bone marrow cellularity is typically hyper / normocellular
  • Significant dysplasia in 1 or more of the myeloid lineages
    • Dysmegakaryopoiesis
    • Dyserythropoiesis
    • Dysgranulopoiesis
  • Reference: Leukemia 2022;36:1703
Microscopic (histologic) images

Contributed by Hatem Kaseb, M.D., Ph.D., M.P.H.
Blast

Blast

Bone marrow with increased blasts

Bone marrow with increased blasts

Dyserythropoiesis

Dyserythropoiesis

Hypercellular bone marrow with dysplasia Hypercellular bone marrow with dysplasia

Hypercellular bone marrow with dysplasia


Hypercellular bone marrow with dysplasia

Hypercellular bone marrow with dysplasia

Increased blasts in bone marrow Increased blasts in bone marrow

Increased blasts in bone marrow

TP53 IHC TP53 IHC

TP53 IHC

Positive stains
N/A
Negative stains
N/A
Flow cytometry description
  • No findings specific to this entity
  • Myeloblasts with aberrant phenotype may be detected
Molecular / cytogenetics description
  • Multihit TP53 is approximated by (Blood Adv 2022;6:2847, Nat Med 2020;26:1549)
    • Detection of ≥ 2 TP53 mutations, usually affecting both alleles that can be considered multihit status (Nat Med 2020;26:1549)
    • After exclusion of constitutional changes, a TP53 VAF > 49% may be regarded as presumptive (not definitive) of copy loss on the trans allele or copy neutral LOH
    • In the presence of one TP53 mutation, evidence of TP53 copy loss or copy neutral LOH is required as concurrent 17p LOH, suggestive of biallelic TP53 alterations (Nat Med 2021;27:1239, Nat Commun 2020;11:4980)
Sample pathology report
  • Bone marrow aspirate, core biopsy, clot section and peripheral blood smear:
    • Myeloid neoplasm with multihit TP53 (see comment)
    • Comment: This neoplasm meets criteria for MDS with increased blasts 2 (MDS IB2) (WHO 5). The presence of TP53 in MDS supersedes other MDS types in classification; therefore, in conjunction with the reported TP53 mutation (VAF: 87%), it is compatible with myelodysplastic syndrome with biallelic TP53 inactivation (MDS biTP53) (WHO 5) and MDS / AML with mutated TP53 (ICC).
Board review style question #1
Which of the following is correct regarding myelodysplastic neoplasm (MDS) with TP53 inactivation / multihit?

  1. Associated with 5q loss
  2. Association with SF3B1 mutation is common
  3. Favorable outcome among MDS types
  4. Rarely transforms to acute myeloid leukemia
  5. TP53 VAF > 49% supports the diagnosis
Board review style answer #1
E. TP53 VAF > 49% supports the diagnosis. This is one of the criteria of MDS with multihit TP53 as laid out by WHO 5th edition. Answer C is incorrect because the disease is associated with unfavorable outcome. Answer D is incorrect because the patients usually progress to acute myeloid leukemia. Answer B is incorrect because the disease is not associated with SF3B1 mutation. Answer A is incorrect because the disease is not associated with 5q loss.

Comment Here

Reference: MDS with biallelic TP53 inactivation
Board review style question #2

Which of the following is correct regarding myelodysplastic neoplasm (MDS) with TP53 inactivation / multihit?

  1. Blast count must be < 10% in bone marrow
  2. Disease shows specific flow cytometry findings
  3. Dysmegakaryopoiesis is the most common dysplastic finding
  4. MDS TP53 monoallelic patients typically show similar prognosis to MDS TP53 multihit patients
  5. Usually associated with complex karyotype
Board review style answer #2
E. Usually associated with complex karyotype. MDS with multihit TP53 is commonly associated with complex karyotype and this association predicts an unfavorable prognosis. Answer B is incorrect because the disease does not show a specific flow cytometry finding. Answer C is incorrect because no distinct dysplasia in a certain lineage has been found to be common in this disease entity. Answer A is incorrect because the blast count must be < 19% in bone marrow. Answer D is incorrect because the majority of papers have shown that the prognosis of MDS TP53 multihit patients is worse than MDS TP53 monoallelic.

Comment Here

Reference: MDS with biallelic TP53 inactivation

MDS with biallelic TP53 inactivation (pending)
[Pending]

MDS with excess blasts
Definition / general
  • Myeloblasts are 5 - 19% of bone marrow differential
  • Usually cytopenias in 2 or 3 lineages
  • Considered high risk MDS
  • Type 1: 5 - 9% blasts in bone marrow or 2 - 4% blasts in peripheral blood, no Auer rods, < 1 billion/L monocytes
  • Type 2: 10 - 19% blasts in bone marrow or 5 - 19% blasts in peripheral blood or Auer rods in any MDS, < 1 billion/L monocytes; more aggressive, greater tendency to progress to AML (Am J Clin Pathol 2005;124:191)
  • Refractory anemia with excess blasts in transformation (RAEB-T): classified as AML under WHO classification
  • Median survival of 16 months for RAEB-1 and 9 months for RAEB-2 (Br J Haematol 2006;132:162)
Laboratory
  • Anemia (normochromic, normocytic or macrocytic), usually neutropenia and thrombocytopenia
Case reports
Microscopic (histologic) description
  • Peripheral blood: nucleated red blood cells, immature granulocytes, neutrophilic hyposegmentation, pseudo-Pelger-Huet cells and hypogranulation, myeloblasts 2 - 4% (RAEB-1) or 5 - 19% (RAEB-2), occasional micromegakaryocytes
  • Bone marrow: normocellular or hypercellular; hyperplasia of granulocytes or erythrocytes; myeloblasts comprise 5 - 9% (RAEB-1) or 10 -19% (RAEB-2) of white blood cells; Auer rods often seen; severe dysplastic changes in all 3 lineages, more severe than other MDS; abnormal localization of immature precursors (ALIP / clusters or aggregates of blasts located away from bone trabeculae and vascular structures); may have increased reticulin fibers
Microscopic (histologic) images

AFIP images

Blast in center with adjacent neutrophils

Promyelocytes and myelocytes

Markedly hypogranular cytoplasm

Auer rod present in blast; neutrophils show nuclear hypolobulation

Bone marrow aspirate


Normocellular biopsy

Normal erythroid precursors

Slightly more cellular marrow

More immature granulocytes

Marked fibrosis with streaming effect



Images hosted on other servers:

RAEB-1

Peripheral smear images

AFIP images

Neutrophil has hypogranular cytoplasm

Hypogranular cytoplasm



Images hosted on other servers:

RAEB-1

RAEB-2: blood, marrow and myocardial infiltration

Molecular / cytogenetics description
Differential diagnosis

MDS with multilineage dysplasia
Definition / general
  • Myelodysplastic syndrome (MDS) with multilineage dysplasia is characterized by 1 or more cytopenias and dysplastic changes in 2 or more of the myeloid lineages (erythroid, granulocytic and megakaryocytic)
  • Myelodysplastic syndrome with ring sideroblasts and multilineage dysplasia (MDS RS MLD) if ring sideroblasts are > 15% (> 5% with SF3B1 mutation)
Essential features
  • Dysplastic changes in 2 or more of the myeloid lineages
  • Blast counts < 1% in the peripheral blood and < 5% in the bone marrow
  • Assess ring sideroblasts (< 15%) and SF3B1 mutation status (if sideroblasts comprise 5 - 15% of erythroid elements) to categorize as MDS MLD or MDS RS MLD
  • Monocytes < 1 x 109/L
  • Exclusion of other causes of dysplasia
  • References: Leukemia 2015;29:66, Leukemia 2007;21:668
Terminology
  • Refractory cytopenia with multilineage dysplasia
  • Myelodysplastic syndrome with ring sideroblasts and multilineage dysplasia (MDS RS MLD)
ICD coding
  • ICD-0: 9985/3 - refractory cytopenia with multilineage dysplasia
  • ICD-10: D46.A - refractory cytopenia with multilineage dysplasia
Epidemiology
Sites
  • Bone marrow
Pathophysiology
  • Cell of origin is a hematopoietic stem cell
  • Complex interplay between genetic and epigenetic alterations and the bone marrow microenvironment lead to dysregulated hematopoietic differentiation, resulting in impaired differentiation, morphological dysplasia and cytopenia
Etiology
  • Age associated risk attributed to hematopoietic stem cell mutations
  • Exposure to toxins (e.g., benzene, cigarette smoking, agricultural chemicals or solvents)
  • Family history of hematopoietic neoplasms
  • Acquired aplastic anemia
  • References: N Engl J Med 2014;371:2477, N Engl J Med 2014;371:2488
Diagrams / tables

Images hosted on other servers:

Pathogenesis of myelodyspastic syndromes

Clinical features
  • Unicytopenia, bicytopenia (common)
  • Pancytopenia, milder cytopenia (less common)
Diagnosis
  • Peripheral blood analysis
  • Bone marrow examination
Laboratory
  • Hemoglobin concentration < 10 g/dL (recommended by 2016 WHO)
  • Platelet count < 100 x 109/L (recommended by 2016 WHO)
  • Standard hematologic values for cytopenia (hemoglobin < 13 g/dL [males], < 12 g/dL [females] and platelets < 150 109/L) are also used by some authors
  • Absolute neutrophil count < 1.8 x 109/L
  • Monocytes < 1 x 109/L
  • Blasts < 1% in the peripheral blood
  • Reference: Blood 2016;128:2096
Prognostic factors
Case reports
Treatment
  • Transfusion
  • Iron chelation
  • Erythropoiesis stimulating factors
  • Immunosuppressive treatment
  • Lenalidomide for lower risk del(5q) MDS
  • Luspatercept for MDS with ring sideroblasts
  • Azacytidine
  • Decitabine
  • Induction chemotherapy
  • Stem cell transplantation
  • Reference: Blood 2013;122:2943
Microscopic (histologic) description
  • Bone marrow findings (Leuk Res 2013;37:64)
    • General
      • Cellularity: majority of cases are normocellular or hypercellular; a subset of MDS-MLD show age adjusted hypocellularity
      • Erythroid hyperplasia
      • Myeloid hyperplasia with left shift
      • Myeloblasts must be < 5%
      • Occasional fibrosis (16% of cases)
    • Erythroid lineage dysplasia
      • Nuclear: nuclear budding, internuclear bridging, karyorrhexis, multinuclearity, megaloblastoid changes
      • Cytoplasmic: ring sideroblasts (< 15%, < 5% with SF3B1 mutation)
      • Vacuolization
    • Myeloid lineage dysplasia
      • Small or unusually large size
      • Nuclear cytoplasmic asynchrony
      • Nuclear hyposegmentation (pseudo Pelger-Huët anomaly)
      • Nuclear hypersegmentation
      • Decreased granules; agranularity
      • Pseudo Chédiak-Higashi granules
      • Döhle bodies
    • Megakaryocyte dysplasia
      • Micromegakaryocytes (most reliable and reproducible)
      • Nuclear hypolobation or nonlobation
      • Binucleation or multinucleation
Microscopic (histologic) images

Contributed by Julia T. Geyer, M.D. and Shajo Kunnath-Velayudhan, M.B.B.S., M.M.S.T.

Hypercellular bone marrow

Dysplastic megakaryocytes

Dysmegakaryopoiesis

Dysgranulopoiesis

Dyserythropoiesis

Peripheral smear description
  • Red blood cells: anisopoikilocytosis with macrocytosis of red blood cells
  • White blood cells
    • Nuclear clumping
    • Hyposegmentation
    • Lack of lobation (pseudo Pelger-Huët anomaly)
    • Cytoplasmic hypogranularity or agranularity
    • Abnormal granular clumping (similar to Chédiak-Higashi syndrome): infrequent
  • No increase in blasts (must be < 1%)
  • Reference: Haematologica 2013;98:568
Peripheral smear images

Contributed by Genevieve M. Crane, M.D., Ph.D.
Dysplastic neutrophils

Dysplastic neutrophils

Positive stains
Flow cytometry description
  • Flow cytometry findings alone are not diagnostic
  • Decreased side scatter of granulocytes
  • Erythroid: increased coefficient of variation and decreased intensity of CD71 or CD36
  • Aberrant expression of CD56 or CD7 on progenitors, granulocytes or monocytes
  • Asynchrony of CD15 and CD16 on granulocytes
  • Altered expression of CD13 in relation to CD11b or CD16
  • Reference: Leukemia 2014;28:1793
Molecular / cytogenetics description
  • Cytogenetics
    • Trisomy 8
    • Monosomy 7
    • del(7q)
    • Monosomy 5
    • del(5q)
    • del(20q)
    • Complex karyotypes
  • Molecular aberrations
    • Cohesion family (STAG2)
    • Chromatin modifiers (ASXL1)
    • Spliceosome genes (SRSF2)
    • Transcription factors (RUNX1)
    • Signaling molecules (CBL)
    • Tumor suppressors (TP53)
    • DNA modifiers (TET2)
    • SF3B1 (in setting of MDS RS MLD with at least 5% ring sideroblasts)
  • Reference: Leuk Res 2013;37:64
Videos

What is multilineage dysplasia?

Sample pathology report
  • Bone marrow, posterior iliac crest, core biopsy, clot section, aspirate smears and touch imprint:
    • Myelodysplastic syndrome with trilineage dysplasia
    • Flow cytometry: abnormal myeloid blasts present
    • Peripheral blood: anemia, moderate, macrocytic
    • Cytogenetics: 46,XY,+8[16]/46,XY[4]
    • Next generation sequencing:
      • SRSF2 P95R with mutant allele frequency of 31%
      • ASXL1 E635%fs*15 with mutant allele frequency of 28%
Differential diagnosis
  • Secondary causes of dysplasia:
    • Nutritional deficiencies, toxic exposures, drugs and biologic agents, infection, congenital disorders and sideroblastic anemias
    • History and physical examination
    • Appropriate laboratory investigations
  • Myelodysplastic syndrome with single lineage dysplasia:
    • Only 1 hematopoietic lineage shows dysplasia
  • Myelodysplastic syndrome with excess blasts:
    • 5 - 19% myeloblasts in the bone marrow or 2 - 19% blasts in the peripheral blood
    • Presence of Auer rods in blasts
  • Myelodysplastic syndrome, unclassifiable:
    • MDS (MDS with excess blasts excluded) associated with 1% blasts in the peripheral blood on 2 separate occasions
    • MDS (MDS with multilineage dysplasia excluded) associated with pancytopenia
    • Persistent cytopenia with < 2% blasts in the blood and < 5% in the bone marrow and no significant dysplasia in any myeloid lineage and the presence of cytogenetic abnormalities considered presumptive evidence of MDS
Board review style question #1

A 60 year old man with anemia underwent bone marrow core biopsy (an H&E image is shown). Which of the following is essential to make the diagnosis of myelodysplastic syndrome?

  1. Absence of secondary causes of dysplasia
  2. Presence of dysplasia in the erythroid lineage
  3. Presence of dysplasia in the megakaryocyte lineage
  4. Presence of dysplasia in the myeloid lineage
Board review style answer #1
A. Absence of secondary causes of dysplasia

Comment Here

Reference: MDS with multilineage dysplasia
Board review style question #2
A 90 year old man presents with fatigue and the peripheral blood smear examination shows dysplastic neutrophils. No blasts are present. The evaluation for secondary causes of anemia and dysplastic neutrophils are negative. The bone marrow examination shows dysplasia in erythroid and myeloid lineages and 2% blasts. The iron stain shows 16% ring sideroblasts. This condition is best classified as

  1. Myelodysplastic syndrome, unclassifiable
  2. Myelodysplastic syndrome with multilineage dysplasia
  3. Myelodysplastic syndrome with ring sideroblasts
  4. Myelodysplastic syndrome with ring sideroblasts and multilineage dysplasia
Board review style answer #2
D. Myelodysplastic syndrome with ring sideroblasts and multilineage dysplasia

Comment Here

Reference: MDS with multilineage dysplasia
Board review style question #3
Which of the following findings is considered the most reliable and reproducible dysplastic feature in the megakaryocyte lineage?

  1. Hyperchromatic nuclei
  2. Micromegakaryocytes
  3. Multinucleation
  4. Nuclear hypolobation
Board review style answer #3
B. Micromegakaryocytes

Comment Here

Reference: MDS with multilineage dysplasia

MDS with mutated SF3B1
Definition / general
  • Myelodysplastic syndrome (MDS) characterized by morphological dysplasia with presence of ring sideroblasts (RS); may or may not have an SF3B1 mutation
Essential features
  • Characterized by cytopenias, morphological dysplasia and ring sideroblasts, often with concomitant SF3B1 mutation
  • In the absence of SF3B1 mutation, ≥ 15% ring sideroblasts are needed, whereas ≥ 5% ring sideroblasts are required for diagnosis in the presence of SF3B1 mutation
  • Ring sideroblasts are erythroid precursors with ≥ 5 iron granules encircling a third or more of the nucleus
  • Lacks Auer rods or del 5q as an isolated cytogenetic abnormality; myeloid blasts are not increased in the bone marrow or peripheral blood
Terminology
  • Formerly called refractory anemia with ring sideroblasts, refractory cytopenia with multilineage dysplasia and ring sideroblasts
  • Current nomenclature: MDS RS SLD (single lineage dysplasia) and MDS RS MLD (multilineage dysplasia)
ICD coding
  • ICD-10: D46.1 - refractory anemia with ring sideroblasts
Epidemiology
  • 3 - 11% of all MDS cases
  • Median age: 60 - 73 years
  • M = F
  • Myelodysplastic syndrome with ring sideroblasts, multilineage dysplasia (MDS RS MLD) is more common than myelodysplastic syndrome with ring sideroblasts, single lineage dysplasia (MDS RS SLD)
  • Reference: Am J Med 2012;125:S2
Pathophysiology and etiology
  • SF3B1 gene encodes a component of U2 small nuclear ribonucleoproteins (snRNP) spliceosome
  • Mutations in the SF3B1 gene lead to altered splicing of mitochondrial iron transporter gene and other metabolic genes, which in turn leads to the ineffective erythropoiesis and formation of ring sideroblasts
  • Ring sideroblasts are erythroid precursors with abnormal iron accumulation within the mitochondria
  • These changes lead to morphologic dysplasia, ineffective erythropoiesis and iron overload
  • References: Blood 2016;128:462, Blood 2012;120:3173
Clinical features
  • Anemia
  • Thrombocytopenia and neutropenia in a minority
  • Hepatosplenomegaly due to iron overload in some
  • Reference: Am J Hematol 2021;96:379
Diagnosis
  • Refractory anemia or bicytopenia / pancytopenia on complete blood count; 1 - 2 cytopenias MDS RS SLD, 1 - 3 cytopenias MDS RS MLD
  • Bone marrow aspiration and biopsy, < 5% bone marrow blasts, < 1% peripheral blood blasts
  • Molecular studies for diagnosis and prognosis
  • Cytogenetic studies: any abnormality unless it fulfills criteria for MDS with isolated del(5q)
  • References: Am J Hematol 2021;96:379, Blood 2016;127:2391
Laboratory
  • Normochromic macrocytic or normochromic normocytic anemia (hemoglobin usually is < 10 g/dL), usually no other cytopenias but rarely may have thrombocytosis (correlate with JAK2 mutation status and diagnosis of MDS / myeloproliferative neoplasm with ring sideroblasts and thrombocytosis [MPN RS T]) (Am J Hematol 2021;96:379)
  • Concurrent iron deficiency can mask the presence of MDS RS (Blood 2011;117:5793)
  • May demonstrate lab values of progressive iron overload - e.g. elevated serum ferritin and transferrin saturation (Hemasphere 2020;4:e357)
Prognostic factors
  • 1 - 2% cases evolve into acute myeloid leukemia (AML)
  • Age > 70 years, hemoglobin < 8 g/dL in women and < 9 g/dL in men and platelet count < 75 x 109/L are poor prognostic factors
  • Absence of SF3B1 mutation, presence of high risk cytogenetics, thrombocytopenia and multilineage dysplasia are poor prognostic factors
  • Monosomal karyotype (MK), non-MK other than single or double del(5q) abnormalities, presence of RUNX1 and ASXL1 mutations (Am J Hematol 2019;94:475)
  • More recently, SF3B1 mutant MDS proposed as a distinct subtype with relatively good prognosis (Blood 2020;136:157, Am J Clin Pathol 2021 May 12 [Epub ahead of print])
  • Reference: Am J Hematol 2021;96:379
Case reports
Treatment
  • Recombinant erythropoietin (EPO) for anemia
  • Iron chelation therapy for iron overload
  • Lusatercept and sotatercept: soluble fusion proteins that increase hemoglobin levels by a different mechanism than EPO (Am J Hematol 2019;94:475)
Microscopic (histologic) description
  • Microscopic criteria
    • Characterized by cytopenias (1 - 2 MDS RS SLD, 1 - 3 MDS RS MLD)
    • Ring sideroblasts and morphological dysplasia (1 lineage [erythroid] MDS RS SLD, 2 - 3 lineages MDS RS MLD), often with concomitant SF3B1 mutations
    • In the absence of SF3B1 mutations, ≥ 15% ring sideroblasts are needed, whereas > 5% ring sideroblasts are required for diagnosis in the presence of SF3B1 mutations (Blood 2012;119:5674)
    • Ring sideroblasts are erythroid precursors with ≥ 5 iron granules encircling a third or more of the nucleus (Haematologica 2008;93:1712)
    • Lacks Auer rods, < 5% bone marrow blasts, < 1% peripheral blood blasts
    • Does not meet criteria for MDS with isolated del (5q); other cytogenetic abnormalities allowed
  • Bone marrow histology
    • Bone marrow general:
      • Hypercellular or normocellular marrow with erythroid hyperplasia
    • Bone marrow erythroid:
      • Erythroid hyperplasia (may be mild to moderate dysplasia, including irregular nuclear contours, nuclear budding, megaloblastic change), markedly increased iron stores
    • Bone marrow myeloid:
      • Myeloblasts < 5% (or call MDS with excess blasts)
    • Bone marrow megakaryocytes:
      • Normal numbers and morphology (in SLD)
    • In addition to presence of ring sideroblasts and erythroid hyperplasia and dysplasia, > 10% dysplastic cells within the myeloid or megakaryocytic series in multilineage dysplasia subtype
  • Reference: Mediterr J Hematol Infect Dis 2015;7:e2015035, Blood 2016;127:2391
Microscopic (histologic) images

Contributed by Cecilia C.S. Yeung, M.D.
Erythroid predominance

Erythroid predominance

Erythroid hyperplasia and dyserythropoiesis

Erythroid hyperplasia and dyserythropoiesis

Multilineage dysplasia

Multilineage dysplasia

Ring sideroblasts

Ring sideroblasts

Cytology description
Peripheral smear description
  • Peripheral blood:
    • Dimorphic red blood cell populations normal and microcytic hypochromic
    • Occasional coarse basophilic stippling and Pappenheimer bodies
    • < 1% myeloblasts
    • Evaluate for cytopenias and granulocytic dysplasia to subclassify
Peripheral smear images

AFIP images
eripheral blood

Peripheral blood

Positive stains
Electron microscopy description
  • Transmission electron microscopy analysis of ring sideroblasts reveals features of apoptosis with condensed nuclear chromatin and fragmentation of the erythroid cells because of iron deposition in mitochondria (Oncogene 2006;25:4757)
Electron microscopy images

AFIP images
Erythroid precursor shows iron deposits

Erythroid precursor shows iron deposits

Molecular / cytogenetics description
  • Mutation in the spliceosome gene SF3B1 is found in 90% cases of MDS RS SLD and in 30 - 70% of MDS RS MLD
  • Clonal cytogenetic abnormalities involve a single chromosome in 50% of MDS RS MLD cases, including high risk loss of chromosome 7
  • Other MDS related gene mutations are identified in more than half of cases of MDS MLD, including mutations in STAG2, ASXL1, SRSF2, RUNZ1, CBL, TP53 and TET2
Sample pathology report
  • Bone marrow, aspiration and biopsy:
  • Final diagnosis
    • Myelodysplastic syndrome with ring sideroblasts with multilineage dysplasia (see comment)
    • Negative for increased blasts
    • Positive for SF3B1 mutation
    • Peripheral blood with normocytic normochromic anemia and pancytopenia
    • Comment: The bone marrow aspirates are cellular and demonstrate trilineage dysplasia with presence of 40% ring sideroblasts and without significant increase in blast count (3% by morphology and flow cytometry). SF3B1 mutations are identified by next generation sequencing. Overall, the morphologic, flow cytometric and molecular features are diagnostic of myelodysplasia with ring sideroblasts with multilineage dysplasia. Cytogenetic studies reveal loss of chromosome 7, which is associated with high risk of leukemic transformation.
  • Cytogenetics
    • Loss of chromosome 7
  • Molecular studies
    • SF3B1 mutation
  • Flow cytometry
    • 3% myeloid blasts (see comment)
      • Comment: Bone marrow flow cytometry reveals presence of 3% myeloid blasts expressing CD33, CD13, CD117 and MPO. There is no abnormal B cell clone identified and B cells constitute approximately 4% of all nucleated cells. Plasma cells comprise 2% of all nucleated cells and normally express CD138 and CD38.
  • Peripheral blood
    • Differential:
      • Complete blood count: 3,500/cumm
      • Neutrophils: 60%
      • Lymphocytes: 30%
      • Eosinophils: 4%
      • Monocytes: 4%
      • Basophils: 2%
      • Blasts: 0%
    • Morphology:
      • Red blood cells: decreased numbers, normocytic normochromic with mild anisopoikilocytosis and polychromasia
      • White blood cells: leukopenia, no blasts, normal morphology
      • Platelets: mild thrombocytopenia, few giant platelets seen
  • Bone marrow aspirate
    • Differential:
      • Erythroid: 65%
      • Myeloid: 32%
      • Lymphocytes: 3%
      • Plasma cells: 2%
      • Blasts: 3%
      • M:E ratio: 1:2
    • Morphology:
      • Erythroid: erythroid hyperplasia with features of dyserythropoeisis - binucleation, nuclear budding, nucleocytoplasmic asynchrony
      • Myeloid: normal maturation with mild dysgranulopoeisis
      • Megakaryocytes: normal in numbers with hypolobated and monolobated forms
      • Plasma cells: normal in number and morphology
      • Lymphocytes: normal in number and morphology
      • Iron stain: 40% ring sideroblasts are identified with presence of perinuclear iron granules
  • Bone marrow biopsy
    • Morphology:
      • Hypercellular (70% cellularity) bone marrow with erythroid hyperplasia, dyserythropoiesis, dysgranulopoeisis and dysmegakaryopoeisis
      • Reticulin stain shows grade I fibrosis
Differential diagnosis
  • MDS / MPN with ring sideroblasts and thrombocytosis:
    • Clonal disorder with diagnostic features of MDS RS SLD
    • Platelet count of ≥ 450 × 109/L (persistent)
    • Large, atypical megakaryocytes
  • Various nonclonal conditions can present with ring sideroblasts in the bone marrow; thorough clinical history and absence of diagnostic criteria for clonal MDS RS (> 15% RS or SF3B1 mutation with > 5% RS) are essential for differential diagnosis
    • Genetically inherited congenital sideroblastic anemia (Br J Haematol 2016;174:847):
      • Characterized by mutations in 1 of 3 mitochondrial pathways: heme synthesis, iron sulfur cluster biogenesis and protein synthesis
      • X linked sideroblastic anemia is the most common form attributed to mutation in aminolevulinic acid (ALA) synthase
      • Microcytic hypochromic anemia and iron overload
    • Acquired sideroblastic anemia due to chronic alcohol use (Postgrad Med 1992;92:147):
      • History of chronic alcoholism
      • Elevated serum aspartate aminotransferase (AST):alanine aminotransferase (ALT) ratio
      • Macrocytic anemia
      • < 15% ring sideroblasts
    • Drug induced sideroblastic anemia - chloramphenicol, penicillamine, linezolid, isoniazid (Haematologica 2013;98:e138):
      • History of medication use
      • May have pure red cell aplasia
      • Reversible after discontinuation of drug
    • Lead poisoning:
      • High serum lead levels, lead lines (Burton lines) on teeth and gingiva
      • < 15% ring sideroblasts
      • Basophilic stippling in red cells on peripheral smear, microcytic and hemolytic anemia
    • Copper deficiency / zinc toxicity (Clin Case Rep 2015;3:325):
      • Low serum copper levels or high serum zinc levels
      • < 15% ring sideroblasts
      • Cytoplasmic vacuoles in erythroid and myeloid cells
  • Reference: Am J Hematol 2021;96:379
Board review style question #1

Bone marrow aspirate with iron stain is shown in the image above. Which molecular alteration is most commonly observed in patients who have myelodysplastic syndrome (MDS) with this histologic feature?

  1. Isolated del 5q
  2. JAK 517F mutation
  3. Monosomy 7
  4. SF3B1 mutation
Board review style answer #1
D. SF3B1 mutation

Comment Here

Reference: MDS with ring sideroblasts
Board review style question #2
How does the finding of an SF3B1 mutation impact the diagnosis of myelodysplastic syndrome (MDS) with ring sideroblasts?

  1. If no SF3B1 mutation is seen, only 5% ring sideroblasts are needed to make the diagnosis of MDS with ring sideroblasts
  2. If SF3B1 mutation is seen, only 5% ring sideroblasts are needed to make the diagnosis of MDS with ring sideroblasts
  3. If SF3B1 mutation is seen, only 10% ring sideroblasts are needed to make the diagnosis of MDS with ring sideroblasts
  4. There is no impact; 15% ring sideroblasts are required for the diagnosis of MDS with ring sideroblasts
Board review style answer #2
B. If SF3B1 mutation is seen, only 5% ring sideroblasts are needed to make the diagnosis of MDS with ring sideroblasts

Comment Here

Reference: MDS with ring sideroblasts

MDS with mutated SF3B1
Definition / general
  • Myelodysplastic syndrome with mutated SF3B1 (MDS SF3B1) is a myeloid neoplasm with cytopenia and dysplasia characterized by SF3B1 mutation and often ring sideroblasts
  • Acceptable definition: MDS with low blasts and ring sideroblasts
  • ICC 2022 Nomenclature: MDS with mutated SF3B1
Essential features
  • Blood counts: characterized by cytopenia of one or more lineages, without thrombocytosis
  • Morphology: erythroid lineage dysplasia, ring sideroblasts are present in over 90% cases but not required for diagnosis in the presence of SF3B1 mutation, blasts < 5% in the bone marrow and < 2% in the peripheral blood
  • Cytogenetics: absence of 5q deletion, monosomy 7/7q deletion or complex karyotype
  • Molecular genetics: SF3B1 mutation at a variant allele frequency (VAF) of > 10% (per International Consensus Classification [ICC]), no multihit TP53 or RUNX1 mutation
  • In the absence of SF3B1 mutation, ≥ 15% ring sideroblasts are needed
  • Reference: Blood 2020;136:157
Terminology
  • WHO 4th edition equivalents: MDS RS SLD (myelodysplastic syndrome with ring sideroblasts and single lineage dysplasia) and MDS RS MLD (multilineage dysplasia)
  • Formerly called refractory anemia with ring sideroblasts, refractory cytopenia with multilineage dysplasia and ring sideroblasts
ICD coding
  • ICD-O: 9982/3 - refractory anemia with sideroblasts
  • ICD-11: 2A4Y - other specified myelodysplastic and myeloproliferative neoplasms
Epidemiology
Sites
No information provided
Pathophysiology
  • SF3B1 gene encodes a component of the U2 small nuclear ribonucleoproteins (snRNP) spliceosome
  • Mutations in the SF3B1 gene lead to altered splicing of mitochondrial iron transporter gene and other metabolic genes, which in turn leads to the ineffective erythropoiesis and formation of ring sideroblasts
  • Ring sideroblasts are erythroid precursors with abnormal iron accumulation within the mitochondria
  • These changes lead to morphologic dysplasia, ineffective erythropoiesis and iron overload
  • References: Blood 2016;128:462, Blood 2012;120:3173
Etiology
  • Driver mutations in SF3B1 gene in hematopoietic stem cells
Diagrams / tables
No information provided
Clinical features
  • Anemia
  • Thrombocytopenia and neutropenia in a minority
  • Hepatosplenomegaly due to iron overload in some
  • Reference: Am J Hematol 2021;96:379
Diagnosis
  • Refractory anemia or bicytopenia / pancytopenia on complete blood count
  • Bone marrow aspiration and biopsy, < 5% bone marrow blasts, < 2% peripheral blood blasts
  • Molecular studies: next generation sequencing (NGS) panel including SF3B1 and associated myeloid neoplasm genes for diagnosis and prognosis; absence of biallelic TP53 inactivation
  • Cytogenetic studies: absence of del(5q), monosomy 7 or 7q deletion, complex karyotype
  • Does not fulfill criteria for other MDS / myeloproliferative neoplasm (MPN) entities
  • References: Am J Hematol 2021;96:379, Blood 2016;127:2391
Laboratory
  • Normochromic macrocytic or normochromic normocytic anemia (hemoglobin usually is < 10 g/dL); usually no other cytopenias
  • Thrombocytosis should be absent as these are classified separately as MDS / MPN with SF3B1 mutation and thrombocytosis; these usually have concurrent JAK2 V617F, CARR or MPL mutations
  • Concurrent iron deficiency can mask the presence of MDS RS (Blood 2011;117:5793)
  • May demonstrate lab values of progressive iron overload (e.g., elevated serum ferritin and transferrin saturation) (Hemasphere 2020;4:e357)
Radiology description
No information provided
Radiology images
No information provided
Prognostic factors
  • Among other WHO 5th edition and ICC 2022 described MDS subtypes, MDS SF3B1 has the best outcomes (Leukemia 2022;36:1703, Blood 2022;140:1200)
  • When SF3B1 mutations are seen, the presence of multilineage dysplasia has not demonstrated significant prognostic impact
  • Transfusion dependence is associated with worse outcomes and higher risk of transformation to acute myeloid leukemia (AML)
  • In comparison to patients with MDS with low blasts and SF3B1 mutations, MDS with low blasts, ring sideroblasts and wild type SF3B1 has a less favorable overall survival and leukemia free survival (Crit Rev Oncol Hematol 2019;133:74)
  • Presence of excess blasts has a worse prognosis even in the presence of SF3B1 mutation
  • Patients with this diagnosis who have additional mutations in TP53, RUNX1, EZH2 mutations have been associated with a worse outcome as compared to patients with SF3B1 mutation alone (Blood 2015;126:233)
Case reports
Treatment
  • Recombinant erythropoietin (EPO) for anemia
  • Iron chelation therapy for iron overload
  • Lusatercept and sotatercept: soluble fusion proteins that increase hemoglobin levels by a different mechanism than EPO (Am J Hematol 2019;94:475)
Clinical images
No information provided
Gross description
No information provided
Gross images
No information provided
Frozen section description
No information provided
Frozen section images
No information provided
Microscopic (histologic) description
  • Microscopic criteria
    • Characterized by cytopenias (1 - 2 MDS RS SLD, 1 - 3 MDS RS MLD)
    • Ring sideroblasts and morphological dysplasia (1 lineage [erythroid] or 2 - 3 lineages), often with concomitant SF3B1 mutations
    • In the absence of SF3B1 mutations, ≥ 15% ring sideroblasts are needed, whereas > 5% ring sideroblasts are required for diagnosis in the presence of SF3B1 mutations
    • Ring sideroblasts are erythroid precursors with ≥ 5 iron granules encircling a third or more of the nucleus (Haematologica 2008;93:1712)
    • Lacks Auer rods, < 5% bone marrow blasts, < 2% peripheral blood blasts
    • Does not meet criteria for MDS with isolated del (5q); other cytogenetic abnormalities allowed
  • Bone marrow histology
    • Bone marrow general
      • Hypercellular or normocellular marrow with erythroid hyperplasia
    • Bone marrow erythroid
      • Erythroid hyperplasia (may be mild to moderate dysplasia, including irregular nuclear contours, nuclear budding, megaloblastic change), markedly increased iron stores
    • Bone marrow myeloid
      • Myeloblasts < 5% (or call MDS with excess blasts)
    • Bone marrow megakaryocytes
      • Normal numbers and morphology (in SLD)
    • In addition to presence of ring sideroblasts and erythroid hyperplasia and dysplasia, > 10% dysplastic cells within the myeloid or megakaryocytic series in multilineage dysplasia subtype
  • References: Mediterr J Hematol Infect Dis 2015;7:e2015035, Blood 2016;127:2391
Microscopic (histologic) images

Contributed by Cecilia C.S. Yeung, M.D.
Erythroid predominance

Erythroid predominance

Erythroid hyperplasia and dyserythropoiesis

Erythroid hyperplasia and dyserythropoiesis

Multilineage dysplasia

Multilineage dysplasia

Ring sideroblasts

Ring sideroblasts



Bone marrow aspirate, iron stain

Bone marrow aspirate, iron stain

Bone marrow aspirate, Wright-Giemsa stain Bone marrow aspirate, Wright-Giemsa stain

Bone marrow aspirate, Wright-Giemsa stain

Virtual slides
No information provided
Cytology description
Cytology images
No information provided
Peripheral smear description
  • Peripheral blood
    • Dimorphic red blood cell populations, normal and microcytic hypochromic
    • Occasional coarse basophilic stippling and Pappenheimer bodies
    • < 1% myeloblasts
    • Evaluate for cytopenias and granulocytic dysplasia to subclassify
  • Reference: Blood 2020;136:157
Peripheral smear images

AFIP images
Peripheral blood

Dimorphic red blood cells

Positive stains
Negative stains
No information provided
Flow cytometry description
No information provided
Flow cytometry images
No information provided
Electron microscopy description
  • Transmission electron microscopy analysis of ring sideroblasts reveals features of apoptosis with condensed nuclear chromatin and fragmentation of the erythroid cells due to iron deposition in mitochondria (Oncogene 2006;25:4757)
Electron microscopy images

AFIP images
Erythroid precursor shows iron deposits

Erythroid precursor shows iron deposits

Molecular / cytogenetics description
  • Mutation in the spliceosome gene SF3B1 is found in 90% of cases, usually at a high VAF (35 - 43%)
  • 50% of SF3B1 mutations affect codon 700, with other hotspots at codons 666, 662, 622, 625
  • Presence of comutations involving BCOR, BCORL1, NRAS, RUNX1, SRSF2 or STAG2 are associated with significantly different outcomes in comparison to mutation in SF3B1 alone (NEJM Evidence 2022;1:EVIDoa2200008)
  • Most cases of MDS SF3B1 have a normal karyotype or abnormalities involving a single chromosome
  • Cases with 5q deletion, monosomy 7/7q deletion or complex karyotype are excluded
Molecular / cytogenetics images
No information provided
Videos
No information provided
Sample pathology report
  • Bone marrow, right posterior iliac crest, aspirate smears, core biopsy, particle prep and peripheral blood smear:
    • Hypercellular marrow with megakaryocytic and erythroid dysplasia, 1+ reticulin deposition and a normal blast count (see comment)
    • Increased ring sideroblasts are present
    • Anemia with no circulating blasts
    • Comment: The sum of the morphological findings (notably, the presence of ringed sideroblasts and a dominant SF3B1 mutation [VAF 44%]) is diagnostic of a myeloid neoplasm and compatible with myelodysplastic neoplasm with low blasts and SF3B1 mutation (MDS SF3B1) under the WHO classification system. Under the International Consensus classification system, this would be called myelodysplastic syndrome with mutated SF3B1 (also called MDS SF3B1). Myelodysplastic / myeloproliferative neoplasm with SF3B1 mutation and thrombocytosis (MDS / MPN SF3B1 T) was a consideration but the lack of thrombocytosis argues against this entity.
  • Cytogenetics
    • Normal male, 46, XY
  • Molecular studies
    • Positive for clinically significant variants: SF3B1 (p.N626S, NM_012433.3:c.1877A>G, VAF 44%)
  • Flow cytometry
    • 3% myeloid blasts (see comment)
    • Comment: Bone marrow flow cytometry reveals presence of 3% myeloid blasts expressing CD33, CD13, CD117 and MPO. There is no abnormal B cell clone identified and B cells constitute approximately 4% of all nucleated cells. Plasma cells comprise 2% of all nucleated cells and normally express CD138 and CD38.
  • Peripheral blood
    • Differential
      • Complete blood count: 3,500/mm³
      • Neutrophils: 60%
      • Lymphocytes: 30%
      • Eosinophils: 4%
      • Monocytes: 4%
      • Basophils: 2%
      • Blasts: 0%
    • Morphology
      • Red blood cells: macrocytic anemia with anisopoikilocytosis
      • White blood cells: leukopenia, no circulating blasts, normal morphology
      • Platelets: mild thrombocytopenia, few giant platelets seen
  • Bone marrow aspirate
    • Differential
      • Erythroid: 65%
      • Myeloid: 32%
      • Lymphocytes: 3%
      • Plasma cells: 2%
      • Blasts: 3%
      • M:E ratio = 1:2
    • Morphology
      • Erythroid: erythroid hyperplasia with features of dyserythropoiesis - binucleation, nuclear budding, nucleocytoplasmic asynchrony
      • Myeloid: normal maturation with mild dysgranulopoiesis
      • Megakaryocytes: normal in numbers with hypolobated and monolobated forms
      • Plasma cells: normal in number and morphology
      • Lymphocytes: normal in number and morphology
      • Iron stain: 40% ring sideroblasts are identified with presence of perinuclear iron granules
  • Bone marrow biopsy
    • Morphology
      • Hypercellular (70% cellularity) bone marrow with erythroid hyperplasia, dyserythropoiesis, dysgranulopoiesis and dysmegakaryopoiesis
      • Reticulin stain shows grade I fibrosis
Differential diagnosis
  • MDS / MPN with SF3B1 mutation and thrombocytosis:
    • Anemia associated with dysplastic erythropoiesis and ≥ 15% ring sideroblasts
    • Persistent thrombocytosis with platelet count ≥ 450 x 109/L
    • Detection of SF3B1 mutation concurrently with JAK2 p.V617F
  • Various nonclonal conditions can present with ring sideroblasts in the bone marrow; thorough clinical history and absence of diagnostic criteria for clonal MDS RS (> 15% RS or SF3B1 mutation) are essential for differential diagnosis
    • Genetically inherited congenital sideroblastic anemia (Br J Haematol 2016;174:847):
      • Characterized by mutations in 1 of 3 mitochondrial pathways: heme synthesis, iron sulfur cluster biogenesis and protein synthesis
      • X linked sideroblastic anemia is the most common form attributed to mutation in aminolevulinic acid (ALA) synthase
      • Microcytic hypochromic anemia and iron overload
    • Acquired sideroblastic anemia due to chronic alcohol use (Postgrad Med 1992;92:147):
      • History of chronic alcoholism
      • Elevated serum aspartate aminotransferase (AST):alanine aminotransferase (ALT) ratio
      • Macrocytic anemia
      • < 15% ring sideroblasts
    • Drug induced sideroblastic anemia - chloramphenicol, penicillamine, linezolid, isoniazid (Haematologica 2013;98:e138):
      • History of medication use
      • May have pure red cell aplasia
      • Reversible after discontinuation of drug
    • Lead poisoning:
      • High serum lead levels, lead lines (Burton lines) on teeth and gingiva
      • < 15% ring sideroblasts
      • Basophilic stippling in red cells on peripheral smear, microcytic and hemolytic anemia
    • Copper deficiency / zinc toxicity (Clin Case Rep 2015;3:325):
      • Low serum copper levels or high serum zinc levels
      • < 15% ring sideroblasts
      • Cytoplasmic vacuoles in erythroid and myeloid cells
  • Reference: Am J Hematol 2021;96:379
Board review style question #1

Bone marrow aspirate with iron stain is shown in the image above. Which molecular alteration is most commonly observed in patients who have myelodysplastic syndrome (MDS) with this histologic feature?

  1. Isolated del 5q
  2. JAK 617F mutation
  3. Monosomy 7
  4. SF3B1 mutation
Board review style answer #1
D. SF3B1 mutation. SF3B1 mutations are associated with ringed sideroblasts in almost 90% of cases. Answer A is incorrect because isolated del 5q may or may not be associated with ring sideroblasts and these MDS cases are classified as MDS with 5q deletion irrespective of ring sideroblasts. Answer B is incorrect because JAK 617F mutation is frequently associated with myeloproliferative neoplasms like polycythemia vera and essential thrombocytosis; however, some cases of myelodysplastic / myeloproliferative neoplasm with SF3B1 mutation and thrombocytosis can have JAK2 V617F comutation and these cases show the presence of ring sideroblasts. Answer C is incorrect because MDS with monosomy 7 is not usually associated with ring sideroblasts.

Comment Here

Reference: MDS with ring sideroblasts
Board review style question #2
Which of the following statements is true regarding myelodysplastic syndrome (MDS) with ring sideroblasts?

  1. MDS with mutated SF3B1 has poor disease outcomes as compared to other subtypes of MDS
  2. Presence of excess blasts has a worse prognosis, even in the presence of SF3B1 mutation
  3. Presence of ring sideroblasts is an essential diagnostic requirement for MDS with mutated SF3B1
  4. Ring sideroblasts as well as SF3B1 mutation are not detected in other MDS types or other myeloid neoplasms
Board review style answer #2
B. Presence of excess blasts has a worse prognosis, even in the presence of SF3B1 mutation. The favorable outcome associated with SF3B1 mutation is lost as soon as an excess of blasts is observed. Answer C is incorrect because the presence of ring sideroblasts is not a diagnostic requirement if SF3B1 mutation is present at allele fraction > 35%. Ring sideroblasts are seen in more than 90% of these cases; however, if SF3B1 is not available, > 15% ring sideroblasts are required for diagnosis of MDS with low blasts and ring sideroblasts. Answer A is incorrect because MDS with low blasts and SF3B1 mutation has a better prognosis than other MDS types with low blasts. Answer D is incorrect because ring sideroblasts as well as SF3B1 mutation can be detected in other MDS types or other myeloid neoplasms, including acute myeloid leukemia and MDS with increased blasts. Cases that fulfill the criteria for MDS with low blasts and 5q deletion should be classified as such, even if ring sideroblasts or SF3B1 mutation are identified. In patients who present with thrombocytosis, detection of SF3B1 mutation concurrently with JAK2 p.V617F, CALR or MPL mutations qualifies for a diagnosis of myelodysplastic / myeloproliferative neoplasm with SF3B1 mutation and thrombocytosis.

Comment Here

Reference: MDS with ring sideroblasts

MDS with mutated TP53
Definition / general
  • Myelodysplastic syndrome (MDS) with mutated TP53 is characterized by ≥ 1 TP53 mutation or mutation with loss of TP53 function resulting in ≥ 1 cytopenia and either blasts in the blood or bone marrow (International Consensus Classification ([ICC]) or single / multilineage dysplasia (WHO 5th edition [WHO HAEM5])
  • Other MDS subtypes should not be diagnosed in patients with multihit TP53 (ICC)
Essential features
  • Myelodysplastic neoplasm (MDS) with multihit TP53 mutation is associated with aggressive behavior with a rapid transformation to acute myeloid leukemia (AML)
  • Poor prognosis, median overall survival of 0.7 years
  • Genomic testing is essential to make the diagnosis
  • No morphologic features specific to MDS with mutated TP53
  • Reference: Cancer Discov 2022;12:2954
Terminology
  • Myelodysplastic neoplasm (MDS) (WHO 5th edition [WHO HAEM5])
  • Myelodysplastic syndrome (MDS) (International Consensus Classification ([ICC])
  • MDS with mutated TP53 (MDS-TP53) (ICC)
  • MDS with biallelic TP53 inactivation (MDS-biTP53) (WHO HAEM5)
ICD coding
  • ICD-10: D46.9 - myelodysplastic syndrome, unspecified
Epidemiology
  • Median age: 69.7 years
  • Increased frequency of TP53 mutation in patients following cytotoxic chemotherapy or radiation therapy
  • ~10% of all cytopenias
  • Reference: Blood Adv 2022;6:2847
Sites
  • Bone marrow and peripheral blood
Pathophysiology
  • Mutations affecting TP53 function, a tumor suppressor gene that plays a critical role in cellular apoptosis (Blood Rev 2023;60:101055)
Etiology
Diagrams / tables
Not relevant to this topic
Clinical features
  • Symptoms associated with cytopenias
    • Anemia
      • Fatigue, shortness of breath, pallor
    • Thrombocytopenia
      • Purpura, bleeding, easy bruising
    • Neutropenia
      • Infections
  • Reference: Blood 2023;142:2247
Diagnosis
  • WHO HAEM5
    • Myelodysplastic neoplasms with defining genetic abnormalities
      • MDS with biallelic TP53 inactivation
        • < 20% blasts in peripheral blood and bone marrow, usually complex cytogenetics, ≥ 2 TP53 mutations or 1 mutation with TP53 copy number loss or copy neutral loss of heterozygosity
  • ICC
    • MDS cases with defining cytogenetic or molecular criteria along with blast percentage and cytopenias, no requirement for dysplasia
      • MDS with mutated TP53
        • 0 - 9% peripheral blood / bone marrow blasts, multihit TP53 mutations or TP53 mutation with variant allele frequency (VAF) > 10% and complex karyotype and often with loss of 17p
      • MDS / AML with mutated TP53
        • 10 - 19% bone marrow or peripheral blood blasts with any somatic TP53 mutation with VAF ≥ 10%
  • Reference: Virchows Arch 2023;482:39, Ann Lab Med 2023;43:503
Laboratory
  • ≥ 1 cytopenia
  • Reticulocytopenia is commonly found in MDS
Prognostic factors
  • TP53 mutation is the strongest prognostic factor in MDS (Cancer Discov 2022;12:2954)
    • Associated with treatment resistant disease, rapid disease progression and low overall survival
    • 3 year transformation to AML rate: 40%
Case reports
Treatment
  • Stem cell transplant associated with increased overall survival (OS = 20.6 months) (N Engl J Med 2017;376:536)
    • Median relapse free survival of 14.5 months
Clinical images
Not relevant to this topic
Gross description
Not relevant to this topic
Gross images
Not relevant to this topic
Frozen section description
Not relevant to this topic
Frozen section images
Not relevant to this topic
Microscopic (histologic) description
  • Multilineage dysplasia in most cases
  • Increased erythroid precursors common
  • Occasional bone marrow fibrosis
  • Reference: Blood 2023;142:2247
Microscopic (histologic) images

Contributed by Hamzah Rehan, D.O.
Hypercellular bone marrow

Hypercellular bone marrow

Dysplastic megakaryocytes

Dysplastic megakaryocytes

Clot section

Clot section

CD34

CD34

Virtual slides
None
Cytology description
  • Multilineage dysplasia
Cytology images

Contributed by Hamzah Rehan, D.O.
Aspirate Smear

Aspirate smear

Erythroid budding

Erythroid budding

Peripheral smear description
  • Red blood cells
    • Anisopoikilocytosis with macrocytosis of red blood cells
  • White blood cells
    • Hyposegmentation, lack of lobation, nuclear clumping, cytoplasmic hypogranularity or agranularity
  • Reference: Blood 2023;142:2247
Peripheral smear images
None
Positive stains
  • p53 IHC staining of > 2% is associated with TP53 mutation (Leuk Res 2016;41:21)
    • Confirmation by molecular and cytogenetic testing
    • p53 good screening tool but not specific
    • MDS with truncation types of mutations may have virtual absence of p53
Negative stains
Not relevant to this topic
Flow cytometry description
  • Flow cytometry alone is not diagnostic
    • Decreased side scatter of granulocytes
    • Erythroid: increased coefficient of variation and decreased intensity of CD71 or CD36
    • Aberrant expression of CD56 or CD7 on progenitors, granulocytes or monocytes
    • Asynchrony of CD15 and CD16 on granulocytes
    • Altered expression of CD13 in relation to CD11b or CD16
  • Reference: Ann Hematol 2023;102:3015
Flow cytometry images
Not relevant to this topic
Electron microscopy description
Not relevant to this topic
Electron microscopy images
None
Molecular / cytogenetics description
  • Cytogenetics
    • Usually accompanied by complex karyotype
      • del(5q), 17p loss of heterozygosity (LOH), -7/del(7q), -18
      • Other mutations as well
  • Molecular genetics
    • TP53 mutations
      • Commonly in DNA binding domain, inactivating
      • Copy number status can be detected by FISH probe
  • Reference: Blood 2022;139:2347
Molecular / cytogenetics images
None
Videos
None
Sample pathology report
  • Bone marrow, left posterior iliac crest; core biopsy, clot section, aspirate smears and touch imprint:
    • Myelodysplastic neoplasm with low blasts
      • Hypercellular marrow with trilineage hematopoiesis showing multilineage dysplasia
      • Aspirate smear: blasts 3%, lymphocytes 11%
      • Cytogenetics: -7/del(7q)
      • Next generation sequencing: multiallelic TP53 mutation, deletion 5q and 20q
Differential diagnosis
Additional references
None
Board review style question #1

A 71 year old man presenting with anemia and thrombocytopenia underwent a bone marrow biopsy (H&E image shown above) of his left posterior iliac crest. Which of the following mutations is associated with the worst prognosis?

  1. 5q deletion
  2. JAK2 V617F
  3. Multihit TP53 inactivation
  4. SF3B1
Board review style answer #1
C. Multihit TP53 inactivation. Of the listed mutations, only TP53 has a very poor prognosis. Myelodysplastic syndrome (MDS) with TP53 mutation is treatment resistant and has a median survival of 0.7 months. Of the listed choices, it has the highest rate of transformation to acute myeloid leukemia (AML). Answer A is incorrect because MDS with 5q deletion without TP53 abnormality has a longer overall survival and a lower risk of transformation to AML. These patients can have near normal levels of bone marrow cellularity. Answer B is incorrect because JAK2 V617F mutation less commonly transforms to AML and has a therapeutic target in JAK2. Answer D is incorrect because SF3B1 mutation is an independent predictor of favorable prognosis, has a 9 year average survival with isolated mutation and has a very low risk of leukemic evolution.

Comment Here

Reference: MDS with mutated TP53
Board review style question #2
A 63 year old man presents with a history of myelodysplastic syndrome (MDS) with mutated TP53. Which of the following is a risk factor for his disease?

  1. 30 pack year smoking history
  2. Brother with 3 prior basal cell carcinomas and a prior meningioma
  3. Deceased donor kidney transplant
  4. HIV infection
Board review style answer #2
B. Brother with 3 prior basal cell carcinomas and a prior meningioma. The patient's brother is described as having multiple soft tissue neoplasms, consistent with germline p53 mutation. The patient's brother has Li-Fraumeni syndrome, an autosomal dominant mutation of the p53 tumor suppressor gene. A mutation of this gene predisposes patients to a wide range of cancers, including hematological malignancies. Answer A is incorrect because tobacco use does not increase the risk of MDS with TP53 mutation. Answer C is incorrect because immunosuppression does not increase the risk of MDS with TP53 mutation. Answer D is incorrect because HIV or antiretroviral therapy does not increase the risk of MDS with TP53 mutation.

Comment Here

Reference: MDS with mutated TP53

MDS with mutated TP53 (pending)
[Pending]

MDS with single lineage dysplasia
Definition / general
  • Myelodysplastic syndrome with single lineage dysplasia (MDS SLD) is defined as persistent unexplained cytopenias (anemia [most common], neutropenia or thrombocytopenia) or bicytopenia with ≥ 10% dysplastic cells in 1 myeloid lineage
Essential features
  • Per 2016 revised WHO classification, MDS SLD includes:
    • Peripheral blood with anemia (Hb < 10 g/dL) or neutropenia (< 1.8 x 109/L) or thrombocytopenia (< 100 x 109/L) or bicytopenia with < 1% blasts
    • Bone marrow (BM) with ≥ 10% dysplastic cells in one myeloid lineage
    • MDS with erythroid dysplasia only and 5 - 14% ring sideroblasts in absence of SF3B1 mutation
  • Reference: Blood 2016;127:2391
Terminology
  • According to 2008 WHO classification, it was known as refractory cytopenia with unilineage dysplasia (refractory anemia, refractory neutropenia or refractory thrombocytopenia)
ICD coding
  • ICD-O: 9980/3 - refractory anemia
  • ICD-10:
    • D46.0 - refractory anemia without ring sideroblasts
    • D46.4 - refractory anemia, unspecified
Epidemiology
  • Incidence: 7 - 20% of all MDS cases
  • Median age of onset: 65 - 70 years
  • M = F
Sites
  • Peripheral blood and bone marrow
Etiology
  • Risk factors:
    • Benzene exposure (J Natl Cancer Inst 2012;104:1724)
    • Cigarette smoking (also in part due to benzene in cigarette smoke)
    • Agricultural solvents or chemicals
    • Family history of hematopoietic neoplasms
    • Inherited hematological disorders, such as Fanconi anemia, dyskeratosis congenita, Shwachman-Diamond syndrome (SDS) and Diamond-Blackfan anemia
    • Acquired aplastic anemia (Blood 2002;100:786)
Diagrams / tables

Contributed by Karan Saluja, M.D. and Beenu Thakral, M.D.

Classification algorithm for MDS SLD

Clinical features
  • Clinical presentation may include fatigue, infection or petechiae related to the type and degree of cytopenia(s)
Diagnosis
  • MDS is a comprehensive diagnosis made by review of complete clinical history, complete blood count, peripheral blood smear findings, bone marrow morphology assessment, flow cytometry immunophenotyping (if available), cytogenetics and molecular findings
  • Review of:
    • Complete clinical history (personal and family history), history of chemotherapy or radiation exposure
    • Medications that can contribute to cytopenia or dysplasia
    • Peripheral blood smear for blasts percentage, absolute monocyte count, monocyte percentage, degree of dysplasia and number of lineages involved
    • Bone marrow morphology should include cellularity, blast percentage, degree of dysplasia, ring sideroblasts percentage (if any), presence or absence of fibrosis
  • Flow cytometry to evaluate for aberrancies in CD34 positive myeloblast population and abnormal myelomonocytic maturation; in addition, any monoclonal B cell population or immunophenotypically abnormal T cells or NK cell population, if present, can also be assessed
  • Cytogenetic abnormality in the absence of morphologic criteria, such as gain of chromosome 8, del 20q and loss of Y chromosome, is not considered definitive evidence of MDS, according to the WHO 2016 revised classification; in the setting of persistent cytopenia of undetermined origin, other MDS defining cytogenetic abnormalities are considered presumptive evidence of MDS, even in the absence of definitive morphologic features
  • MDS associated somatic mutations identified alone in the absence of morphologic features of dysplasia are not considered diagnostic evidence of MDS, according to the current WHO 2016 revised classification
  • References: Hematology Am Soc Hematol Educ Program 2015;2015:294, Pathobiology 2019;86:7
Laboratory
  • Peripheral blood cytopenias with anemia (Hb < 10 g/dL) or neutropenia (< 1.8 x 109/L) or thrombocytopenia (< 100 x 109/L) or bicytopenia and < 1% blasts
Prognostic factors
  • Prognosis in MDS is determined using IPSS-R diagnostic scoring system, which takes into account cytogenetic findings, bone marrow blast percentage, hemoglobin, platelet count and absolute neutrophil count (ANC)
  • 5 risk groups:
    • Very low risk: ≤ 1.5
    • Low: > 1.5 - 3.0
    • Intermediate: > 3.0 - 4.5
    • High: > 4.5 - 6.0
    • Very high: > 6.0
  • References: Blood 2012;120:2454, J Natl Compr Canc Netw 2015;13:261


IPPS-R scoring for MDS
Prognostic variables Score value
0 0.5 1.0 1.5 2.0 3.0 4.0
Karyotype Very good - Good - Intermediate Poor Very poor
Percentage of bone marrow blasts ≤ 2% - > 2% to < 5% - 5 - 10% > 10% -
Hemoglobin (Hb) (g/dL) ≥ 10 - 8 to < 10 < 8 - - -
Platelets (x109/L) ≥ 100 50 to < 100 < 50 - - - -
Absolute neutrophil count (ANC) (x109/L) ≥ 0.8 < 0.8 - - - - -
Cytogenetic prognostic group:
  • Very good: loss of Y and del(11q)
  • Good: normal, del(5q), del(12p), del(20q), double, including del(5q)
  • Intermediate: del(7q), gain of chromosome 8, 19, i17q, ≥ 2 independent noncomplex clones
  • Poor: loss of chromosome 7, inv3, t(3q), del(3q), double, including loss of chromosome 7 or del(7q), complex (3 abnormalities)
  • Very poor: complex (> 3 abnormalities)
Case reports
  • 46 year old man and his daughter presented with a novel germline frameshift GATA2 mutation presenting as congenital sensorineural hearing loss and familial MDS (Int J Hematol 2021;114:286)
  • 53 year old man with history of shortness of breath, dizziness and fatigue (pernicious anemia with spuriously normal vitamin B12 level might be misdiagnosed as myelodysplastic syndrome) (Clin Lymphoma Myeloma Leuk 2014;14:e141)
  • 55 year old man with history of fatigue and generalized weakness (zinc induced copper deficiency: a diagnostic pitfall of myelodysplastic syndrome) (Pathology 2014;46:246)
Treatment
  • Low grade MDS can be observed if the patient is asymptomatic with mild cytopenias
  • ESA (erythropoietin stimulating agent) to treat anemia
  • Hypomethylating agents (azacitidine or decitabine) if the patient is symptomatic, with or without moderate to severe cytopenias
  • Reference: Cancers (Basel) 2021;13:784
Gross description
  • Adequate morphologic assessment for MDS requires high quality, good stained aspirate smears prepared from fresh aspirate specimen (< 2 hours); bone marrow core biopsy should be of adequate length (≥ 1.5 cm) and good quality peripheral blood smear
Microscopic (histologic) description
  • Morphologic features of dysplasia in erythroid, myeloid or megakaryocytic lineages

Dyserythropoiesis Dysgranulopoiesis Dysmegakaryopoiesis
Nuclear budding or multinucleation Small or unusually large size granulocytes Micromegakaryocytes
Internuclear bridging Nuclear hyposegmentation (pseudo Pelger-Huët) Nuclear hypolobation
Karyorrhexis Nuclear hypersegmentation

Multinucleation or widely
separated nuclear lobes
Megaloblastoid changes Hypogranularity; agranularity
Presence of ring sideroblasts Pseudo Chediak-Higashi granules
Cytoplasmic vacoulization Auer rod
Microscopic (histologic) images

Contributed by Beenu Thakral, M.D.

Hypercellular BM and megakaryocytic dysplasia

Erythroid dysplasia

Ring sideroblasts (Pearl stain)


Myeloid dysplasia

Myeloid dysplasia


Megakaryocytic dysplasia

Megakaryocytic dysplasia

Peripheral smear description
  • Red blood cells are usually normocytic or macrocytic and normochromic with anisopoikilocytsois
  • Granulocytes may show hypogranularity, uneven distribution of cytoplasmic granules, pseudo Pelger-Huët anomaly
  • Blasts are usually < 1%
  • Subset of platelets may be hypogranular
Positive stains
  • CD34 highlights scattered blasts are not increased (< 5%)
  • CD117 highlights no increase in immature cells
  • CD61 / CD42b highlights megakaryocytes (useful in highlighting increased dysplastic hypolobated megakaryocytes)
Negative stains
  • CD3 highlights scattered T cells
  • CD20 highlights scattered B cells
Flow cytometry description
Flow cytometry images

Contributed by Beenu Thakral, M.D.

CD45 versus side scatter

CD19 versus CD33

CD7 versus CD10

CD117 versus CD34

Molecular / cytogenetics description
  • Cytogenetics findings (J Clin Oncol 2012;30:820):
    • Conventional karyotyping should be performed in all MDS cases at diagnosis, as it defines prognosis
    • Cytogenetic abnormalities are seen in 50 - 60% of MDS; these abnormalities usually include del20q, gain in chromosome 8 or abnormalities in chromosome 5 and 7
    • MDS SLD with isolated thrombocytopenia, the presence of cytogenetic or molecular abnormality is helpful in distinguishing MDS from immune mediated thrombocytopenia

MDS defining cytogenetic abnormalities
Unbalanced cytogenetic abnormalities
  • del(5q) or t(5q)
  • i(17q) or t(17p)
  • -13 or del(13q)
  • del(11q)
  • del12p or t(12p)
  • del(9q)
  • idic(X)(q13)
Balanced cytogenetic abnormalities
  • t(11;16)(q23; p13.3)
  • t(3;21)(q26.2;q22.1)
  • t(1;3)(p36.3;q21.2)
  • t(2;11)(p21;q23)
  • inv(3)(q21q26.2)/t(3;3)(q21q26.2)
  • t(6;9)(p23;q34)

  • Molecular findings (Int J Lab Hematol 2020;42:671, Blood 2012;119:3578, Leuk Res 2015;39:6):
    • Recurrent somatic mutations are identified in 80 - 90% of MDS cases
    • Most commonly mutated genes in MDS include RNA splicing (SF3B1, SRSF2, U2AF1 and ZRSR2) or epigenetic regulation of gene expression via DNA methylation (TET2, DNMT3A, IDH1 and IDH2) or genes involved in histone modification (ASXL1 and EZH2)
    • Specific mutations associated with morphologic features and MDS include SF3B1 mutation with ring sideroblasts and mutations in ASXL1, RUNX1, TP53 and SRSF2 are associated with severe granulocytic dysplasia
    • Acquired clonal mutations identical to those seen in MDS (TET2, DNMT3A, SF3B1, ASXL1 and JAK2) can also occur in hematopoietic cells of apparently healthy older individuals without MDS; thus, the presence of MDS associated somatic mutations alone are not considered diagnostic of MDS even in patients with unexplained cytopenia


Genes commonly mutated in MDS
Pathway
Frequency
Prognosis
SF3B1 RNA splicing 20 - 30% Favorable
TET2 DNA methylation 20 - 30% Neutral
ASXL1 Histone modification 15 - 20% Adverse
SRSF2 RNA splicing 15% Adverse
DNMT3A DNA methylation 10% Adverse
RUNX1 Transcription factor 10% Adverse
U2AF1 RNA splicing 5 - 10% Adverse
TP53 Tumor suppressor 5 - 10% Adverse
EZH2 Histone modification 5 - 10% Adverse
ZRSF2 RNA splicing 5 - 10% Neutral
STAG2 Cohesion complex 5 - 7% Adverse
IDH1 / IDH2 DNA methylation 5% Neutral
CBL Signaling 5% Adverse
NRAS Transcription factor 5% Adverse
BCOR Transcription factor 5% Adverse
Sample pathology report
  • Bone marrow, right posterior iliac crest, core biopsy, clot section, aspirate smears, touch imprint and peripheral blood smear:
    • Myelodysplastic syndrome with single lineage dysplasia, 1% blast (see comment)
    • Comment: Flow cytometry immunophenotyping performed on bone marrow aspirate shows an aberrant myeloblast population (0.8% of total events) with the following immunophenotype: CD7 partial, CD13+, CD33+, CD34+, CD117 inc and CD38 dec. Hematogones are absent. No monoclonal B cells or immunophenotypically abnormal T cells are identified.
    • Review of peripheral blood smear shows moderate macrocytic anemia with slight anisopoikilocytosis and no circulating blast.
    • Cytogenetic study shows a normal female karyotype: 46,XX[20]
    • Next generation sequencing identified mutations in U2AF1 and TET2 genes.
    • The current bone marrow specimen shows hypercellular (80%) bone marrow with erythroid predominant trilineage hematopoiesis, dyserythropoiesis, 3% ring sideroblasts and no increase in blasts; this is consistent with myelodysplastic syndrome with single lineage dysplasia (WHO 2016 revised classification).
Differential diagnosis

Characteristics of ICUS, CHIP / ARCH, CCUS and MDS
Idiopathic cytopenia of uncertain significance
(ICUS)
Clonal hematopoiesis of indeterminate potential or age related clonal hematopoiesis
(CHIP / ARCH)
Clonal cytopenia of uncertain significance
(CCUS)

Myelodysplastic syndrome
(MDS)
Clonality
-
+
+
+
Cytopenia
+
-
+
+
Dysplasia
-
-
-
+
Bone marrow blasts
< 5%
< 5%
< 5%
< 20%
Cytogenetic abnormality
No
No
Can be seen but not MDS defining
Present in 50 - 60% of cases, can be MDS defining
Typical number of mutated genes
0
1
1.5 - 2
2.6 - 17
Commonly mutated genes

NA
DNMT3A, TET2, ASXL1 (DTA) genes
DNMT3A, TET2, ASXL1, SRSF2, TP53 or U2AF1
SF3B1, SRSF2, U2AF1, TET2, DNMT3A, ASXL1, EZH2, IDH1, IDH2, BCOR, BCORL1, RUNX1, TP53, PHF6, NRAS, KRAS
Typical VAF (variants with allele frequency)
NA
2 - 10%
20 - 30%
> 30%
Risk of developing MDS or acute myeloid leukemia

Very low

Low

High

NA

Board review style question #1
A 50 year old man presented with macrocytic anemia with no known medical history. Serum vitamin B12 and folate levels are normal. A bone marrow biopsy shows hypercellular (80%) bone marrow with erythroid predominance, dyserythropoiesis and 10% ring sideroblasts with 2% blasts on aspirate smears. Peripheral blood smear review shows macrocytic anemia and no circulating blasts. Cytogenetics shows a diploid karyotype. Next generation sequencing identified mutation in the U2AF1 gene. According to the current WHO 2016 revised classification, how should this case of MDS be classified?

  1. Myelodysplastic syndrome, unclassifiable
  2. Myelodysplastic syndrome with del5q
  3. Myelodysplastic syndrome with ring sideroblasts
  4. Myelodysplastic syndrome with single lineage dysplasia
Board review style answer #1
D. Myelodysplastic syndrome with single lineage dysplasia

Comment Here

Reference: MDS with single lineage dysplasia
Board review style question #2
Which of the following RNA splicing gene is commonly mutated in MDS?

  1. ASXL1
  2. RUNX1
  3. SF3B1
  4. STAG2
  5. TP53
Board review style answer #2

MDS, hypoplastic (pending)
[Pending]

MDS-general
Definition / general
  • Myelodysplastic neoplasms (MDS), previously called myelodysplastic syndromes, are clonal hematopoietic stem cell neoplasms characterized by ineffective marrow hematopoiesis that results in morphologic dysplasia and peripheral cytopenia; there is increased risk of transformation to acute myeloid leukemia (AML)
  • This topic discusses myelodysplastic syndromes / neoplasms based on the WHO 2022 classification; differences between WHO 2022 and the ICS classification system are noted in Diagrams / tables below
Essential features
  • The definition of cytopenia comes from clonal cytopenia of undetermined significance (CCUS), MDS and myelodysplastic / myeloproliferative neoplasm (MDS / MPN)
  • 2 main categories are accepted for subclassification of MDS
    • MDS with defining genetic abnormalities, which is encompassed by MDS-5q, MDS-SF3B1 and MDS with biallelic TP53
    • MDS, morphologically defined, consisting of MDS with low blasts, MDS with increased blasts and MDS, hypoplastic
    • There is also a third category named MDS of childhood
  • The previous subclassification of MDS-U has been eliminated from the new version of WHO classification
  • Clarifications for the 2022 WHO MDS diagnoses
    • Criteria for cytopenia
      • Anemia: hemoglobin < 13 g/dL in men, < 12 g/dL in women
      • Leukopenia: absolute neutrophil count < 1.8 x 109/L
      • Thrombocytopenia: low platelet count < 150 x 109/L
      • Threshold for dysplasia remains at 10% dysplastic cells
  • Reference: Blood 2016;127:2391
Terminology
  • Per the fifth edition of WHO published in 2022, the terminology of myelodysplastic neoplasms has been introduced to replace the previous term of myelodysplastic syndromes (MDS), in parallel to myeloproliferative neoplasm and to emphasize the neoplastic course of the disease; however, the abbreviation of MDS is kept for myelodysplastic neoplasm
  • Increased blasts is a term that has been adopted to replace excess blasts as previously used in the fourth edition of WHO classification
ICD coding
  • ICD-10: D46 - myelodysplastic syndromes
WHO classification (2022) - details
  • Myelodysplastic neoplasms with defining genetic abnormalities
    • Myelodysplastic neoplasm with low blasts and 5q deletion (MDS-5q)
      • Anemia, commonly macrocytic, transfusion dependent, with or without other cytopenias; thrombocytopenia is uncommon, instead, 33% of patients present with thrombocytosis (platelet count ≥ 450 x 109)
      • Blasts < 5% in bone marrow and < 2% in peripheral blood
      • Normo or hypercellular marrow
      • Dysplasia involving megakaryocytes with characteristic features (megakaryocytes, intermediate in size, with single lobation), with or without dysplasia involving other lineages (see Microscopic (histologic) images)
      • Often associated with erythroid hypoplasia
      • Ring sideroblasts can be seen
      • Detection of 5q deletion, isolated or with one other cytogenetic aberration but not including monosomy 7 or 7q deletion; not fulfilling diagnostic criteria of AML, MDS with biallelic TP53 inactivation, MDS with increased blasts or MDS / MPN
      • MDS-5q with SF3B1 mutation (20% of the cases) are included in this category, as the SF3B1 mutation is likely a secondary event in this context
    • Myelodysplastic neoplasm with low blasts and SF3B1 mutation (MDS-SF3B1)
      • Detection of SF3B1 mutation with variant allele frequency (VAF) ≥ 5%; lower than the threshold is not considered diagnostic
      • Cytopenia involving ≥ 1 lineages, most commonly anemia, without thrombocytosis (presence of thrombocytosis and SF3B1 without del(5q) belongs to MDS / MPN with SF3B1 mutation)
      • Blasts < 5% in bone marrow and < 2% in peripheral blood
      • Erythroid preponderance, mild left shifted, with megaloblastoid maturation or dysplasia; single or multilineage dysplasia can be observed in the subcategory, without impact on diagnosis or prognosis (Blood 2015;126:233, Blood 2020;136:157)
      • Ring sideroblasts are often identified; however, an absence of ring sideroblasts in the setting does not exclude the diagnosis
      • If SF3B1 mutation analysis is not available, demonstration of ring sideroblasts comprising ≥ 15% of erythroid precursors can be used as a surrogate
      • Absence of 5q deletion, monosomy 7 / 7q deletion or complex karyotype
      • Not fulfilling diagnostic criteria of AML, MDS with low blasts and 5q deletion, MDS with biallelic TP53 inactivation, MDS with increased blasts or any MDS / MPN type
    • Myelodysplastic neoplasm with biallelic TP53 inactivation (MDS-biTP53)
      • Detection of ≥ 1 TP53 mutations, meeting molecular diagnostic criteria of biallelic TP53 mutations (see Diagnosis)
      • Cytopenia, dysplasia and < 20% blasts or 30% erythroblasts in bone marrow
      • Next generation sequencing analysis or Sanger sequencing covering at least exons 4 - 11 of TP53 gene is required for detection of biallelic TP53 alterations, coupled with a technique to detect copy number status, usually fluorescence in situ hybridization (FISH) with a probe set specific for the TP53 locus on 17p13.1 (Leukemia 2014;28:241, Blood 2013;122:3616)
      • Detection of ≥ 2 TP53 mutations, usually affecting both alleles that can be considered multihit status (Nat Med 2020;26:1549)
      • After exclusion of constitutional changes, a TP53 VAF > 49% may be regarded as presumptive (not definitive) of copy loss on the trans allele or copy neutral loss of heterozygosity (LOH)
      • In the presence of one TP53 mutation, evidence of TP53 copy loss or copy neutral LOH is required as concurrent 17p LOH, suggestive of biallelic TP53 alterations (Nat Med 2021;27:1239, Nat Commun 2020;11:4980)
      • Characterized by increased blast counts, higher risk of leukemic transformation and higher risk of mortality
  • Myelodysplastic neoplasms, morphologically defined
    • Myelodysplastic neoplasm with low blasts (MDS-LB)
      • Low blasts are defined as < 5% in bone marrow and < 2% in peripheral blood
      • Cytopenia involving ≥ 1 lineages
      • Dysplastic changes in ≥ 1 lineages, involving at least 10% of cells
      • Blasts < 5% in bone marrow and < 2% in peripheral blood
      • Need to exclude folate and vitamin B12 deficiency
      • No fulfilling diagnostic criteria of MDS with defining genetic alterations or hypoplastic MDS
    • Myelodysplastic neoplasm, hypoplastic (h-MDS)
      • Hypocellular bone marrow (adjusted for age of the patient); drug / toxin exposure or pertinent nutritional deficiency need to be excluded
      • Cytopenia involving ≥ 1 lineages
      • Dysplasia involving myeloid or megakaryocytic lineages
      • Blasts < 5% in bone marrow and < 2% in peripheral blood
      • Not meeting criteria for MDS with defining genetic abnormalities or MDS with increased blasts
    • Myelodysplastic neoplasm with increased blasts (MDS-IB)
      • Cytopenia involving ≥ 1 lineages
      • Dysplastic changes in ≥ 1 lineages, involving at least 10% of cells
      • Blasts ≥ 5% in bone marrow or ≥ 2% in peripheral blood
      • No fulfilling diagnostic criteria of MDS with biallelic TP53 inactivation or AML
      • Subclassification
        • MDS with increased blasts 1 (MDS-IB1): 5 - 9% blasts in bone marrow or 2 - 4% blasts in peripheral blood, without significant reticulin fibrosis
        • MDS with increased blasts 2 (MDS-IB2): 10 - 19% blasts in bone marrow or 5 - 19% blasts in peripheral blood, without significant reticulin fibrosis; or with presence of Auer rods
        • MDS with increased blasts and fibrosis (MDS-F): 5 - 19% blasts in bone marrow or 2 - 19% blasts in peripheral blood, with significant reticulin fibrosis (defined as grade 2 or 3)
  • Myelodysplastic neoplasms of childhood
    • Childhood myelodysplastic neoplasm with low blasts (cMDS-LB)
      • Myeloid neoplasm with cytopenia and dysplasia arising in children and adolescents (< 18 years of age)
      • Cytopenia involving ≥ 1 lineage
      • Dysplastic changes in ≥ 1 lineage, involving at least 10% of cells
      • Blasts < 5% in bone marrow and < 2% in peripheral blood
      • Meeting at least 1 of the following criteria
        • Detection of clonal cytogenetic or molecular abnormality; monosomy 7 is the most common cytogenetic abnormality
        • Need to exclude other causes of cytopenia (nonneoplastic and some germline mutations)
      • Subclassification
        • cMDS-LB, hypocellular
        • cMDS-LB, not otherwise specified
    • Childhood myelodysplastic neoplasm with increased blasts (cMDS-IB)
      • Myeloid neoplasm with cytopenia and dysplasia arising in children and adolescents (< 18 years of age)
      • Cytopenia involving ≥ 1 lineages
      • Dysplastic changes in ≥ 1 lineages, involving at least 10% of cells
      • Blasts 5 - 19% in bone marrow or 2 - 19% in peripheral blood
      • Need to exclude Down syndrome, juvenile myelomonocytic leukemia and AML with defining genetic abnormalities
ICC classification (2022)
WHO classification (2016)
Epidemiology
  • Median age of MDS is 77 years at diagnosis; < 10% are < 50 years old
  • Incidence of MDS increases progressively with age from 60 to 90 years of age (from 2.2 to 56.8 cases per 100,000) (Blood Rev 2019;34:1)
  • M > F except in MDS with low blasts and 5q deletion (MDS-5q) where F > M (Am J Med 2012;125:S2)
Diagrams / tables

Comparison of guidelines WHO 4th edition, 2016 WHO 5th edition, 2022 ICC 2022
Name of the entity Myelodysplastic syndrome (MDS) Myelodysplastic neoplasms (MDS) Myelodysplastic syndromes (MDS)
Morphologically defined
Lineage MDS-SLD
MDS-MLD
Subclassification using dysplastic lineages removed, replaced with MDS-LB (MDS with low blasts) MDS, NOS-SLD
MDS, NOS-MLD
Excessive blast counts in bone marrow (5 - 9%) or in peripheral blood (2 - 4%) MDS-EB-I# MDS-IB1 MDS-EB*
Excessive blast counts in bone marrow (10 - 19%) or in peripheral blood (5 - 19%) MDS-EB-II# MDS-IB2 MDS-AML**
Others MDS-U cMDS (MDS of childhood)
MDS-h (hypoplastic MDS)
MDS-f (MDS with fibrosis, blasts 5 - 19% in bone marrow, 2 - 19% in peripheral blood)
MDS, NOS (without dysplasia)
Defining genetic abnormalities
SF3B1 mutation MDS-RS-SLD
MDS-RS-MLD
(5% ring sideroblasts if SF3B1 mutated, 15% ring sideroblasts if SF3B1 wild type)
MDS-SF3B1 (MDS with low blasts and SF3B1 mutation or MDS with ring sideroblasts if SF3B1 wild type) MDS-SF3B1 (MDS with mutated SF3B1) or MDS, NOS (with ring sideroblasts and SF3B1 wild type)
del(5q) MDS with isolated del(5q) MDS-5q
(MDS with low blasts and 5q deletion)
[5q deletion alone or with 1 other genetic aberration other than del(7q) / -7]
MDS with del(5q)
[del(5q) is isolated or with up 1 genetic aberration except del(7q) / -7 or multihit TP53]
TP53 Not included in classification MDS-biTP53
(MDS with biallelic TP53 inactivation)
MDS with mutated TP53
Multihit TP53
Includes MDS, MDS / AML, AML

  • WHO 4th edition, 2016
    • MDS-SLD: MDS with single lineage dysplasia
    • MDS-MLD: MDS with multilineage dysplasia
    • MDS-RS-SLD: MDS with ring sideroblasts and single lineage dysplasia
    • MDS-RS-MLD: MDS with ring sideroblasts and multilineage dysplasia
    • MDS-EB-I: MDS with excess blast I
    • MDS-EB-II: MDS with excess blast II
    • MDS del(5q): MDS with isolated del(5q)
    • MDS-U: MDS, unclassifiable
    • RCC: refractory cytopenia of childhood
  • WHO 5th edition, 2022
    • MDS-LB: MDS with low blasts
    • MDS-IB1: MDS with increased blasts I
    • MDS-IB2: MDS with increased blasts II
    • cMDS: MDS of childhood
    • MDS-h: hypoplastic MDS
    • MDS-f: MDS with fibrosis
    • MDS-SF3B1: MDS with low blasts and SF3B1 mutation or MDS with RS if SF3B1 wild type
    • MDS-5q: MDS with low blasts and 5q deletion
    • MDS-biTP53: MDS with biallelic TP53 inactivation
  • ICC 2022
    • MDS, NOS-SLD: MDS, not otherwise specified with single lineage dysplasia
    • MDS, NOS-MLD: MDS, not otherwise specified with multilineage dysplasia
    • MDS-EB: MDS with excess blasts
    • MDS-AML: myelodysplastic syndrome / acute myeloid leukemia
    • MDS-SF3B1: MDS with mutated SF3B1
    • MDS, NOS: MDS, not otherwise specified
  • * ICC MDS-EB, MDS with excess blasts (blast count 5 - 9% in bone marrow, 2 - 4% in peripheral blood)
  • ** ICC MDS-AML: myelodysplastic syndrome / acute myeloid leukemia (blast count 10 - 19% in bone marrow and 2 - 4% in peripheral blood)
  • # WHO 4th edition, 2016: MDS-EB including MDS-EB and erythroid preponderance and MDS-EB and fibrosis
Diagnosis
  • If a patient presented with sustained cytopenia and suspected MDS, an extensive clinical and laboratory workup is necessary to exclude other etiologies that could clinically or morphologically mimic MDS (e.g., nutritional deficiency [e.g., folate acid, vitamin B12, copper deficiency], infection [e.g., HIV], medication induced marrow suppression [e.g., cytoreduction therapy], autoimmune disorders [e.g., rheumatoid arthritis, SLE])
  • Imaging study
    • To detect hepatomegaly or splenomegaly, which is usually not present in MDS patients but often seen with primary myelofibrosis or chronic myelomonocytic leukemia
  • Cytogenetic consulting
    • For the patients with germline gene mutations (e.g., DDX41, GATA2, RUNX1) or family history of MDS or other bone marrow failure syndromes
  • Reference: Blood 2016;127:2391
Laboratory
  • A series of laboratory tests should be ordered
    • Routine complete blood count (CBC) data with differential count
    • Reticulocyte percentage: reticulocytopenia is commonly found in bone marrow failure, including MDS
    • Serum vitamin B12, folate level: within normal limits
    • Erythropoietin (EPO) levels are usually lower than normal serum ferritin; elevated in the presence of inflammatory conditions such as rheumatoid arthritis
    • Serum iron levels and total iron binding capacity, along with serum ferritin, may be helpful to exclude iron deficiency anemia
    • HIV screening
    • Screening for paroxysmal nocturnal hemoglobinuria (PNH) and T cell large granular lymphocytic leukemia
    • Microscopic examination of bone marrow and peripheral blood smears to identify morphologic dysplasia and enumerate blasts
    • Chromosome analysis, e.g., conventional karyotyping, fluorescence in situ hybridization using specific probes, most commonly including del(5q) / -5, del(7q) / -7, +8, del(17p) / TP53 and del(20q)
    • Mutation analysis (e.g., next generation sequencing myeloid targeted mutation profiling certain gene mutations are often seen in MDS cells including DNMT3A, TET2, ASXL1, TP53, RUNX1, SRSF2 and SF3B1)
    • Optional flow cytometric analysis for immunophenotyping for monocyte subset, blasts or altered myeloid maturation (Blood 2014;123:3675)
    • Immunohistochemical staining for assessment of blasts, abnormal localization of immature precursors, dysplastic megakaryocytes, p53 expression and altered trilineage distribution
Prognostic factors
  • Myelodysplastic neoplasm with low blasts and 5q deletion (MDS-5q)
  • Myelodysplastic neoplasm with low blasts and SF3B1 mutation (MDS-SF3B1)
    • Favorable outcome among MDS types (Blood 2020;136:157)
    • Presence of other molecular mutations (e.g., BCOR, BCORL1, NRAS, RUNX1, SRSF2 or STAG2) also alter clinical outcome when compared with single SF3B1 mutation
  • Myelodysplastic neoplasm with biallelic TP53 inactivation (MDS-biTP53)
    • Higher risk of leukemic transformation and higher risk of mortality, independent of Revised International Prognostic Scoring System risk categorization and independent of treatment (Nat Med 2020;26:1549)
    • Similar adverse clinical outcomes are observed in MDS patients with mutated TP53 (VAF ≥ 40%) or complex cytogenetics
  • Myelodysplastic neoplasm with low blasts (MDS-LB)
  • Myelodysplastic neoplasm, hypoplastic (h-MDS)
  • Myelodysplastic neoplasm with increased blasts (MDS-IB)
  • Childhood myelodysplastic neoplasm with low blasts (cMDS-LB)
    • Patients with monosomy 7, 7q deletion or complex karyotype usually progress to AML and typically treated with hematopoietic stem cell transplantation (Blood 2018;131:1406)
  • Childhood myelodysplastic neoplasm with increased blasts (cMDS-IB)
Microscopic (histologic) images

Contributed by Ling Zhang, M.D.

Dysplastic erythroid precursors

Dysplastic myeloid
and megaloblastic
erythroid precursors

Bone marrow core biopsy, MDS with del(5q)

Aspirate smear, MDS with del(5q)


MDS, SF3B1 mutated with increased ring sideroblasts

Dysplastic
megakaryocytes
and granulocytes
with biallelic TP53

MDS transformation to AML-MR

ALIP in bone marrow biopsy

ALIP CD34 immunostaining

Peripheral smear images

Contributed by Ling Zhang, M.D.

Dysplastic granulocytes and peripheral blood blast

Sample pathology report
  • Bone marrow, right posterior iliac crest, biopsy and aspiration:
    • Hypercellular marrow (90%) with trilineage dyspoiesis and increased blasts (7%) consistent with myelodysplastic syndrome with increased blasts (MDS-IB) (see comment)
    • Comment: Immunochemistry with CD34 and CD117 confirms the presence of increased proportion of blasts. Mild diffuse reticulin fibrosis is noted. FISH MDS panel shows both del(5q) and del(7q). No BCR::ABL1 rearrangement is identified. Karyotyping shows 46,XY,del(5q)(27;q33),del(7q)(q22),+21[10]/46,XY[10] and NGS study reveals several tier 1 pathologic mutations involving ASXL1 (VAF of 27%), DNMTA (VAF of 33%) and RUNX1 (VAF of 6.5%). There are no JAK2, NPM1 or TP53 mutations detected.
Board review style question #1
Which of the following is a correct statement regarding myelodysplastic neoplasm (MDS) with low blasts and SF3B1 mutation?

  1. Absence of ring sideroblasts excludes the diagnosis
  2. Blasts are > 5% in bone marrow
  3. Detection of SF3B1 mutation with any percentage of variant allele frequency is considered diagnostic
  4. Favorable outcome among MDS types
  5. If SF3B1 mutation analysis is not available, demonstration of ring sideroblasts comprising ≥ 5% of erythroid precursors can be used as a surrogate
Board review style answer #1
D. Favorable outcome among MDS types. Answer C is incorrect because detection of SF3B1 mutation with variant allele frequency (VAF) should be ≥ 5%. Lower than the threshold is not considered diagnostic. Answer A is incorrect because ring sideroblasts are often identified, however, an absence of ring sideroblasts in the setting does not exclude the diagnosis. Answer B is incorrect because blasts are < 5% in bone marrow and < 2% in peripheral blood in MDS-SF3B1. Answer E is incorrect because if SF3B1 mutation analysis is not available, demonstration of ring sideroblasts comprising ≥ 15% of erythroid precursors can be used as a surrogate (Blood 2020;136:157).

Comment Here

Reference: MDS-general
Board review style question #2
Which of the following is correct regarding myelodysplastic neoplasm (MDS) with TP53 inactivation?

  1. Associated with low risk disease
  2. Presence of biallelic TP53 mutations is required for diagnosis
  3. Rarely transforms to acute myeloid leukemia
  4. Sensitive to conventional therapies
Board review style answer #2
B. Presence of biallelic TP53 mutations is required for diagnosis. It is classified as MDS with biallelic TP53 inactivation. Answer A is incorrect because MDS with biallelic TP53 inactivation is associated with a worse prognosis. Answer C is incorrect because MDS with biallelic TP53 inactivation has a higher risk to transform to acute myeloid leukemia. Answer D is incorrect because MDS with biallelic TP53 inactivation is usually resistant to conventional therapies.

Comment Here

Reference: MDS-general

MGUS - non-IgM
Definition / general
  • The term monoclonal gammopathy of undetermined significance (MGUS) was introduced in 1978 by Robert Kyle to describe benign asymptomatic patients having an M protein that may progress to other malignancies
  • By definition, no evidence of multiple myeloma, other B cell lymphoproliferative disorders or other diseases known to produce M protein
  • 2 distinct types of MGUS: non-IgM MGUS (the focus of this topic) and IgM MGUS
    • Non-IgM MGUS and IgM MGUS (lymphoplasmacytic lymphoma / Waldenström macroglobulinemia) are considered separate entities by the WHO / ICC classification due to the differences in cell of origin, clinical picture and disease progression (Lancet Oncol 2014;15:e538, Leukemia 2022;36:1720, Blood 2022;140:1229)
    • Non-IgM MGUS (IgG, IgA, IgD or light chain MGUS) accounts for most MGUS cases and is characterized by a monoclonal plasma cell proliferation
    • Light chain MGUS (a subset) has only monoclonal light chains with complete loss of heavy chain expression
  • Non-IgM MGUS is a benign premalignant plasma cell disorder that is characterized by the presence of serum M protein < 30 g/L, bone marrow clonal plasma cells < 10%, absence of plasma cell myeloma related end organ damage and amyloidosis
  • Non-IgM MGUS is usually asymptomatic; conditions with symptoms should be categorized as monoclonal gammopathy of renal significance (MGRS) or monoclonal gammopathy of clinical significance (MGCS) based on symptoms (Blood 2022;140:1229)
  • Rarely, plasma cell MGUS produces an IgM M protein (1% of cases)
Essential features
  • Non-IgM MGUS is a benign, asymptomatic premalignant plasma cell disorder that is characterized by the presence of serum M protein < 30 g/L, bone marrow clonal plasma cells < 10% and absence of plasma cell myeloma related end organ damage or amyloidosis
  • Non-IgM MGUS may progress to a malignant plasma cell neoplasm (rate of 1% per year)
ICD coding
  • ICD-10: D47.2 - monoclonal gammopathy of undetermined significance (MGUS)
Epidemiology
  • Most common plasma cell neoplasm (Blood 2009;113:5412)
  • Incidence increases with age and is found in ~2 - 3% of adults over age 50 and 5% of adults over age 70 (N Engl J Med 2018;378:241)
  • More common than multiple myeloma (1 million versus 13,000 cases/year in the U.S.)
  • More prevalent in African Americans than in Caucasians (2 - 3 fold) (Leukemia 2014;28:1537)
  • More common in men than women (1.5:1)
  • Median age at diagnosis: 70 years
  • Non-IgM MGUS represents up to 85% of MGUS cases
Pathophysiology
  • Originates from clonal mature plasma cells residing in the bone marrow that harbor somatic hypermutation errors or IgH class switched variable regions; these events occur during antigen selection in germinal centers
  • Most non-IgM MGUS cases are IgG, followed by IgA, biclonal and IgD
  • Random secondary genetic alterations have been associated with increased risk of progression to plasma cell myeloma
  • Reference: Clin Lymphoma Myeloma Leuk 2023;23:e195
Etiology
Clinical features
  • Most patients are asymptomatic
  • Usually diagnosed as an incidental finding on protein electrophoresis performed to evaluate peripheral neuropathy, vasculitis, hemolytic anemia, skin rashes, hypercalcemia or elevated erythrocyte sedimentation rate
  • Diagnosis should be established only if the disease persists and is not transient (CMAJ 2013;185:1345)
  • Paraprotein incidence: 70% IgG, 15% IgM, 12% IgA, 3% biclonal, 1% IgD, 1% IgE; monoclonal light chain in urine in 33% of cases; 20% with only light chain (N Engl J Med 2006;354:1362)
  • No myeloma related organ or tissue impairment, such as hypercalcemia (> 1 mg/dL higher than the upper limit of normal or > 11 mg/dL), renal insufficiency (Cr clearance < 40 mL/min or serum Cr > 2 mg/dL), anemia (Hb > 20 g/L below lower limit of normal or > 100 g/L) or bone lesions (≥ 1 by skeletal radiography, CT or PET / CT) (= CRAB features of myeloma)
  • 3 diagnostic criteria
    1. Serum monoclonal protein < 30 g/L
    2. < 10% clonal plasma cells in the bone marrow
    3. Absence of end organ damage attributable to the plasma cell disorder (CRAB features) and absence of amyloidosis
  • Light chain only MGUS diagnostic criteria: abnormal light chain ratio (< 0.26 or > 1.65), urinary light chain must be < 0.5 g/24 h, < 10% plasma cells, end organ damage or amyloidosis
Laboratory
  • Need thorough investigation to exclude other plasma cell neoplasms
    • Complete blood count / peripheral smear: usually normal but some cases show rouleaux formation
      • Plasma cells are associated with high risk MGUS
    • Bone marrow with < 10% clonal plasma cells
      • Plasma cells are usually mature with an interstitial distribution
    • Serum calcium and creatinine: important in differentiating non-lgM MGUS from plasma cell myeloma
    • Serum protein electrophoresis and immunofixation, serum free light chains, urine protein electrophoresis and immunofixation assay, quantitation of IgM
  • Quantification of M protein is useful to monitor disease course (Hematol Oncol Clin North Am 2014;28:775)
    • Serum protein electrophoresis can be performed on agarose gel or capillary zone electrophoresis
    • Serum free light chains: measures κ and λ light immunoglobulin chains that circulate unbound to heavy chains in the serum; the normal ratio for κ/λ is 0.26 - 1.65
    • Light chain MGUS: no immunoglobulin heavy chain on immunofixation electrophoresis
Radiology description
  • Bone survey of skull, spine and pelvis (Xray, CT, MRI and PET with differing sensitivities)
    • Important in differentiating non-lgM MGUS from other plasma cell neoplasm entities and other B cell lymphoproliferative disorders
    • Osteolytic bone destruction (≥ 5 mm in size) on imaging is considered positive for bone involvement of plasma cell myeloma or plasmacytoma (Lancet Oncol 2014;15:e538)
Prognostic factors
Case reports
Treatment
  • Management requires an understanding of the risk of progression
  • Follow up of MGUS patients depends on the risk assessment
  • Risk stratification (Blood 2005;106:812)
    • Serum monoclonal protein level ≥ 15 g/L
    • Non-IgG MGUS (i.e., IgA, IgM and IgD MGUS)
    • Abnormal serum free light chain ratio
  • Patients are categorized by number of risk factors present
    • 3 risk factors: high risk MGUS
    • 2 risk factors: high intermediate risk MGUS
    • 1 risk factor: low intermediate risk MGUS
    • 0 risk factors: low risk MGUS
  • Despite close observation, can progress abruptly to plasma cell myeloma
  • Follow up
Microscopic (histologic) description
  • < 10% clonal plasma cells in bone marrow aspirate / biopsy
  • Plasma cells evenly scattered or in occasional small clusters
  • Plasma cells lack nucleoli
  • Reference: Clin Lymphoma Myeloma Leuk 2023;23:e195
Microscopic (histologic) images

Contributed by Hatem Kaseb, M.D., Ph.D., M.P.H.
Bone biopsy Bone biopsy

Bone biopsy

Interspersed plasma cells Interspersed plasma cells Interspersed plasma cells

Interspersed plasma cells

Few mature plasma cells

Few mature plasma cells

Positive stains
Negative stains
  • CD19 (in contrast to normal plasma cells)
Flow cytometry description
  • Monoclonal plasma cells that are CD38+ (bright) cells, which may express aberrant CD56 or CD117
Molecular / cytogenetics description
  • Usually shows a normal karyotype because of the relatively small number of plasma cells
  • Possible chromosomal alterations include t(11;14), t(4;14), t(14;16), deletions of 13q and hyperdiploidy; no clinical correlation for these genetic alterations have been found in non-IgM MGUS
  • Reference: Clin Lymphoma Myeloma Leuk 2023;23:e195
Sample pathology report
  • Bone marrow, biopsy and aspirate:
    • Plasma cell neoplasm with 5% plasma cells (see comment)
    • Comment: This is a 70 year old man with a medical history of MGUS, currently presenting with congestive heart failure and anemia. The bone marrow shows increased plasma cells (5%); these cells show monotypic kappa light chain restriction by immunohistochemistry. Flow cytometry studies show 0.1% of the total events consistent with kappa restricted plasma cells. Further classification of plasma cell neoplasms requires correlation with clinical, laboratory and radiologic findings.
Differential diagnosis
Board review style question #1
What is the rate of progression of non-IgM monoclonal gammopathy of uncertain significance (MGUS) to more advanced disease?

  1. 0.5% per year
  2. 1% per year
  3. 1.5% per year
  4. 2% per year
Board review style answer #1
B. 1% per year

Comment Here

Reference: MGUS - non-IgM
Board review style question #2

Which of the following factors is associated with increased risk of progression in non-IgM monoclonal gammopathy of uncertain significance (MGUS)?

  1. Generalized lymphadenopathy
  2. Lambda restriction of plasma cells in bone marrow aspirate
  3. M protein of 5 g/L
  4. M protein of 15 g/L
Board review style answer #2
D. M protein of 15 g/L. The risk stratification (Blood 2005;106:812) for non-IgM MGUS includes serum monoclonal protein level ≥ 15 g/L, non-IgG MGUS (i.e., IgA, IgM and IgD MGUS) and abnormal serum free light chain ratio. Generalized lymphadenopathy and type of light chain restriction are not considered as risk factors for non-IgM MGUS.

Comment Here

Reference: MGUS - non-IgM

MGUS-IgM
Definition / general
  • IgM monoclonal gammopathy of undetermined significance (MGUS) is characterized by < 3 g/dL of monoclonal IgM protein, < 10% clonal lymphoplasmacytic or clonal plasma cells, with no anemia, constitutional symptoms, hyperviscosity, lymphadenopathy or hepatosplenomegaly
Essential features
  • Serum IgM monoclonal protein < 3 g/dL
  • Bone marrow clonal lymphoplasmacytic cells or clonal plasma cells < 10% of cellular elements
  • No anemia, constitutional symptoms, hyperviscosity, lymphadenopathy or hepatosplenomegaly caused by the underlying disorder
  • Can progress to lymphoplasmacytic lymphoma / Waldenström macroglobulinemia, IgM multiple myeloma or AL amyloidosis
ICD coding
  • ICD-O: 9761/1 - IgM monoclonal gammopathy of undetermined significance
  • ICD-10: D47.2 - monoclonal gammopathy
Epidemiology
Sites
  • Monoclonal IgM kappa and lambda proteins are found in the serum or urine
  • Clonal plasma cells or lymphocytes can be seen in the bone marrow
Pathophysiology
  • Initial step is an abnormal immune response to an antigenic stimulus (F1000Res 2017;6:2142)
  • MYD88 (42 - 54%) and CXCR4 (7%) mutations are seen in IgM MGUS
  • Additional steps (such as the 6q deletion) may be involved in progression to Waldenström macroglobulinemia (F1000Res 2017;6:2142)
  • Noncoding RNAs may drive development of Waldenström macroglobulinemia (Cancers (Basel) 2020;12:320)
  • IgM MGUS patients with t(11:14) may progress to IgM plasma cell myeloma
Etiology
  • Unknown
Clinical features
  • Patients with IgM MGUS do not have constitutional symptoms (Clin Lymphoma Myeloma 2009;9:17)
  • Progression among IgM MGUS patients is 2% per year in the first 10 years and 1% per year thereafter (N Engl J Med 2018;378:241)
  • Risk of progression is higher than non-IgM MGUS
  • Some patients have associated peripheral neuropathy
Diagnosis
  • Diagnosis of IgM MGUS is based on the presence of a serum IgM monoclonal protein < 3 g/dL, fewer than 10% clonal lymphoplasmacytic or plasma cells in the bone marrow and lack of evidence of progression to lymphoplasmacytic lymphoma / Waldenström macroglobulinemia, IgM multiple myeloma or AL amyloidosis (Blood 2018;131:163)
Laboratory
  • Serum electrophoresis shows a restricted band (< 3 g/dL)
  • Serum immunofixation shows an IgM kappa or IgM lambda monoclonal protein
Radiology description
  • Skeletal survey does not show lytic lesions
  • CT / PET scans are negative for lymphoma
Prognostic factors
  • Risk factors for progression are initial concentration of the monoclonal protein (≥ 1.5 g/dL) and abnormal serum free light chain ratio (N Engl J Med 2018;378:241)
  • Overall survival rate at 30 years is 4% for patients with IgM MGUS and 7% for non-IgM MGUS, compared with 12% in an age and sex matched control population (N Engl J Med 2018;378:241)
Case reports
  • 62 year old immunocompetent woman was admitted with cytomegalovirus (CMV) infection, pulmonary embolism, splenic vein thrombosis and monoclonal gammopathy of undetermined significance (MGUS) (BMJ Case Rep 2019;12:e226448)
  • 70 and 84 year old Caucasian men showed an M spike by protein electrophoresis (J Med Case Rep 2020;14:75)
  • 72 year old Japanese man with diabetes mellitus and hypertension presented with an acutely elevated serum creatinine level (CEN Case Rep 2020;9:109)
  • 72 year old Japanese woman was hospitalized because of massive proteinuria (Front Med (Lausanne) 2021;8:608741)
  • 75 year old woman presented with maculopapular purpura and erythematous (inflammatory) retiform purpura on her lower leg (JAAD Case Rep 2019;5:288)
Treatment
  • Treatment is not recommended for asymptomatic patients with IgM MGUS (F1000Res 2017;6:2142)
  • "Wait and watch" approach is usually followed
  • Patients with peripheral neuropathy are treated with IVIG, rituximab, steroids or plasma exchange
Microscopic (histologic) description
  • Bone marrow biopsy is usually normocellular with normal trilineage hematopoiesis
  • Plasma cells and CD20+ B cells may be slightly increased but do not form prominent aggregates
Microscopic (histologic) images

Contributed by Ameet R. Kini, M.D., Ph.D. and Maryam F. Raouf, M.D.
Increased CD20+ B cells

Increased CD20+ B cells

Increased plasma cells

Increased plasma cells

Cytology description
  • Bone marrow aspirate smears show normal trilineage hematopoiesis
  • Plasma cells and lymphocytes may be slightly increased but are < 10% of cells
Peripheral smear description
  • Peripheral blood smear shows no abnormal findings
Positive stains
Negative stains
Flow cytometry description
Flow cytometry images

Contributed by Ameet R. Kini, M.D., Ph.D. and Maryam F. Raouf, M.D.
Abnormal plasma cells

Abnormal plasma cells

Molecular / cytogenetics description
  • FISH analysis may detect presence of t(11;14), CCND1-IGH in plasma cells
  • MYD88 mutation may be detected (Blood 2013;121:2051)
Molecular / cytogenetics images

Contributed by Ameet R. Kini, M.D., Ph.D. and Maryam F. Raouf, M.D.
CCND1-IGH fusion

CCND1-IGH fusion

Videos

Introduction to MGUS

MGUS diagnosis

MGUS management


MGUS associated diseases

IgM MGUS pathophysiology

Sample pathology report
  • Bone marrow, right posterior iliac crest, core biopsy, clot section, aspirate smears and touch imprint:
    • Normocellular bone marrow (50%) with trilineage hematopoiesis and slightly increased plasma cells (4%) (see comment)
    • Comment: Review of laboratory data shows a monoclonal protein in the serum (IgM lambda, 1.2 g/dL). The CBC is within normal limits. Imaging studies reveal bone marrow lytic lesions and no lymphadenopathy or organomegaly.
    • The bone marrow core biopsy is normocellular for age (50%). Trilineage hematopoiesis is present. Megakaryocytes are adequate and appear normal in morphology.
    • The aspirate clot section is adequate and shows features similar to the core biopsy.
    • The bone marrow aspirate smear is adequate. The myeloid and erythroid precursors show progressive and orderly maturation. Megakaryocytes are adequate in number and show normal morphology. Blasts are not increased. Plasma cells are slightly increased (4% plasma cells based on a 500 cell differential count performed on the bone marrow aspirate smear).
    • The touch preparation findings are similar to the aspirate smear.
    • Flow cytometry shows a small abnormal plasma cell population (CD45 negative, bright CD38 positive) with expression of CD20, CD56 and CD117. These plasma cells show lambda light chain restriction. Gating on the lymphocytes shows no evidence of a monoclonal B cell population or T cell abnormality based on the markers assayed. There is no increase in blasts.
    • FISH analysis performed on an enriched plasma cell population reveals presence of t(11;14). Molecular analysis for the MYD88 mutation is negative.
    • The findings are consistent with IgM MGUS. IgM MGUS patients can progress to lymphoplasmacytic lymphoma / Waldenström macroglobulinemia, IgM multiple myeloma or AL amyloidosis.
Differential diagnosis
  • IgM myeloma:
    • IgM protein monoclonal ≥ 3 g/dL or ≥ 10% clonal plasma cells
    • Lytic bone lesions may be seen
  • Lymphoplasmacytic lymphoma:
    • IgM protein monoclonal ≥ 3 g/dL or ≥ 10% clonal lymphoplasmacytic cells
    • Hyperviscosity and lymphadenopathy may be seen
  • AL amyloidosis:
    • Presence of amyloid deposits that display apple green birefringence under polarized microscopy on Congo red stain
Additional references
Board review style question #1

A 58 year old woman with a past medical history significant for type 2 diabetes and hypertension presents to her PCP office for routine care. The patient's diabetes and hypertension are well controlled and she reports no significant constitutional symptoms. Physical exam shows no lymphadenopathy or organomegaly. Laboratory studies show her CBC within normal limits. Serum electrophoresis shows a restricted band at 1.5 g/dL. A serum immunofixation is shown above (ELP = serum electrophoresis, G = IgG, A = IgA, M = IgM, K = kappa, L = lambda). This patient's most likely diagnosis is

  1. IgG MGUS
  2. IgM MGUS
  3. Polyclonal gammopathy
  4. Smoldering myeloma
  5. Waldenström macroglobulinemia
Board review style answer #1
B. IgM MGUS

Comment Here

Reference: MGUS-IgM
Board review style question #2
A 69 year old man with a history of acute myeloid leukemia presents to his oncologist for follow up. The patient was diagnosed 5 years ago and after treatment, has been in complete remission. His CBC is within normal limits. A serum immunofixation shows a small IgM lambda monoclonal protein. A bone marrow biopsy was performed and shows slightly increased lymphocytes, some of which are plasmacytoid. Flow cytometry shows a small B cell population with expression of CD19, CD20 and lambda light chain restriction. Blasts are not increased and show normal immunophenotype. A mutation in which gene is mostly likely to be detected by molecular studies?

  1. CALR
  2. FLT3
  3. JAK2
  4. MYD88
  5. SF3B1
Board review style answer #2
D. MYD88

Comment Here

Reference: MGUS-IgM

MPN unclassifiable
Definition / general
  • Has clinical phenotype of myeloproliferative neoplasms (MPN) but does not meet diagnostic criteria for any classic or nonclassic myeloproliferative neoplasms
Molecular / cytogenetics description

Mast cell sarcoma (pending)
[Pending]

Mixed phenotype
Definition / general
  • De novo acute leukemia containing separate populations of blasts of more than one lineage (bilineal or bilineage), or a single population of blasts co-expressing antigens of more than one lineage (biphenotypic)

Excludes:
  • Acute myeloid leukemia (AML) with recurrent translocations t(8;21), t(15;17) or inv(16)
  • Leukemias with FGFR1 mutations
  • Chronic myelogenous leukemia (CML) in blast crisis
  • Myelodysplastic syndrome (MDS) related AML and therapy related AML, even if they have MPAL immunophenotype

Criteria for biphenotypic leukemia:
  • Score of 2 or more for each of two separate lineages (shown below)
Diagrams / tables

Contributed by Daniela Mihova, M.D.
Missing Image

The European Group
for the Immunological
Classification of Leukemias
(EGIL) scoring system

Missing Image

2008 WHO
classification of
acute leukemias of
ambiguous lineage

Prognosis and treatment
  • Poor, overall survival of 18 months
  • Young age, normal karyotype and ALL induction therapy are associated with favorable survival
  • Ph+ is a predictor for poor prognosis
  • Bone marrow transplantation should be considered in first remission
MPAL with t(9;22)(q34;q11.2); BCR-ABL1
  • 20% of all MPAL
  • Blasts with t(9;22)(q34;q11.2) translocation or BCR-ABL1 rearrangement (Ph+) without history of CML
  • Majority in adults
  • High WBC counts

  • Most of the cases B/myeloid phenotype
  • Rare T/myeloid, B and T lineage, or trilineage leukemias

Morphology:
  • Many cases show a dimorphic blast population, one resembling myeloblasts and the other lymphoblasts

Cytogenetic abnormalities:
  • Conventional karyotyping for t(9;22), FISH or PCR for BCR-ABL1 translocation
  • Additional complex karyotypes
  • Ph+ is a poor prognostic factor for MPAL with a reported median survival of 8 months
  • Worse than patients of all other types of MPAL
MPAL with t(v;11q23); MLL rearranged
  • Meeting the diagnostic criteria for MPAL with blasts bearing a translocation involving the 11q23 breakpoint (MLL gene)
  • MPAL with MLL rearranged rare
  • More often seen in children and relatively common in infancy
  • High WBC counts
  • Poor prognosis
  • Dimorphic blast population with one resembling monoblasts and the other resembling lymphoblasts
  • Lymphoblast population often shows a CD19+, CD10- B precursor immunophenotype, frequently CD15+
  • Expression of other B markers usually weak
  • Translocations involving MLL gene include t(4;11)(q21;q23), t(11;19)(q23;p13) and t(9;11)(p22;q23)
  • Cases with chromosome 11q23 deletion should not be classified in this category
Molecular / cytogenetics images

AFIP images
Missing Image

t(4;11)(q21;23) in bilineal leukemia

MPAL B/Myeloid, NOS
  • Meets the diagnostic criteria for MPAL with both B and myeloid lineages without Ph+ and MLL rearrangement
  • B/myeloid acute leukemia accounts for 1% of all leukemias
  • More common in adults but also seen in children

Morphology:
  • Dimorphic populations, resembling lymphoblasts and myeloblasts, or a single population resembling ALL

Genetics:
  • Multiple different cytogenetic changes have been demonstrated, however none is proven to be specific in this subtype
MPAL T/Myeloid, NOS
  • Meets the diagnostic criteria for MPAL with both T and myeloid lineages without Ph+ and MLL rearrangement
  • More often in children, though in adults as well
  • Dimorphic populations, resembling lymphoblasts and myeloblasts, or a single population resembling ALL
  • Other commonly expressed T-lineage markers include CD2, CD7
  • Myeloid markers MPO, CD117, TdT, CD13 and CD33
  • T cell plus myeloid cases may have 2p13 translocations or other unrelated anomalies (Leukemia 2007;21:2264)

Monoclonal immunoglobulin deposition diseases (pending)
[Pending]

Myeloid leukemia associated with Down syndrome
Definition / general
  • Myeloid proliferations seen in individuals with Down syndrome (DS) include:
    • Transient abnormal myelopoiesis (TAM) is a hematopoietic stem cell disorder of newborns with Down syndrome which presents with clinical and morphological findings of acute myeloid leukemia but is self limiting; this entity is discussed separately
    • Myeloid leukemia associated with Down syndrome
Essential features
  • Myeloid proliferations associated with Down syndrome include transient abnormal myelopoiesis which is self limiting and acute myeloid leukemia, specifically, acute megakaryoblastic leukemia
  • Trisomy 21 and GATA1 mutations are implicated in pathogenesis
  • Prognosis is better than in cases with other pediatric acute leukemias
ICD coding
  • ICD-10: C92.Z - Other myeloid leukemia
  • ICD-O: 9898/3 - Myeloid leukemia associated with Down syndrome
Epidemiology
  • Individuals with Down syndrome have 10 - 100 times the risk of acute leukemia as compared with the general population (Swerdlow: WHO Classification of Tumours of Haematopoietic and Lymphoid Tissues, Revised Edition, 2017)
  • 1 - 2% of children with Down syndrome will develop acute myeloid leukemia before 5 years of age
  • If patient is a neonate, consider the diagnosis transient abnormal myelopoiesis associated with Down syndrome
  • 20 - 30% of children with history of transient abnormal myelopoiesis get acute myeloid leukemia within 1 - 3 years following transient abnormal myelopoiesis
Sites
  • Primary sites: peripheral blood and bone marrow
  • Extramedullary: spleen and liver, usually present
Pathophysiology
  • Trisomy 21 in fetal hematopoietic precursors followed by acquired GATA1 mutations is implicated in the development of myeloid proliferations associated with Down syndrome
  • Secondary epigenetic and genetic events lead to acute myeloid leukemia in individuals with transient abnormal myelopoiesis
  • Trisomy 21 affects fetal hematopoietic stem cell biology and leads to defective megakaryocytic, erythroid and B lymphoid lineage development
  • N terminal truncating mutations in the key megakaryocyte erythroid transcription factor, GATA1, are commonly seen in association with trisomy 21; mutated GATA1 gene affects megakaryocytic lineage in the fetal liver
  • Some of the secondary events that lead to acute leukemia in patients with transient abnormal myelopoiesis are gain of function mutation in myeloid cytokine receptor CSF2RB and tyrosine kinases such as JAK1, JAK2 and JAK3; loss of function mutations in genes encoding cohesion core subunits - CTCF, NIPBL, JAK1-3, SRSF2 and SF3B (Cancer Cell 2019;36:123)
Clinical features
  • Often a prolonged preleukemic or myelodysplastic phase may be present when patient presents with refractory cytopenias
Prognostic factors
  • Individuals with Down syndrome and GATA1 mutated acute myeloid leukemia have a better prognosis than those with other pediatric acute myeloid leukemias (must use Down specific treatment protocols)
  • Older age at diagnosis is associated with poor prognosis
Case reports
Microscopic (histologic) description
  • Preleukemic phase has morphologic features of refractory cytopenia of childhood; no increase in blasts
  • Acute megakaryoblastic leukemia accounts for ≥ 50% of cases
  • Bone marrow:
    • Variable blast count
    • Leukemic blasts have round to slightly irregular nuclei and a moderate amount of basophilic cytoplasm, basophilic granules; cytoplasmic blebs (megakaryoblastic features)
    • Erythroid, granulocytic and megakaryocytic dysplasia
    • Variable degree of reticulin fibrosis
  • Reference: Arch Pathol Lab Med 2020;144:466
Microscopic (histologic) images

Contributed by Alexa J. Siddon, M.D.

Hypercellular bone marrow

Bone marrow with CD34+ blasts

Bone marrow with CD61+ megakaryocytes and blasts

Peripheral smear description
  • Circulating blasts are typically present in blood; may have cytoplasmic blebs
  • Macrocytic red cells with anisopoikilocytosis including dacrocytes
  • Thrombocytopenia with giant platelets
Peripheral smear images

Contributed by Alexa J. Siddon, M.D.

Megakaryoblasts in peripheral blood

Positive stains
Negative stains
Flow cytometry description
  • Similar to stains
  • Can be helpful to show that the blasts are positive for both CD34 and platelet markers
Molecular / cytogenetics description
Sample pathology report
  • Bone marrow, biopsy, aspirate and peripheral smear:
    • Acute megakaryoblastic leukemia (see comment)
    • Comment: The overall features, including smear morphology, immunophenotype of the blast population and detection of a GATA1 mutation are most consistent with myeloid leukemia associated with Down syndrome in this 3 year old child with trisomy 21.
  • Peripheral smear:
    • The peripheral blood shows anemia and scattered intermediate sized blasts with basophilic cytoplasm, a subset of which has cytoplasmic blebs.
  • Bone marrow biopsy:
    • Cellular marrow for age (100% cellular). The marrow shows marked fibrosis, clusters of immature cells and frequent dysplastic megakaryocytes which are small with hypolobated nuclei. Residual trilineage hematopoiesis is markedly decreased. A reticulin stain reveals that reticulin is markedly increased (MF 3 of 3). Trabecular bone is unremarkable and there are no lymphoid aggregates. The clot section consists predominantly of clotted blood admixed with rare marrow elements.
  • Immunostains:
    • CD34 highlights abundant blasts, accounting for 50% of cellularity, which are negative for myeloperoxidase. CD61 highlights abundant dysplastic megakaryocytes as well as blasts.
  • Bone marrow aspirate:
    • The aspirate smear is aparticulate hemodilute; a formal differential count was not performed. Frequent blasts similar to those in blood are noted. They are medium in size and have a high N/C ratio. The nuclei are round or irregular with moderately delicate chromatin and inconspicuous nucleolus. The cytoplasm is scant and blue staining with occasional cytoplasmic blebs. Prussian blue iron stain on diluted smear is inconclusive for storage iron assessment; no increase in ring sideroblasts is seen.
  • Flow cytometry:
    • Flow cytometric analysis reveals approximately 44% of analyzed cells are blasts with a megakaryoblastic immunophenotype: CD45dim+ CD34+ CD117+ CD7dim+ CD41a(var)+ CD42+ CD61+ CD14- CD64- CD13dim+ CD33dim CD4- HLADR- CD19- CD10- CD79a- TDT- surfaceCD3- CD56(var) CD235a- MPO- cytoCD3-. The differential based on immunophenotype alone includes acute megakaryoblastic leukemia (AMKL) and transient abnormal myelopoiesis of Down syndrome. Blasts cannot be reliably differentiated by flow cytometry for these two entities but age is more consistent with AMKL. Please correlate with core biopsy analysis and molecular/cytogenetic results as available.
  • Cytogenetics:
    • Karyotype: 47,XY,+21[20]
Differential diagnosis
Board review style question #1

A 29 month old child with Down syndrome presents to the clinic with easy bruising. Peripheral blood smear is significant for thrombocytopenia, anisopoikilocytosis and blasts (~ 30%). A flow cytometry analysis of peripheral blood specimen is pending. What is the most likely immunophenotype of the blasts?

  1. CD34-, CD117+, HLADR-, MPO++, CD33,
  2. CD34+, CD33+, CD11b+, CD41+, CD61+, MPO-, CD71-
  3. CD34+, CD117+, CD33+, HLA-DR+, CD41-, CD61-
  4. CD34-, CD4+, CD7+, CD33+, CD123+, CD56+
  5. CD34+, CD19+, CD20-, CD10+, TdT+
Board review style answer #1
B. CD34+, CD33+, CD11b+, CD41+, CD61+, MPO-, CD71-

Comment Here

Reference: Myeloid leukemia associated with Down syndrome
Board review style question #2
The 29 month old child has been diagnosed with myeloid leukemia associated with Down syndrome. Which of the following will likely be positive?

  1. Monosomy 7
  2. t(1;22)
  3. t(1;19)
  4. t(2;5)
  5. GATA1 mutation
Board review style answer #2

Myeloid neoplasms with ANKRD26 mutation (pending)
[Pending]

Myeloid neoplasms with germline GATA2 mutation
Definition / general
  • Classified as one of the myeloid neoplasms with germline predisposition and other organ dysfunction in the revised 2016 WHO classification (Blood 2016;127:2391)
  • Now known to underlie several syndromes: MonoMAC syndrome, dendritic cell, monocyte, B and NK lymphoid (DCML) deficiency and Emberger syndrome
Essential features
  • Counts for the cause of 7 - 15% of primary myelodysplastic syndrome cases in children and adolescents and for approximately 37% of myeloid neoplasms with monosomy 7 (-7) or other forms of 7q- / -7 in adolescents and young adults
  • Differentiating somatic from germline GATA2 mutations is necessary
  • Genetic counseling of family members and timely screening of donors for presence of predisposition germline mutations are necessary prior to hematopoietic stem cell transplant for optimal patient outcomes
  • Skin fibroblasts are the gold standard for obtaining germline DNA in patients with myeloid neoplasms
Terminology
  • Myeloid neoplasms with germline GATA2 mutation
  • Familial myeloid neoplasms
  • Familial myelodysplastic syndrome (MDS) / acute myeloid leukemia (AML)
ICD coding
  • Coding not available for this entity, however:
    • ICD-O: 984-993 - myeloid leukemias
    • ICD-10: C92.A0 - acute myeloid leukemia with multilineage dysplasia, not having achieved remission
Epidemiology
Sites
  • Myeloid neoplasms mainly involve the peripheral blood and bone marrow
  • Many other organs other than bone marrow (e.g. ear, lung, heart, joints, skin, blood vessels and immune system) can also be involved, especially if a germline GATA2 mutation associated syndrome exists (Blood 2014;123:809)
Pathophysiology
  • GATA2 is a member of the GATA2 family of zinc finger transcription factors that regulate transcription of many genes
    • Plays an essential role in genesis and function of hematopoietic stem and progenitor cells and thus all subsequent blood cell lineages
    • Also involved in autoimmunity, inflammation and developmental processes
  • Germline GATA2 mutations may result in loss of function of the mutated allele, which leads to GATA2 haploinsufficiency and then affecting function of multiple cell lineages (Hematol Oncol Clin North Am 2018;32:713)
  • 4 types of germline GATA2 mutations are categorized:
    • Missense mutations, including all mutations resulting in a mutant protein
    • Null mutations, including all nonsense, frameshift mutations and large deletions that result in no protein product
    • Regulatory mutations, including all mutations within the enhancer region of intron 5
    • Uniallelic mutations, including cases with phenotypical GATA2 deficiency but without a defined gene mutation
    • According to Spinner et al, null mutations are closely associated with severe viral infections and earlier onset of those infections, while lymphedema was only observed in patients with null or regulatory mutations (Blood 2014;123:809)
    • Large deletions are associated with extramedullary developmental defects
    • Somatic ASXL1 mutations are commonly observed in patients with germline GATA2 mutations and are considered as an important second hit for myeloid transformation observed in these patients (Haematologica 2014;99:276)
  • Inheritance: spontaneously arising but transmitted mainly as autosomal dominant (approximately 80%) (Blood 2014;123:809, Best Pract Res Clin Haematol 2020;33:101197)
    • Penetrance was estimated at 90% by the age of 60 years; cases with sporadic germline GATA2 mutations have been reported as well (Br J Haematol 2013;161:701)
Clinical features
  • Onset of myeloid neoplasm in patients with germline GATA mutation(s) is between ages of 5 months and 78 years (median 20 years) (Blood 2014;123:809, Best Pract Res Clin Haematol 2020;33:101197)
    • Spectrum of the diseases include pediatric MDS or familial MDS / AML, chronic myelomonocytic leukemia (CMML) and myeloproliferative neoplasms (MPNs); especially, germline GATA2 mutations count for approximately 7% of all primary MDS cases in children but are absent in children with secondary MDS
  • Initial presentations in these patients are heterogeneous, ranging from asymptomatic, mild or moderate cytopenia (e.g. refractory cytopenia of childhood, RCC) to mild to severe immunodeficiency (e.g. lymphedema, pulmonary diseases, vascular problems, virus or bacteria infections, skin lesions or congenital deafness) (Best Pract Res Clin Haematol 2020;33:101197)
    • Can present or be diagnosed as a standalone disease / syndrome prior to myeloid neoplasm as well as coexist with myeloid neoplasm
Diagnosis
  • The following conditions can be indications to start a workup for potential germline GATA2 mutations that are associated with existing or preexisting myeloid neoplasm:
    • Idiopathic cytopenia (especially neutropenia, CD4 lymphocytopenia, depletion of monocytes, B cells, NK cells and dendritic cells), idiopathic bone marrow failure, susceptibility to refractory virus or other pathogen infection (e.g. human papillomavirus [HPV], herpes virus, nontuberculous mycobacteria [NTM] and Histoplasma capsulatum, etc.), pulmonary alveolar proteinosis (PAP) and ventilatory defects, skin lesions (e.g. flat warts), idiopathic lymphedema, solid tumor, especially those being associated with HPV, EBV infections and so on
  • Diagnostic criteria for each type of germline GATA2 mutation associated myeloid neoplasm (e.g. MDS, MPN or AML) are the same as that for cases without germline GATA2 mutation; the prevalence of MDL, AML and CMML in a study of 57 patients with GATA2 deficiency were 84%, 14% and 8% respectively and all the AML cases were evolved from MDS (Blood 2016;127:1387)
  • Different from de novo MDS cases with a bone marrow feature of hypercellularity, germline GATA2 mutation associated MDS patients mostly present with hypocellular bone marrow for their ages, also frequently with increased fibrosis and dysplastic magakryocytes (Blood 2016;127:1387)
Prognostic factors
Case reports
Treatment
Clinical images

Images hosted on other servers:

Recalcitrant periungual warts

Microscopic (histologic) images

Contributed by Wei Wang, M.D., Ph.D.

Megakaryocytic dysplasia in MDS

Virtual slides

Images hosted on other servers:

Alveolar proteinosis in lung tissue, GATA mutation positive

H&E staining

PAS staining

Peripheral smear images

Contributed by Wei Wang, M.D., Ph.D.

Pancytopenia in MDS

Positive stains
  • Variable, depending on the type and stages of myeloid neoplasm but not specifically associated with the underlying germline GATA2 mutation
Negative stains
  • Variable, depending on the type and stages of myeloid neoplasm but not specifically associated with the underlying germline GATA2 mutation
Flow cytometry description
  • Variable, depending on the type and stages of myeloid neoplasm but not specifically associated with the underlying germline GATA2 mutation; for example, in childhood myelodysplastic syndrome with germline GATA2 mutation, characteristic immunophenotyping changes include but are not limited to granulocytopenia, monocytopenia, decreased B cells and NK cells, presence of CD56+ plasma cells, etc. (Haematologica 2011;96:1221, Hematol Oncol Clin North Am 2018;32:713)
Flow cytometry images

Images hosted on other servers:

Flow cytometry study of peripheral blood

Molecular / cytogenetics description
  • Molecular findings:
    • Mutation of various types (missense, nonsense, indels, etc.), involving exon(s) or intron(s) of GATA2 gene; differentiating somatic from germline GATA2 mutations is necessary (see below)
  • Cytogenetic findings:
  • Next generation sequencing (NGS): in 10 - 15% cases, large deletions or mutations in intronic region of GATA2 gene may be involved and they can be missed with standard gene panel based NGS; therefore, NGS based whole exome (WES) or even whole genome (WGS) sequencing can be applicable to identify rare mutation(s) or those located within an intron (Leuk Lymphoma 2020;61:3010, JCO Precis Oncol 2019;3:PO.18.00301)
  • Sanger sequencing, especially for known mutation(s)
  • Conventional cytogenetic analysis or FISH, with focus on monosomy 7 or 7q abnormality, trisomy 8 and trisomy 21
  • Note: somatic GATA2 mutations reportedly coexist with other germline mutations (e.g. ASXL1, CEBPA) or other acquired drive mutations in myeloid neoplasm cases; differentiating somatic from germline GATA2 mutations is necessary
  • Additional reference: Blood 2014;123:809
Molecular / cytogenetics images

Contributed by Zhenya Tang, M.D., Ph.D.

Abnormal karyotype with -7

FISH analysis

Sample pathology report
  • Bone marrow biopsy, right posterior iliac crest:
    • Gross measurement: 1.2 cm in aggregate. Quality: limited, subcortical. Cellularity: near empty marrow with only 2 small foci of cellular areas with erythroid predominant. Megakaryocytes: present. Infiltrate: no increase in immature cells.
  • Bone marrow clot:
    • Quality: adequate, 10 - 20% cellularity, morphology similar to biopsy.
  • Bone marrow smear / touch preparations:
    • Quality: adequate. Granulocytes: decreased with maturation and dysplasia. Erythrocytes: relatively increased with maturation and occasional dyserythropoiesis. Megakaryocytes: rare. Lymphocytes: not increased. Plasma cells: not increased. Blasts: not increased.
  • Stains of bone marrow smears, clots and biopsy:
    • Iron: slightly increased, no ringed sideroblasts. Reticulin / trichrome: inadequate for reticulin and trichrome stain evaluation. MPO: positive in scattered cells, decreased. CD71 / glycophorin A: highlight increased erythroids. CD61: highlights few scattered megakaryocytes.
  • Bone marrow diagnosis:
    • Hypocellular marrow with erythroid predominance and granulocytic dysplasia, 2% blasts, morphologically and immunophenotypically consistent with myelodysplastic syndrome (MDS). An underlying congenital condition should be considered (see comment).
    • Comment: 23 year old man with a history of recurrent skin infections and warts, CBC with mild pancytopenia and marrow biopsy showed hypocellularity. Concurrent flow cytometry analysis demonstrated aberrant myeloblasts, lack of hematogones, virtual absence of monocytes, minimal NK cells and only 1.1% of B cells. In addition, a tiny atypical T cells population is also detected, CD2+, CD3+, CD5 dim, CD4-, CD8 partial, CD56+, CD57 bright. The overall picture is highly suspicious for a congenital GATA2 mutation. Correlation with pending NGS result will be helpful for confirmation. Correlation with pending ancillary study results is recommended for complete evaluation.
Differential diagnosis
  • Germline versus somatic GATA2 mutation
  • Syndromic versus nonsyndromic germline GATA2 mutation
  • 1 case with somatic GATA2 mutation but mimicking clinical presentation of GATA2 deficiency of germline GATA2 mutations has been reported (Blood Adv 2018;2:904)

Table 1. Differentials of Germline and Somatic GATA2 mutations
Germline
Somatic
# of mutations identified
~ 150 ~ 50
Locations
Whole GATA2 gene, ZF2 especially ZF1
Types
Multiple (see pathophysiology) Mostly point mutations
Underlying mechanism(s)
GATA2 deficiency of loss of function of affected copy Gain of function
Clinical presentation
Syndromic and nonsyndromic with or without myeloid neoplasm; myeloid neoplasm as standalone, especially MDS and AML in children and young adults Mainly MDS, AML, CML blast phase in all age groups
Coexisting mutations
Other germline mutation, somatic ASXL1, etc. Biallelic CEBPA mutations; other germline mutations (e.g. RUNX1)

Positive testing

Both disease affected and nonaffected tissues; especially skin biopsy
Disease affected tissue only (e.g. bone marrow and skin biopsy will be mostly negative)
References: Blood 2014;123:809, Hematol Oncol Clin North Am 2018;32:713, Best Pract Res Clin Haematol 2020;33:101197
Board review style question #1
What is the gold standard to obtain reliable germline DNA in a patient with a hematologic malignancy?

  1. Bone marrow biopsy and cell culture
  2. Buccal swabs and saliva
  3. Nails or hair
  4. Peripheral blood and cell culture
  5. Skin biopsy and cell culture
Board review style answer #1
E. Skin biopsy and cell culture. Genetic testing of cultured skin fibroblasts is the gold standard for obtaining germline DNA in patients with hematologic malignancies. DNA from nails or hair can be also used if skin biopsy and cell culture are not feasible. Peripheral blood and bone marrow can be affected by myeloid neoplasm or other hematological malignancies; they should be used cautiously as a source of germline DNA. Buccal swabs and saliva are often contaminated with blood cells and they should also be cautiously used as resource of germline DNA (2016 WHO classification).

Comment Here

Reference: Myeloid neoplasms with germline GATA2 mutation

Myeloid neoplasms with germline SAMD9 mutation (pending)
[Pending]

Myeloid neoplasms with germline SAMD9L mutation (pending)
[Pending]

Myeloid or lymphoid neoplasms with germline ETV6 mutation (pending)
[Pending]

Myeloid or lymphoid neoplasms with germline RUNX1 mutation (pending)
[Pending]

Myeloid or lymphoid neoplasms with germline RUNX1 mutation
Definition / general
  • Myeloid or lymphoid neoplasms with germline RUNX1 variant
Essential features
  • Confirmation of germline origin of RUNX1 variant in skin fibroblasts or persistent presence of RUNX1 variant at ~50% allelic frequency in all complete remission samples
Terminology
  • Hematologic neoplasms with RUNX1 germline variant
ICD coding
N/A
Epidemiology
Sites
  • Variable tissues, including skin, peripheral blood and bone marrow
  • Frequency of tissue and germline mosaicism is unknown
Pathophysiology
Etiology
Diagrams / tables
None
Clinical features
Diagnosis
  • Detection of heterozygous germline P / LP variant in RUNX1 by molecular genetics test
Laboratory
  • Mild to moderate thrombocytopenia (50 - 150 x 109/L)
  • Qualitative platelet defect with impaired platelet aggregation and dense granule deficiency
Prognostic factors
Not provided
Case reports
  • 10 year old boy with history of thrombocytopenia, germline RUNX1 variant and new diagnosis of T cell lymphoblastic lymphoma (Pediatr Blood Cancer 2023;70:e30184)
  • 12 year old girl with revertant mosaicism for the familial RUNX1 mutation (Haematologica 2020;105:e535)
  • 16 year old girl and 16 year old boy (unrelated patients) with history of mild thrombocytopenia, germline RUNX1 variants and new diagnosis of B cell acute lymphoblastic leukemia (Blood Adv 2021;5:3199)
Treatment
  • Best practice consensus guidelines from the UK Cancer Genetics Group, CanGene-CanVar and the NHS England Hematological Oncology Working group recommend CBC every 3 - 4 months and, if abnormal, thorough clinical and bone marrow examination for all carriers of familial RUNX1 P / LP variants (Br J Haematol 2023;201:25)
  • For bleeding: recommendation to use clotting promoters during procedures with bleeding risk
  • For early onset of hematologic malignancy: consideration for allogeneic stem cell transplantation from related donor who does not carry RUNX1 germline variant or matched unrelated donor
  • For skin manifestations: topical steroids
  • Genetic counseling to identify unaffected relatives at risk of being a carrier of germline RUNX1 variant for further surveillance
Clinical images
None
Gross description
Not provided
Gross images
Not provided
Frozen section description
Not provided
Frozen section images
Not provided
Microscopic (histologic) description
Microscopic (histologic) images

Contributed by Barina Aqil, M.D.
Small hypolobated megakaryocyte Small hypolobated megakaryocyte Small hypolobated megakaryocyte

Small hypolobated megakaryocyte

Erythroid dysplasia

Erythroid dysplasia

Scattered blasts

Scattered blasts

CD61 highlights small megakaryocytes

CD61 highlights small megakaryocytes

Virtual slides
None
Cytology description
Not provided
Cytology images
None
Peripheral smear description
Not provided
Peripheral smear images
None
Positive stains
Negative stains
  • Reticulin does not show increased fibrosis
  • Prussian blue shows decreased iron storage with no ring sideroblasts identified
Flow cytometry description
Not provided
Flow cytometry images
None
Electron microscopy description
Not provided
Electron microscopy images
None
Molecular / cytogenetics description
Molecular / cytogenetics images

Contributed by Madina Sukhanova, Ph.D.
<i>RUNX1</i> whole gene deletion

RUNX1 whole gene deletion



Images hosted on other servers:
<i>RUNX1</i> germline mutations

RUNX1 germline mutations

Reverse mosaicism due to LOH

Reverse mosaicism due to LOH

Videos
None
Sample pathology report
  • Peripheral blood, bone marrow aspirate and bone marrow core biopsy:
    • Myelodysplastic syndrome involving a variably cellular bone marrow (see comment)
    • Comment: There is dysplasia in the granulocyte, erythroid and megakaryocyte lineages. Immunochemical stain for CD34 highlights 3 - 4% blasts. No ring sideroblasts are identified. Myeloid NGS detected DNMT3A (Q656*, VAF 47%) and TET2 (Q1828*, VAF 46%) mutations. The overall findings are consistent with myelodysplastic syndrome with low blasts (MDS LB) per the WHO, 5th edition, 2022, online beta version and myelodysplastic syndrome, not otherwise specified (MDS, NOS), with multilineage dysplasia per ICC (Blood 2023;141:437)
    • Peripheral blood smear
      • Peripheral blood smear shows mild normocytic anemia
      • Red blood cells show anisopoikilocytosis including occasional teardrop forms, ovalocytes and acanthocytes
      • Absolute neutropenia with shift to immaturity
      • Thrombocytopenia with unremarkable morphology
    • Bone marrow aspirate smear / touch preparation
      • Bone marrow aspirate smear slides show several cellular spicules for evaluation
      • Myeloid precursors show shift to immaturity with many hypolobated / monolobated neutrophils
      • Erythroid precursors show progressive maturations with megaloblastic changes and occasional nuclear budding and irregular nuclear contour
      • Megakaryocytes are decreased but occasional small monolobated megakaryocytes are present
      • Touch preparations are cellular with similar findings to the bone marrow aspirates
      • Small hypolobated / monolobated megakaryocytes are present
    • Bone marrow core biopsy / particle clot
      • Bone marrow core biopsy is fragmented and variable cellularity, overall normocellular
        • Myeloid precursors show progressive maturation with unremarkable morphology
        • Small megakaryocytes are present
      • Particle clot sections show many bone marrow particles for evaluation
        • Marrow is hypercellular for age (~80% cellular)
        • Myeloid precursors show progressive maturation with many hypolobated / monolobated neutrophils
        • Erythroid precursors are relatively decreased and show progressive maturation with unremarkable morphology
        • Megakaryocytes are present with small hypolobated / monolobated forms
Differential diagnosis
Board review style question #1

Which of the following is a characteristic phenotype of megakaryocytes in bone marrow biopsy of an asymptomatic individual with familial RUNX1 mutation (shown in the image above)?

  1. Atypical megakaryocytes with separated nuclear lobes
  2. Dysplastic megakaryocytes with nonlobated or bilobed nuclei
  3. Numerous micromegakaryocytes
  4. Small hypolobated / monolobated megakaryocytes
Board review style answer #1
D. Small hypolobated / monolobated megakaryocytes is a characteristic finding during bone marrow examination of individuals with germline RUNX1 pathogenic or likely pathogenic (P / LP) variant. It should be noted that megakaryocytes are atypical but not dysplastic (Pediatr Dev Pathol 2019;22:315, Haematologica 2017;102:1661); thus, answer B is incorrect because nonlobated or bilobed megakaryocytes are common in AML with MECOM rearrangements. Answer A is incorrect because megakaryocytes with separated nuclear lobes are typical for GATA2 deficiency syndrome. Answer C is incorrect because the presence of numerous micromegakaryocytes in the bone marrow is frequently observed in MDS with isolated del(5q) syndrome.

Comment Here

Reference: Myeloid or lymphoid neoplasms with germline RUNX1 mutation
Board review style question #2
A 48 year old patient with a history of lifelong thrombocytopenia and family history of thrombocytopenia and myeloid malignancies in mother and maternal uncle was found to have pancytopenia during routine monitoring. Bone marrow examination revealed 3 - 4% blasts, erythroid precursors with unremarkable morphology and megakaryocytes with small monolobated forms. Conventional cytogenetic analysis did not reveal any chromosomal aberrations. SNP array analysis detected intragenic deletion of RUNX1 and NGS revealed pathogenic mutations in TET2, DNMT3A and ASXL1. Which of detected genetic abnormalities is most likely a germline one?

  1. ASXL1 mutation
  2. DNMT3A mutation
  3. RUNX1 intragenic deletion
  4. TET2 mutation
Board review style answer #2
C. RUNX1 intragenic deletion. The patient's personal history and family history as well as small monolobated megakaryocytes are concerning for RUNX1 pathogenic or likely pathogenic (P / LP) variant, in this case intragenic deletion, which should be confirmed in cultured skin fibroblasts (Pediatr Dev Pathol 2019;22:315, Haematologica 2017;102:1661). Answers A, B and D are incorrect because mutations in TET2, DNMT3A and ASXL1 are common during oncogenic progression and are somatic events.

Comment Here

Reference: Myeloid or lymphoid neoplasms with germline RUNX1 mutation

Myeloid or lymphoid neoplasms with germline TP53 mutation (pending)
[Pending]

Myeloid sarcoma
Definition / general
  • Tumor mass consisting of myeloblasts occurring at an anatomical site other than the bone marrow and distorts the normal tissue architecture
  • Although commonly associated with concurrent acute myeloid leukemia, myeloid sarcoma can occur in isolation
  • Diagnosis of myeloid sarcoma should be considered an equivalent to a diagnosis of acute myeloid leukemia (AML)
Essential features
  • Based on the WHO classification, myeloid sarcoma must present as a tumor mass with tissue architectural effacement
  • Important distinction
    • Leukemia cutis (LC) is a somewhat nonspecific term defined as a cutaneous infiltration by neoplastic leukocytes (myeloid or lymphoid; mature or immature) (Am J Clin Pathol 2008;129:130)
    • Leukemia cutis is not an equivalent of myeloid sarcoma without fulfilling the myeloid sarcoma diagnostic criteria by WHO classification
    • Leukemia cutis could be seen in non-AML patients with myeloproliferative neoplasm (MPN), myelodysplastic syndrome (MDS) or myelodysplastic / myeloproliferative neoplasm (MDS / MPN), such as chronic myelomonocytic leukemia (CMML)
  • Can present as de novo lesions, as therapy related myeloid neoplasms or as disease progression of MPN, MDS or MDS / MPN
Terminology
  • Granulocytic sarcoma
  • Chloroma
  • Extramedullary myeloid tumor
ICD coding
  • ICD-10: C92.3 - myeloid sarcoma
Epidemiology
  • Can occur at any age
Sites
  • Can involve any site; the most common sites of involvement are lymph nodes, skin, soft tissue, GI tract, bone and testicles
Pathophysiology
  • Exact mechanism is not fully understood
  • Chemokine / chemokine receptor interactions are implicated in the migration of leukemia cells migration to skin (Pediatr Blood Cancer 2010;55:344)
  • Matrix metalloproteinases (MMPs) and integrin are implicated in the leukemia cells migration and the blood brain barrier disruption (Blood 2009;114:3008, PLoS One 2011;6:e20599)
  • CXCR4, CXCL12 and CD56 have been previously reported to be associated with specific sites of myeloid sarcoma, which was not confirmed by a more recent study (Am J Surg Pathol 2016;40:1473)
Clinical features
  • Variable clinical presentation depending on the location of tumor (Clin Lymphoma Myeloma Leuk 2017;17:263)
  • Primary myeloid sarcoma (de novo): no pre-existing diagnosis of myeloid neoplasm or AML
  • Secondary myeloid sarcoma: concurrent or pre-existing diagnosis of myeloid neoplasm or AML
  • Isolated myeloid sarcoma: primary or secondary myeloid sarcoma with no bone marrow involvement
Diagnosis
  • Broad panel of immunohistochemical stains required to avoid misdiagnosis (Am J Clin Pathol 2015;144:219)
  • Cases without concurrent bone marrow involvement may have no fresh material for karyotyping and flow cytometric studies; fluorescence in situ hybridization (FISH) studies and mutation analysis should still be attempted on formalin fixed, paraffin embedded (FFPE) samples
Laboratory
  • Findings vary, depending on the location of the tumor and the bone marrow involvement status
Prognostic factors
Case reports
Treatment
  • No universal consensus on treatment (Leukemia 2021;35:1193)
  • Treatment strategies may depend on the disease presentation (initial or relapse) and vary from institution to institution
  • Treatment includes systemic chemotherapy, local therapy (surgery and radiation therapy), bone marrow transplant and target therapy
Microscopic (histologic) description
  • At least partial effacement of the architecture
  • Neoplastic cells can be seen forming cohesive nests, mimicking carcinoma or lymphoma or infiltrating as single files
  • Granulocytic, monocytic or rarely, erythroblastic or megakaryoblastic differentiation can be seen
  • Neoplastic cells usually demonstrate dispersed chromatin with high N/C ratio
  • Mitotic figures and apoptosis are commonly identified indicating a high proliferation rate
Microscopic (histologic) images

Contributed by Yen-Chun Liu, M.D., Ph.D.
Architecture effaced by neoplasm

Architecture effaced by neoplasm

Sheets of myeloid blasts

Sheets of myeloid blasts

Blasts positive for CD34

Blasts positive for CD34

Blasts positive for CD33

Blasts positive for CD33


Nasopharyngeal mass

Nasopharyngeal mass

Mitosis and apoptosis identified

Mitosis and apoptosis identified

Blasts positive for CD117

Blasts positive for CD117

Blasts positive for MPO

Blasts positive for MPO

Positive stains
Negative stains
  • Negative for lymphoid lineage markers, such as CD20 and CD3
  • Usually negative for CD19 and PAX5 but association with t(8;21)(q22;q22.1) RUNX1-RUNX1T1 can show positive expression (Blood 1992;80:470)
Flow cytometry description
  • Workup similar to that for acute leukemia but fresh material is usually lacking
Molecular / cytogenetics description
  • Roughly half of cases show cytogenetic abnormalities
  • Rare cases may show discrepancies from the karyotype obtained from the corresponding bone marrow, which may suggest clonal evolution
  • Cytogenetic and molecular abnormalities seen in myeloid sarcoma are similar to those seen in AML
  • Common translocations include t(8;21)(q22;q22) RUNX1-RUNX1T1 and inv(16)(p13.1q22) CBFB-MYH11
  • Complex karyotype can be seen, usually in secondary myeloid sarcoma (Eur J Haematol 2018;100:603)
  • Common mutations include NRAS, NPM1, FLT3-ITD, KRAS and IDH2 (Blood Cancer J 2018;8:43)
  • JAK2 V617F mutation can be seen, especially in cases with pre-existing MPN
  • BCR-ABL1 fusion can be seen and may represent disease progression of chronic myeloid leukemia (CML)
Sample pathology report
  • Nasopharyngeal mass, biopsy:
    • Myeloid sarcoma (see comment)
    • Comment: The histologic sections demonstrate sheets of immature mononuclear cells. The immature cells are positive for CD34, CD33, CD117, MPO and negative for CD19 and CD3 by immunohistochemical stains. The morphologic and immunohistologic findings in the current sample support a diagnosis of myeloid sarcoma. A bone marrow examination to investigate for potential involvement by a myeloid neoplasm / acute myeloid leukemia will be of interest if clinically indicated. If no fresh diagnostic material can be retrieved from additional tissue / bone marrow examination, fluorescence in situ hybridization studies and molecular studies can be requested on the current sample for further characterization.
Differential diagnosis
  • Lymphoblastic lymphoma:
    • Can demonstrate similar blast-like morphology
    • Relies on immunophenotyping to differentiate
    • Positive for T cell or B cell lineage markers; commonly positive for TdT
  • Burkitt lymphoma:
    • Usually has a prominent starry sky appearance, though the starry sky appearance is not entirely specific
    • Positive for B cell lineage markers, not myeloid markers
    • MYC rearrangement present
  • Diffuse large B cell lymphoma:
    • Predominately large cells; cytologic features are less likely blastoid
    • Positive for B cell lineage markers, not myeloid markers
  • Histiocytic sarcoma:
    • Can be a very challenging differential diagnosis due to significant overlapping immunophenotypic features
    • Less likely to demonstrate bone marrow involvement
  • Nonhematopoietic tumor:
    • Relies on immunophenotyping in some scenarios
    • Commonly used markers for screening include cytokeratin, CD45 and S100
  • Extramedullary hematopoiesis:
    • More likely occurs in patients with MPN
    • Trilineage hematopoiesis with maturation is usually present
  • Blastic plasmacytoid dendritic cell neoplasm (BPDCN):
Board review style question #1
A 49 year old man with a history of MPN presented with small intestinal obstruction and was found to have a soft tissue mass on abdominal CT scan. Excisional biopsy showed sheets of immature mononuclear cells with fine chromatin. Tumor cells were positive for MPO, CD34, CD117 and CD33 and negative for TdT. A diagnosis of myeloid sarcoma was rendered. Which of the following statements is true about myeloid sarcoma?

  1. Always presents as a solitary mass
  2. Can occur with a concurrent myeloid disorder
  3. CD43 is not a good marker for the diagnosis
  4. Only occurs in the GI tract
Board review style answer #1
B. Can occur with a concurrent myeloid disorder (Clin Lymphoma Myeloma Leuk 2017;17:263)

Comment Here

Reference: Myeloid sarcoma
Board review style question #2
Nasopharyngeal mass Blasts positive for MPO


Which of the following statements is true about myeloid sarcoma?

  1. Always shows megakaryoblastic differentiation
  2. Can only be identified with concurrent acute myeloid leukemia in bone marrow
  3. Never shows cytogenetic abnormalities
  4. NPM1 mutation can be found in myeloid sarcoma and lesions with NPM1 mutations tend to demonstrate myelomonocytic morphology
Board review style answer #2
D. NPM1 mutation can be found in myeloid sarcoma and lesions with NPM1 mutations tend to demonstrate myelomonocytic morphology (Clin Lymphoma Myeloma Leuk 2017;17:263, Blood 2016;127:2391)

Comment Here

Reference: Myeloid sarcoma

Myeloma prognostic probe sets (pending)
[Pending]

NK lymphoblastic leukemia/lymphoma (pending)
[Pending]

Non secretory myeloma (pending)
[Pending]

PDGFRA rearrangement
Definition / general
  • Myeloid, lymphoid or combined myeloid / lymphoid neoplasms often accompanied by eosinophilia and associated with FIP1L1-PDGFRA fusion, other PDGFRA gene rearrangements or an activating mutation of the PDGFRA gene
Essential features
  • Myeloid or lymphoid neoplasm, often accompanied by prominent eosinophilia
  • Involved PDGFRA gene, cytogenetic location 4q12 (N Engl J Med 2003;348:1201)
  • Rearrangement of PDGFRA must be defined by FISH or reverse transcription PCR
  • Multiple fusion partners are identified
  • Can have different bone marrow pictures, most commonly chronic eosinophilic leukemia (CEL) or other diseases (e.g., acute transformation to acute myeloid leukemia or T cell acute lymphoblastic leukemia [T ALL])
  • Very sensitive to imatinib therapy
  • Associated mast cell hyperplasia
Terminology
  • Myeloid and lymphoid neoplasms with PDGFRA rearrangement
  • Myeloid and lymphoid neoplasms associated with PDGFRA rearrangement
ICD coding
  • ICD-11: 2A50 - myeloid neoplasm associated with PDGFRA rearrangement
Epidemiology
Sites
  • Usually, peripheral blood and bone marrow
Pathophysiology
  • Most common translocation partner is FIP1L1; FIP1L1-PDGFRA is generated from a submicroscopic, 800 kb interstitial deletion on chromosome 4, del(4)(q12q12), a cryptic deletion of CHIC2
  • Other fusion partners have been identified:
  • t(4;22)(q12;q11) BCR-PDGFRA translocation is a neoplasm with features intermediate between chronic myeloid leukemia and chronic eosinophilic leukemia (CEL) (Immunol Allergy Clin North Am 2007;27:377)
  • Extramedullary myeloid tumor can harbor PDGFRA rearrangement despite the bone marrow not displaying the rearrangement (Cancer Genet 2018;220:13)
  • Activating mutations of PDGFRA can play a role in tumorigenesis (e.g., Y288C mutation) (Nat Commun 2018;9:4583)
Etiology
Clinical features
  • Always involves peripheral blood and bone marrow; however, other organs can be affected (e.g., lungs, CNS, skin and GI tract)
  • Usually presents as chronic eosinophilic leukemia (CEL); can present as acute myeloid leukemia, T acute lymphoblastic leukemia or myeloid sarcoma (Leukemia 2007;21:1183, Head Neck Pathol 2021;15:1399)
  • Symptoms range from asymptomatic to generalized fatigue, respiratory (dyspnea, cough) or GI symptoms (Leuk Lymphoma 2013;54:897)
  • Can present with cardiac symptoms, such as endomyocardial fibrosis, Loeffler endocarditis and valvular regurgitation (Korean J Intern Med 2018;33:642)
  • Splenomegaly is more common than hepatomegaly
  • Can present as arterial or venous thrombosis or thrombotic thrombocytopenic purpura (TTP) (Case Rep Hematol 2019;2019:2820954)
  • May present with anemia and thrombocytopenia
Diagnosis
  • Peripheral blood and bone marrow examination both usually show eosinophilia (sometimes normal)
  • FISH is the most used method for diagnosis (also known as CHIC2 deletion) and chromosomal microarray; however, reverse transcription PCR is more sensitive (Oncol Lett 2020;19:3587)
  • Phosphoflow cytometry can be used for detection of PDGFRA rearrangement; however, it is not very sensitive in patients on treatment (Cytometry A 2021;99:784)
  • Next generation sequencing can show additional mutations associated with this rearrangement (e.g., somatic KIT and CSF3R mutations) (Haematologica 2018;103:e348, Am J Hematol 2016;91:E10)
  • Diagnostic criteria for myeloid / lymphoid neoplasms with eosinophilia associated with FIP1L1-PDGFRA or a variant fusion gene (A):
    • Myeloid or lymphoid neoplasm, usually with prominent eosinophilia and presence of FIP1L1-PDGFRA fusion gene or a variant fusion gene with rearrangement of PDGFRA or an activating mutation of PDGFRA (B):
      • A) Cases presenting as a myeloproliferative neoplasm, acute myeloid leukemia or lymphoblastic leukemia / lymphoma with eosinophilia and FIP1L1-PDGFRA gene fusion are assigned to this category
      • B) If appropriate molecular analysis is not possible, this diagnosis should be suspected if there is a myeloproliferative neoplasm with no Philadelphia chromosome and with the hematological features of chronic eosinophilic leukemia associated with splenomegaly, marked elevation of serum vitamin, elevation of serum tryptase (> 12 ng/dL and < 20 ng/dL) and an increased number of bone marrow mast cells
Laboratory
  • Complete blood count usually shows eosinophilia, usually > 1,500/uL; however, normal counts can be found
  • Increased vitamin B12 levels > 1,900 ng/L in all cases with FIP1L1-PDFGRA due to increased levels of haptocorrins (Am J Hematol 2019;94:1149)
  • Increased tryptase levels > 12 ng/dL and < 20 ng/dL (minor criteria for systemic mastocytosis)
Prognostic factors
  • Good prognosis with absence of cardiac involvement
Case reports
  • 51 year old woman with cutaneous T cell lymphoma with PDGFRA rearrangement and absence of eosinophilia (Am J Dermatopathol 2018;40:610)
  • 60 year old man with chronic eosinophilic leukemia with PDGFRA rearrangement associated with tumor lysis syndrome as a complication of imatinib therapy (J Glob Oncol 2018;4:1)
  • 70 year old man presenting with myeloid sarcoma in the retromolar area with PDGFRA rearrangement (Head Neck Pathol 2021;15:1399)
Treatment
  • Usually sensitive to imatinib; alternative tyrosine kinase inhibitors (TKIs; e.g., midostaurin or sorafenib) may be effective
  • Treatment with imatinib even in cases presenting with acute myeloid leukemia or T cell acute lymphoblastic leukemia
  • However, T674I mutation within the adenosine triphosphate (ATP) binding domains of PDGFRA is resistant to imatinib and second generation tyrosine kinase inhibitors; a panresistant D842 variant was also identified (Hemasphere 2019;3:e182, Leukemia 2009;23:845)
  • Third generation TKI ponatinib is active against both FIP1L1-PDGFRα p.T674I and p.D842 V (Hemasphere 2019;3:e182)
    • Midostaurin and sorafenib are alternative regimens
  • Some activating mutations of PDGFRA (e.g., Y288C mutations) are resistant to PDGFRA inhibitors; however, they are sensitive to PI3K / mTOR inhibitors (Nat Commun 2018;9:4583)
  • Cases of acute myeloid leukemia with PDGFRA rearrangement and another mutation can achieve complete remission with tyrosine kinase inhibitors (Oncol Lett 2020;19:3587)
  • End point of treatment is unclear; in general, imatinib can effectively suppress but not eliminate the FIP1L1-PDGFRA clone in most patients (Am J Hematol 2019;94:1149)
  • Allogeneic hematopoietic stem cell transplantation would be the option for those resistant / refractory cases (Blood 2017;129:704)
Microscopic (histologic) description
  • Monocytes and basophil counts are normal
  • Hypercellular marrow with increased eosinophils and its precursors (see Microscopic [histologic] images); clusters or scattered, increased spindle shaped CD25+ or CD2- mast cells present (Blood 2003;101:4660)
  • Reticulin fibrosis is identified; sometimes can be normal (Blood 2004;103:473)
  • Increased blasts identified in accelerated / blastic phase of myeloproliferative neoplasms or de novo acute myeloid leukemia or T cell acute lymphoblastic leukemia (see Microscopic [histologic] images)
  • Tissue eosinophilic infiltrate forming eosinophilic microabscesses or identifiable Charcot-Leyden crystals
Microscopic (histologic) images

Contributed by Ismail Elbaz Younes, M.D. and Ling Zhang, M.D.
Core biopsy Core biopsy Core biopsy

Core biopsy

Peripheral smear description
  • Peripheral blood eosinophilia, occasionally neutrophilia; eosinophils are usually mature
  • Variable morphologic abnormalities can be present in the eosinophils, including cytoplasmic vacuolation, nuclear hypersegmentation / hyposegmentation; morphologic changes are not unique as reactive conditions can have these changes as well (Foucar: Bone Marrow Pathology, 4th Edition, 2019)
  • Some cases will present with increased blasts if the patient has acute lymphocytic leukemia or acute myeloid leukemia
Peripheral smear images

Contributed by Ismail Elbaz Younes, M.D. and Ling Zhang, M.D.
Increased eosinophils with abnormal granulation Increased eosinophils with abnormal granulation

Increased eosinophils with
abnormal granulation

Molecular / cytogenetics description
  • FISH shows loss of CHIC2 gene
Molecular / cytogenetics images

Contributed by Ismail Elbaz Younes, M.D. and Ling Zhang, M.D.
Deletion of <i>CHIC2</i>

Deletion of CHIC2

Sample pathology report
  • Peripheral blood:
    • Microcytosis with mild anisopoikilocytosis
    • Moderate neutropenia with marked eosinophilia
    • Moderate thrombocytopenia
  • Bone marrow, left posterior iliac crest, aspirate and biopsy:
    • Hypercellular marrow with marked hypereosinophilia and FIP1L1-PDGFRA rearrangement (see comment)
    • Marked reticulin and diffuse collagen fibrosis
    • Absent iron stores
    • Comment: Cytogenetics confirms a normal male karyotype (46,XY). FISH identifies a microdeletion of the CHIC2 gene on chromosome 4q12, resulting in fusion of FIP1L1-PDGFRA. No abnormality of PDGFRB (5q33) is identified.
  • Microscopic description:
    • Peripheral smear: The hemoglobin is normal, with microcytic indices and mild anisopoikilocytosis. The reticulocyte count is decreased for the degree of anemia. The total white cell count is moderately increased, with a moderately severe neutropenia and a marked eosinophilia composed of relatively mature appearing eosinophils with normal granulation. Circulating blasts are not seen. The platelet count is moderately decreased, with relatively normal platelet morphology.
    • Aspirate smear: The Wright stained aspirate smears are hypercellular with several poorly spread spicules, adequate for evaluation. Megakaryocytes are noted in moderately increased numbers, including many large hyperlobulated forms and few hypolobulated forms with increased cytoplasm. Myeloid and erythroid precursors are evenly distributed, with an M:E ratio calculated at 2.6. Myeloid precursors show adequate maturation and relatively normal morphology. There is no increase in blast cells. There is a marked eosinophilia, with predominantly mature segmented eosinophils showing normal granulation. Erythroid precursors are admixed, with adequate maturation and mild megaloblastic change. There is no significant increase in mast cells.
    • Clot section and core biopsy: The clot section (A) and core biopsy (B) are fixed in B-Plus Fix (BBC) fixative. The core biopsy (B) is decalcified in Nitrical (5% nitric acid). 1 level of the clot section (A) and 1 level of the core biopsy (B) are stained with PAS to evaluate myeloid maturation and more accurately estimate an M:E ratio. 1 level of the clot section (A) and 1 level of the core biopsy (B) are stained with Prussian blue to evaluate iron stores and the average value reported. 1 level of the core biopsy (B) is stained with reticulin stain to evaluate marrow fibrosis.
    • Thickened bony trabeculae show extensive evidence of bony remodeling and surround a marrow that is variably densely fibrotic and variably hypercellular, with prominent dilatation of the marrow sinuses. The overall cellularity approaches 100%. An occasional interstitial benign lymphoid aggregate is noted. Megakaryocytes showing the aforementioned abnormalities are present, forming distinct clusters and few small aggregates. Large erythroid islands are noted, with interspersed marrow parenchyma containing predominantly mature eosinophils. Admixed neutrophils are also seen. An M:E ratio is estimated at 10:1. Stainable iron is absent. There is a diffuse marked reticulin fibrosis associated with the aforementioned collagen fibrosis. Collagen accounts for approximately 20% of the total marrow space.
Differential diagnosis
  • Chronic eosinophilic leukemia, NOS:
    • Sustained eosinophilia (> 1.5 x 109/L) with other reactive processes excluded
    • < 20% blasts in peripheral blood or bone marrow, no inv(16)(p13.1q22), t(16;16)(p13.1q22), t(8;21)(q22;q22.1)
    • Bone marrow aspirate is hypercellular with abundant eosinophils and eosinophilic precursors with dysplastic features
    • Clonal cytogenetic (e.g., +8, -20q) or molecular genetic abnormalities (e.g., TET2, ASXL1 and DNMT3A); if no clonal abnormalities are found, then blast count must be > 2% in peripheral blood or > 5% in bone marrow aspirate
    • No PDGFRA, PDGFRB, FGFR rearrangement, BCR-ABL1, PCM1-JAK2, ETV6-JAK2 or BCR-JAK2 fusions
    • No lymphoid or myeloid neoplasms (e.g., myelodysplastic syndrome, myeloproliferative neoplasms, systemic mastocytosis or acute myeloid leukemia) are identified
  • Hypereosinophilic syndrome:
    • Sustained eosinophilia (> 1.5 x 109/L) for at least 6 months with other reactive processes excluded
    • Tissue damage must be present
    • Exclude cytokine related lymphoid variant hypereosinophilia
    • Bone marrow shows increased eosinophils with no dysplastic features and no increase in blast cells
    • No cytogenetic or molecular alteration is identified
    • No PDGFRA, PDGFRB, FGFR rearrangement, BCR-ABL1, PCM1-JAK2, ETV6-JAK2 or BCR-JAK2 fusions; no inv(16)(p13.1q22), t(16;16)(p13.1q22) or t(8;21)(q22;q22.1)
    • No lymphoid or myeloid neoplasms (e.g., myelodysplastic syndrome, myeloproliferative neoplasms, systemic mastocytosis or acute myeloid leukemia) are identified
  • Reactive eosinophilia:
    • Can present with the increased eosinophils and can have abnormal eosinophils in the peripheral blood
    • Secondary causes (infection, allergies, drugs, skin diseases, asthma, adrenal insufficiency, gastrointestinal eosinophilia) are present
    • No evidence of PDGFRA rearrangement or active mutation or other cytogenetic / molecular abnormalities
  • Idiopathic hypereosinophilia:
    • Sustained eosinophilia (> 1.5 x 109/L) for at least 6 months
    • No tissue damage
    • No reactive causes, lymphoid or myeloid neoplasms are identified
    • Bone marrow shows increased eosinophils with no dysplastic features and no increase in blast cells
    • No molecular alteration is identified
    • No PDGFRA, PDGFRB, FGFR rearrangement, BCR-ABL1, PCM1-JAK2, ETV6-JAK2 or BCR-JAK2 fusions
  • Systemic mastocytosis with eosinophilia:
    • Major diagnostic criteria: 4 minor criteria or multifocal infiltrates of mast cells (≥ 15 mast cells in aggregates) + 1 minor criterion
    • Minor criteria:
      • > 25% of mast cells with atypical morphology (e.g., spindle shape or immature nuclear features)
      • Presence of KIT D816V
      • CD2 or CD25 positive on mast cells
      • Tryptase levels are usually more than 20 ng/dL
    • Bone marrow shows eosinophilia with variable dysplastic features
    • No PDGFRA rearrangement
Board review style question #1

What is the most common translocation partner in myeloid and lymphoid neoplasms with PDGFRA rearrangement?

  1. BCR
  2. ETV6
  3. FIP1L1
  4. KIF5B
  5. STRN
Board review style answer #1
C. FIP1L1

Comment Here

Reference: PDGFRA rearrangement
Board review style question #2
What is the most deleted gene in myeloid and lymphoid neoplasms with PDGFRA rearrangement?

  1. ABL
  2. BCR
  3. CHIC2
  4. FGFR
  5. TNK2
Board review style answer #2

PDGFRB rearrangement
Definition / general
  • Myeloid or lymphoid neoplasm with frequent eosinophilia, sometimes neutrophilia or monocytosis; associated with rearrangement of PDGFRB, at chromosome 5, long arm (5q32)
  • Discovered in 1994 by Golub TR, Barker GF, Lovett M, Gilliland DG (Cell 1994;77:307)
  • To date, more than 40 fusion partners identified
Essential features
  • Can have different presentation; e.g., myeloproliferative neoplasm (MPN), myelodysplastic syndrome / myeloproliferative neoplasm (MDS / MPN), acute myeloid leukemia (AML), with MDS / MPN overlap syndrome being more common
  • Conventional karyotyping is the most common used modality for diagnosis, confirmation by FISH PDGFRB (break apart probe) or RT-PCR is recommended due to multiple other genes being located at 5q31-33 (e.g., IL3, IL5, GM-CSF)
Terminology
  • Chronic myelomonocytic leukemia with eosinophilia associated with t(5;12); myeloid neoplasms with PDGFRB rearrangement
  • Myeloid neoplasms associated with PDGFRB rearrangement
ICD coding
  • ICD-11: 2A51 - myeloid neoplasm associated with PDGFRB rearrangement
Epidemiology
Sites
  • Peripheral blood and bone marrow is always involved
  • Spleen; other organs can be involved as well
Pathophysiology
  • Most common translocation t(5;12)(q32;p13.2) ETV6::PDGFRB fusion gene
  • More than 40 different partners are identified (Histopathology 2020;76:1042)
    • t(1;3;5)(p36;p21;q33) WDR48::PDGFRB
    • t(1;5)(q21;q33) TPM3::PDGFRB
    • t(1;5)(q23;q33) PDE4DIP::PDGFRB
    • t(5;7)(q33;q11.2) HIP1::PDGFRB
    • t(5;10)(q33;q21) CCDC6 (H4/D10S170)::PDGFRB
  • Some fusions are typically associated with BCR::ABL1-like B cell acute lymphoblastic leukemia (B ALL) (e.g., PDGFRB fused with EBF1, SSBP2, TNIP1, ZEB2 and ATF7IP)
Etiology
  • Unknown
Clinical features
  • Can present with MPN, MDS / MPN (e.g., atypical chronic myeloid leukemia [CML], chronic myelomonocytic leukemia [CMML] or juvenile myelomonocytic leukemia [JMML])
  • Rarely myeloid blast phase / secondary AML or de novo B ALL (Histopathology 2020;76:1042, Blood 2017;129:704)
  • Can present with B symptoms
  • Hepatosplenomegaly
  • Cardiac and skin damage
  • Good response to imatinib or other tyrosine kinase inhibitor (TKI) therapy
Diagnosis
  • Peripheral blood and bone marrow examination both usually show eosinophilia (sometimes normal) and other features representative of MPN, MDS / MPN or transformed AML
  • Conventional karyotyping
  • FISH PDGFRB for confirmation, especially cryptic cases
  • RT-PCR has limited utility due to multiple fusion partners (Foucar: Bone Marrow Pathology, 4th Edition, 2019)
  • Next generation sequencing (NGS) can detect cases missed by FISH (Cancer Genet 2020;244:55)
  • According to WHO, diagnostic criteria for myeloid / lymphoid neoplasms associated with ETV6::PDGFRB or other rearrangement of PDGFRB:
    • Myeloid or lymphoid neoplasm often with prominent eosinophilia and sometimes with neutrophilia or monocytosis and presence of t(5;12)(q32;p13.2) or a variant translocationa,b or demonstration of ETV6::PDGFRB fusion gene or other rearrangement of PDGFRBb
      1. Cases with fusion genes typically associated only with BCR::ABL1-like B lymphoblastic leukemia are specifically excluded
      2. Because t(5;12)(q32;p13.2) does not always result in ETV6::PDGFRB fusion, molecular confirmation is highly desirable; if molecular analysis is not possible, this diagnosis should be suspected if there is a myeloproliferative neoplasm associated with eosinophilia, with no Philadelphia (Ph) chromosome and with a translocation with a 5q32 breakpoint
Laboratory
  • Complete blood count (CBC) usually shows eosinophilia; however, normal counts can be found
  • Increased tryptase levels (> 12 ng/dL); however, < 20 ng/dL (Expert Rev Hematol 2014;7:683)
  • Vitamin B12 levels may be increased
Prognostic factors
  • Median overall survival is < 2 years before introduction of tyrosine kinase inhibitor; a case series of 26 cases treated with imatinib showed a longer survival (up to 6.6 years)
Case reports
  • 4 month old girl with refractory juvenile xanthogranuloma with a novel MRC1::PDGFRB fusion and a dramatic response induced by dasatinib (Blood Adv 2020;4:2991)
  • 22 year old woman with relapsed BCR::ABL1-like ALL with CCDC88C::PDGFRB fusion and complete remission after treatment with tyrosine kinase inhibitor (Int J Hematol 2021;113:285)
  • 22 year old man with a cryptic imatinib sensitive G3BP1::PDGFRB rearrangement in a myeloid neoplasm with eosinophilia (Blood Adv 2020;4:445)
  • 70 year old man with asymptomatic leukocytosis, thrombocytosis with PDGFRB rearrangement and absence of eosinophilia (Leukemia 2016;30:1402)
  • 76 year old man with myeloid / lymphoid neoplasms associated with eosinophilia, with negative FISH and positive next generation sequencing for PDGFRB rearrangement (Cancer Genet 2020;244:55)
Treatment
Microscopic (histologic) description
  • Peripheral blood shows leukocytosis including eosinophilia, neutrophilia or monocytosis; sometimes circulating immature myeloid precursors (e.g., metamyelocytes, myelocytes or promyelocytes and occasional blasts / blast equivalents)
  • Increased blasts up to 20% seen in blastic phage
  • Anemia and thrombocytopenia can be seen
  • Bone marrow is hypercellular with myeloid preponderance including many eosinophils and neutrophils
  • Rarely there might be increase in basophils
  • Can show increase in mast cells with identified abnormal spindle shaped forms and expression of CD2 and CD25 (Am J Hematol 2007;82:77, Haematologica 2007;92:163)
  • Reticulin fibrosis (Blood 2008;111:1855)
Microscopic (histologic) images

Contributed by Ismail Elbaz Younes, M.D. and Ling Zhang, M.D.
Increased blasts in bone marrow core biopsy Increased blasts in bone marrow core biopsy

Increased blasts in bone marrow core biopsy

Dysplastic megakaryocytes and neutrophils

Dysplastic megakaryocytes and neutrophils

Increased blasts

Increased blasts

Peripheral smear description
  • Peripheral blood eosinophilia and occasionally neutrophilia, eosinophils are usually mature
  • Variable morphologic abnormalities can be present in the eosinophils, including cytoplasmic vacuolation, nuclear hyper / hyposegmentation
Peripheral smear images

Contributed by Ismail Elbaz Younes, M.D. and Ling Zhang, M.D.
Increased myeloid blasts

Increased myeloid blasts

Molecular / cytogenetics description
  • FISH rearrangement with 5q31~33 breakpoint, typically t(5;12)(q31~33;p12) creating ETV6::PDGFRB
Molecular / cytogenetics images

Contributed by Ismail Elbaz Younes, M.D. and Ling Zhang, M.D.
Rearrangement with 5q31~33 breakpoint

Rearrangement with 5q31~33 breakpoint



Images hosted on other servers:

FISH assays indicate rearrangement of PDGFRB

Sample pathology report
  • Peripheral blood:
    • Mild macrocytic anemia
    • Neutrophilia with left shifted granulocyte maturation
    • Monocytosis
    • Mild thrombocytopenia
  • Bone marrow, left posterior iliac crest, aspirate and biopsy:
    • Hypercellular marrow with myeloid hyperplasia and monocytosis, consistent with a myeloid neoplasm with PDGFRB rearrangement (see comment)
    • Mildly increased reticulin fibers
    • Comment: Cytogenetics reported 46,XY,t(5;14)(q33;q32)[18]/46,XY[2]. FISH was reported positive for PDGFRB gene rearrangement. An MDS FISH panel and FISH for BCR::ABL were reported negative. Flow cytometry reported atypical myeloid maturation with decreased CD10, atypical maturation of CD64 / CD14 and increased HLA-DR expression. Given the cytogenetic findings, this myeloid neoplasm with PDGFRB rearrangement is associated with a hematologic diagnosis of chronic myelomonocytic leukemia with eosinophilia. The morphology is compatible with chronic myelomonocytic leukemia and there are focal areas of slightly increased eosinophils seen on the core biopsy.
  • Microscopic description:
    • Peripheral smear: There is a mild macrocytic anemia with mild anisopoikilocytosis. The total white blood cell count is elevated, with a neutrophilia and monocytosis. Left shifted granulocytes are seen, including an occasional circulating blast. The monocytes are maturing. The granulocytes have variable cytoplasmic granulation. Platelets are mildly decreased.
    • Aspirate smear: The Wright stained aspirate smear is cellular, with spicules available for evaluation. Megakaryocytes are seen in adequate number, with an occasional hypolobated form. Myeloid and erythroid precursors are seen. The myeloid precursors show slight left shifted maturation, with approximately 4% blasts. The myeloid precursors have variable toxic granulation and include a few precursors with mild nuclear:cytoplasmic asynchrony. Monocytes comprise approximately 25% of the differential and are unevenly distributed. The monocytes include approximately 5% promonocytes and many immature monocytes. Eosinophils include forms with some large basophilic granules. Erythroid precursors show maturation. Lymphocytes and scattered plasma cells are seen in the background.
    • Aspirate iron: An aspirate smear is stained for iron by the contributor. Stainable iron stores are present. A rare ring sideroblast is seen.
    • Clot section and core biopsy: Subcortical bone and normal bony trabeculae surround the marrow space with a cellularity approaching 100%. Megakaryocytes are seen in slightly decreased numbers, with occasional hypolobated forms. Small erythroid islands with admixed myeloid precursors are seen, with an M:E ratio estimated at greater than 5:1. The myeloid precursors have focal areas of left shifted maturation. No clusters of blasts are seen on the H&E stained section. Focal areas of slightly increased eosinophils are noted. Stainable iron stores are adequate. Reticulin fibers are mildly increased.
Differential diagnosis
  • Chronic eosinophilic leukemia, NOS:
    • Sustained eosinophilia (> 1.5 x 109/L) with other reactive processes excluded
    • < 20% blasts in peripheral blood or bone marrow, no inv(16)(p13.1q22), t(16;16)(p13.1q22), t(8;21)(q22;q22.1)
    • Bone marrow aspirate is hypercellular with abundant eosinophils and eosinophilic precursors with dysplastic features
    • Clonal cytogenetic (e.g., +8, -20q) or molecular genetic abnormalities (e.g., TET2, ASXL1 and DNMT3A)
      • If no clonal abnormalities are found, then blast count must be > 2% in peripheral blood or > 5% in bone marrow aspirate
    • No PDGFRA, PDGFRB, FGFR rearrangement, BCR::ABL1, PCM1::JAK2, ETV6::JAK2 or BCR::JAK2 fusions
    • No lymphoid or myeloid neoplasms (e.g., MDS, MPNs, systemic mastocytosis [SM] or AML) are identified
  • Hypereosinophilic syndrome:
    • Sustained eosinophilia (> 1.5 x 109/L) for at least 6 months with other reactive processes excluded
    • Tissue damage must be present
    • Exclude cytokine related lymphoid variant hypereosinophilia
    • Bone marrow shows increased eosinophils with no dysplastic features and no increase in blast cells
    • No cytogenetic or molecular alteration is identified
    • No PDGFRA, PDGFRB, FGFR rearrangement, BCR::ABL1, PCM1::JAK2, ETV6::JAK2 or BCR::JAK2 fusions
    • No inv(16)(p13.1q22), t(16;16)(p13.1q22) or t(8;21)(q22;q22.1)
    • No lymphoid or myeloid neoplasms (e.g., MDS, MPNs, SM or AML) are identified
  • Reactive eosinophilia:
    • Can present with increased eosinophils and can have abnormal eosinophils in the peripheral blood
    • Secondary causes are present: infection (e.g., parasites; the most common cause), connective tissue disorders, certain cutaneous lesions (e.g., dermatitis herpetiform), paraneoplastic (e.g., Hodgkin lymphoma, T cell lymphoma, colon carcinoma), certain lung diseases (e.g., Churg-Strauss syndrome, aspergillosis), autoimmune disorder or on IL2 therapy (Hematology Am Soc Hematol Educ Program 2018;2018:326)
    • No evidence of PDGFRB rearrangement
  • Idiopathic hypereosinophilia:
    • Sustained eosinophilia (> 1.5 x 109/L) for at least 6 months
    • No tissue damage
    • No reactive causes, lymphoid or myeloid neoplasms are identified
    • Bone marrow shows increased eosinophils with no dysplastic features and no increase in blast cells
    • No molecular alteration is identified
    • No PDGFRA, PDGFRB, FGFR rearrangement, BCR::ABL1, PCM1::JAK2, ETV6::JAK2 or BCR::JAK2 fusions
  • Systemic mastocytosis with eosinophilia:
    • Major diagnostic criteria: 4 minor criteria or multifocal infiltrates of mast cells (≥ 15 mast cells in aggregates) + 1 minor criterion
    • Minor criteria
      • > 25% of mast cells with atypical morphology (e.g., spindle shape or immature nuclear features)
      • Presence of KIT D816V
      • CD2 or CD25 positive on mast cells
      • Tryptase levels are usually more than 20 ng/dL
    • Bone marrow shows eosinophilia with variable dysplastic features
    • No PDGFRB rearrangement
Board review style question #1

What is the most common translocation in myeloid / lymphoid neoplasms with eosinophilia and PDGFRB rearrangement?

  1. t(1;3;5)(p36;p21;q33) WDR48::PDGFRB
  2. t(1;5)(q21;q33) TPM3::PDGFRB
  3. t(1;5)(q23;q33) PDE4DIP::PDGFRB
  4. t(5;7)(q33;q11.2) HIP1::PDGFRB
  5. t(5;12)(q32;p13.2) ETV6::PDGFRB
Board review style answer #1
E. t(5;12)(q32;p13.2) ETV6::PDGFRB

Comment Here

Reference: PDGFRB rearrangement
Board review style question #2
Which of the following is the least useful in diagnosing in myeloid / lymphoid neoplasms with eosinophilia and PDGFRB rearrangement?

  1. FISH
  2. Karyotyping
  3. RT-PCR
  4. Snap array
Board review style answer #2
C. RT-PCR

Comment Here

Reference: PDGFRB rearrangement

POEMS syndrome (osteosclerotic myeloma)
Definition / general
  • Paraneoplastic syndrome due to an underlying plasma cell neoplasm
  • POEMS = polyneuropathy, organomegaly, endocrinopathy, M protein spike and skin changes
  • But the acronym does not include several of the common clinical manifestations
Essential features
  • Major criteria for diagnosis are:
    • Polyradiculopathy
    • Clonal plasma cell disorder
    • Sclerotic bone lesions
    • Elevated vascular endothelial growth factor
    • Castleman disease (also known as angiofollicular hyperplasia, 11 - 30% of cases, most commonly multicentric Castleman disease)
Terminology
  • POEMS acronym coined in 1980 by Bardwick et al. (Medicine (Baltimore) 1980;59:311)
  • Other names: osteosclerotic myeloma, Takatsuki syndrome, Crow-Fukase syndrome
ICD coding
  • ICD-10: D47.7 - other neoplasms of uncertain behavior of lymphoid, hematopoietic and related tissue
  • See also WHO coding
Epidemiology
  • Prevalence of approximately 0.3 per 100,000 reported in Japan but occurs worldwide
Pathophysiology
  • VEGF levels are high and correlate with disease activity but may not drive the process
  • IL12 also correlates with disease activity; IL6 produced by plasma cells may play a role
Clinical features
  • In addition to essential features above, other clinical features include:
    • Organomegaly
    • Endocrinopathy
    • Characteristic skin changes
    • Papilledema
    • Extravascular volume overload / anasarca
    • Thrombocytosis, with propensity for thrombosis
    • Abnormal pulmonary function tests, pulmonary fibrosis, pulmonary hypertension
    • Diarrhea, calciphylaxis (rare)
    • Generally no bone pain
Diagnosis
  • Diagnosis is made by fulfilling at least 3 major criteria, 2 of which must include polyradiculoneuropathy and a clonal plasma cell disorder, along with at least 1 minor criterion (Am J Hematol 2017;92:814)
  • Mandatory criteria (must have both): polyneuropathy and monoclonal plasma cell disorder
  • Other major criteria (at least 1 present): Castleman disease, sclerotic bone lesions, increased VEGF levels
  • Minor criteria (at least 1 present):
    • Organomegaly
    • Extravascular volume overload (edema, pleural effusion, ascites),
    • Endocrinopathy (adrenal, thyroid, pituitary, gonadal, parathyroid, pancreatic); hypogonadism most common
    • Skin changes (hyperpigmentation, hypertrichosis, glomeruloid angiomata, plethora, acrocyanosis, white nails, sclerodermoid changes, clubbing)
    • Papilledema
    • Thrombocytosis / polycythemia

Laboratory
  • Monoclonal light chain (especially lambda)
  • VEGF levels are high
  • Thrombocytosis or erythrocytosis, absence of cytopenias
Radiology description
  • Bone lesions
  • Signs of volume overload (e.g. pleural effusion, ascites)
Prognostic factors
  • Risk associated with the extent of the plasma cell disorder
  • Bone marrow plasmacytosis increases risk of cerebrovascular event
  • Number of clinical criteria met is not prognostic
  • Generally better overall survival than myeloma, older data shows median survival of 14 years
Case reports
Treatment
  • If no bone marrow involvement by plasma cell clone, local radiation therapy may be adequate (e.g. sclerotic plasmacytoma)
  • If disseminated disease or progression following local radiation, systemic therapy advised
  • Corticosterioids as a temporizing measure
  • Alkylating agents a mainstay, including high dose with stem cell transplant
  • Thalidomide, bortezomib can be considered but may worsen peripheral neuropathy
  • Prompt treatment recommended
Clinical images

Images hosted on other servers:

Classic findings of POEMS syndrome

Microscopic (histologic) description
  • Bone marrow biopsy may show megakaryocyte hyperplasia and clustering (no JAK2 mutation)
  • Lymphoid aggregates (50% of cases) with clonal plasma cell rimming
Microscopic (histologic) images

Contributed by Genevieve M. Crane, M.D., Ph.D. and Sarah Elsoukkary, M.D.

A pyogenic granuloma arising in a patient with POEMS syndrome

Biopsy of sclerotic bone lesions in a patient with POEMS syndrome

Bone marrow clot section


Bone marrow aspirate smear

CD138

CD138 / Ki67

CD56+ clone

Lambda

IHC for kappa and lambda of the bone marrow clot

Positive stains
  • Lambda clones predominate (95% of cases)
  • Other features are as for clonal plasma cell populations
Electron microscopy description
  • No features of macrophage associated demyelination in contrast to chronic inflammatory demyelinating polyradiculopathy
Molecular / cytogenetics description
  • Tend to have restricted immunoglobulin variable chain usage (IGLV1)
  • Translocations and deletion of chromosome 13 have been described
Differential diagnosis
Board review style question #1
Which 2 of the following clinical features are more commonly associated with POEMS syndrome than multiple myeloma?

  1. Bone pain
  2. Demonstration of clonal plasma cells
  3. Neuropathy
  4. Pleural effusion
  5. Renal failure
Board review style answer #1
C and D. Neuropathy and pleural effusion; volume overload and neuropathy are characteristic features of POEMS syndrome. POEMS syndrome is less frequently associated with bone pain and renal failure as compared to multiple myeloma. Both require demonstration of clonal plasma cells.

Comment Here

Reference: POEMS syndrome (osteosclerotic myeloma)

Plasma cell leukemia
Definition / general
  • Rare subtype of multiple myeloma characterized by the presence of clonal plasma cells in the peripheral blood and an aggressive clinical course
  • Current definition: ≥ 5% circulating plasma cells, detected by manual differential of peripheral blood (recently updated from prior cutoff of ≥ 20% or ≥ 2.0 x 109/L) (Blood Cancer J 2021;11:192)
Essential features
  • Detection of ≥ 5% circulating plasma cells by manual differential of peripheral blood, in a patient with newly diagnosed or relapsed / refractory multiple myeloma
  • Aggressive clinical course marked by high tumor burden and shortened overall survival
Terminology
  • Primary plasma cell leukemia (pPCL): ≥ 5% circulating plasma cells by manual differential of peripheral blood, in a patient with newly diagnosed multiple myeloma
  • Secondary plasma cell leukemia (sPCL): ≥ 5% circulating plasma cells by manual differential of peripheral blood, in a patient with relapsed / refractory multiple myeloma
ICD coding
  • ICD-10:
    • C90.0 - multiple myeloma
    • C90.1 - plasma cell leukemia
Epidemiology
Sites
  • Peripheral blood: by definition, plasma cells are ≥ 5% of circulating white blood cells
  • Bone marrow: variable, often extensive involvement by sheets of plasma cells with varying degrees of cytologic atypia
  • Extramedullary sites: liver, spleen, lymph nodes and CNS are common sites of extramedullary involvement (Curr Oncol Rep 2019;21:8, Leuk Lymphoma 2022;63:2955)
Pathophysiology
  • Pathophysiology is incompletely understood
  • Alterations in the bone marrow microenvironment and downregulated expression of cell adhesion molecules may decrease neoplastic cell dependence on the bone marrow and promote extramedullary involvement (Leuk Lymphoma 2017;58:1538, Leukemia 2013;27:780)
    • Loss of CD56 / NCAM expression disrupts adhesion of plasma cells to bone marrow stroma (Int J Hematol Oncol 2022;11:IJH39)
    • Decreased expression of chemokine receptors such as CXCR4 may disrupt plasma cell localization to bone marrow stroma
    • Expression of CD27 is associated with antiapoptotic processes via the nuclear factor Κβ pathway (which can be targeted with antiproteosome agents such as bortezomib) (Blood 2014;123:3770)
Etiology
  • Exact etiology of primary and secondary PCL remains under investigation
  • Altered expression of adhesion molecules and extracellular matrix play an incompletely understood role in pathogenesis (Blood Cancer J 2020;10:70, Leukemia 2020;34:1866)
Clinical features
Diagnosis
  • pPCL is defined by detection of ≥ 5% circulating plasma cells, identified by manual differential of the peripheral blood, in a patient with a new diagnosis of multiple myeloma (Blood Cancer J 2021;11:192)
  • sPCL is defined by detection of ≥ 5% circulating plasma cells, identified by manual differential of the peripheral blood, in a patient with relapsed or refractory multiple myeloma
  • Detection of ≥ 2% circulating plasma cells by flow cytometry in patients with multiple myeloma portends a pPCL-like clinicopathologic course but is not yet recognized as pPCL (J Clin Oncol 2023;41:1383)
    • When present, the percentage of circulating plasma cells and the method used for detection (e.g., manual differential, flow) should be included in the pathology report
Laboratory
  • Monoclonal gammopathy by serum or urine protein electrophoresis
    • IgG is most common, followed by IgA, IgD, light chain only and nonsecretory
  • Serum and urine immunofixation to characterize the monoclonal gammopathy
  • Elevated levels of serum or urine free light chains
  • Abnormal kappa:lambda free light chain ratio
  • Anemia and thrombocytopenia
  • Renal dysfunction and elevated creatinine
  • Elevated serum LDH and beta 2 microglobulin
  • Bone marrow with variable, often extensive involvement by plasma cells
  • ≥ 5% circulating plasma cells by morphologic examination of concurrent peripheral blood
  • Flow cytometry of peripheral blood and bone marrow shows aberrant plasma cells with monotypic, cytoplasmic light chain expression
    • Background B lymphocytes are polytypic and lack abnormalities
  • Hypodiploidy and high risk cytogenetic abnormalities, by fluorescence in situ hybridization
  • Mutation of TP53 is frequently observed in both pPCL and sPCL
  • Frequent detection of t(11;14) and cyclin D1 expression observed in both pPCL and sPCL
  • References: Leuk Lymphoma 2017;58:1538, Leuk Lymphoma 2022;63:2955, Leukemia 2013;27:780, J Clin Oncol 2023;41:1342
Prognostic factors
  • pPCL has an unfavorable prognosis with decreased overall survival as compared to multiple myeloma
  • Overall survival (using > 20% circulating plasma cells as cutoff): ~6.8 - 12.6 months (Leukemia 2013;27:780)
  • Overall and progression free survival is approximately doubled by use of lower cutoffs, novel therapeutic agents and autologous stem cell transplant
  • Presence of any amount of circulating plasma cells in patients with multiple myeloma negatively affects prognosis and survival rates progressively decline with increasing amounts of circulating cells (J Clin Oncol 2023;41:1342)
  • A recent study using flow cytometry based identification and quantitation of circulating plasma cells reported that achievement of minimal residual disease (MRD) negativity improved progression free survival in patients with multiple myeloma and circulating tumor cells (J Clin Oncol 2022;40:3120)
  • Majority of patients with pPCL have high (II or III) Revised International Staging System (R-ISS) scores due to elevated beta 2 microglobulin and LDH, indicating a high tumor burden and poor prognosis (Leuk Lymphoma 2022;63:2955, Curr Oncol Rep 2019;21:8, J Clin Oncol 2015;33:2863)
  • Prognostic influence of various cytogenetic abnormalities, molecular genetic abnormalities and prognosis remains under investigation
Case reports
  • 60 year old woman with secondary plasma cell leukemia, successfully treated with chimeric antigen receptor T cell therapy targeting B cell maturation antigen (BCMA; CD269) (Front Oncol 2022;12:901266)
  • 62 year old man presenting with hypercalcemia, tumor lysis syndrome and primary plasma cell leukemia (Clin Case Rep 2022;10:e05933)
Treatment
Microscopic (histologic) description
  • Peripheral blood
    • White blood cells may be decreased or markedly increased with ≥ 5.0% plasma cells detected by manual differential
    • Plasma cells are usually larger than background lymphocytes with eccentric nuclei, condensed chromatin with peripherally located chromatin clumps imparting a clock face-like appearance, abundant, basophilic cytoplasm and a perinuclear hof of clear cytoplasm
    • Anemia and thrombocytopenia are often present
    • Rouleaux formation is often present
  • Bone marrow aspirate
    • Aspirate contains increased plasma cells
    • Background hematopoietic elements may be markedly reduced
  • Bone marrow core biopsy and clot section
    • Marrow usually shows infiltration by sheets and clusters of plasma cells
    • CD138 immunohistochemistry is used to estimate the percentage of bone marrow involvement and to confirm diagnosis in cases with atypical morphology
  • Bone lesions and extramedullary lesions
    • Sheets and clusters of plasma cells (morphologic features of osseous and nonosseous plasmacytoma, respectively)
  • Reference: Semin Diagn Pathol 2003;20:211
Microscopic (histologic) images

Contributed by Jesse Manuel Jaso, M.D.

Sheets of plasma cells

Bone marrow core

Pleomorphic plasma cells

Plasma cells


CD138

CD56 negative plasma cells

Cyclin D1 expression in plasma cells

Cytology description
  • Medium sized cells with eccentric nuclei, condensed, clock face or spoke wheel-like chromatin, abundant basophilic cytoplasm and a perinuclear clearing
  • Cells may show variable amounts of morphologic atypia
  • Intranuclear (Dutcher bodies), cytoplasmic (Russell bodies) or extracellular immunoglobulin may be present
Peripheral smear description
  • Peripheral blood
    • White blood cells may be decreased or markedly increased with ≥ 5.0% plasma cells detected by manual differential
    • Plasma cells are usually larger than background lymphocytes with eccentric nuclei, condensed chromatin with peripherally located chromatin clumps imparting a clock face-like appearance, abundant, basophilic cytoplasm and a perinuclear hof of clear cytoplasm
  • Reference: Semin Diagn Pathol 2003;20:211
Peripheral smear images

Contributed by Jesse Manuel Jaso, M.D.

Circulating plasma cells

Positive stains
Negative stains
  • Neoplastic cells are characteristically negative for CD56 by flow cytometry and immunohistochemistry
  • Neoplastic cells are frequently negative for CD19, CD27, CD71, CD117 and HLA-DR by flow cytometry
Flow cytometry description
Flow cytometry images

Contributed by Franklin Fuda, D.O.
Increased forward and side scatter

Increased forward and side scatter

Heterogeneous CD45 expression

Heterogeneous CD45 expression

Negative for CD19 and CD10

Negative for CD19 and CD10

Negative for CD19 and CD5

Negative for CD19 and CD5

Bright expression of CD38

Bright expression of CD38


Negative for CD22

Negative for CD22

Lack of CD56 expression

Lack of CD56 expression

Lack of surface light chain expression

Lack of surface light chain expression

Cytoplasmic kappa expression

Cytoplasmic kappa expression

Molecular / cytogenetics description
Sample pathology report
  • Bone marrow, aspirate, clot, core biopsy and peripheral blood:
    • Primary plasma cell leukemia, 25% circulating plasma cells (see comment)
    • Plasma cell neoplasm, involving > 95% of bone marrow
    • Comment:
      • The peripheral blood shows leukocytosis with numerous circulating plasma cells (25%). The bone marrow is hypercellular (95 - 100%) and shows near total replacement by sheets of atypical, pleomorphic plasma cells (67% by manual differential; > 95% by immunohistochemistry for CD138).
      • Concurrent flow cytometry shows an aberrant plasma cell population with monotypic, cytoplasmic light chain expression.
      • Review of the electronic medical record indicates that the patient does not have a prior diagnosis of plasma cell neoplasm or other malignancy.
      • The combined findings are diagnostic of primary plasma cell leukemia. Clinical correlation and correlation with pending ancillary and staging studies is recommended.

  • Bone marrow, aspirate, clot, core biopsy and peripheral blood:
    • Secondary plasma cell leukemia, 35% circulating plasma cells (see comment)
    • Relapsed plasma cell myeloma, involving 80 - 90% of bone marrow
    • Comment:
      • The peripheral blood shows leukocytosis with numerous circulating plasma cells (35%). The bone marrow is hypercellular (95 - 100%) and shows near total replacement by sheets of atypical, pleomorphic plasma cells (45% by manual differential; 80 - 90% by immunohistochemistry for CD138).
      • Concurrent flow cytometry shows an aberrant plasma cell population with monotypic, cytoplasmic light chain expression.
      • The combined findings are diagnostic of relapse of the patient’s previously diagnosed plasma cell myeloma with development of secondary leukemic transformation (secondary plasma cell leukemia). Clinical correlation and correlation with pending ancillary and staging studies is recommended.
Differential diagnosis
  • Florid polytypic plasmacytosis:
    • In rare cases, patients with autoimmune disease, HIV and other infectious / inflammatory disorders may have a florid increase in reactive bone marrow plasma cells
    • Plasma cells will lack immunophenotypic aberrancies by flow cytometry or immunohistochemistry
    • Plasma cells will show polytypic, cytoplasmic kappa and lambda light chain expression by flow cytometry or immunohistochemistry
  • B cell lymphoma with plasmacytic differentiation:
    • B cell non-Hodgkin lymphoma shows a variable propensity for plasmacytic differentiation (e.g., marginal zone lymphoma and lymphoplasmacytic lymphoma)
      • Flow cytometry will demonstrate the presence of 2 monoclonal cell populations: an aberrant B cell and plasma cell population with monotypic expression of the same light chain (assessed on the cell surface for B cells and within cytoplasm / intracellularly for plasma cells)
      • Neoplastic B cells will express CD19, CD20, CD22 and monotypic, surface light chain expression
      • Neoplastic plasma cells will show monotypic, cytoplasmic expression of the same light chain as the B cells
      • Neoplastic plasma cells often express CD19, CD20 (partial) and CD45 and lack expression of CD56
      • Neoplastic plasmacytic cells often show monotypic, surface light chain expression similar to the neoplastic B lymphocytes
Board review style question #1


A 45 year old patient presents with weight loss, leukocytosis, anemia and thrombocytopenia. Serum protein electrophoresis shows a monoclonal IgG kappa protein. Bone marrow examination shows ~90% involvement by sheets of atypical plasma cells with monotypic, intracytoplasmic kappa light chain expression. Review of a concurrent peripheral blood smear shows ~6% atypical cells (see images above). Which of the following statements regarding these cells is most correct?

  1. Neoplastic cells have monotypic, surface light chain expression by flow cytometry
  2. Neoplastic cells typically lack expression of CD38 by flow cytometry
  3. Neoplastic cells typically lack expression of CD56 by flow cytometry
  4. Neoplastic cells with bright expression of CD19 by flow cytometry
  5. Neoplastic cells with bright expression of CD45 by flow cytometry
Board review style answer #1
C. Neoplastic cells typically lack expression of CD56 by flow cytometry. The clinical and pathologic features are consistent with a diagnosis of primary plasma cell leukemia. Answer D is incorrect because by flow cytometry, the neoplastic plasma cells are characteristically negative for expression of CD19, CD56 and surface immunoglobulin expression. Answers A, B and E are incorrect because the neoplastic plasma cells express monotypic, cytoplasmic light chain and they are positive for CD38 and CD138 with variable expression of CD20, CD23 and CD45.

Comment Here

Reference: Plasma cell leukemia
Board review style question #2
A 56 year old patient is diagnosed with secondary plasma cell leukemia. Which of the following laboratory values would provide the best assessment of tumor burden and disease prognosis?

  1. Serum beta 2 microglobulin
  2. Serum calcium
  3. Serum creatinine
  4. Urine immunofixation electrophoresis
  5. White blood cell count
Board review style answer #2
A. Serum beta 2 microglobulin. Serum beta 2 microglobulin, LDH and albumin level have been shown to correlate with tumor burden and disease prognosis in multiple myeloma and plasma cell leukemia and are major components of the Revised International Staging System (R-ISS) for multiple myeloma. Answers B - E are incorrect because serum creatinine, calcium, white blood cell count and immunoglobulin heavy or light chain isotype assist with diagnosis but do not correlate with tumor burden.

Comment Here

Reference: Plasma cell leukemia

Plasma cell myeloma (multiple myeloma)
Definition / general
  • Bone marrow based, multifocal plasma cell neoplasm usually associated with a monoclonal immunoglobulin (M protein) in serum or urine and evidence of organ damage related to the plasma cell neoplasm (J Natl Compr Canc Netw 2019;17:1154)
Essential features
  • Diagnosis requires synthesis of clinical, laboratory, radiologic and histologic findings
  • Assessment of serum or urine M protein, clonal plasma cells in bone marrow and presence of end organ damage related to the plasma cell neoplasm critical for diagnosis
Terminology
  • Multiple myeloma
  • Plasma cell myeloma
  • Myeloma
  • Medullary plasmacytoma
  • Myelomatosis
  • Kahler disease (no longer used) (Recent Results Cancer Res 2011;183:3)
  • Smoldering (asymptomatic) plasma cell myeloma
  • Nonsecretory myeloma
  • Plasma cell leukemia
ICD coding
  • ICD-O:
  • ICD-10:
    • C90.0 - multiple myeloma
    • C90.1 - plasma cell leukemia
    • C90.2 - extramedullary plasmacytoma
    • C90.3 - solitary plasmacytoma
Epidemiology
Pathophysiology
  • Bone marrow site of origin for nearly all cases
  • Interactions between bone marrow stroma and neoplastic plasma cells directly influences disease with a potential key role of IL6 to support survival and expansion of myeloma cells (Leukemia 2014;28:1647, Cancer 2003;97:2440, Cancers (Basel) 2021;13:216)
  • Role of IL6 and other cytokines in promoting osteoclastic activity and lytic bone lesions
  • Multiple myeloma cells suppress the differentiation and proliferation of osteoblasts while inducing osteoclast differentiation and hyperfunction (Cancers (Basel) 2021;13:216)
Etiology
Clinical features
  • Bone disease is the most frequent disease defining clinical feature (Am Soc Clin Oncol Educ Book 2018;38:638)
  • Often presents with bone pain due to lytic bone lesions (thoracic vertebrae most common, also ribs, skull, shoulders, pelvis and long bones); spinal cord compression or peripheral neuropathy are less common presenting symptoms
  • Renal failure / elevated creatinine / hyperuricemia (monoclonal light chain proteinuria results in renal tubular damage); hypercalcemia; hypoalbuminemia
  • Recurrent infections due to impaired humoral immunity (immunoglobulin production, often < 50% normal)
  • Anemia (bone marrow infiltration often in areas of most active hematopoiesis and renal failure causing loss of erythropoietin)
  • Extramedullary involvement generally associated with advanced disease
  • Historical fact: Bence Jones proteins were the first tumor marker (Clin Kidney J 2012;5:478)
Clinical variants
  • Smoldering (asymptomatic):
    • More likely to progress to symptomatic myeloma than monoclonal gammopathy of uncertain significance (MGUS)
    • Both show gammopathy without myeloma defining events (hypercalcemia, anemia, bone lesions and renal insufficiency)
    • Risk of progression 10% per year for the first 5 years
    • Lower risk if no progression in the first 5 years after diagnosis (N Engl J Med 2007;356:2582)
  • Nonsecretory myeloma (~1% of cases):
    • Serum protein electrophoresis (SPE) / immunofixation electrophoresis (IFE) negative, 85% with impaired secretion and have cytoplasmic immunoglobulin (Ig) by IHC (Am J Clin Pathol 2011;136:168)
    • 15% nonproducers, serum free light chain may still be detected
    • Lower incidence of renal insufficiency, hypercalcemia and depression of normal IgG
    • Must be distinguished from the rare IgD and IgE myelomas
  • Plasma cell leukemia:
    • Typically aggressive with short survival < 1 year
    • > 2 x 109/L or 20% of the leukocyte count on differential are monoclonal plasma cells
    • Primary plasma cell leukemia (0.6% of myeloma) or develop as a late stage transformation (secondary) (Curr Oncol Rep 2019;21:8)
    • Usually lack CD56 (80% of PCLs), more frequent high risk genetic findings
    • Bone pain and osteolytic lesions less common
    • Typically associated with extramedullary lesions (e.g., body cavity effusions, lymphadenopathy and organomegaly)
Diagnosis
  • Division into these categories will guide plan for therapy (Am Soc Clin Oncol Educ Book 2016;35:e418):
    • Multiple (symptomatic) myeloma (J Natl Compr Canc Netw 2019;17:1154):
      • Clonal bone marrow plasma cells ≥ 10% or biopsy proven bony or extramedullary plasmacytoma
      • And ≥ 1 of the following myeloma defining events:
        • Calcium > 1 mg/dL above upper limit of normal or > 11 mg/dL
        • Renal insufficiency*; creatinine > 2 mg/dL or creatinine clearance < 40 mL/min (preferred)
        • Hemoglobin < 10 g/dL or > 2 g/dL below the lower limit of normal
        • 1 or more osteolytic bone lesions on skeletal radiography, CT or FDG PET / CT
        • Clonal bone marrow plasma cells ≥ 60% of bone marrow cellularity
        • Involved / uninvolved serum free light chain (FLC) ratio ≥ 100 and involved free light chain concentration 10 mg/dL or higher
        • > 1 focal lesion on MRI studies ≥ 5 mm
    • These include traditional CRAB features associated with end organ damage (hypercalcemia, renal failure, anemia and osteolytic bone lesions) and biomarkers associated with ~80% risk of progression to end organ damage (≥ 60% clonal plasma cells in bone marrow, serum FLC ≥ 100 with FLC level ≥ 10 mg/dL or > 1 focal lesion by MRI) (Am Soc Clin Oncol Educ Book 2016;35:e418)
    • *Only suspected or proven light chain cast nephropathy is considered a multiple myeloma defining event, consider renal biopsy to clarify if FLC < 500 mg/L (Am Soc Clin Oncol Educ Book 2016;35:e418)
    • Smoldering (asymptomatic) myeloma (J Natl Compr Canc Netw 2019;17:1154):
      • M protein in serum (IgG or IgA) at ≥ 3 g/dL
      • Or Bence Jones protein ≥ 500 mg/24 h urine
      • Or 10 - 59% clonal plasma cells in bone marrow
      • And no related tissue damage / myeloma defining event or amyloidosis; if bone survey negative, bone disease should be assessed with whole body MRI, FDG PET / CT or low dose CT scan
Laboratory
  • 97% have an M protein in serum or urine, 3% are nonsecretory
  • IgG (50%), IgA (20%), light chain (20%), others < 10% (IgD, IgE, IgM and biclonal)
  • SPEP: serum proteins normally separate into 5 major fractions based on electric charge and size (Am Fam Physician 2005;71:105):
    • Albumin
    • Alpha 1 globulins
    • Alpha 2 globulins
    • Beta 1 and beta 2 globulins
    • Gamma globulins
  • Gamma globulins include polyclonal antibodies and light chains, with a normal gamma zone appearing as a symmetrical smear; myeloma may appear as a spike in this region
  • Urine protein electrophoresis (UPEP): monoclonal light chains in urine = Bence Jones protein
  • IFE: used to characterize the M spike, by reacting with specific antisera to heavy chains IgG, IgA, IgM, IgD, IgE and kappa and lambda light chains
  • Serum free light chain assay (SFLCA) (Freelite): more sensitive for monitoring light chain disease and nonsecretory myeloma (Blood Cancer J 2020;10:2)
  • Mass spectrometry may have equivalent performance to IFE and better differentiate M protein from therapeutic antibodies (Blood Cancer J 2021;11:24)
Laboratory images

Images hosted on other servers:

Serum protein immunofixation shows IgD lambda myeloma

IgD lambda myeloma

Another IgD myeloma

Radiology description
  • Evidence of 1 or more sites of osteolytic bone destruction (at least 5 mm in size)
  • Advanced methods include low dose whole body CT, MRI and (18F) fluorodeoxyglucose PET (FDG PET), and FDG PET with PET / CT (Am Soc Clin Oncol Educ Book 2016;35:e418)
  • Increased uptake on PET / CT alone is not adequate without evidence of underlying osteolytic bone destruction; bone biopsy recommended if any doubt
Radiology images

Contributed by Mark R. Wick, M.D. and AFIP images

Skull Xray

Tibia Xray

Solitary plasma cell myeloma


Plasma cell myeloma



Images hosted on other servers:

Prominent skull defect

Prognostic factors
  • Usually incurable with median survival of ~5.5 years with a 5 year survival rate of 54% (ASCO: Multiple Myeloma - Statistics [Accessed 26 May 2022])
  • Higher risk:
    • Elevated beta 2 microglobulin, lactate dehydrogenase, C reactive protein, serum soluble receptor for IL6 and plasma cell proliferation or bone marrow infiltration
    • Reduced polyclonal (uninvolved) serum immunoglobulins
    • Plasmablastic morphology
    • Abnormalities by conventional cytogenetics
  • Active myeloma can be staged using the International Staging System (ISS) or revised International Staging System (R-ISS) (J Natl Compr Canc Netw 2019;17:1154)
  • Revised International Staging System:
    • Stage I: serum beta 2 microglobulin < 3.5 mg/L, serum albumin ≥ 3.5 g/dL, standard risk chromosomal abnormalities by FISH [absence of del(17p), t(4:14) or t(14;16)] and normal serum LDH
    • Stage II: not R-ISS stage I or III
    • Stage III: serum beta 2 microglobulin ≥ 5.5 mg/L or either high risk chromosomal abnormalities [del(17p), t(4:14) or t(14;16)] and serum LDH > upper limit of normal
  • Additional prognostic chromosomal abnormalities:
    • Worse: t(4;14), MAF translocations t(14;16) and t(14;20), del(17p), del 13, aneuploidy, hypodiploidy
    • Better: hyperdiploidy, t(11;14), t(6;14), cyclin D1 or D3 positive
  • References: analysis of prognostic value of most frequent chromosomal changes in a large series of patients with newly diagnosed symptomatic myeloma (Blood 2007;109:3489)
  • Model for identifying patients with increased risk of progression of smoldering myeloma (Mayo 2018 criteria or 20/2/20 criteria) with 3 independent risk factors:
    • Serum monoclonal protein > 2 g/dL
    • Involved to uninvolved serum free light chain ratio > 20
    • Bone marrow plasma cells > 20%
    • Low (0 factor), intermediate (1 factor) and high risk (2 - 3 factors) shown to have 2 year rates of progression to multiple myeloma of 5%, 17% and 46%, respectively (Am Soc Clin Oncol Educ Book 2020;40:1)
Case reports
Treatment
Minimal residual disease (MRD) testing
  • Sensitive measure of response to therapy to guide treatment decisions
  • Flow cytometry (Blood 2013;122:1088)
  • Molecular methods monitoring immunoglobulin rearrangement by NGS (Blood Adv 2020;4:4573)
  • Clonotypic peptides by mass spectrometry by liquid chromatography with tandem mass spectrometry (LC-MS / MS) (Clin Chem 2016;62:243)
  • MRD negativity associated with better progression free survival (Blood Adv 2020;4:4573)
Gross description
  • Bone defects are filled with a soft, gelatinous "fish flesh" hemorrhagic tissue
Gross images

Images hosted on other servers:

Vertebrae with myeloma lesions

Skull lesions

Frozen section description
  • Osseous or extraosseous plasmacytomas, particularly if the patient is not known to have a history of plasma cell myeloma, may be sent for frozen section evaluation (Borczuk: Frozen Section Pathology, 1st Edition, 2021)
  • Sites can include: mucosa of the upper respiratory tract, lymph nodes, thyroid, testes, breast, salivary gland and CNS (Int J Otolaryngol 2010;2010:302656)
  • Morphology may range from monotonous plasma cells to more irregular, multinucleated or pleomorphic forms in more advanced myeloma
Frozen section images

Contributed by Genevieve M. Crane, M.D., Ph.D.

Touch preparation plasmacytoma

Frozen section plasmacytoma

Microscopic (histologic) description
  • Core biopsy (Am J Clin Path 1987;87:342):
    • Interstitial clusters, nodules or sheets of plasma cells
    • Areas of bone marrow may be spared with preserved hematopoiesis, other cases may have diffuse involvement and markedly suppressed hematopoiesis
    • Prominent osteoclastic activity may be seen
    • IHC to quantify plasma cells (CD138), stains for Ig kappa and lambda to establish clonality
Microscopic (histologic) images

Contributed by Genevieve M. Crane, M.D., Ph.D. and Tapan Bhavsar, M.D., Ph.D.

Amyloid deposition (Congo red stain)

Amyloid deposition
(Congo red stain
with birefringence)

Focal sheets of plasma cells

Extensive amyloid, bone marrow

Extensive bone marrow involvement by myeloma


Trephine biopsy

Trephine biopsy

Myeloma with plasmablastic transformation

CD138


In situ hybridization for kappa

In situ hybridization for lambda

Myeloma involving bone marrow


Bone marrow touch preparation



Images hosted on other servers:

Bone marrow aspirate has plasmacytoid cells

Virtual slides

Images hosted on other servers:

Plasmacytoma involving thyroid

Myeloma with pleomorphic features involving lymph node

Plasmacytoma

Myeloma with
pleomorphic features
diffusely involving
bone marrow

Cytology description
  • Cytology can assess plasma cell morphology (e.g., mature, immature, plasmablastic) but number of plasma cells present may vary substantially from the core biopsy
  • Mature plasma cells: oval with abundant basophilic cytoplasm, perinuclear hof, round eccentric nuclei, clock face chromatin and indiscernible nucleoli
    • Immature plasma cells: higher nuclear to cytoplasmic ratio, more abundant cytoplasm and hof region compared to plasmablastic, more dispersed chromatin, often prominent nucleoli
  • Plasmablastic: less abundant cytoplasm with little or no hof region, fine reticular chromatin, large nucleus (> 10 microns) or large nucleolus (> 2 microns) (Blood 1998;91:2501)
  • Pleomorphic: multinucleated, polylobated
    • Rare cases may have small, lymphoid appearing plasma cells or plasma cells with marked nuclear lobation
  • Immature or pleomorphic features are rare in reactive plasma cell proliferations
  • Morphologic features:
    • Mott cells / morula cells: multiple grape-like cytoplasmic inclusions comprised of crystalized immunoglobulin
    • Russell bodies: hyaline intracytoplasmic inclusions
    • Flame cells: vermillion staining glycogen rich IgA in cytoplasmic projections (American Society of Hematology: Flame Cells in Multiple Myeloma [Accessed 26 May 2022])
    • Pseudo-Gaucher cells / thesaurocytes: overstuffed fibrils (J Clin Pathol 1976;29:916)
    • Cytoplasmic crystals: occasional in myeloma, common in adult Fanconi syndrome (Am J Clin Pathol 1983;80:224)
    • Dutcher body: pale staining immunoglobulin filled cytoplasm invaginating into the nucleus and appearing as an intranuclear inclusion, single and usually large, more common in IgA myeloma
Cytology images

Contributed by Genevieve M. Crane, M.D., Ph.D.

Extensive amyloid, aspirate

Peripheral smear description
  • Rouleaux formation: erythrocytes resemble stacked coins; related to quantity and type of M protein, not specific and may be caused by alterations in other plasma proteins (Biophys J 2000;78:2470, American Society of Hematology: Rouleaux Formation [Accessed 26 May 2022])
  • Leukoerythroblastic reaction can occur with extensive marrow involvement
  • Circulating plasma cells can be seen in ~15% of cases, usually small numbers not meeting criteria for plasma cell leukemia (> 2 x 109/L or 20% of the leukocyte count)
Peripheral smear images

Contributed by Genevieve M. Crane, M.D., Ph.D.

Peripheral blood rouleaux

Rouleaux formation



Images hosted on other servers:

Peripheral blood shows plasma cells (right: with blastic features)

Positive stains
Negative stains
Flow cytometry description
Flow cytometry images

Contributed by Genevieve M. Crane, M.D., Ph.D.

Dim kappa

Strong cytoplasmic expression

Neoplastic plasma cells

Myeloma with t(11;14) and partial CD19



Images hosted on other servers:

Prominent monotypic
pattern (lambda,
with minimal kappa)

Electron microscopy description
Molecular / cytogenetics description
  • Immunoglobulin heavy and light chain genes are clonally rearranged with a high load of IGHV gene somatic hypermutation; abnormalities in IGH rearrangement may be seen with light chain only disease (Blood 2004;103:3869)
  • Cytogenetics: abnormalities by conventional cytogenetics seen in 33% of cases but identified by FISH in > 90%, including trisomies, deletions and translocations (Blood Cancer J 2015;5:e365)
  • FISH studies often performed by enriching the plasma cell fraction using magnetic cell sorting (MACS) and fluorescence activated cell sorting (FACS) to increase detection of cytogenetic abnormalities in interphase cells (Am J Clin Pathol 2011;136:712)
  • IGH (14q32) translocations in 40% of tumors, recurrent oncogenes:
    • t(4;14) FGFR3 / MMSET (NSD2) (4p16.3); 15%
    • t(6;14) cyclin D3 (6p21); 4%
    • t(11;14) cyclin D1 (11q13); 20%
    • t(14;16) MAF (16q23); 4%
    • t(14;20) MAFB (20q11); 1%
    • MAFA (8q24)
    • Cyclin D2 (12p13)
  • Tumors without 1 of the frequent IGH are more often hyperdiploid with gains of odd numbered chromosomes (3, 5, 7, 9, 11, 15, 19, 21); 50%
  • Monosomy or partial deletion of chromosome 13 (13q14) found in nearly 50% by FISH
  • MYC rearrangements in nearly half of tumors
  • Activating mutations of KRAS, NRAS or BRAF in 40% of tumors
  • Other: TP53 deletion or mutation, gain of 1q, loss of 1p, NF kappa B pathway activation, inactivation of CDKN2C, RB1, FAM46C, DIS3 and DNA methylation changes
Videos

Video on myeloma MRD

Plasma cell neoplasms

Podcast on SPEP

Introduction to bone marrow interpretation

Sample pathology report
  • Bone marrow aspirate smears, touch imprints, core biopsy and clot section with peripheral smear:
    • Plasma cell myeloma (see comment)
    • Normocytic anemia, rouleaux formation
    • Comment: The patient is a 65 year old man who recently presented with back pain, anemia and hypercalcemia. Xrays demonstrated evidence of lytic bone lesions and serum protein analysis demonstrated an IgG kappa M protein. The bone marrow shows sheets of atypical plasma cells in a hypercellular bone marrow overall comprising 80% of the intertrabecular space. Flow cytometry as reported separately, demonstrates a CD56+, CD19- kappa restricted plasma cell population. Please correlate with forthcoming cytogenetics and myeloma FISH panel, which will be reported as an addendum.
Differential diagnosis
Board review style question #1

Which of the following findings would be compatible with a diagnosis of smoldering myeloma?

  1. 70% plasma cells in bone marrow
  2. Free light chain ratio is 0.01
  3. IgG lambda M protein in serum of 3.5 g/dL
  4. 1 osteolytic lesion on skeletal radiography
  5. Serum calcium of 11.2 mg/dL
Board review style answer #1
C. IgG lambda M protein in serum of 3.5 g/dL. In smoldering myeloma, end stage organ damage should be absent (no CRAB features), so the elevated serum calcium excludes answer E and the osteolytic lesion excludes option D. Plasma cells in excess of 60% or light chain ratio (involved to uninvolved) of ≥ 100 would qualify as active plasma cell myeloma excluding answers A and B. In the diagnosis of smoldering myeloma the patient may have M protein in serum (IgG or IgA) at ≥ 3 g/dL or Bence Jones protein ≥ 500 mg/24 h urine or 10 - 59% clonal plasma cells in bone marrow.

Comment Here

Reference: Plasma cell myeloma (multiple myeloma)
Board review style question #2

Clonal plasma cells in the setting of multiple myeloma are typically negative for CD19. What cytogenetic abnormality is typically associated with CD19 or CD20 positive plasma cells in the setting of a plasma cell neoplasm?

  1. Deletion 13
  2. Hypodiploidy
  3. t(4:14)
  4. t(11;14)
  5. t(14:16)
Board review style answer #2
D. t(11:14). The presence of CD19 positive clonal plasma cells may favor the presence of a small B cell lymphoma with plasmacytic differentiation as opposed to a plasma cell neoplasm in many cases. These entities are important to differentiate as the treatments are generally distinct. However, in the setting of a t(11;14) translocation, the plasma cells are cyclin D1+, typically retain CD19 expression and may express additional B cell markers including CD20. This can create potential diagnostic pitfalls (Leuk Res 2013;37:1251).

Comment Here

Reference: Plasma cell myeloma (multiple myeloma)
Board review style question #3


Which of the following findings would most strongly favor the presence of a neoplastic plasma cell process rather than a reactive plasma cell proliferation?

  1. Interstitial plasma cells comprising 10% of cellularity
  2. Plasma cells with prominent clock face chromatin
  3. Russell bodies
  4. Scattered immature plasma cells
  5. Scattered Mott cells with grape-like inclusions
Board review style answer #3
D. Scattered immature plasma cells are more specific to a neoplastic process compared to binucleation, Russell bodies or mild plasmacytosis.

Comment Here

Reference: Plasma cell myeloma (multiple myeloma)

Plasmacytoma
Definition / general
  • Solitary lesion of clonal plasma cells that are cytologically, immunophenotypically and genetically similar to plasma cell myeloma
  • Solitary plasmacytoma of bone: localized bone tumor consisting of monoclonal plasma cells with no other radiographic lesions and no evidence of other bone marrow involvement
  • Extraosseous (extramedullary) plasmacytoma: localized tumor of plasma cells (not in bone)
Essential features
  • Monoclonal plasma cells, no associated clonal B cell population, isolated lesion without evidence of additional bone marrow plasmacytosis
Terminology
  • Solitary plasmacytoma of bone
  • Extraosseous (extramedullary) plasmacytoma
ICD coding
  • ICD-10:
    • C90.20 - extramedullary plasmacytoma not having achieved remission
    • C90.30 - solitary plasmacytoma not having achieved remission
Epidemiology
  • Bone and extramedullary tumors each comprise 3 - 5% of plasma cell neoplasms
  • Both are more common in men (65%), median age 55 years
Sites
  • Extraosseous: 80% in upper respiratory tract (15% spread to cervical lymph nodes)
  • Other: GI, lymph nodes (Am J Clin Pathol 2001;115:119), bladder, CNS, breast, thyroid, testis, parotid, skin
  • Bone:
    • Areas of active hematopoiesis
    • In order of decreasing frequency: vertebrae, ribs, skull, pelvis, femur, clavicle, scapula, rare in distal long bones
Clinical features
Bone:
  • Most present with bone pain
  • Vertebral lesions may cause cord compression
  • Palpable mass due to soft tissue extension
  • Up to 2/3 progress to myeloma or additional plasmacytomas; 1/3 remain disease free for > 10 years following local (radiation) control
  • 5% may have multiple or recurrent plasmacytomas but no evidence of myeloma

Extraosseous (clinical features depend on site):
  • Rhinorrhea, epistaxis, nasal obstruction
  • Approximately 15% progress to myeloma, 70% are disease free at 10 years
  • 25% have local recurrence, may spread to regional lymph nodes or metastasize to distant sites
Diagnosis
  • Monoclonal plasma cells, less frequently plasmablastic or anaplastic morphology
  • Histologic findings are combined with laboratory and radiographic findings (below) showing absence of additional disease
Laboratory
  • M protein in serum or urine (absent or lower than in myeloma):
    • Bone: approximately 50%, usually IgG
    • Extraosseous: 20%, often small, IgA
  • No CRAB (hypercalcemia, renal failure, anemia, additional bone lesions)
  • Unlike myeloma, normal levels of uninvolved immunoglobulins
  • Monitor free light chain to measure progression
Radiology description
  • MRI may be preferred method to exclude additional bone lesions
  • Solitary bone lesions are usually purely lytic with a narrow zone of transition to normal bone
  • Abnormalities may persist after successful treatment
Prognostic factors
Bone - poor prognostic features (in some series):
  • Older patients
  • Plasmacytoma > 5 cm
  • Persistence of M protein following radiotherapy
  • Low polyclonal immunoglobulins (also raises concern for myeloma)
  • Osteopenia
Case reports
Treatment
  • Local radiation therapy
Microscopic (histologic) description
  • Similar to myeloma, may contain mature, immature, plasmablastic or anaplastic plasma cells
  • Amyloid deposits may appear in extraosseous tumors as pink amorphous material with scattered multinucleated giant cells
Microscopic (histologic) images

Contributed by Genevieve M. Crane, M.D., Ph.D. and Kelly Magliocca, D.D.S., M.P.H.

Plasmacytoma

Plasmacytoma of bone with osteolytic destruction

CD138 positive and MUM1 positive

CD79 variable and CD20 negative


Core biopsy section

Core biopsy, kappa and lambda

Solitary plasmacytoma of bone

Solitary plasmacytoma of bone

Virtual slides

Contributed by Genevieve M. Crane, M.D., Ph.D.

EBV+ tumor

Cytology description
  • Similar to myeloma
  • Mature plasma cells: oval with abundant basophilic cytoplasm, perinuclear hof, round eccentric nuclei, "clock face" chromatin and indiscernible nucleoli
  • Immature plasma cells: higher nuclear / cytoplasmic ratio, more abundant cytoplasm and hof region compared to plasmablastic, more dispersed chromatin, often prominent nucleoli
  • Plasmablastic: less abundant cytoplasm with little or no hof region, fine reticular chromatin, large nucleus ( > 10 microns) or large nucleolus ( > 2 microns) (Blood 1998;91:2501)
  • Pleomorphic: multinucleated, polylobated
    • Rare cases may have small, lymphoid appearing plasma cells or plasma cells with marked nuclear lobation
  • Morphologic features:
    • Mott cells / morula cells: multiple grape-like cytoplasmic inclusions comprised of crystalized Ig
    • Russell bodies: hyaline intracytoplasmic and intranuclear inclusions
    • Flame cells: vermillion staining glycogen rich IgA in cytoplasmic projections
    • Gaucher-like cells / thesaurocytes: overstuffed fibrils
    • Cytoplasmic crystals: occasional in myeloma, common in adult Fanconi syndrome
    • Dutcher bodies: pale staining nuclear inclusions, single and usually large, more common in IgA myeloma
Cytology images

Contributed by Genevieve M. Crane, M.D., Ph.D.

Plasmacytoma touch preparation

Positive stains
Negative stains
Flow cytometry description
Molecular / cytogenetics description
Differential diagnosis
Board review style question #1
What is the best way to distinguish plasma cell myeloma from solitary plasmacytoma?

  1. CD19 expression
  2. Cytogenetics demonstrating a t(11;14) translocation
  3. Plasmablastic features
  4. Radiographs
Board review style answer #1
D. Radiographs

Comment Here

Reference: Plasmacytoma
Board review style question #2
When considering the diagnosis of a solitary plasmactyoma in a cervical lymph node, which factor(s) are most important to consider?

  1. Lymph node involvement by myeloma
  2. Nodal marginal zone lymphoma with extensive plasmacytic differentiation
  3. Potential spread from the upper respiratory tract
  4. All of the above
Board review style answer #2
D. All of the above

Comment Here

Reference: Plasmacytoma

Polycythemia vera
Definition / general
  • BCR::ABL1 negative myeloproliferative neoplasm (MPN) characterized by peripheral erythrocytosis frequently accompanied by leukocytosis or thrombocytosis
Essential features
  • Relatively common MPN characterized by increased red blood cell (RBC) production with elevated red blood cell count and hemoglobin level, which is not proportional to erythropoietin (EPO) production (usually has markedly decreased EPO level)
  • Diagnosis requires all 3 major criteria or 2 major plus the minor criterion (see 2016 and 2022 WHO classification in Diagnosis below)
  • JAK2 V617F (exon 14, valine to phenylalanine) and JAK2 exon 12 mutations are identified in > 95% and ~5% of patients, respectively (Blood 2006;108:1865, Hum Pathol 2006;37:1458, N Engl J Med 2007;356:459)
  • Occurs in 3 phases
    • Prodromal / prepolycythemic
    • Overt polycythemic
    • Postpolycythemic myelofibrosis / spent
  • < 20% undergo blastic transformation within 15 years
Terminology
  • Polycythemia vera (PV)
  • Not accepted: Vaquez disease, Osler-Vaquez disease, polycythemia rubra vera, primary polycythemia, polycythemia
ICD coding
  • ICD-O: 9950/3 - polycythemia vera
  • ICD-11: 2A20.4 - polycythemia vera
Epidemiology
Sites
  • Peripheral blood and bone marrow are major sites of involvement
  • Spleen and liver are common sites of extramedullary hematopoiesis in later stages (Blood 2016;128:5491)
Pathophysiology
  • Clonal hematopoietic stem cell disorder (Blood 2003;102:3793)
  • Point mutation in Janus kinase 2 (JAK2) gene at 9p24 results in constitutional activation of transcription factors of the STAT family, which promote growth factor independent proliferation and survival (Blood 2006;108:1865)
  • Mechanisms of thrombosis and bleeding are due to increased circulating red blood cells and elevated platelets (StatPearls: Polycythemia Vera [Accessed 12 September 2023])
  • 3 phases
    • Prodromal / prepolycythemic: characterized by borderline to mild erythrocytosis
    • Overt polycythemic: associated with significantly increased red cell mass
    • Postpolycythemic myelofibrosis / spent: cytopenias, including anemia, are associated with ineffective hematopoiesis, bone marrow fibrosis, extramedullary hematopoiesis and hypersplenism
Etiology
  • Underlying cause is unknown
  • Presence of JAK2 V617F mutation is detected in over 95% of patients with PV (Blood 2008;111:1686)
  • Abnormal karyotype in hematopoietic progenitor cells has been found in ~34% of patients with PV (J Clin Oncol 2011;29:761)
Clinical features
Diagnosis
  • Diagnosis criteria (WHO 5th edition) for polycythemia vera requires meeting either all 3 major criteria or the first 2 major criteria and the minor criterion
    • Major criteria
      • Hemoglobin > 16.5 g/dL in men, > 16.0 g/dL in women; hematocrit (HCT) > 49% in men, > 48% in women
      • Bone marrow biopsy showing hypercellularity for age with trilineage growth (panmyelosis), including prominent erythroid, granulocytic and megakaryocytic proliferation with pleomorphic, mature megakaryocytes (differences in size)
      • Presence of JAK2 V617F or JAK2 exon 12 mutation
    • Minor criterion
      • Subnormal serum erythropoietin level (normal range of EPO in adults: 4.1 - 19.5 mU/mL)
    • Major criterion 2 may not be required in cases with sustained absolute erythrocytosis
      • Sustained absolute erythrocytosis is defined as hemoglobin levels greater than 18.5 g/dL in men or 16.5 g/dL in women or hematocrit greater than 55.5% in men and 49.5% in women
      • Major criterion 3 (presence of JAK2 V617F or JAK2 exon 12 mutation) plus the minor criterion must also be present
    • Accelerated phase: presence of 10 - 19% blasts in bone marrow or peripheral blood (Best Pract Res Clin Haematol 2022;35:101379)
    • Blast phase: presence of ≥ 20% blasts in bone marrow or peripheral blood (Best Pract Res Clin Haematol 2022;35:101379)

  • Diagnostic criteria for postpolycythemic myelofibrosis (post-PV MF)
    • Criteria based on International Working Group for Myeloproliferative Neoplasms Research and Treatment (IWG-MRT) and WHO (Blood 2016;127:2391, Am J Hematol 2017;92:94)
      • Required criteria
        • Documentation of a previous diagnosis of PV according to 2016 or 2022 WHO criteria
        • Presence of grade 2 or 3 bone marrow fibrosis, based on either 2016 or 2022 WHO classification or the European consensus on grading of bone marrow fibrosis (Haematologica 2005;90:1128)
      • Additional criteria (2 required)
        • Anemia
        • Leukoerythroblastosis
        • Splenomegaly - either development of newly palpable splenomegaly or increased splenomegaly of > 5 cm from baseline
        • Development of any 2 (or all 3) of the following constitutional symptoms
          • Night sweats
          • > 10% weight loss within past 6 months
          • Unexplained fevers greater than 37.5 °C
Laboratory
  • Laboratory studies usually demonstrate erythrocytosis
  • May also present with asymptomatic leukocytosis or thrombocytosis
  • Bone marrow biopsy may show hypercellularity and panmyelosis
  • Iron deficiency can be seen in many cases (Hematology 2023;28:2204621)
  • JAK2 V617F mutation is detectable in over 95% of patients with PV (Blood 2008;111:1686)
  • JAK2 exon 12 mutations may be seen in 2 - 5% of patients who are negative for the JAK2 V617F mutation (Ann Hematol 2020;99:983)
Prognostic factors
  • Leukocytosis may predict inferior survival and leukemic transformation
  • Risk of PV progressing to postpolycythemic myelofibrosis is 4.9% at 10 years and 9.4% at 15 years (Hematol J 2003;4:198)
    • 15% develop spent phase with marrow fibrosis and marked cytopenias after 10 years; death in months if no treatment
  • JAK2 V617F mutant allele burden of > 50% has been associated with fibrotic transformation (Am J Hematol 2017;92:94)
  • Leukemic transformation rates at 20 years are estimated at < 10% for PV (Am J Hematol 2017;92:94)
    • Risk factors for transformation include advanced age, leukocytosis and abnormal karyotype
    • Leukemic transformation has been associated with treatment exposure to pipobroman or P32 / chlorambucil (Leukemia 2013;27:1874)
  • Recent study showed older age, leukocytosis, venous thrombosis and abnormal karyotype are associated with adverse clinical outcomes (Leukemia 2013;27:1874)
  • 3 risk groups are proposed, including low risk (0 points), intermediate risk (1 or 2 points) and high risk (3 points)
    • Adverse points are assigned to age 67 years and older (5 points), age 57 - 66 years (2 points), leukocyte count 15 x 109/L (1 point) and venous thrombosis (1 point) with median survival of 28 years (low risk), 19 years (intermediate risk) and 11 years (high risk) (Leukemia 2013;27:1874)
  • Next generation sequencing (NGS) study indicated that ASXL1, SRSF2 and IDH2 mutations are negative prognostic parameters associated with shorter overall, leukemia free and fibrosis free survival (Am J Hematol 2017;92:94)
  • Cytogenetic abnormalities can be detected in up to 20% of patients at the time of initial diagnosis (Haematologica 2017;102:1511)
    • Abnormalities may vary in different phases of disease
      • Isolated del(20q), +8 and +9 are shown to be the most common abnormalities in polycythemic phase
      • Del(20q) and +1q have been shown to be the most common abnormalities in postpolycythemic myelofibrosis
      • Complex karyotypes have been shown to be the most common in accelerated and blast phases
Case reports
Treatment
  • Risk of PV progressing to postpolycythemic myelofibrosis is 4.9% at 10 years and 9.4% at 15 years (Hematol J 2003;4:198)
    • Due to increased red blood cell production independent of mechanisms that typically regulate erythropoiesis
    • 15% develop spent phase with marrow fibrosis and marked cytopenias after 10 years; death in months if no treatment
  • Leukemic transformation rates at 20 years are estimated at < 10% for PV (Am J Hematol 2017;92:94)
    • 2% develop acute myeloid leukemia with phlebotomy treatment but 15% with alkylating agents or radioactive phosphorus (no longer used)
    • Risk of leukemic transformation (ALL) is rare, except in patients treated with cytotoxic regimens
  • Therapeutic goals
    • Decrease thrombosis without increasing bleeding
    • Prevent progression
    • Ameliorate symptoms
  • Treatment guidelines (Ann Oncol 2015;26:v85, Blood 2014;124:3212, N Engl J Med 2004;350:114)
    • Low / intermediate risk for thrombosis
      • First line: phlebotomy
      • Low dose aspirin
      • Intensive management of cardiovascular risk factors
    • High risk for thrombosis
      • Cytoreductive therapy (hydroxyurea)
      • Adjunct phlebotomy
      • Low dose aspirin
    • Pregnant women
      • First line: phlebotomy
      • Low dose aspirin
      • Low molecular weight heparin in postpartum period
  • European collaboration on low dose aspirin in polycythemia vera (ECLAP) trial (2004): multicenter project that investigated the benefits and risks of low dose aspirin in patients with PV
    • Study showed that daily aspirin (100 mg) lowered the risk of cardiovascular death, nonfatal myocardial infarction, nonfatal stroke and total mortality
    • Treatment also was known to nonsignificantly increase the risk of major bleeding (Am Heart J 2004;148:1068)
  • Randomized study of efficacy and safety in polycythemia vera with JAK inhibitor ruxolitinib versus best available care (RESPONSE) trial (2020): longterm efficacy and safety of ruxolitinib versus best available therapy in polycythaemia vera (Lancet Haematol 2020;7:e226)
    • 5 year follow up of a phase 3 trial comparing patients with failed response to hydroxyurea
    • Study results showed that ruxolitinib is a safe and effective longterm treatment option for patients with polycythemia vera who are resistant to or intolerant of hydroxyurea
Microscopic (histologic) description
  • Bone marrow (prepolycythemic and overt polycythemic phases) (see Microscopic (histologic) images 1, 4 & 7)
    • Hypercellular with panmyelosis; notable predominance of erythroid and megakaryocytic lineages
    • Complete and progressive maturation of all 3 hematopoietic lineages
    • Abnormal megakaryocyte morphology and architecture is a prominent feature
      • Mekagaryocytes are variably hyperlobulated and often seen in loose clusters; frequently seen close to bony trabeculae
    • Minority of cases (< 20%) may show mild increase in reticulin fibrosis (Blood 2012;119:2239)
    • Iron stores are decreased, often absent
    • Reactive lymphoid aggregates may be seen in up to 20% of cases (Histol Histopathol 2005;20:317)
    • Blasts can be seen during accelerated or blast phase
  • Post-PV / spent phase (see Peripheral smear image and Microscopic (histologic) images 2, 5 & 6)
Microscopic (histologic) images

Contributed by Erika M. Baardsen, D.O. and Ling Zhang, M.D.
Panmyelosis

Panmyelosis

Fibrosis and erythroid islands

Fibrosis and erythroid islands

Leukemic transformation (AML, M5)

Leukemic transformation (AML, M5)

Mild increase in reticulin fibers

Mild increase in reticulin fibers


Core biopsy with dense fibrosis

Core biopsy with dense fibrosis

Core biopsy with extensive fibrosis

Core biopsy with extensive fibrosis

Hypercellular bone marrow

Hypercellular bone marrow

Leukemic transformation (AML, M5)

Leukemic transformation (AML, M5)

Peripheral smear description
Peripheral smear images

Contributed by Erika M. Baardsen, D.O. and Ling Zhang, M.D.
Leukoerythroblastosis

Leukoerythroblastosis

Positive stains
Negative stains
  • MPL (thrombopoietin receptor) on megakaryocytes is usually moderate to strong in normal controls and secondary erythrocytosis but is not used for diagnosis (Blood 2000;96:771)
Molecular / cytogenetics description
  • Most frequent somatic activating mutations of JAK2 are located in exon 14 (JAK2 V617F) (> 95%) followed by exon 12 (~5%)
    • JAK2 V617F is not specific and is found at a lower frequency in other MPN
  • Patients with JAK2 V617F are usually elderly with higher level of hemoglobin, white blood cell (WBC) count and lower level of platelets, while patients with JAK2 exon 12 mutation are frequently younger with erythroid hyperplasia but low erythropoietin production (low EPO level) (Am J Hematol 2017;92:94)
  • Cytogenetic abnormalities are detectable in about 20% of patients and most commonly include del(20q), +8, +9 and +1q (Haematologica 2017;102:1511)
  • Cytogenetic abnormalities increase in frequency when patients with PV transform to acute myeloid leukemia (AML), myelodysplastic syndrome (MDS) or myelofibrosis (MF) (Leuk Lymphoma 2013;54:2667)
  • Next generation sequencing identified several gene mutations in PV in addition to JAK2, which includes most frequently TET2 and ASXL1 followed by SRSF2 and IDH2, with combined prevalence of 15% (Blood Adv 2016;1:21)
Sample pathology report
  • Bone marrow, left posterior iliac crest, aspirate and biopsy:
    • Myeloproliferative neoplasm (see comment)
    • Comment: The patient has a history of persistent erythrocytosis with a decreased erythropoietin level. A JAK2 V617F mutation was identified in the peripheral blood (VAF: 10%). The combined morphologic and ancillary laboratory findings are diagnostic of polycythemia vera.
    • Hypercellular bone marrow (80 - 90%) with panmyelosis
    • Microscopic examination
      • Peripheral blood
        • Erythrocytosis
        • Normal WBCs and platelets
      • Peripheral smear: normocytic erythrocytosis
        • Cytopenias present: none
        • Red cell morphology: normocytic erythrocytosis
        • Reticulocytosis: not increased
        • Other significant findings: none
      • Bone marrow aspirate smear
        • Adequacy: spicular, cellular
        • M:E ratio: 2:1
        • Bone marrow differential count: blasts: 2.1 %, promyelocytes: 1.0%, myelocytes: 14.2%, metamyelocytes: 15.8%, band neutrophils: 3.5%, segmented neutrophils: 9.8%, eosinophils: 11.8%, basophils: 1.2%, monocytes: 2.0%, lymphocytes: 10.0%, early normoblasts: 2.5%, late normoblasts: 24.0%
        • Significant findings: maturing trilineage hematopoiesis, no overt dysplasia, megakaryocytes are relatively increased and show variable morphology, including small forms and large hyperlobulated forms
      • Bone marrow clot section and core biopsy
        • Bony trabeculae: unremarkable
        • Cellularity: 80 - 90%
        • Megakaryocytes: increased, with occasional loose clustering
        • Estimated M:E ratio: 3:1
        • Iron stores: decreased
        • Reticulin fibrosis: none (marrow fibrosis 0/3)
        • Significant findings: hypercellular bone marrow with panmyelosis
Differential diagnosis
  • Chronic myelogenous leukemia (CML):
    • Leukocytosis (WBC ranges from 12 k/L to 1,000 k/L; median 100 k/L), basophilia, thrombocytosis, monocytosis (often accounts for < 2 - 3% of WBC)
    • No evidence of true erythrocytosis
    • Positive for BCR::ABL1 gene rearrangement
    • Negative for JAK2 V617F or JAK2 exon 12 or 13 mutation
  • Essential thrombocythemia (ET) and early phase of primary myelofibrosis (PMF):
    • Following the adoption of the updated 2016 WHO classification of PV or when a patient presents with thrombocytosis, it is particularly important to differentiate masked / prodromal polycythemia from JAK2 positive ET by careful examination of bone marrow morphology
      • In contrast to PV, ET and PMF show distinct clinical and pathologic features (bone marrow biopsy of PV shows panmyelopoiesis with more atypical or pleomorphic appearing mature megakaryocytes) (Am J Hematol 2017;92:1062, Blood 2016;127:2391)
    • ET: peripheral thrombocytosis but no evidence of erythrocytosis; bone marrow with essentially normocellular marrow, hyperlobated megakaryocytes
    • PMF, early phase: cytosis but no evidence of erythrocytosis; bone marrow with atypical megakaryocytic proliferation, often associated with clustering, accompanied by myeloid hyperplasia
    • PMF fibrotic phase: peripheral cytopenia; bone marrow with atypical megakaryopoiesis, syncytial clustering of megakaryocytes, reticulin fibrosis (marrow fibrosis 2 - 3/3)
  • Secondary erythrocytosis / polycythemia:
    • In contrast to primary polycythemia vera, EPO levels are elevated, usually as a secondary response to chronic hypoxemia
    • May be compensatory to various physiologic or environmental processes
      • Lung disease, high altitude living, cyanotic heart disease, heavy smoking, carbon monoxide poisoning, body builder or hormone replacement (e.g., testolone)
    • May be caused by EPO secreting tumors (i.e., renal cell carcinoma, hepatocellular carcinoma, cerebellar hemangioblastoma)
    • May be caused by an inherited defect that stabilizes HIF1A (prolyl hydroxylase mutations, homozygous VHL mutations, etc.)
    • Negative JAK2 mutation and normal serum EPO level help to differentiate from PV
    • Gaisbock syndrome: normal megakaryocytes, less cellular marrow, no increase in reticulin, more normal iron stores, normal or high erythropoietin levels (Angiology 1978;29:520)
Board review style question #1
Which of the following is the most helpful when differentiating between polycythemia vera (PV) and secondary erythrocytosis?

  1. Grade of marrow fibrosis
  2. Increased erythroid precursors seen in the bone marrow
  3. JAK2 V617F mutation status
  4. Presence of BCR::ABL1 gene fusion
  5. Red blood cell morphology
Board review style answer #1
C. JAK2 V617F mutation status. Detection of the JAK2 V617F mutation is a key criterion for diagnosing PV; therefore, out of the answer choices provided, JAK2 V617F mutation status would be the most helpful in differentiating PV from secondary erythrocytosis. Keep in mind the mutation can also be associated with essential thrombocythemia (ET) and primary myelofibrosis or other myeloid neoplasms. Answer A is incorrect because bone marrow fibrosis is a nonspecific response to chronic stimulation of hematopoiesis and cannot be used to differentiate between PV and secondary erythrocytosis. Answer B is incorrect because both PV and secondary erythrocytosis will show an increase in erythroid precursors on bone marrow biopsy. Answer E is incorrect because both PV and secondary erythrocytosis can exhibit variations in red blood cell morphology; there is no specific parameter for distinguishing between these entities on morphology alone. Answer D is incorrect because BCR::ABL1 gene fusion products are usually not detected in PV but have been rarely reported in certain cases of PV; however, it is not specific and does not help to differentiate PV from secondary erythrocytosis.

Comment Here

Reference: Polycythemia vera
Board review style question #2

A patient underwent bone marrow biopsy for unexplainable pruritus and erythromelalgia. The bone marrow findings are depicted in the image above. What is a notable microscopic feature of polycythemia vera (PV) that can be seen in the bone marrow during the prepolycythemic and overt polycythemic phases?

  1. Decreased erythroid precursors
  2. Hypercellularity with panmyelosis
  3. Hypocellularity with sparse megakaryocytes
  4. Increased reticulin fibrosis
  5. Presence of collagen fibrosis highlighted by trichrome stain
Board review style answer #2
B. Hypercellularity with panmyelosis. During the prepolycythemic and overt polycythemic phases of PV, the bone marrow is hypercellular with notable panmyelosis. Answer C is incorrect because the bone marrow is hypercellular, rather than hypocellular, in prepolycythemic and overt polycythemic phases of PV and megakaryocytes are often increased, rather than sparse. Answer A is incorrect because PV involves an increase in erythroid production; therefore, in both prepolycythemic and overt polycythemic phases of PV, there are increased (rather than decreased) erythroid precursor cells seen in the bone marrow. Answer D is incorrect because fibrosis is typically a feature of the postpolycythemic myelofibrosis phase of PV and not the prepolycythemic or overt polycythemic phase. Answer E is incorrect because the presence of trichrome positive collagen fibrosis is usually not observed in prefibrotic PV bone marrow and is more often seen in the postpolycythemic myelofibrosis phase.

Comment Here

Reference: Polycythemia vera
Board review style question #3
A 55 year old man is found to have a significantly elevated hemoglobin level. He has no significant medical history and no significant surgical history. He has a body mass index (BMI) of 21 and runs 40 minutes every day. He takes no medications. Vital signs are within normal limits. Physical exam is notable for a palpable spleen. Laboratory data shows a hematocrit of 53%, hemoglobin of 18 g/dL, leukocyte count of 8 x 109/L and platelet count of 800 x 1009/L. Genetic testing is positive for JAK2 V617F mutation. Presence of the JAK2 V617F mutation is one of the major criteria for diagnosing polycythemia vera (PV). Which of the following is also a major criteria requirement for diagnosis based on WHO classification?

  1. Bone marrow hypocellularity for age
  2. Hemoglobin greater than 16.5 g/dL
  3. Platelet count less than 100 x 103/μL
  4. Presence of extramedullary hematopoiesis
Board review style answer #3
B. Hemoglobin greater than 16.5 g/dL. One of the major criteria for diagnosing PV includes a hemoglobin greater than 16.5 g/dL in men and greater than 16.0 g/dL in women (see Diagnosis). Answer A is incorrect because bone marrow hypercellularity (not hypocellularity) is also a major criterion. Answer D is incorrect because, while extramedullary hematopoiesis may occur in PV, it is not a major criterion for diagnosis. Answer C is incorrect because decreased platelet count (thrombocytopenia) is not a major criterion for diagnosis and is seen less frequently than increased platelet count (thrombocytosis) in PV.

Comment Here

Reference: Polycythemia vera

Primary amyloidosis
Definition / general
  • Light chain (AL) amyloidosis is the most common form of systemic amyloidosis (Clin J Am Soc Nephrol 2006;1:1331)
  • Monoclonal immunoglobulin deposition disease with visceral and soft tissue Ig deposition resulting in organ dysfunction
  • Underlying plasma cell dyscrasia or (less commonly, 5 - 7%) lymphoplasmacytic neoplasm (Expert Rev Hematol 2018;11:117)
  • Fibrils consist of whole or fragments of immunoglobulin light chains
  • Form beta pleated sheets (AL amyloid) in tissue that bind Congo red dye and show characteristic apple green birefringence in polarized light (Clin J Am Soc Nephrol 2006;1:1331)
Essential features
  • M protein is found in the serum or urine in > 80% of patients
  • Diagnostic criteria for myeloma met in a subset of cases (12 - 15%), typically Ig molecule accumulates before there is a large tumor burden (N Engl J Med 2003;349:583)
  • Primary amyloidosis and light chain and heavy chain deposition disease (nonamyloid immunoglobulin deposition in tissues) appear to be chemically different manifestations of similar pathologic processes
  • Rare and may present with a broad range of symptoms, some vague, making it challenging to recognize (Clin J Am Soc Nephrol 2006;1:1331)
Terminology
  • Accepted nomenclature for Amyloid is AX, where A stands for Amyloid and X represents the protein in the fibril
  • Historically light chain amyloidosis (AL) referred to as primary amyloidosis
  • Overall, > 30 different proteins can cause amyloidosis (Blood 2009;114:4957, Clin J Am Soc Nephrol 2010;5:2180)
  • Those involving kidney recently grouped into the category of monoclonal gammopathies of renal significance (MGUS) to separate them from MGUS based on need for treatment (Blood 2012;120:4292)
ICD coding
  • ICD-O: 9769/1 - immunoglobulin deposition disease
  • ICD-10:
    • E85.9 - amyloidosis, unspecified (general, primary)
    • E85.81 - light chain (AL) amyloidosis
Epidemiology
  • Incidence: approximately 1 case per 100,000; approximately 3,000 cases in the U.S. per year
  • Most common type of systemic amyloidosis in developed countries (75%) (Blood 2016;128:159)
  • Median age 64, > 95% are older than 40
  • Predominant in men (65 - 70%)
Sites
Pathophysiology
  • Abnormal folding of a protein that is normally soluble as a result of a proteolytic event or mutation that renders it unstable / prone to self aggregation (Clin J Am Soc Nephrol 2006;1:1331)
  • Aggregates form protofilaments that associate into amyloid fibrils
  • Interactions with the extracellular environment may result in proteolytic cleavage and matrix components such as glycosaminoglycans and collagen may serve as scaffolds to facilitate aggregation and fibril buildup (Sci Rep 2016;6:29096)
  • Only a subset of Ig light chains are amyloidogenic (12 - 15% of myeloma) (N Engl J Med 2003;349:583)
  • Repertoire of variable region germline genes involved in systemic amyloidosis is restricted and certain gene usage may affect organ targeting; e.g. use of IGVL1-44 had 5 fold odds of dominant cardiac involvement (Blood 2012;119:144)
  • Amyloid precursor proteins directly impair cardiac function independent of fibril formation, invoking oxidative stress, cellular dysfunction, apoptosis (Proc Natl Acad Sci U S A 2010;107:4188)
Etiology
  • Clonal expansion of plasma cells or low grade lymphoproliferative disorder secreting monoclonal immunoglobulin light chains which misfold, aggregate and deposit as interstitial fibrils of amyloid
Clinical features
  • ~30% present with advanced, irreversible organ damage (Blood 2016;128:159)
  • General: dyspnea, fatigue, weight loss
  • GI: macroglossia (highly suggestive of AL but only present in 10 - 15%), hepatomegaly (25 - 30%), malabsorption (5%)
  • Renal: nephrotic syndrome (28%)
  • Heart: restrictive cardiomyopathy, congestive heart failure (17%), amyloid precursor proteins with direct cardiotoxic effect (Proc Natl Acad Sci U S A 2010;107:4188)
  • Neural: peripheral sensory neuropathy (17%), carpal tunnel syndrome (21%), autonomic dysfunction
  • Bone pain (5%), periorbital or facial purpura
  • Hemorrhage: can result from factor X binding to amyloid (Blood 2001;97:1885)
  • Patients with underlying IgM-secreting lymphoplasmacytic lymphoma are typically older, higher prevalence of neuropathy and lymph node involvement and lower level of cardiac involvement than non IgM AL amyloidosis (Expert Rev Hematol 2018;11:117)
Diagnosis
  • Tissue diagnosis is the gold standard
  • Laser microdissection with analysis by mass spectrometry is the most reliable method of characterizing amyloid type (Blood 2009;114:4957)
  • Abdominal fat pad aspirate: most common and easily accessible tissue (positive in more than 80% of patients with primary amyloidosis)
  • Rectal biopsies diagnostic in 80% of cases
  • Biopsy of kidney, heart, liver
  • Bone marrow: 60% of cases show amyloid; detection improved if adequate sized vessels present
Laboratory
  • Serum and urine protein electrophoresis and immunoelectrophoresis identifies M spike (99% of cases); IgG most frequent (followed by light chain only, IgA, IgM and IgD); 80% lambda (Expert Rev Hematol 2018;11:117)
  • Critical to test for free light chain as some patients lack circulating intact immunoglobulin
  • Nephrotic range proteinuria and hypoalbuminemia with renal involvement
  • Elevated erythrocyte sedimentation rate (ESR)
  • Elevated brain natriuretic peptide (BNP), proBNP and troponin in cardiac involvement
  • Cholestasis with elevated alkaline phosphatase with liver involvement
  • AL / primary amyloidosis may also present with hypothyroidism / hypoadrenalism or hypopituitarism
  • Scintigraphy with radiolabeled serum amyloid P(SAP) can be used to test for amyloid where SAP binds to the amyloid and reveals its presence (Annu Rev Med 2006;57:223)
Radiology description
  • Bone lesions restricted to patients with myeloma
  • Uncommon to see splenomegaly or lymphadenopathy
Prognostic factors
  • Extent of cardiac involvement a major determinant of outcome and most frequent cause of death (40% of cases)
  • Higher risk: monoclonal plasma cells > 10% bone marrow involvement, high serum free light chain, elevated beta-2-microglobulin, multiple organ involvement, elevated uric acid level
  • Median survival time of 12 months if untreated
  • Multiple staging systems have been proposed based on cardiac markers or renal function (Blood 2016;128:159)
Case reports
Treatment
  • Antiplasma cell chemotherapy to reduce concentration of toxic light chains based on myeloma regimens
  • Myeloma treatment regimens are discussed in the NCCN guidelines (J Natl Compr Canc Netw 2016;14:389)
  • Not solely a hematologic malignancy with amyloid related organ dysfunction limiting access to aggressive treatment; clinical trials are recommended with close monitoring of response to treatment (Blood 2016;128:159)
  • Common regimens include melphalan or dexamethasone; thalidomide and its analogues may also be used
  • May have increased sensitivity to proteasome inhibitors such as bortezomib due to presence of misfolded light chain (Blood 2017;129:2132)
  • Autologous stem cell transplant may also be considered with caution for transplant related mortality; not solely a hematologic malignancy with amyloid related organ dysfunction limiting access to aggressive treatment (Blood 2016;128:159)
  • Targeting amyloid deposits to interfere with organ damage is a newer strategy: the small molecule anthracycline 49-iodo-49-deoxy-doxorubicin and related molecule doxycycline (antibiotic) may disrupt amyloid fibrils and improve clinical status (Proc Natl Acad Sci U S A 1995;92:2959, Blood Cancer J 2017;7:e546) and other approaches as (Blood 2016;128:159)
Clinical images

Images hosted on other servers:

Systemic AL amyloidosis

Gross description
  • Amyloidosis may or may not be apparent on macroscopic examination
  • Classic gross appearance porcelain-like or waxy
  • Involved organ may appear enlarged, gray, firm and waxy when amyloid accumulates in larger amounts
Microscopic (histologic) description
  • Pale pink, extracellular, glassy, hyaline material on H&E staining, mostly deposited in vascular or perivascular location
  • Characteristic cracking artifact
  • Macrophages and foreign body giant cells may be found around deposits
Microscopic (histologic) images

Contributed by Meenakshi Bansal, M.D.

Bone marrow with extensive involvement by amyloid


Gastric biopsies from the same patient showing extensive involvement by amyloid

Cytology description
  • Marrow aspirate smears typically contain > 10% plasma cells
  • Plasma cells may show a range of morphologies including those seen in myeloma, vacuolated as seen in mu heavy chain disease may be seen
  • If extensive amyloid deposition, proteinaceous clumps (lightly eosinophilic to basophilic) may be present
Positive stains
  • Congo red: ordinary light imparts a pink to red color to amyloid deposits; the beta pleated sheet structure of amyloid produces an apple green birefringence under polarized microscopy when stained by Congo red or sirius red stain
  • PAS: moderately positive
  • Iodine: stains deep brown but turns blue after treatment with concentrated sulfuric acid ("iodine sulphuric reaction" historical review Histochemistry 1976;49:131)
  • Crystal violet and toluidine blue: metachromatic staining
  • Thioflavin T and thioflavin S: exhibits fluorescence
  • Clonal plasma cells can also be identified
Negative stains
Electron microscopy description
  • Extracellular, amorphous, straight nonbranching thin fibrils of 4 - 9 nm
Electron microscopy images

Contributed by Karen Vanderbilt and Bruce Goldman, M.D.

Amyloid by electron micrograph

Molecular / cytogenetics description
  • Genetic abnormalities similar to non-IgM MGUS; e.g. 13q14 deletion, 1q21 gain
  • t(11;14) overrepresented (present > 40% of cases) and may be associated with worse survival in contrast to the favorable prognosis in myeloma (Hematology Am Soc Hematol Educ Program 2010;2010:287)
  • Not typically seen are t(4;14) and del 17p
Differential diagnosis
  • Light chain or heavy chain deposition disease: plasma cell tumors that secrete abnormal light or heavy chain which deposit in tissues and cause organ dysfunction
    • Do NOT have amyloid beta pleated sheets or bind Congo red
    • Often do have monoclonal gammopathy (75%), most often kappa (80%), variable myeloma
  • Secondary or hereditary amyloidosis: laser microdissection and analysis by mass spectrometry most reliable to distinguish; IHC with anti-amyloid fibril antibodies can also be used; presence of MGUS does not exclude concurrent secondary or familial amyloidosis (Engl J Med 2002;346:1786)
  • Serous fat atrophy: extensive extravascular deposits may resemble serous fat atrophy (clinical history, Congo red staining will help distinguish)
Board review style question #1
Which parameter is most closely linked to a poorer prognosis for primary amyloidosis?

  1. Extent of cardiac involvement
  2. Liver involvement
  3. Macroglossia
  4. Nephrotic range proteinuria
  5. Presence of myeloma
Board review style answer #1
A. The best answer is extent of cardiac involvement, which also results in 40% of deaths due to primary amyloidosis. Increased bone marrow plasma cells (> 10%) and involvement of multiple organs are also associated with a worse prognosis.

Comment Here

Reference: Primary amyloidosis
Board review style question #2
Which genetic alteration is most overrepresented in primary amyloidosis compared to plasma cell myeloma?

  1. del 17p
  2. Hyperdiploidy
  3. Monosomy 13q14
  4. t(11;14)
  5. TP53 mutation
Board review style answer #2
D. t(11;14)

While all of these alterations have been described in plasma cell myeloma and may be seen in primary amyloidosis, the t(11;14) translocation is particularly overrepresented in primary amyloidosis (> 40% of cases).13q14 deletion is also common but del 17p is rarely seen.

Comment Here

Reference: Primary amyloidosis

Primary myelofibrosis
Definition / general
  • Primary myelofibrosis is an uncommon myeloproliferative neoplasm characterized by a proliferation of predominantly abnormal megakaryocytes and granulocytes in the bone marrow, which in fully developed disease is associated with reactive deposition of fibrous connective tissue and extramedullary hematopoiesis
Essential features
  • Shows a stepwise evolution from an initial prefibrotic / early stage to overt fibrotic stage
  • Is a clonal stem cell defect characterized by:
    • Impaired medullary hematopoiesis
      • Granulocytic and megakaryocytic proliferation in the bone marrow
      • Gradual increase in bone marrow reticulin and collagen fibrosis
    • Extramedullary hematopoiesis in the spleen, liver and other organs
Terminology
  • Chronic idiopathic myelofibrosis, myelofibrosis / sclerosis with myeloid metaplasia, agnogenic myeloid metaplasia, megakaryocytic myelosclerosis, idiopathic myelofibrosis, myelofibrosis with myeloid metaplasia, myelofibrosis as a result of myeloproliferative disease (Swerdlow: WHO Classification of Tumours of Haematopoietic and Lymphoid Tissues, 4th Edition, 2017)
  • Myelofibrosis refers to the increase in the amount and density of reticulin fibers in the bone marrow (can be caused by infections, inflammatory, neoplasms, etc)
ICD coding
  • ICD-10: D75.81 - myelofibrosis
Epidemiology
  • Least common of all myeloproliferative neoplasms
  • Estimated annual incidence of overt phase is 0.5 - 1.5 cases per 100,000 population
  • Prefibrotic / early phase accounts for 30 - 50% of all cases
  • Increasing prevalence due to earlier diagnosis (pre-primary myelofibrosis)
  • Equal gender predisposition
  • Disease of older adults (mean age 60 years)
  • Extremely rare in children; however, some childhood cases may be inherited and associated with other anomalies (Jaffe: Hematopathology, 2nd Edition, 2016)
  • Increased incidence in Ashkenazi Jews (Hsi: Hematopathology - Foundations in Diagnostic Pathology, 3rd Edition, 2017)
Pathophysiology
  • Clonal stem cell defect
  • Fibrosis is due to neoplastic megakaryocytes releasing platelet derived growth factor, basic fibroblast growth factor, transforming growth factor beta or other cytokines, which causes nonneoplastic fibroblasts in marrow to deposit collagen
    • Myelofibrosis and osteosclerosis are secondary changes due to the abnormal release of growth factors and fibrogenic cytokines
  • Prominent angiogenesis in the bone marrow and spleen due to increased serum levels of vascular endothelial growth factor
  • Teardrop red blood cells and leukoerythroblastosis are caused by extramedullary hematopoiesis
Clinical features
  • Presentation
    • Asymptomatic (30%)
    • Symptomatic (70%)
      • Splenomegaly (90%): often considered a hallmark
      • Hepatomegaly (50%): portal hypertension can develop as a complication of hepatomegaly and may precede the onset of the disease (Gastroenterology 1988;94:1063)
      • Constitutional symptoms such as fatigue, dyspnea, weight loss, night sweats, low grade fever and cachexia
        • Fatigue is the most common presenting symptom
      • Anemia
      • Leukocytosis or thrombocytosis is common; leukopenia or thrombocytopenia is less common
      • Gouty arthritis and renal stones
      • Marked thrombocytosis may lead to thrombosis or hemorrhagic episodes
      • Uncommon symptoms include pruritus and pulmonary hypertension (Am J Hematol 2012;87:136, Leukemia 2008;22:646)
  • 2 phases (Swerdlow: WHO Classification of Tumours of Haematopoietic and Lymphoid Tissues, 4th Edition, 2017):
    • Prefibrotic:
      • Usually presents with thrombocytosis
      • No blasts
      • No / borderline anemia
      • No / borderline splenomegaly
      • Normal or borderline increased lactate dehydrogenase
      • Misdiagnosis of essential thrombocythemia is possible if the bone marrow biopsy is not carefully examined
    • Overt:
      • Thrombocytopenia
      • Leukoerythroblastosis
      • Anemia
      • Splenomegaly
      • Increased lactate dehydrogenase
      • Increased myeloblasts can be seen (< 20%)
      • Transformation to acute myeloid leukemia occurs in ~5 - 20% at a median of 3 years after the diagnosis of the overt phase (Jaffe: Hematopathology, 2nd Edition, 2016)
        • 2 factors at the time of diagnosis are considered predictors of leukemic transformation: circulating blasts ≥ 3% and platelet count < 100,000/μL (Cancer 2008;112:2726)
  • Extramedullary hematopoiesis can form masses
  • May coexist with systemic mastocytosis (J Mol Diagn 2008;10:58)
  • Survival 3.5 - 5.5 years
Diagnosis
  • Diagnostic criteria for primary myelofibrosis, prefibrotic / early stage: must meet all 3 major criteria plus at least 2 minor criteria (Swerdlow: WHO Classification of Tumours of Haematopoietic and Lymphoid Tissues, 4th Edition, 2017)
    • Major
      • Major criterion 1: megakaryocyte proliferation and atypia in the absence of reticulin fibrosis, accompanied by increased marrow cellularity, granulocytic proliferation and often decreased erythropoiesis
      • Major criterion 2: not meeting WHO criteria for other myeloproliferative neoplasms or myelodysplastic syndromes
      • Major criterion 3: demonstration of JAK2, CALR or MPL mutation OR other clonal marker OR no evidence of reactive marrow fibrosis
    • Minor
      • Minor criterion 1: leukoerythroblastosis
      • Minor criterion 2: increased serum lactate dehydrogenase
      • Minor criterion 3: anemia
      • Minor criterion 4: palpable splenomegaly
    • Comparing WHO 2017 to WHO 2008, only minor changes to the diagnostic criteria; the change involved the addition of CALR or MPL mutations in addition to the JAK2 mutation
  • Diagnostic criteria for primary myelofibrosis, overt fibrotic stage: all 3 major criteria and at least 1 minor criterion must be confirmed in 2 consecutive determinations (Swerdlow: WHO Classification of Tumours of Haematopoietic and Lymphoid Tissues, 4th Edition, 2017)
    • Major
      • Major criterion 1: megakaryocytic proliferation and atypia, accompanied by reticulin or collagen fibrosis grades 2 or 3
      • Major criterion 2: not meeting WHO criteria for other myeloproliferative neoplasms or myelodysplastic syndromes
      • Major criterion 3: demonstration of JAK2, CALR or MPL mutation OR other clonal marker OR no evidence of reactive marrow fibrosis
    • Minor
      • Minor criterion 1: anemia not attributed to a comorbid condition
      • Minor criterion 2: leukocytosis
      • Minor criterion 3: palpable splenomegaly
      • Minor criterion 4: increased serum lactate dehydrogenase
      • Minor criterion 5: leukoerythroblastosis
Laboratory
  • Complete blood count / peripheral blood smear
    • Prefibrotic:
      • Initially normal or increased blood counts
        • Mild neutrophilia with a left shift
        • Thrombocytosis (mild / moderate)
      • No / borderline anemia
      • No myeloblasts
      • No leukoerythroblastosis
    • Overt:
      • Thrombocytopenia with bizarre abnormal large platelets with altered granulation; in addition, fragmented megakaryocytes can be seen on the peripheral smear
      • Leukoerythroblastosis and anemia
      • Myeloblasts (usually > 5%)
  • Lactate dehydrogenase
    • Prefibrotic: normal or borderline increased lactate dehydrogenase
    • Overt: increased lactate dehydrogenase
  • Alkaline phosphatase, uric acid, leukocyte alkaline phosphatase and vitamin B12 are increased (Leuk Lymphoma 1996;22:303)
  • Bone marrow aspirate / biopsy
    • Prefibrotic:
      • Hypercellular bone marrow with granulocytic and atypical megakaryocytic proliferations
        • Usually morphologically more variable in size, atypical and bizarre than other myeloproliferative neoplasms
        • Bulbous hypolobated nucleus
      • Erythropoiesis is reduced
      • Absent or only slight reticulin fibrosis
    • Overt:
      • Hypocellular bone marrow with usually alternating cellular and hypocellular regions
      • Atypical megakaryocytes can form clusters or sheets
        • More variable in size, atypical and bizarre than other myeloproliferative neoplasms
        • Bulbous hypolobated nucleus
      • Marked reticulin or collagen fibrosis
      • New bone formation and osteosclerosis
    • Bone marrow aspirate is often difficult (dry tap)
  • See also Molecular description
Prognostic factors
  • Has the least favorable prognosis among the myeloproliferative neoplasms
  • Patients are at risk of death due to disease progression, leukemic transformation, thrombohemorrhagic complications and infections
  • Survival
    • Prefibrotic: 10 - 15 years
    • Overt: 3 - 5 years
  • Dynamic International Prognostic Scoring System (DIPSS plus) (Swerdlow: WHO Classification of Tumours of Haematopoietic and Lymphoid Tissues, 4th Edition, 2017)
    • Includes 8 predictors of inferior survival:
      • Patient age > 65 years
      • Red blood cell transfusion dependency
      • Unfavorable karyotype
      • Hemoglobin < 10 g/dL
      • White blood cells > 25 × 109/L
      • Constitutional symptoms (fever, night sweats, weight loss)
      • Circulating blasts ≥ 1%
      • Platelet count < 100 × 109/L
    • Risk status is defined by the number of adverse prognostic factors present: 0 (low risk), 1 (intermediate - 1 risk), 2 or 3 (intermediate - 2 risk) or > 4 (high risk)
  • Absolute monocyte count ≥ 1 × 109/L confers an unfavorable outcome
  • Unfavorable karyotype
    • Complex karyotype, sole abnormality
    • 2 abnormalities that include +8, -7 / 7q-, i(17q), inv(3), -5 / del(5q), 12p- or 11q23.3 rearrangement
  • CALR mutation confers a better prognosis compared to other mutations
Case reports
Treatment
  • Management based on the risk stratification by
    • GIPSS (Genetically Inspired Prognostic Scoring System): uses karyotyping, driver mutations and HMA mutations
    • MIPSS70+ v2.0 (Mutation and Karyotype Enhanced International Prognostic Scoring System) criteria: uses clinical information, karyotyping and mutations
  • Higher risk:
    • Eligible for transplantation: allogeneic hematopoietic cell transplantation
    • Ineligible for transplantation: enrollment in a clinical trial or treatment with ruxolitinib or hydroxyurea for relief of symptoms (N Engl J Med 2012;366:787)
  • Lower risk:
    • Asymptomatic: observation
    • Symptomatic: treatment with ruxolitinib or hydroxyurea
Microscopic (histologic) description
  • Bone marrow:
    • Prefibrotic stage:
      • Hypercellular with large, dysplastic, clustered (loose or tight) megakaryocytes and excess granulocytes
      • Increased reticulin is present around clusters of megakaryocytes; megakaryocytes have aberrant nuclear/cytoplasmic ratios and hyperchromatic, bulbous or irregularly folded nuclei
      • Often bare megakaryocytic nuclei
      • Megakaryocytic features are most useful to distinguish this stage of primary myelofibrosis from essential thrombocythemia
    • Fibrotic phase:
      • Hypocellular and diffusely fibrotic bone marrow with atypical streaming megakaryocytes
      • Marrow osteosclerosis with irregular, broad bony trabeculae
      • Markedly dilated sinuses; associated with dry bone marrow taps
  • Spleen: red pulp sinuses contain megakaryocytes, granulocyte precursors, nucleated red cells; may be nodules of extramedullary hematopoiesis
Microscopic (histologic) images

Contributed by Hatem Kaseb, M.D., Ph.D., M.P.H. and AFIP images

Atypical megakaryocytes

Fibrotic bone marrow

Blast transformation

Peripheral smear description
  • Leukoerythroblastosis (immature granulocytes and normoblasts in peripheral blood) is common in later phases; also dacrocytes (teardrop erythrocytes)
Peripheral smear images

Contributed by Hatem Kaseb, M.D., Ph.D., M.P.H. and AFIP images

Dacrocytes

Cell intermediate

Blast transformation

Positive stains
Flow cytometry description
  • Normal number and phenotype myeloblasts with normal myeloid scatter by CD45 / side scatter
  • Normal CD10 / CD13 / CD16 / CD11b myeloid maturation pattern and all other myeloid markers are normally expressed, hence there is no immunophenotypic evidence of myelodysplasia
  • No evidence for monoclonal B cell lymphoproliferative disease
  • Although no unusual phenotype T cells are identified, surface markers will not routinely detect monoclonal T lymphocytes
Flow cytometry images

Contributed by Hatem Kaseb, M.D., Ph.D., M.P.H.

Normal myeloid maturation

Molecular / cytogenetics description
  • No specific genetic marker
  • No BCR-ABL fusion gene (or classify as chronic myeloid leukemia); presence of del(13)(q12-22) or der(6)t(1;6)(q21-23;p21.3) is strongly suggestive of primary myelofibrosis
  • Shows karyotypic abnormalities in up to 50% of cases
    • Frequent abnormalities include del(20q), del(13q), +8, +9 and abnormalities of 1q
  • Driver mutations
    • JAK2 V617F mutation (50 - 60%)
    • CALR mutations (25 - 30%)
    • MPL mutations (5 - 10%)
    • Triple negative for mutations in JAK2, CALR and MPL (8 - 12%)
    • Other gene mutations include ASXL1, EZH2, TET2, IDH1 / IDH2, SRSF2 or SR3B1
  • Driver mutational profile is associated with overall survival
    • CALR mutated cases have better survival compared to others
    • Triple negative cases have the shortest survival compared to others
Sample pathology report
  • Bone marrow, biopsy and aspirate:
    • Myeloproliferative neoplasm, favoring prefibrotic primary myelofibrosis (see comment)
    • Comment: The marrow biopsy shows morphologic features of a myeloproliferative neoplasm, in conjunction with thrombocytosis. The major differential diagnosis includes essential thrombocythemia and prefibrotic stage of primary myelofibrosis. Overall, we favor a diagnosis of the latter, due to hypercellular marrow, distribution of megakaryocytes (focal clusters) and markedly dysplastic cytology of megakaryocytes. Recommend correlation with clinical findings (presence of splenomegaly and disease progression) and molecular studies (JAK2, CALR and MPL mutational studies).
    • Complete blood count (9/17/2018), by report: hemoglobin - 11.4 g/dL, mean corpuscular volume - 100 fL, white blood cells - 21.7 K/uL, platelets - 698 K/uL
  • Bone marrow biopsy microscopic description:
    • Hypercellular marrow for age (80% cellular). Megakaryocytes are increased in number, forming focal loose clusters and showing abnormal morphology including hypersegmented nuclei with hyperchromasia. The myeloid:erythroid ratio is markedly increased. Erythroid elements exhibit maturation. Myeloid elements exhibit maturation. Myeloblasts appear to be increased. No lymphoid aggregates are identified. Granulomas are absent. Reticulin stain reveals that reticulin is mildly increased (MF1/3). Trabecular bone is normal.
  • Bone marrow aspirate microscopic description:
    • The marrow aspirate smears are spicular and cellular. Megakaryocytes are increased and show prominent dysmegakaryopoiesis, including hyper and hyposegmented nuclei and increased nuclear/cytoplasmic ratio. The myeloid:erythroid ratio is approximately 4:1. Erythroid maturation is identified. Myeloid maturation is slightly left shifted. Prussian blue iron stain shows # storage iron and no significant increase in ring sideroblasts.
    • Aspirate cell count: A 207 cell count reveals 1% blasts, 15% promyelocytes / myelocytes, 57% maturing granulocyte forms, 20% erythroid forms, 3% lymphocytes, 2% eosinophils, 1% plasma cells, 1% basophils / mast cells and 2% monocytes.
Differential diagnosis
  • Other causes of leukoerythroblastosis or dry taps:
  • Other myeloid disorders:
    • Myeloproliferative neoplasms:
    • Myelodysplastic / myeloproliferative neoplasms:
      • Show hybrid myelodysplastic and myeloproliferative features manifested by at least 1 cytopenia and at least 1 cytosis in blood
      • Myeloid maturation is intact and usually mature cells predominate
      • Exact subtype should be diagnosed based on the specific criteria of the entity
    • Myelodysplastic syndromes are a heterogeneous group of entities that demonstrate:
      • Variable degrees of ineffective hematopoiesis
      • < 20% blasts in marrow
      • Peripheral blood with unexplained and persistent cytopenia(s), < 20% blasts, < 1.0 x 109/L monocytes and dysplasia in 1 or more myeloid lineages
    • Mastocytosis:
      • Shows focal, compact aggregates of mast cells in bone marrow core biopsy
      • Cells can show the following patterns of infiltration: paratrabecular, perivascular or parafollicular
      • Presence of atypical mast cell supports the diagnosis
      • Most cases will harbor a KIT D816V mutation
  • Reactive thrombocythemia:
    • Causes include exercise, allergic reaction, reaction to medications, inflammatory disorders, asplenism, infection, connective tissue disorders, metastatic cancer, lymphoproliferative disorders and iron deficiency
  • Acute myelofibrosis:
    • Subtype of acute myeloid leukemia that shows > 20% blasts in peripheral blood or bone marrow
    • Patients also present with bone marrow fibrosis, fever and pancytopenia
  • Autoimmune myelofibrosis: generally shows diffuse reticulin fibrosis and an associated autoimmune disease such as rheumatoid arthritis
  • Primary hyperparathyroidism: rarely causes myelofibrosis and pancytopenia (Int J Lab Hematol 2007;29:464)
Board review style question #1

A 57 year old man presented with fatigue, splenomegaly and hepatomegaly. A complete blood count demonstrated anemia and thrombocytosis. A bone marrow aspirate and biopsy were ordered (shown above). What is the most likely diagnosis?

  1. Acute myeloid leukemia
  2. Chronic myelogenous leukemia
  3. Myelodysplastic syndrome
  4. Primary myelofibrosis
Board review style answer #1
D. Primary myelofibrosis

The symptoms and bone marrow biopsy support the diagnosis of primary myelofibrosis. The biopsy above shows hypercellular bone marrow with granulocytic and atypical megakaryocytic proliferations.

Comment Here

Reference: Primary myelofibrosis
Board review style question #2
What is the most common genetic mutation in primary myelofibrosis?

  1. ABL
  2. CALR
  3. JAK2
  4. MPL
Board review style answer #2
C. JAK2

Primary myelofibrosis cases harbor a phenotypic driver mutation in JAK2 V617F mutation (50 - 60%), CALR mutations (25 - 30%), MPL mutations (5 - 10%), triple negative for mutations in JAK2, CALR and MPL (8 - 12%).

Comment Here

Reference: Primary myelofibrosis

Primary myelofibrosis
Definition / general
  • Primary myelofibrosis is an uncommon myeloproliferative neoplasm characterized by a proliferation of predominantly abnormal megakaryocytes and granulocytes in the bone marrow, which in fully developed disease is associated with reactive deposition of fibrous connective tissue and extramedullary hematopoiesis
Essential features
  • Shows a stepwise evolution from an initial prefibrotic / early stage to overt fibrotic stage
  • Is a clonal stem cell defect characterized by
    • Impaired medullary hematopoiesis
      • Granulocytic and megakaryocytic proliferation in the bone marrow
      • Gradual increase in bone marrow reticulin and collagen fibrosis
    • Extramedullary hematopoiesis in the spleen, liver and other organs
  • The International Consensus Classification (ICC) 2022 distinguishes prefibrotic from overtly fibrotic PMF (Blood 2022;140:1200)
Terminology
  • Myelofibrosis refers to the increase in the amount and density of reticulin fibers in the bone marrow (can be caused by infections, inflammatory, neoplasms, etc.)
ICD coding
  • ICD-10: D75.81 - myelofibrosis
Epidemiology
Sites
  • Mainly blood and bone marrow
  • Other variably included sites: liver, spleen
Pathophysiology
  • Clonal stem cell defect
  • Fibrosis is due to neoplastic megakaryocytes releasing platelet derived growth factor, basic fibroblast growth factor, transforming growth factor beta or other cytokines, which causes nonneoplastic fibroblasts in marrow to deposit collagen
    • Myelofibrosis and osteosclerosis are secondary changes due to the abnormal release of growth factors and fibrogenic cytokines
  • Prominent angiogenesis in the bone marrow and spleen due to increased serum levels of vascular endothelial growth factor
  • Teardrop red blood cells and leukoerythroblastosis are caused by extramedullary hematopoiesis
  • References: Int J Lab Hematol 2023:45:59, Leukemia 2022;36:1703
Etiology
No information provided
Diagrams / tables
None
Clinical features
  • Presentation
    • Asymptomatic (30%)
    • Symptomatic (70%)
      • Splenomegaly (90%): often considered a hallmark
      • Hepatomegaly (50%): portal hypertension can develop as a complication of hepatomegaly and may precede the onset of the disease (Gastroenterology 1988;94:1063)
      • Constitutional symptoms such as fatigue, dyspnea, weight loss, night sweats, low grade fever and cachexia
        • Fatigue is the most common presenting symptom
      • Anemia
      • Leukocytosis or thrombocytosis is common; leukopenia or thrombocytopenia is less common
      • Gouty arthritis and renal stones
      • Marked thrombocytosis may lead to thrombosis or hemorrhagic episodes
      • Uncommon symptoms include pruritus and pulmonary hypertension (Am J Hematol 2012;87:136, Leukemia 2008;22:646)
  • 2 phases
    • Prefibrotic
      • Usually presents with thrombocytosis
      • No blasts
      • No / borderline anemia
      • No / borderline splenomegaly
      • Normal or borderline increased lactate dehydrogenase
      • Misdiagnosis of essential thrombocythemia is possible if the bone marrow biopsy is not carefully examined
    • Overt / fibrotic
      • Thrombocytopenia
      • Leukoerythroblastosis
      • Anemia
      • Splenomegaly
      • Increased lactate dehydrogenase
      • Increased myeloblasts can be seen (< 20%); 10 - 19% blasts indicates progression to accelerated phase
      • Transformation to acute myeloid leukemia occurs in ~5 - 20% at a median of 3 years after the diagnosis of the overt phase (Jaffe: Hematopathology, 2nd Edition, 2016)
        • 2 factors at the time of diagnosis are considered predictors of leukemic transformation: circulating blasts ≥ 3% and platelet count < 100,000/μL (Cancer 2008;112:2726)
  • Extramedullary hematopoiesis can form masses
  • Survival: 3.5 - 5.5 years
Diagnosis

Diagnostic criteria for prefibrotic primary myelofibrosis Diagnostic criteria for primary myelofibrosis, fibrotic stage
Diagnosis of prefibrotic primary myelofibrosis requires that all 3 major criteria and at least 1 minor criterion are met Diagnosis of overt primary myelofibrosis requires that all 3 major criteria and at least 1 minor criterion are met
Major criteria
  1. Megakaryocytic proliferation and atypia, without reticulin fibrosis grade > 1a, accompanied by increased age adjusted bone marrow cellularity, granulocytic proliferation and (often) decreased erythropoiesis
  2. WHO criteria for BCR::ABL1 positive chronic myeloid leukemia, polycythemia vera, essential thrombocythemia, myelodysplastic syndromes or other myeloid neoplasms are not met
  3. JAK2, CALR or MPL mutation
    Or presence of another clonal marker
    Or absence of minor reactive bone marrow reticulin fibrosis
  1. Megakaryocytic proliferation and atypia, accompanied by reticulin or collagen fibrosis grades 2 or 3a
  2. WHO criteria for essential thrombocythemia, polycythemia vera, BCR::ABL1 positive chronic myeloid leukemia, myelodysplastic syndrome or other myeloid neoplasms are not met
  3. JAK2, CALR or MPL mutation
    Or presence of another clonal marker
    Or absence of reactive myelofibrosis
Minor criteria
Presence of at least 1 of the following, confirmed in 2 consecutive determinations
  • Anemia not attributed to a comorbid condition
  • Leukocytosis ≥ 11 x 109/L
  • Splenomegaly detected clinically or by imaging
  • Lactate dehydrogenase level above the upper limit of the institutional reference range
  • Leukoerythroblastosis
Presence of at least 1 of the following, confirmed in 2 consecutive determinations
  • Anemia not attributed to a comorbid condition
  • Leukocytosis ≥ 11 x 109/L
  • Splenomegaly detected clinically or by imaging
  • Lactate dehydrogenase level above the upper limit of the institutional reference range
  • Leukoerythroblastosis
Laboratory
  • Complete blood count / peripheral blood smear
    • Prefibrotic
      • Initially normal or increased blood counts
        • Mild neutrophilia with a left shift
        • Thrombocytosis (mild / moderate)
      • No / borderline anemia
      • No myeloblasts
      • No leukoerythroblastosis
    • Overt
      • Thrombocytopenia with bizarre abnormal large platelets with altered granulation; in addition, fragmented megakaryocytes can be seen on the peripheral smear
      • Leukoerythroblastosis and anemia
      • Myeloblasts (usually > 5%)
  • Lactate dehydrogenase (LDH)
    • Prefibrotic: normal or borderline increased lactate dehydrogenase
    • Overt: increased lactate dehydrogenase
  • Alkaline phosphatase, uric acid, leukocyte alkaline phosphatase and vitamin B12 are increased (Leuk Lymphoma 1996;22:303)
  • Bone marrow aspirate / biopsy
    • Prefibrotic
      • Hypercellular bone marrow with granulocytic and atypical megakaryocytic proliferations
        • Usually morphologically more variable in size, atypical and bizarre than other myeloproliferative neoplasms
        • Bulbous hypolobated nucleus
      • Erythropoiesis is reduced
      • Absent or only slight reticulin fibrosis
    • Overt
      • Hypocellular bone marrow with usually alternating cellular and hypocellular regions
      • Atypical megakaryocytes can form clusters or sheets
        • More variable in size, atypical and bizarre than other myeloproliferative neoplasms
        • Bulbous hypolobated nucleus
      • Marked reticulin (MF2 / 3) or collagen fibrosis
      • New bone formation and osteosclerosis
    • Bone marrow aspirate is often difficult (dry tap)
  • See also Molecular description
Radiology description
No information provided
Radiology images
None
Prognostic factors
  • Has the least favorable prognosis among the myeloproliferative neoplasms
  • Patients are at risk of death due to disease progression, leukemic transformation, thrombohemorrhagic complications and infections
  • Survival
    • Prefibrotic: 10 - 15 years
    • Overt: 3 - 5 years
  • Dynamic International Prognostic Scoring System (DIPSS plus)
    • Includes 8 predictors of inferior survival:
      • Patient age > 65 years
      • Red blood cell transfusion dependency
      • Unfavorable karyotype
      • Hemoglobin < 10 g/dL
      • White blood cells > 25 × 109/L
      • Constitutional symptoms (fever, night sweats, weight loss)
      • Circulating blasts ≥ 1%
      • Platelet count < 100 × 109/L
    • Risk status is defined by the number of adverse prognostic factors present
      • 0 (low risk), 1 (intermediate - 1 risk), 2 or 3 (intermediate - 2 risk) or > 4 (high risk)
  • Genetics inspired International Prognostic Scoring System (Leukemia 2018;32:1631)
    • Points are scored as follows
      • Very high risk karyotype f (2 points)
      • Unfavorable karyotype g (1 point)
      • ASXL1 mutation (1 point)
      • SRSF2 mutation (1 point)
      • U2AF1 Q157 mutation (1 point)
      • Type 1-like CALR absent (1 point)
    • Risk status is defined by the points
      • 0 (low), 1 (intermediate - 1), 2 (intermediate - 2), > 3 (high)
  • Absolute monocyte count ≥ 1 × 109/L confers an unfavorable outcome
  • Unfavorable karyotype
    • Complex karyotype, sole abnormality
    • 2 abnormalities that include +8, -7 / 7q-, i(17q), inv(3), -5 / del(5q), 12p- or 11q23.3 rearrangement
  • CALR mutation confers a better prognosis compared to other mutations
Case reports
Treatment
  • Management based on the risk stratification by
    • GIPSS (Genetically Inspired Prognostic Scoring System): uses karyotyping, driver mutations and HMA mutations
    • MIPSS70+ v2.0 (Mutation and Karyotype Enhanced International Prognostic Scoring System) criteria: uses clinical information, karyotyping and mutations
  • Higher risk
    • Eligible for transplantation: allogeneic hematopoietic cell transplantation
    • Ineligible for transplantation: enrollment in a clinical trial or treatment with ruxolitinib or hydroxyurea for relief of symptoms (N Engl J Med 2012;366:787)
  • Lower risk
    • Asymptomatic: observation
    • Symptomatic: treatment with ruxolitinib or hydroxyurea
Clinical images
None
Gross description
No information provided
Gross images
None
Frozen section description
No information provided
Frozen section images
None
Microscopic (histologic) description
  • Bone marrow
    • Prefibrotic stage
      • Hypercellular with large, dysplastic, clustered (loose or tight) megakaryocytes and excess granulocytes
      • Increased reticulin is present around clusters of megakaryocytes; megakaryocytes have aberrant N:C ratios and hyperchromatic, bulbous or irregularly folded nuclei
      • Often bare megakaryocytic nuclei
      • Megakaryocytic features are most useful to distinguish this stage of primary myelofibrosis from essential thrombocythemia
    • Fibrotic phase
      • Hypocellular and diffusely fibrotic bone marrow with atypical streaming megakaryocytes
      • Marrow osteosclerosis with irregular, broad bony trabeculae
      • Markedly dilated sinuses; associated with dry bone marrow taps
  • Spleen: red pulp sinuses contain megakaryocytes, granulocyte precursors, nucleated red cells; may be nodules of extramedullary hematopoiesis
  • References: Int J Lab Hematol 2023:45:59, Leukemia 2022;36:1703
Microscopic (histologic) images

Contributed by Hatem Kaseb, M.D., Ph.D., M.P.H. and AFIP

Atypical megakaryocytes

Fibrotic bone marrow


Blast transformation Blast transformation

Blast transformation

Cell intermediate

Cell intermediate

Virtual slides
None
Cytology description
No information provided
Cytology images
None
Peripheral smear description
  • Leukoerythroblastosis (immature granulocytes and normoblasts in peripheral blood) is common in later phases; also dacrocytes (teardrop erythrocytes)
Peripheral smear images

Contributed by Hatem Kaseb, M.D., Ph.D., M.P.H.

Dacrocytes

Positive stains
Negative stains
No information provided
Flow cytometry description
  • Normal number and phenotype myeloblasts with normal myeloid scatter by CD45 / side scatter
  • Normal CD10 / CD13 / CD16 / CD11b myeloid maturation pattern and all other myeloid markers are normally expressed, hence there is no immunophenotypic evidence of myelodysplasia
  • No evidence for monoclonal B cell lymphoproliferative disease
  • Although no unusual phenotype T cells are identified, surface markers will not routinely detect monoclonal T lymphocytes
  • Reference: Clin Lab Med 2023;43:411
Flow cytometry images

Contributed by Hatem Kaseb, M.D., Ph.D., M.P.H.

Normal myeloid maturation

Electron microscopy description
No information provided
Electron microscopy images
None
Molecular / cytogenetics description
  • No specific genetic marker
  • No BCR::ABL fusion gene (or classify as chronic myeloid leukemia); presence of del(13)(q12-22) or der(6)t(1;6)(q21-23;p21.3) is strongly suggestive of primary myelofibrosis
  • Shows karyotypic abnormalities in up to 50% of cases
    • Frequent abnormalities include del(20q), del(13q), +8, +9 and abnormalities of 1q
  • Unfavorable karyotype (complex karyotype; > 3 abnormalities) (Virchows Arch 2023;482:53)
    • Isolated +8, isolated -7/7q-, i(17q), -5/5q-, 12p-, 11q23 rearrangement or inv(3) and an abnormal karyotype with abnormalities of chromosomes 5, 7, 17 or 12p-
  • Mutations (Am J Hematol 2023;98:801)
    • Driver mutations
      • JAK2 V617F mutation (50 - 60%)
      • CALR mutations (25 - 30%)
      • MPL mutations (5 - 10%)
      • Triple negative for mutations in JAK2, CALR and MPL (8 - 12%)
    • Other mutations
      • Include ASXL1, EZH2, TET2, IDH1 / IDH2, SRSF2 or SR3B1
  • Driver mutational profile is associated with overall survival
    • CALR mutated cases have better survival compared to others
    • Triple negative cases have the shortest survival compared to others
Molecular / cytogenetics images
None
Videos
None
Sample pathology report
  • Bone marrow, biopsy and aspirate:
    • Myeloproliferative neoplasm, favoring prefibrotic primary myelofibrosis (see comment)
    • Comment: The marrow biopsy shows morphologic features of a myeloproliferative neoplasm, in conjunction with thrombocytosis. The major differential diagnosis includes essential thrombocythemia and prefibrotic stage of primary myelofibrosis. Overall, we favor a diagnosis of the latter, due to hypercellular marrow, distribution of megakaryocytes (focal clusters) and markedly dysplastic cytology of megakaryocytes. Recommend correlation with clinical findings (presence of splenomegaly and disease progression) and molecular studies (JAK2, CALR and MPL mutational studies).
    • Complete blood count (9/17/2018), by report: hemoglobin - 11.4 g/dL, mean corpuscular volume - 100 fL, white blood cells - 21.7 K/uL, platelets - 698 K/uL
  • Bone marrow biopsy microscopic description:
    • Hypercellular marrow for age (80% cellular). Megakaryocytes are increased in number, forming focal loose clusters and showing abnormal morphology including hypersegmented nuclei with hyperchromasia. The myeloid:erythroid ratio is markedly increased. Erythroid elements exhibit maturation. Myeloid elements exhibit maturation. Myeloblasts appear to be increased. No lymphoid aggregates are identified. Granulomas are absent. Reticulin stain reveals that reticulin is mildly increased (MF1/3). Trabecular bone is normal.
  • Bone marrow aspirate microscopic description:
    • The marrow aspirate smears are spicular and cellular. Megakaryocytes are increased and show prominent dysmegakaryopoiesis, including hyper and hyposegmented nuclei and increased N:C ratio. The myeloid:erythroid ratio is approximately 4:1. Erythroid maturation is identified. Myeloid maturation is slightly left shifted. Prussian blue iron stain shows # storage iron and no significant increase in ring sideroblasts.
    • Aspirate cell count: A 207 cell count reveals 1% blasts, 15% promyelocytes / myelocytes, 57% maturing granulocyte forms, 20% erythroid forms, 3% lymphocytes, 2% eosinophils, 1% plasma cells, 1% basophils / mast cells and 2% monocytes.
Differential diagnosis
  • Other causes of leukoerythroblastosis or dry taps:
  • Other myeloid disorders:
    • Myeloproliferative neoplasms:
    • Myelodysplastic / myeloproliferative neoplasms:
      • Show hybrid myelodysplastic and myeloproliferative features manifested by at least 1 cytopenia and at least 1 cytosis in blood
      • Myeloid maturation is intact and usually mature cells predominate
      • Exact subtype should be diagnosed based on the specific criteria of the entity
    • Myelodysplastic syndromes are a heterogeneous group of entities that demonstrate:
      • Variable degrees of ineffective hematopoiesis
      • < 20% blasts in marrow
      • Peripheral blood with unexplained and persistent cytopenia(s), < 20% blasts, < 1.0 x 109/L monocytes and dysplasia in 1 or more myeloid lineages
    • Mastocytosis:
      • Shows focal, compact aggregates of mast cells in bone marrow core biopsy
      • Cells can show the following patterns of infiltration: paratrabecular, perivascular or parafollicular
      • Presence of atypical mast cell supports the diagnosis
      • Most cases will harbor a KIT D816V mutation
  • Reactive thrombocythemia:
    • Causes include exercise, allergic reaction, reaction to medications, inflammatory disorders, asplenism, infection, connective tissue disorders, metastatic cancer, lymphoproliferative disorders and iron deficiency
  • Acute myelofibrosis:
    • Subtype of acute myeloid leukemia that shows > 20% blasts in peripheral blood or bone marrow
    • Patients also present with bone marrow fibrosis, fever and pancytopenia
  • Autoimmune myelofibrosis:
    • Generally shows diffuse reticulin fibrosis and an associated autoimmune disease such as rheumatoid arthritis
  • Primary hyperparathyroidism:
Board review style question #1

A 57 year old man presented with fatigue, splenomegaly and hepatomegaly. A complete blood count demonstrated anemia and thrombocytosis. A bone marrow aspirate and biopsy were ordered (shown above). What is the most likely diagnosis?

  1. Acute myeloid leukemia
  2. Chronic myelogenous leukemia
  3. Myelodysplastic syndrome
  4. Primary myelofibrosis
Board review style answer #1
D. Primary myelofibrosis. The symptoms and bone marrow biopsy support the diagnosis of primary myelofibrosis. The biopsy above shows hypercellular bone marrow with granulocytic and atypical megakaryocytic proliferations. Answer A (acute myeloid leukemia) is incorrect because there is no increase in blasts apparent. Answer B (chronic myelogenous leukemia) is incorrect because there is markedly atypical megakaryocytes in clusters, which is not seen in chronic myelogenous leukemia. Answer C (myelodysplastic syndrome) is incorrect because dysplastic megakaryocytes are usually small and hypolobated; they usually do not show clustering.

Comment Here

Reference: Primary myelofibrosis
Board review style question #2
What is the most common genetic mutation in primary myelofibrosis?

  1. ABL
  2. CALR
  3. JAK2
  4. MPL
Board review style answer #2
C. JAK2. Primary myelofibrosis cases harbor a phenotypic driver mutation in JAK2 V617F mutation (50 - 60%), CALR mutations (25 - 30%), MPL mutations (5 - 10%), triple negative for mutations in JAK2, CALR and MPL (8 - 12%). Answers B and D are incorrect because CALR and MPL are less common than JAK2 mutations. Answer A is incorrect because ABL mutations have not been described in primary myelofibrosis.

Comment Here

Reference: Primary myelofibrosis

Refractory cytopenia of childhood (pending)
[Pending]

Severe congenital neutropenia (pending)
[Pending]

Systemic mastocytosis
Definition / general
  • Mastocytoses are a heterogeneous group of disorders characterized by abnormal growth and accumulation of mast cells in 1 or more organ systems
  • Mast cells are cytologically and immunophenotypically abnormal in neoplastic mast cell proliferations
Essential features
  • Systemic mastocytosis is a neoplastic condition in which histomorphologically atypical mast cells involve tissue sites including bone marrow, skin and other tissues
  • Occurs primarily in adults and is strongly associated with c-KIT mutations with the majority being KIT D816V
  • Also associated with myeloid neoplasms
  • Major or minor criteria are required for diagnosis and include identification of atypical spindled mast cell clusters, aberrant expression of CD25, CD2 or CD30, the presence of a KIT mutation or elevated serum tryptase
  • Categories of disease are based upon sites of involvement, related symptoms and laboratory findings
Terminology
  • According to the 2017 WHO classification, mastocytosis is classified into the following categories
    • Cutaneous mastocytosis
      • Urticaria pigmentosa / maculopapular cutaneous mastocytosis
      • Diffuse cutaneous mastocytosis
      • Mastocytoma of skin
    • Systemic mastocytosis
      • Indolent systemic mastocytosis
      • Bone marrow mastocytosis
      • Smoldering systemic mastocytosis
      • Systemic mastocytosis with an associated hematological neoplasm
      • Aggressive systemic mastocytosis
      • Mast cell leukemia
    • Diagnosis of these entities requires correlation with burden of disease (B findings) and cytoreduction requiring (C findings), which indicate organ involvement with and without organ dysfunction, respectively (see Clinical features)
ICD coding
  • ICD-10:
    • D47.02 - systemic mastocytosis
    • C96.21 - aggressive systemic mastocytosis
    • C94.3 - mast cell leukemia
  • ICD-11:
    • 2A21.00 - mast cell leukemia
    • 2A21.0Y - other specified systemic mastocytosis
    • 2A21.0Z - systemic mastocytosis, unspecified
Epidemiology
  • Prevalence estimate of 10 - 13 per 100,000 (Transl Res 2016;174:86, Cancers (Basel) 2021;13:6380)
  • Indolent / smoldering types present at median age of ~50 years; more aggressive types present at older age with a median of ~70 years
  • Pediatric systemic mastocytosis is extremely rare
    • Mastocytoses in pediatric patients are predominantly cutaneous limited (not systemic) with a bimodal incidence of birth to 2 years and over 15 years
    • ~50% of cases are self limited, resolving after onset of puberty
  • M ≈ F; most studies show no significant difference or only slight differences in incidence among men and women (Transl Res 2016;174:86)
  • Familial disease is rare, accounting for ~1 - 2% of all cases (Br J Haematol 2021;193:845)
  • Bone marrow or bone involvement is present in up to 90% of systemic mastocytosis cases
Sites
Pathophysiology
  • Clonal expansion of cells with mast cell differentiation having c-KIT mutation
  • Mast cell proliferation and degranulation results in destructive lesions
Etiology
  • Generally thought to differentiate from a CD34+, CD117 / KIT+, CD13+ hematopoietic progenitor cells or a mast cell committed precursor
  • Proto-oncogene c-KIT (Leukemia 2015;29:1223)
    • Chromosome 4
    • Codes for tyrosine protein kinase KIT, CD117 or mast / stem cell growth factor receptor (SCFR)
    • KIT is dimerized by stem cell growth factor, ultimately inducing cell survival, proliferation and differentiation
    • In mastocytosis, c-KIT mutation leads to stem cell growth factor independent activation of KIT
    • The most common c-KIT mutation is KIT D816V, while cutaneous mastocytosis has more frequent non-D816V codon 816 mutations
    • Mast cell release of mediators including histamine may contribute to increased osteoblast activity, activation of osteoclasts and formation of sclerotic lesions
Clinical features
  • Course is variable from benign and self limited to aggressive with a variety of symptoms which differ based on disease subtype (Am J Hematol 2021;96:508)
  • Most commonly affected sites are skin (urticaria pigmentosa) and bone marrow
  • 4 categories of systemic mastocytosis presenting symptoms
    • Constitutional: weight loss, fever and diaphoresis, fatigue
    • Skin manifestations: urticaria and pruritus, flushing, dermatographism
    • Mediator related systemic events: abdominal pain, headache, hypotension, syncope
    • Musculoskeletal symptoms: osteoporosis, osteopenia, bone pain and fracture, arthralgia and myalgia
  • Other presentation findings may include organ impairment associated with infiltration, especially in aggressive systemic mastocytosis and mast cell leukemia
  • Symptoms related to mast cell degranulation with associated increased serum tryptase may be diagnosed as mast cell activation syndrome
    • Mast cell activation syndrome is not considered a subtype of systemic mastocytosis and may be diagnosed in the absence of mastocytosis if criteria are not fulfilled
    • In the absence of all required diagnostic criteria, a clonal mast cell population with KIT D816V mutation or aberrant surface CD25 warrants a diagnosis of monoclonal mast cell activation syndrome
  • Patients with mediator related symptoms and systemic mastocytosis are designated as having primary (clonal) mast cell activation syndrome, which has diagnostic and therapeutic implications
  • Systemic mastocytosis with an associated hematological neoplasm (AHN): associated and often clonally related secondary myeloid neoplasm, most commonly chronic myelomonocytic leukemia
  • C (cytoreduction requiring) findings:
    • Neoplastic mast cell infiltration impacting bone marrow function is defined as ≤ 1 cytopenia, absolute neutrophil count < 1.0 x 109/L, hemoglobin < 10 g/dL or platelet count < 1.0 x 109/L
    • Abnormal liver function with ascites and elevated liver enzymes with or without hepatomegaly, cirrhosis, portal hypertension
    • Large osteolytic lesions (≥ 20 mm) with or without bone pain, pathologic fractures (excepting osteoporosis related fractures)
    • Palpable splenomegaly with hypersplenism with or without weight loss, hypalbuminemia
    • Gastrointestinal mast cell infiltrates causing malabsorption with or without weight loss
  • B (burden of disease) findings:
    • > 30% of mast cells on bone marrow biopsy and serum total tryptase > 200 ng/mL or KIT D816V mutation with VAF ≥ 10% in bone marrow cells or peripheral blood leukocytes (Leukemia 2022;36:1703)
      • International Consensus Classification (ICC): high mast cell burden, > 30% of bone marrow cellularity by mast cell aggregates (assessed on bone marrow biopsy) and serum tryptase > 200 ng/mL)
    • Dysplastic changes or myeloproliferation in nonmast cell lineages, without meeting criteria for associated hematological neoplasm, with normal or only slightly abnormal blood counts
      • ICC: cytopenia (not meeting criteria for C findings) or cytosis excluding reactive causes and other myeloid neoplasm criteria are not met
    • Hepatomegaly without liver dysfunction, palpable splenomegaly without hypersplenism or lymphadenopathy
    • Additional provisional B finding proposed for the upcoming WHO fifth edition update
      • KIT D816V mutation with variant allele fraction ≥ 10% in bone marrow cells or peripheral blood leukocytes (Leukemia 2022;36:1703)
Diagnosis
  • 2017 WHO classification diagnostic criteria for systemic mastocytosis; must demonstrate major criterion and at least 1 minor criterion or ≥ 3 minor criteria
  • ICC diagnostic criteria are very similar to the WHO 5th edition criteria; variation from WHO 5th edition are noted
    • Major criterion
      • Multifocal dense mast cell infiltrates detected in sections of bone marrow or extracutaneous organs; infiltrate is defined as ≥ 15 mast cells in aggregate (ICC specifies that the mast cells must express tryptase or CD117)
    • Minor criteria
      • > 25% of mast cells in bone marrow biopsy are spindled, have abnormal morphology of bone marrow aspirate / tissue infiltrate mast cells are immature or atypical
      • Detection of KIT mutation in blood, bone marrow or other nonskin tissue (activating point mutation at codon 816 or any KIT mutation conferring ligand independent activation (Leukemia 2022;36:1703)
      • Expression of one or more of the following antigens in the mast cell population in addition to normal mast cell markers (by flow cytometry or IHC): CD25, CD2, CD30 (Leukemia 2022;36:1703)
      • Persistent serum total tryptase > 20 ng/mL (except in context of associated myeloid neoplasm)
  • Diagnostic criteria for variants of systemic mastocytosis (in addition to general criteria above)
    • Indolent systemic mastocytosis
      • No C findings indicative of organ involvement
      • No evidence of associated hematological neoplasm
      • Low mast cell burden
      • Skin lesions are almost always present
    • Bone marrow mastocytosis
      • Similar criteria as indolent variant with bone marrow involvement
      • No skin lesions
      • Proposed fifth edition WHO criteria refinement includes tryptase < 125 ng/mL (Leukemia 2022;36:1703)
    • Smoldering systemic mastocytosis
      • ≥ 2 B findings in absence of C findings
      • No evidence of associated hematological neoplasm
      • High mast cell burden
      • Does not meet mast cell leukemia criteria
    • Systemic mastocytosis with an associated hematological neoplasm
      • Associated hematological neoplasm including myelodysplastic syndrome, myeloproliferative neoplasm, acute myeloid leukemia, lymphoma or other WHO classified distinct hematological neoplasm
    • Aggressive systemic mastocytosis
      • ≥ 1 C finding
      • Does not meet mast cell leukemia criteria
      • Skin lesions are usually absent
    • Mast cell leukemia
      • Diffuse infiltration of bone marrow by dense aggregates of atypical, immature mast cells
      • ≥ 20% mast cells in bone marrow aspirate
      • Classic: ≥ 10% white blood cell count is mast cells in peripheral blood
      • Aleukemic: < 10% peripheral blood mast cells (more common than classic)
        • Skin lesions are usually absent
  • Additional morphologic pattern proposed for the upcoming WHO fifth edition update:
    • Well differentiated systemic mastocytosis (Leukemia 2022;36:1703)
      • May occur in any subtype with mast cells lacking atypical histomorphologic characteristics, may not have expression of CD25, CD2 or CD30, predominantly lacking KIT D816V mutation (J Allergy Clin Immunol 2016;137:168)
      • High percentage of bone marrow mast cells
      • Predominantly in women, childhood onset and familial aggregation
Laboratory
  • Complete blood count (CBC) may demonstrate anemia, thrombocytopenia, thrombocytosis, leukocytosis or eosinophilia
  • Serum tryptase > 20 ng/mL (fluoro immunoenzymatic assay [Pharmacia, Uppsala, Sweden]) is a minor diagnostic criterion (Arch Pathol Lab Med 2007;131:784)
  • Elevated histamine metabolites in 24 hour urine sample may be present in patients with cutaneous lesions
Radiology description
  • Skeletal involvement reported in 70 - 90% of patients (Annu Rev Med 2004;55:419)
  • Osteoporosis reported in up to 40% of patients; dual energy Xray absorptiometry (DEXA) of hip and spine should be performed (Cancers (Basel) 2021;13:6380)
  • Most common skeletal findings:
    • Multiple focal sclerotic axial and appendicular bone lesions
    • Diffuse, circumscribed, sclerotic foci with alternating zones having normal or reduced density in axial skeleton, ribs, humerus, femur
Prognostic factors
  • Prognosis is linked to disease subclassification (Am J Hematol 2021;96:508)
    • Favorable prognosis
      • Indolent / smoldering systemic mastocytosis
        • 3% progression to more aggressive forms in study with median follow up of > 10 years; life expectancy is approximately normal
    • Poor prognosis
      • Aggressive systemic mastocytosis
        • Overall median survival of ~40 months
      • Systemic mastocytosis with an associated hematological neoplasm
        • Overall median survival of ~24 months (longer in patients with myeloproliferative neoplasms, shorter in those with chronic myelomonocytic leukemia, myelodysplastic syndrome or acute leukemia)
      • Mast cell leukemia
        • ~2 year overall median survival
  • Prognosis is linked to independent risk factors
    • Age: > 60 years
    • Platelet count: < 150 x 109/L
    • Serum alkaline phosphatase above reference interval
    • Presence of adverse mutation in ASXL1, RUNX1, SRSF2 or NRAS
    • Increased plasma IL2Rɑ / CD25 confers worse prognosis in indolent SM
Case reports
  • 33 year old man with abdominal pain and blood per rectum (Ann Gastroenterol 2019;32:208)
  • 50 year old man with foamy mastocytosis who presented with flushing, fever, intestinal disorder, skin lesions and splenomegaly (Blood 2018;131:586)
  • 62 year old man with systemic mastocytosis and essential thrombocythemia who presented with thrombocytosis and hepatosplenomegaly with cutaneous papules (Medicina (Kaunas) 2019;55:528)
  • 83 year old man with systemic mastocytosis and multiple myeloma who presented with hematuria and bone pain (Blood 2018;132:1545)
Treatment
  • Indolent disease may not require treatment or treatment may be aimed at symptomatic management, including antihistamines, leukotrienes, mast cell stabilizers, proton pump inhibitors, calcium vitamin D, bisphosphonates; epinephrine may be needed for patients having risk of anaphylaxis (Am J Hematol 2021;96:508)
  • Avoidance of exposures that aggravate mast cell degranulation including temperature extremes, mechanical irritation, medications and alcohol
  • Aggressive disease requiring cytoreductive therapy may be treated with the following agents
    • Interferon alpha is often paired with prednisone; may have severe side effects including myelosuppression
    • 2-chlorodeoxyadenosine issued as a second line therapy for interferon alpha refractory disease; may have severe side effects including myelosuppression, lymphopenia and risk of opportunistic infection
    • Cladribine purine analog with mechanism independent of tyrosine kinase inhibition
    • Tyrosine kinase inhibitors
      • Avapritinib targets PDFRɑ and KIT (Int J Mol Sci 2021;22:2983)
      • Midostaurin targets FLT3, KIT D816V mutation (Blood 2020;135:1365)
      • Imatinib targets BCR::ABL, wild type KIT
      • Nilotinib targets BCR::ABL, wild type KIT
      • Dasatinib dual SRC / ABL inhibitor
    • Chemotherapy regimens employed in acute myeloid leukemia
    • Allogeneic stem cell transplantation in rare cases
Microscopic (histologic) description
  • Neoplastic mast cells may be spindled to round / oval (similar to normal mast cells) with variable cytoplasmic granularity
  • Spindled mast cell aggregates may invoke a streaming-like appearance
  • Bone marrow aspirate sampling of mast cells may be limited due to associated reticulin or collagen fibrosis
  • Characteristically, mast cells have a light blue-gray cytoplasm; granules may be inconspicuous on H&E sections but are more prominent with regional distribution on Wright-Giemsa stained bone marrow aspirate and peripheral blood smears
  • Basophilic to amphipathic cytoplasmic granules may obscure mast cell nuclei on Wright-Giemsa stain
  • On H&E sections, round mast cells with central nuclei may give a fried egg-like appearance at low power, raising a differential that includes hairy cell leukemia (Arch Pathol Lab Med 2007;131:784)
  • Mast cell nuclei may have 1 to a few prominent nucleoli and relatively smooth nuclear chromatin
  • Mast cell aggregates may be intermixed with lymphohistiocytic proliferations, often with interspersed eosinophils and plasma cells
  • Atypical mast cell features include spindle morphology, hypogranulation and atypical nuclear borders
  • Bone may demonstrate osteosclerosis, increased trabeculation volume, increased cortical thickness and narrowing of marrow spaces
Microscopic (histologic) images

Contributed by Mark Girton, M.D. and AFIP
Marrow mast cell cluster

Marrow mast cell cluster

Intermixed with lymphoid aggregate

Intermixed with lymphoid aggregate

Aspirate with mast cells Aspirate with mast cells Aspirate with mast cells

Aspirate with mast cells

Mast cell cluster

Mast cell cluster


Hypercellular bone marrow biopsy

Hypercellular bone marrow biopsy

Bone marrow biopsy with marrow replacement

Bone marrow biopsy with marrow replacement

Cytologic spectrum

Cytologic spectrum

CD117 positive mast cells

CD117 positive mast cells

CD25 positive mast cells

CD25 positive mast cells


CD2 positive mast cells

CD2 positive mast cells

CD30 positive mast cells

CD30 positive mast cells

CD117 positive mast cells

CD117 positive mast cells

Mast cell tryptase positive

Mast cell tryptase positive

CD25 positive mast cells

CD25 positive mast cells


Perifollicular distribution

Perifollicular distribution

Characteristic sclerosis

Characteristic sclerosis

Mast cell infiltrate

Mast cell infiltrate

Delicate collagen fibrils and eosinophils

Delicate collagen fibrils and eosinophils

Mast cell staining

Mast cell staining


Interfollicular pattern

Interfollicular pattern

Diffuse involvement

Diffuse involvement

Ovoid nuclei with eosinophilic cytoplasm

Ovoid nuclei with eosinophilic cytoplasm

Spindle shaped mast cells

Spindle shaped mast cells

Spindly configuration

Spindly configuration

Peripheral smear images

Contributed by Mark Girton, M.D.
Peripheral blood mast cell Peripheral blood mast cell Peripheral blood mast cell

Peripheral blood mast cell

Positive stains
Negative stains
Flow cytometry description
  • Neoplastic mast cell populations may be difficult to sample because of associated reticulin or collagen fibrosis; therefore, the population of interest may be underrepresented in flow cytometry assays
  • Antibodies against CD45, CD13, CD33, CD117, CD25 and CD2 are typically positive in neoplastic mast cells, while CD34 is generally expected to be negative, except in mast cell precursors
  • Nonneoplastic mast cells lack expression of CD25 and CD2
  • CD38 expression may be seen in normal mast cell precursors
Molecular / cytogenetics description
Sample pathology report
  • Blood, peripheral smear:
    • Unremarkable peripheral blood smear (see comment)
    • Comment: Microscopic examination of the peripheral blood smear demonstrates a normal red blood cell count with erythrocytes, which are morphologically unremarkable. The white blood cell count is normal and leukocytes are morphologically unremarkable. Platelets are normal in number and morphology.

  • Bone marrow, left, aspirate smear:
    • Trilineage hematopoiesis with occasional mast cells present (see comment)
    • Comment: Microscopic examination of the bone marrow aspirate demonstrates a spicular and cellular specimen which is adequate for evaluation. There is trilineage hematopoiesis. Erythroid precursors demonstrate normoblastic maturation. Myeloid precursors demonstrate the full spectrum of maturation. The myeloid to erythroid (M:E) ratio is normal at 2.3:1. Megakaryocytes are present and appear normal in number and morphology. There are occasional mast cells present, some of which demonstrate a spindled morphology. There is no increase in blasts. An iron stain demonstrates the presence of storage iron and no significant increase in ring sideroblasts.

  • Bone marrow, left, core biopsy:
    • Limited subcortical specimen showing trilineage hematopoiesis with involvement by systemic mastocytosis (see comment)
    • Comment: Microscopic examination of the core biopsy demonstrates a subcortical bone marrow specimen limited sampling, which precludes evaluation of the cellularity. There is trilineage hematopoiesis and small, ill defined lymphoid aggregates. Immunohistochemical staining for CD117 and mast cell tryptase demonstrate clusters of spindle shaped mast cells. Staining for CD25 is positive and staining for CD2 is negative. The lymphoid aggregates are composed of a mixture of B cells and T cells as demonstrated by CD20 and CD3 positivity, respectively. The myeloid to erythroid ratio appears normal. Megakaryocytes are present and appear normal in number and morphology. A PAS stain highlights the myeloid elements and megakaryocytes. An iron stain shows the presence of storage iron. A reticulin stain is not evaluable due to the limited specimen size.

  • Bone marrow, left, clot section:
    • Normocellular bone marrow with trilineage hematopoiesis and involvement by systemic mastocytosis (see comment)
    • Comment: Microscopic examination of the clot preparation demonstrates a cellular specimen that is adequate for evaluation. The clot section appears normocellular. There is trilineage hematopoiesis. The myeloid to erythroid ratio appears normal. Megakaryocytes are present and appear normal in number with unremarkable morphology. There are multiple lymphoid aggregates present with mast cell clusters having focal areas of scattered eosinophils. Immunohistochemical staining for CD117 and mast cell tryptase demonstrate clusters of spindle shaped mast cells. Staining for CD25 is positive and staining for CD2 is negative. The lymphoid aggregates are a mixture of B cells and T cells as demonstrated by CD20 and CD3 positivity, respectively. There is no increase in blasts. An iron stain shows the presence of storage iron and no ring sideroblasts.
    • In summary, the patient's specimen demonstrates an overall normocellular bone marrow with trilineage hematopoiesis and involvement by systemic mastocytosis. Correlation with molecular and genetic studies is recommended.
Differential diagnosis
Board review style question #1
Regarding systemic mastocytosis, which of the following is the most commonly associated hematological neoplasm (AHN)?

  1. Acute myeloid leukemia (AML)
  2. Chronic myelomonocytic leukemia (CMML)
  3. Chronic myelocytic leukemia (CML)
  4. Plasma cell myeloma
  5. Polycythemia vera (PV)
Board review style answer #1
B. Chronic myelomonocytic leukemia (CMML). Associated hematological neoplasms are reported to include CMML, non-Hodgkin lymphoma, refractory anemia with ring sideroblasts and thrombocytosis, essential thrombocythemia, multiple myeloma, monoclonal gammopathy of undetermined significance, myelodysplastic syndrome, myelofibrosis and acute myeloid leukemia. CMML is the most commonly reported associated hematological neoplasm (Am J Hematol 2016;91:692).

Comment Here

Reference: Systemic mastocytosis
Board review style question #2
Which of the following features fulfills a fifth edition WHO classification minor diagnostic criterion for systemic mastocytosis?

  1. Identification of a BRAF V600E mutation in blood, bone marrow aspirate or nonskin tissue
  2. Mast cell expression of CD25, with or without CD2, in combination with normal mast cell markers
  3. Mast cells expressing CD117 or tryptase by immunohistochemical staining
  4. Multiple dense mast cell groups as defined by ≥ 15 mast cells per aggregate in bone marrow or other extracutaneous organ
Board review style answer #2
B. Mast cell expression of CD25, with or without CD2, in combination with normal mast cell markers. The fifth edition WHO’s diagnostic criteria for systemic mastocytosis must demonstrate major criterion and at least 1 minor criterion or ≥ 3 minor criteria. Major criterion includes: multifocal dense mast cell infiltrates detected in sections of bone marrow or extracutaneous organs (infiltrate is defined as ≥ 15 mast cells in aggregate). International Consensus Classification (ICC) specifies that the mast cells must express tryptase or CD117. Minor criteria include: > 25% of mast cells in bone marrow biopsy are spindled, have abnormal morphology of bone marrow aspirate / tissue infiltrate mast cells are immature or atypical; detection of KIT mutation in blood, bone marrow or other nonskin tissue (activating point mutation at codon 816); expression of one or more of the following antigens in the mast cell population in addition to normal mast cell markers (by flow cytometry or IHC): CD25, CD2, CD30; and persistent serum total tryptase > 20 ng/mL (except in context of associated myeloid neoplasm; may be adjusted in this context according to ICC). Additional provisional minor criteria proposed for the fifth edition WHO include CD30 expression and any KIT mutation conferring ligand independent activation (Leukemia 2022;36:1703).

Comment Here

Reference: Systemic mastocytosis

T lymphoblastic lymphoma / leukemia
Definition / general
  • Neoplasm of T lineage lymphoblasts which may form lymphomatous masses, involve blood and bone marrow (Stanford School of Medicine: Precursor T Lymphoblastic Leukemia / Lymphoma [Accessed 13 April 2018])
  • Also called pre T cell acute lymphocytic leukemia / lymphoma (pre-T ALL), T lymphoblastic leukemia / lymphoma (T LBL)
  • Teens and young men (older than B ALL)
  • Most cases begin after birth (Blood 2007;110:3036)
  • T ALL versus T LBL: T ALL has more immature phenotype, CD47 expression, no 11q23 rearrangement, different gene expression profile and may derive from T cell progenitor of bone marrow; T LBL is derived from thymocytes (Pediatr Blood Cancer 2006;47:130, Leuk Lymphoma 2007;48:1745)
  • T ALL constitutes 15% of childhood and 20 - 25% of adult ALL cases
  • T LBL constitutes 85 - 95% of LBL, usually presents as mediastinal mass with no / minimal marrow involvement
  • CNS involvement if untreated
  • T LBL frequently presents with mass in anterior mediastinum, rapid growth, respiratory emergency, pleural effusion
  • Younger (age 16 - 60 years) patients compared to older (61+ years) patients have more hepatosplenomegaly, present with mediastinal mass and lymphadenopathy; myeloid antigens and lineage inappropriate gene rearrangements are less common (Am J Clin Pathol 2002;117:252)
Diagrams / tables

Images hosted on other servers:
Missing Image

Rearrangements involving T cell receptor genes

Diagnosis
  • T ALL if lymphoblasts are 25% or more of marrow cells; T LBL otherwise
Prognostic factors
Treatment
  • Chemotherapy cures 60%
  • Patients have earlier relapse, induction failure and isolated CNS relapse compared to pre-B ALL
Microscopic (histologic) description
  • Similar to B cell disease; scant cytoplasm, delicate chromatin, indistinct nucleoli, convoluted nuclear membrane and grooves
  • Frequent mitotic figures; starry sky pattern produced by interspersed benign macrophages
  • Usually features of FAB L1 or L2; pattern in marrow is usually interstitial
  • Lymph nodes: complete architectural effacement or partial involvement with paracortical infiltrate with germinal center sparing
  • Thymus: replacement of normal parenchyma
  • Occasionally eosinophilia and myeloid hyperplasia with variable t(8;13)(p11.2;q11-22) involving FGFR1 gene; some develop myeloid malignancy (MDS, AML or myeloid sarcoma)
Microscopic (histologic) images

AFIP images
Missing Image

Bone marrow biopsy

Missing Image

Focal paranuclear acid phosphatase staining

Peripheral smear images

AFIP images
Missing Image

Markedly elevated leukocyte count

Missing Image

L2 type (blood smears)

Positive stains
Negative stains
  • CD19, CD20; double negative CD4 / CD8 (40%)
  • Note: ALL with aberrant myeloid antigen expression is correct name, not biphenotypic leukemia
Molecular / cytogenetics description

Genes:
  • MYC (8q24.1)
  • TAL1 (1p32)
  • RBTN1 (LMO1) (11p15)
  • RBTN2 (LMO2) (11p13)
  • HOX11 (TLX1) (10q24)
  • HOX11L2 (TLX3) (5q35)
  • LYL1 (19p13)
  • LCK (1p34.3-35)
Molecular / cytogenetics images

Images hosted on other servers:
Missing Image

FISH: t(5;14)(q35;q32)

Differential diagnosis

TCF3::HLF (pending)
[Pending]

TEMPI syndrome (pending)
[Pending]

Telomere biology disorders including dyskeratosis congenita (pending)
[Pending]

Therapy related B ALL
Definition / general
  • B lymphoblastic leukemia / lymphoma arising after cytotoxic or radiation therapy for a prior malignancy

Notes:
  • Therapy related T lymphoblastic leukemia is not discussed in this topic; rare reports of such cases have been published but understanding of this entity remains limited
  • Secondary B lymphoblastic leukemia / lymphoma is not discussed in this topic
    • The term secondary B lymphoblastic leukemia / lymphoma is preferred when the patient has a history of prior malignancy but has not received cytotoxic or radiation therapy
Essential features
  • B lymphoblastic leukemia / lymphoma arising after cytotoxic or radiation therapy for a prior malignancy
  • Most common prior malignancy is breast cancer, followed by multiple myeloma and lymphoma
  • No distinct morphologic or immunophenotypic features but is commonly associated with KMT2A rearrangement, del(5 / 5q), del(7 / 7q), hypodiploidy, TP53 mutation and BCR::ABL1 rearrangement
Terminology
  • Therapy related B lymphoblastic leukemia / lymphoma (t-B ALL)
ICD coding
  • ICD-10: C91.00 - acute lymphoblastic leukemia not having achieved remission
Epidemiology
Sites
Pathophysiology
Clinical features
  • History of cytotoxic or radiation for prior malignancy
  • Older age at diagnosis (fifth - sixth decade)
  • Patients may present with cytopenia(s), fatigue, weight loss and fever or may be asymptomatic
Diagnosis
  • Examination of blood or bone marrow
Laboratory
  • Complete blood cell count (CBC):
    • Patients present with lower white blood cell (WBC) counts than patients with de novo disease (Br J Haematol 2022;196:963)
      • WBC count ranges widely (may be decreased, normal or increased)
      • Anemia, thrombocytopenia or pancytopenia are frequently encountered
Prognostic factors
  • Previously mentioned cytogenetic abnormalities, as well as prior exposure to mutagenic agents, confer a worse prognosis when compared to de novo disease
  • KMT2A rearrangement is associated with faster time to development of t-B ALL, as well as worse prognosis
  • Autologous hematopoietic stem cell transplant may improve survival (Blood Rev 2019;37:100584, Br J Haematol 2022;196:963)
Case reports
  • 43 and 64 year old men with B ALL arising during lenalidomide maintenance (Am J Clin Pathol 2020;154:816)
  • 53 and 69 year old women with B ALL arising in the context of lenalidomide maintenance (Case Rep Hematol 2018;2018:9052314)
  • 60 year old woman with B ALL, 3 years after treatment for multiple myeloma with cytotoxic agents and autologous stem cell transplant with high dose melphalan conditioning (Leukemia 2005;19:299)
  • 68 year old Caucasian man and 83 year old woman, the first case series to report development of t-B ALL during lenalidomide therapy for myelodysplastic syndrome with isolated del(5q) (Int J Clin Exp Pathol 2011;4:322)
  • 69 year old man with B ALL with mutated EZH2, arising 27 months after initiation of lenalidomide therapy for myelodysplastic syndrome with isolated del(5q) (Medicine (Baltimore) 2019;98:e14011)
  • 72 year old man with B ALL, 6 years after treatment with cytotoxic agents and lenalidomide maintenance therapy for multiple myeloma (Ann Clin Lab Sci 2013;43:176)
  • 74 year old woman with B ALL, 56 months after treatment for AL amyloidosis with lenalidomide (Int J Clin Exp Pathol 2014;7:2683)
  • 80 year old man with B ALL, > 20 years after treatment with alkylating agent (chlorambucil) for Waldenström macroglobulinemia (Leukemia 2004;18:1433)
  • Early case series of B ALL arising in the context of lenalidomide maintenance (Hematol Oncol 2017;35:130)
Treatment
  • Intensive chemotherapy induction regimens, such as R hyper CVAD (rituximab, cyclophosphamide, vincristine, doxorubicin and dexamethasone)
  • Allogeneic hematopoietic stem cell transplant
Microscopic (histologic) description
  • Peripheral blood:
    • Circulating blasts are present and can range from rare to numerous
    • Blasts have irregular nuclei, variably dispersed chromatin, inconspicuous nucleoli and basophilic vacuolated cytoplasm
  • Bone marrow:
    • Aspirate smears and touch preparations contain a variable percentage of blasts (rare to numerous)
    • Core biopsy and clot sections show variable cellularity (hypocellular, normocellular or hypercellular)
    • Blasts are increased and are present in sheets / clusters or as heterogeneously distributed, clusters of immature cells in the interstitium (Am J Clin Pathol 2020;154:816)
    • Background trilineage hematopoiesis is variably diminished
  • References: Am J Clin Pathol 2020;154:816, Jaffe: Hematopathology, 2nd Edition, 2016
Microscopic (histologic) images

Contributed by Jesse Manuel Jaso, M.D.

Bone marrow with increased blasts

CD34 highlighting clusters of blasts

Positive stains
Flow cytometry description
Molecular / cytogenetics description
  • Shows several genetic abnormalities that are also seen in therapy related myeloid neoplasms, suggesting a similar pathogenesis (Blood Rev 2019;37:100584)
  • KMT2A (formerly MLL) rearrangement:
    • Most frequently identified genetic abnormality
    • Associated with patients treated for early stage breast cancer with alkylating agents or topoisomerase inhibitors
    • Shorter latency interval than other forms of t-B ALL (Blood Rev 2019;37:100584)
  • Hypodiploidy:
    • Hypodiploidy is a common finding, often with deletion of chromosomes 5 / 5q, 7 / 7q, 13 / 13q, 17 and others
  • Low hypodiploid, near triploid karyotype may be seen in t-B ALL (Br J Haematol 2022;196:963)
    • Previously described in pediatric patients and associated with aggressive disease (Genes Chromosomes Cancer 2014;53:524)
    • Subset of B ALL with low hypodiploidy (31 - 39 chromosomes) may undergo aberrant endoduplication, resulting in a seemingly high hyperdiploid (51 - 65) or near triploid (66 - 79) karyotype on conventional cytogenetic analysis
    • Copy number analysis (CNA) can identify this masked hypodiploidy, which is also associated with P53 mutation and loss of chromosome (7p12) (IKZF1 locus), often concurrently (Genes Chromosomes Cancer 2014;53:524)
      • IKZF1 encodes Ikaros, a transcription factor involved in B cell development, among other functions
    • Relationship between these cytogenetic and molecular findings and t-B ALL pathogenesis remains incompletely understood
    • Pediatric patients with this karyotype should also be screened for Li-Fraumeni and other inherited disorders
  • BCR::ABL1 rearrangement:
    • Seen in a subset of cases
    • Incidence and role in t-B ALL pathogenesis is incompletely understood
Sample pathology report
  • Bone marrow, aspirate, clot, core biopsy and peripheral blood:
    • Therapy related B lymphoblastic leukemia / lymphoma (see comment)
    • Comment: The bone marrow shows increased blasts (***% by manual differential; *** by immunohistochemistry for CD34 / TdT) with atypical morphology and an aberrant B lymphoblast immunophenotype by flow cytometry.
    • The findings, in combination with the history of cytotoxic / radiation therapy for prior malignancy (list malignancy) are consistent with therapy related B lymphoblastic leukemia / lymphoma. Correlation with cytogenetic studies is recommended. If clinically indicated, molecular testing for germline mutations should be considered.
Differential diagnosis
Additional references
Board review style question #1
A 65 year old woman presents with a new diagnosis of B lymphoblastic leukemia / lymphoma. She has a history of breast cancer 10 years prior, which was treated with alkylating agents and topoisomerase inhibitors. Which of the following genetic abnormalities is most likely to be identified?

  1. BCR::ABL1 rearrangement
  2. CREBBP mutation
  3. ETV6::RUNX1 mutation
  4. IKZF1 mutation
  5. KMT2A rearrangement
Board review style answer #1
E. KMT2A rearrangement. Therapy related B lymphoblastic leukemia with KMT2A rearrangement is frequently associated with exposure to alkylating agents and topoisomerase inhibitors as treatment for prior breast cancer.

Comment Here

Reference: Therapy related B ALL
Board review style question #2


A 20 month old patient presents with lethargy and fever. CBC shows pancytopenia. A bone marrow examination is performed (see image 1 for bone marrow aspirate and image 2 for bone marrow core biopsy with CD34 immunostain). Flow cytometry shows an aberrant B lymphoblast population (15% of total events). Molecular genetic studies identify a mutation in TP53. Which of the following statements is the most correct?

  1. A concurrent mutation in KMT2A is likely to be identified
  2. Conventional karyotype and copy number analysis studies are not indicated in these cases
  3. Germline molecular testing and geneticist consultation should be considered
  4. The disease is associated with in utero exposure to alkylating agents
  5. The molecular findings imply a good prognosis
Board review style answer #2
C. Germline molecular testing and geneticist consultation should be considered. Hypodiploidy with concurrent TP53 mutation may be seen in children and young patients with B lymphoblastic leukemia / lymphoma and a masked hypodiploid karyotype; however, testing for germline mutation of TP53 and other genes should be considered to rule out the presence of an inherited disorder (e.g., Li-Fraumeni) that predisposes to multiple cancers.

Comment Here

Reference: Therapy related B ALL

Therapy related myeloid neoplasm (therapy related AML)
Definition / general
  • Therapy related myeloid neoplasm refers to cases of acute myeloid leukemia (AML), myelodysplastic syndrome (MDS) and myelodysplastic / myeloproliferative neoplasms (MDS / MPN) occurring as a late complication of cytotoxic chemotherapy or radiation therapy
  • Established cases of MPN, MDS or MDS / MPN progressing into AML should not be regarded as therapy related but rather as secondary AML
Essential features
  • Occurs as a late complication of cytotoxic therapy or radiation therapy administered for neoplastic and nonneoplastic conditions
  • Can present as either a chronic phase characterized by progressive bone marrow failure (i.e. t-MDS or t-MDS / MPN) or overt leukemic phase (i.e. t-AML)
  • Most common etiologies are alkylating agents, topoisomerase II inhibitors and ionizing radiation therapy; other agents are less commonly implicated
  • Diagnosed and classified using analogous criteria to the corresponding de novo diseases
  • Prognosis is generally poor, compared to the corresponding de novo malignancies
Terminology
  • Therapy related myelodysplastic syndrome (t-MDS); therapy related myelodysplastic / myeloproliferative neoplasm (t-MDS / MPN); therapy related acute myeloid leukemia, alkylating agent related; therapy related acute myeloid leukemia (t-AML)
ICD coding
  • ICD-O: 9920/3 - therapy related myeloid neoplasm
Epidemiology
Pathophysiology
  • t-MDS, t-MDS / MPN and t-AML occur as the consequence of acquired somatic alterations induced by cytotoxic or radiation therapy
  • Alterations occur within hematopoietic stem cells and progenitor cells and confer a proliferative or survival advantage
  • Alkylation produces inaccurate base pairing during replication and breaks in the DNA double helix as the alkylated bases are repaired (Mutat Res 1996;355:41)
  • Topoisomerase II inhibitors cause chromosomal breakages which favor the development of chromosomal translocations (Chem Res Toxicol 1993;6:585)
  • As only a minority of patients treated eventually develop a therapy related myeloid neoplasm, other factors may contribute to the pathogenesis
  • Mutations in DNA damage sensing or repair genes (e.g. BRCA1/2 or TP53) or polymorphisms in genes affecting drug metabolism or DNA repair mechanisms are the most common secondary events (Proc Natl Acad Sci U S A 2001;98:11592, Cancer 2016;122:304, Leukemia 2007;21:1413, JAMA 2011;305:1568)
Etiology
  • t-MDS, t-MDS / MPN and t-AML develop mostly following the exposure to (Swerdlow: WHO Classification of Tumours of Haematopoietic and Lymphoid Tissues, 4th Edition, 2017):
    • Alkylating agents: melphalan, cyclophosphamide, nitrogen mustard, chlorambucil, busulfan, carboplatin, cisplatin, dacarbazine, procarbazine, carmustine, mitomycin C, thiotepa, lomustine
    • Topoisomerase II inhibitors: etoposide, teniposide, doxorubicin, daunorubicin, mitoxantrone, actinomycin
    • Large field ionizing radiation therapy
    • Antimetabolites: thiopurines, mycophenolate mofetil, fludarabine
    • Antitubulin agents (often in combination with other agents): vincristine, vinblastine, paclitaxel, docetaxel
  • Other agents (hydroxyurea, radioisotopes, L-asparaginase, purine analogues, mycophenolate mofetil) possibly involved but their primary role is unknown
Clinical features
  • Majority of t-MDS, t-MDS / MPN and t-AML develop within 10 years after the initial exposure
  • Cases related to alkylating agents:
    • Represent the majority of cases
    • Occur 5 - 10 years after initial exposure
    • Are characterized by a chronic phase of bone marrow failure and progressive cytopenia(s) which correspond to t-MDS
  • Cases related to topoisomerase II inhibitors:
    • Represent a minority of the cases
    • Occur after a short latent period (< 5 years)
    • Present as overt t-AML, without a preceding chronic phase
  • If latency period over 10 years, the hematological malignancy may not be related to therapy (J Clin Oncol 2015;33:3641)
  • Signs and symptoms are related to the presence and severity of cytopenia(s) (i.e. anemia, thrombocytopenia, leukopenia)
Diagnosis
  • Complete blood count with differentials and morphologic review of blood smears to assess for cytopenia(s), dysplasia and blast count
  • Bone marrow aspirate and biopsy with features of either MDS, MDS / MPN or AML
  • Cytogenetic tests (i.e. conventional karyotyping, FISH) and molecular analysis (i.e. next generation sequencing) for diagnosis, clinical management and risk stratification
  • Reference: Foucar: Diagnostic Pathology - Blood and Bone Marrow, 2nd Edition, 2017
Laboratory
  • Complete blood counts with differentials:
    • Variable cytopenia(s) following exposure to alkylating agents
    • Leukocytosis more common after exposure to topoisomerase II inhibitors
  • Additional tests to exclude other underlying causes of cytopenia(s), as clinically appropriate (i.e. vitamin B12, folate and iron, Coombs test, etc.)
  • Reference: Foucar: Diagnostic Pathology - Blood and Bone Marrow, 2nd Edition, 2017
Prognostic factors
  • Advanced age, poor performance status and low hemoglobin are unfavorable prognostic factors (Am J Hematol 2015;90:E80)
  • Cytogenetic findings are key prognostic factors (Blood 1995;86:3542)
    • Abnormalities on chromosome 5 or 7, TP53 mutation or loss and complex karyotype predict poor prognosis with median survival time < 1 year from diagnosis
    • Balanced translocations including t(15;17), inv(16) or t(16;16) generally harbor better prognosis
    • Therapy related t(15;17), inv(16) or t(16;16) have shorter median survival compared with their de novo counterparts
Case reports
Treatment
Microscopic (histologic) description
  • t-MDS, t-MDS / MPN and t-AML due to alkylating agents (Foucar: Diagnostic Pathology - Blood and Bone Marrow, 2nd Edition, 2017):
    • Increased cellularity
    • Multilineage dysplasia, commonly striking
    • Increased blast count
    • Variable fibrosis
  • t-MDS, t-MDS / MPN and t-AML due to topoisomerase II inhibitors (Foucar: Diagnostic Pathology - Blood and Bone Marrow, 2nd Edition, 2017):
    • Increased cellularity
    • Frequent features of acute monoblastic, acute monocytic or acute myelomonocytic leukemia
    • Common rearrangements involving 11q23
    • Other AML with recurrent cytogenetic alterations
      • Acute promyelocytic leukemia with t(15;17)(q24.1;q21.2)
      • Acute myeloid leukemia with t(8;21)(q22;q22.1)
      • Acute myeloid leukemia with inv16(p13.1q22)
  • Myeloblasts, monoblasts and megakaryoblasts are included in the blast count
  • Promonocytes are considered blasts equivalent in the setting of acute monoblastic, acute monocytic and chronic / acute myelomonocytic leukemia
  • Blast count on bone marrow samples (alongside peripheral smears) used to drive final diagnosis (i.e. t-MDS and t-MDS / MPN: < 20%; t-AML: ≥ 20%)
  • Primary neoplasm may be manifest (i.e. plasma cell neoplasms, lymphomas): typically form discrete lesions
Microscopic (histologic) images

Contributed by Valentina Sangiorgio, M.D. and AFIP

t-MDS: hypercellular bone marrow

t-AML: hypercellular bone marrow with impaired maturation

t-MDS: bland reticulin fibrosis


t-MDS: myeloid hypoplasia with left shift

t-AML: clusters of blasts

Large erythroid precursors with lobulated nuclei and karyorrhexis

PAS+ erythroid precursors in bone marrow smear

Peripheral smear description
Peripheral smear images

AFIP images

Postradiochemotherapy
and splenectomy for
Hodgkin disease

Immunohistochemistry & special stains
  • CD34 to assess number and distribution of hematopoietic precursors
    • CD34 can highlight abnormal localization of immature precursors (ALIP)
  • Not all myeloblasts express CD34 alongside monoblasts
    • CD117 can be used alternatively
    • For diagnostic purposes, blast count should be performed on bone marrow aspirate smears and peripheral smears
  • Megakaryocytic markers (CD61, CD42b, factor VIII) used to highlight number and distribution of megakaryocytes and micromegakaryocytes
  • Erythroid markers (hemoglobin A, glycophorin A, CD71, E-cadherin) to assess erythroid lineage and colony formation
  • Myeloperoxidase highlights myeloid cells and ALIP, if present (Foucar: Diagnostic Pathology - Blood and Bone Marrow, 2nd Edition, 2017)
  • Strong p53 immunostaining (≥ 1% of bone marrow cells) is predictive of a TP53 gene mutation and correlates with the presence of a high risk karyotype (Mod Pathol 2015;28:552)
Flow cytometry description
  • There are no specific immunophenotypic findings in t-MDS, t-MDS / MPN and t-AML compared with the de novo counterparts
  • Blasts generally express CD34 alongside other myeloid markers (i.e. CD13, CD33, CD117, myeloperoxidase)
Flow cytometry images

Contributed by Valentina Sangiorgio, M.D.

t-AML

Molecular / cytogenetics description
  • t-MDS, t-MDS / MPN and t-AML show cytogenetic abnormalities in as many as 90% of cases
  • Around 70% of cases harbor unbalanced chromosomal aberrations:
    • Most common alterations affect chromosome 5 (i.e. -5q or -5) and 7 (i.e. -7q or -7)
    • These generally occur in the context of a complex karyotype or co-occur with mutations or loss of TP53
    • Such changes correlate with a long latent period, a preceding phase of t-MDS and exposure to alkylating agents or radiation therapy (Leukemia 2008;22:240)
  • Around 30% have balanced translocations:
    • 11q23 (KMT2A gene) is frequently involved
    • Others rearrangements: t(8;21)(q22;q22.1), t(15;17)(q24.1;q21.1) and inv16(p13.1q22)
    • These cases develop shortly after exposure to the etiologic agent(s) and present with overt t-AML without a preceding chronic phase
    • Common following topoisomerase II inhibitors or radiation therapy alone (J Clin Oncol 1998;16:1897)
  • A minority of cases have a normal karyotype
  • TP53 is the most commonly mutated gene, occurring in as many as 50% of cases (Haematologica 2013;98:908)
  • NPM1 and FLT3 mutations are less common in t-AML compared with de novo AML
Molecular / cytogenetics images

Contributed by Valentina Sangiorgio, M.D.

G banding karyotype of
t-AML

Videos

Role of bone marrow transplant in therapy related AML and MDS

Sample pathology report
  • Bone marrow, biopsy and aspirate (t-AML):
    • Hypercellular for age, with infiltration (around 60%) by acute myeloid leukemia and background of multilineage dysplasia (see comment)
    • Comment: In light of the current histologic findings within the bone marrow alongside the detection of a complex karyotype and of the patient's known past medical history (breast carcinoma treated with chemo and radiation therapy), the case is best classified as therapy related acute myeloid leukemia. Clinical correlation is advised.
    • Bone marrow aspirate microscopic description (t-AML): Multiparticulate sample showing hypercellular marrow for the patient's age (cellularity around 80%). Erythropoiesis is increased and shows prominent megaloblastoid changes; dysplastic changes in the form of multinucleation and nuclear bridging are seen. Myelopoiesis is increased with left-shifted maturation. Majority of neutrophils shows pseudo-Pelger-Huët changes. Megakaryocytes are seen in adequate number but are overtly dysplastic, the majority being small and hypolobated. There is prominent increase in blasts which are medium in size with scant cytoplasm and open chromatin with variably prominent nucleoli. A cell count (performed on 150 cells) reveals: 39% blasts, 20% promyelocytes / myelocytes, 11% maturing granulocyte forms, 27% erythroid precursors, 1% lymphocytes, 1% plasma cells, 1% monocytes. Prussian blue iron stain shows normal storage iron; no ring sideroblasts are noted.
    • Bone marrow biopsy microscopic description (t-AML): The description of bone marrow trephine findings is similar to the aspirate sample (see above). Additional comments should be as follow: "CD34 highlights an increase in hematopoietic precursors which account for around 30% of the total nucleated cells. Aberrant expression of CD34 within the megakaryocytes is also seen (only if seen)." Reticulin stain reveals an increased network of reticulin fibers with many intersections (WHO MF1).
    • Peripheral smear microscopic description (t-AML): Pancytopenia with dysplastic neutrophils. 15% of the nucleated cells are blasts.
Differential diagnosis
  • Chemotherapy induced dysplasia and bone marrow suppression:
    • Recent rather than remote chemoradiation therapy
    • Usually resolves after chemotherapy discontinued
    • Good response to hematopoietic growth factors (i.e. erythropoietin, G-CSF)
  • Coincidental myeloid neoplasms in patients with history of chemotherapy:
    • Lacks prototypic morphologic or genetic features of t-MDS, t-MDS / MPN and t-AML (i.e. JAK2 positive MPN unrelated to previous therapy)
Board review style question #1
Which of the following minimum blast count on blood smear, bone marrow aspirate or trephine supports a diagnosis of therapy related AML?

  1. 10% if prior treatment with alkylating agents
  2. 10% if prior treatment with topoisomerase II inhibitors
  3. 20%
  4. 30%
Board review style answer #1
Board review style question #2

Is the presence of these small and hypolobated megakaryocytes expected in therapy related myeloid neoplasms?

  1. No; small and hypolobated megakaryocytes are not a common finding in therapy related myeloid neoplasms
  2. No; small and hypolobated megakaryocytes, especially if many, suggest a myeloproliferative neoplasm
  3. Yes; small and hypolobated megakaryocytes are a common manifestation of dysplasia and can be seen in therapy related myeloid neoplasms
  4. Yes; small and hypolobated megakaryocytes are required for t-MDS
Board review style answer #2
C. Yes; small and hypolobated megakaryocytes are a common manifestation of dysplasia and can be seen in therapy related myeloid neoplasms

Comment Here

Reference: AML therapy related

Transient abnormal myelopoiesis associated with Down syndrome
Definition / general
  • Transient disorder of newborns with Down syndrome or phenotypically normal neonates with trisomy 21 mosaicism
  • Presents within 3 - 5 days of birth and resolves spontaneously within 3 months
  • Proliferation of nonerythroid blasts (commonly megakaryoblasts) in the peripheral blood or organs
  • Morphologically indistinguishable from acute myeloid leukemia (AML), specifically acute megakaryoblastic leukemia (AMKL)
  • Unique distinguishing clinical, immunophenotypic and molecular genetic features from AML not associated with Down syndrome
Essential features
  • 10% of newborns with Down syndrome or trisomy 21 mosaicism
  • Proliferation of nonerythroid blasts in the peripheral blood, bone marrow or organs
  • Morphologically indistinguishable from other forms of AML
  • Presents within first week of life and resolves within 3 months
  • 20 - 30% may progress to nontransient AML (i.e. AMKL) within 1 - 3 years
  • Associated with acquired GATA1 mutation
Terminology
  • Transient abnormal myelopoiesis associated with Down syndrome (TAM)
  • Transient myeloproliferative disorder (TMD)
  • Transient leukemia (TL)
ICD coding
  • ICD-O: 9898/1 - Transient abnormal myelopoiesis
Epidemiology
  • Manifests in approximately 10% of neonates with Down syndrome
  • 7 - 16% of TAM is seen in phenotypically normal neonates with trisomy 21 mosaicism
Sites
  • Peripheral blood
    • Common site of blasts, as fetal hematopoiesis occurs predominantly in the liver
    • Peripheral blood involvement > bone marrow
  • Bone marrow
    • Less common site of fetal hematopoiesis
  • Other organs
    • Liver, spleen, skin, pancreas, kidneys, pleural fluid, pericardial fluid
Pathophysiology
  • 3 step process (Curr Hematol Malig Rep 2016;11:333):
    • Perturbation of fetal liver hematopoiesis by trisomy 21
    • Acquired or somatic mutation of GATA1 (chromosome X), hematopoietic transcription factor
    • 20 - 30% progress to AML with further acquisition of oncogenic mutations
Etiology
  • Risk factors for Down syndrome
  • Advanced maternal age
Clinical features
  • Most diagnosed at 3 - 5 days of age
  • Most patients are asymptomatic but may present with myeloblast organ infiltration:
    • Hepatosplenomegaly (common)
    • Ascites, pericardial or pleural effusions, hepatic fibrosis, disseminated intravascular coagulopathy (less common)
    • Severe organ dysfunction causing renal, hepatic or cardiopulmonary failure (rare)
  • If in utero, may present as hydrops fetalis secondary to cardiopulmonary failure and anemia
  • References: Curr Hematol Malig Rep 2016;11:333, Silberstein: Hematology - Basic Principles and Practice, 7th Edition, 2017
Diagnosis
  • No universal diagnostic criteria
  • Children's Oncology Group (COG) (Blood 2011;118:6752):
    • Detection of nonerythroid blasts in peripheral blood or organs
    • Newborns < 90 days old
    • Trisomy 21 or mosaicism
    • Confirmation with:
      • Second blood sample
      • > 5% nonerythroid blasts in bone marrow
      • Hepatosplenomegaly, lymphadenopathy or pericardial / pleural effusions
Laboratory
Prognostic factors
  • Majority resolve spontaneously over several weeks to 3 months
  • Approximately 15 - 23% of patients may die as a result of secondary organ failure:
    • Hepatic fibrosis
    • Cardiopulmonary failure
  • Poor prognostic factors for early death include:
    • High WBC count
    • Increased bilirubin
    • Elevated liver function tests (LFTs)
    • Failure to normalize blood counts
  • 20 - 30% may develop nontransient AML (usually AMKL) within 1 - 3 years
  • Greatest risk factors for AML progression are karyotypic abnormalities in addition to +21 in blast cells
  • References: Curr Hematol Malig Rep 2016;11:333, Mycopathologia 2016;181:909
Case reports
Treatment
  • Treatment is often supportive due to spontaneous remission
  • In severe organ dysfunction, exchange transfusion, leukapheresis or chemotherapy may be necessary
  • In TAM progression to AML, cytosine arabinoside is the most commonly used chemotherapeutic agent
    • Favorable prognosis
  • References: Curr Hematol Malig Rep 2016;11:333, Pediatr Int 2019;61:222
Microscopic (histologic) description
  • Blasts are morphologically indistinguishable from those in AMKL associated with Down syndrome
  • Features of megakaryoblasts
    • Increased nuclear:cytoplasmic (N:C) ratio
    • Dispersed nuclear chromatin
    • Basophilic cytoplasm
    • Coarse basophilic cytoplasmic granules
    • Cytoplasmic blebbing
  • Erythroid and megakaryocytic dysplasia are also seen
    • Dyserythropoiesis with bi and trinucleated forms
    • Dysmegakaryopoiesis with dysplastic small forms and micromegakaryocytes
Microscopic (histologic) images

Contributed by Julia T. Geyer, M.D. and Tayler A. van den Akker, M.D.

Bone marrow biopsy

Blasts of TAM-DS

Blasts with hematopoietic elements

Cytoplasmic blebs

Peripheral smear description
  • Features of megakaryoblasts in peripheral blood
    • Increased N:C ratio
    • Dispersed nuclear chromatin
    • Basophilic cytoplasm
    • Coarse basophilic cytoplasmic granules
    • Cytoplasmic blebbing
  • Basophilia
Peripheral smear images

Contributed by Julie Teruya-Feldstein, M.D.

Peripheral blood blasts

Positive stains
Flow cytometry description
Flow cytometry images

Contributed by Tayler A. van den Akker, M.D.

CD45+ dim population

CD34+, CD33+ dim population

CD34+,
HLA-DR+ partial population

CD33+ dim, CD117+ population


CD34+, CD235A- population

CD7+ heterogeneous, CD15- population

CD34+, CD38+ population

CD34+, MPO- population

Molecular / cytogenetics description
Sample pathology report
  • Bone marrow, right posterior iliac crest and aspirate smears:
    • Hypercellular marrow (100%), consisting predominantly of left shifted granulocytic precursors with focally interspersed small hypolobated megakaryocytes. Increased blasts (20%) with cytoplasmic blebs and prominent nucleoli. These findings are suspicious for transient abnormal myelopoiesis versus congenital AML (see comment).
    • Comment: Flow cytometric analysis demonstrates an aberrant blast population (approximately 35% of total), exhibiting the following immunophenotype: CD45 moderate+, CD34+, CD33 dim+, HLA-DR partial+, CD117+, CD38+. CD7 heterogeneous+, CD4 dim+. These findings do not definitively distinguish between transient abnormal myelopoiesis and acute myeloid leukemia; clinical correlation is required.
    • Bone marrow, left posterior iliac crest, trephine biopsy and clot: Quality: adequate. Cellularity: 100%. Normocellular marrow for age (~100%) consisting predominantly of left shifted granulocytic precursors. Megakaryocytes are increased, predominantly hypolobated forms and not seen in dense clusters. No granulomas. The bony trabeculae are unremarkable.
    • Bone marrow aspirate smears: Quality: adequate. Blasts are increased, approximately 20% of total by manual differential count. Blasts with cytoplasmic blebs and prominent nucleoli.
Differential diagnosis
  • AMKL in Down syndrome:
    • Anemia, with preserved WBC count
    • Occurs after the first year of life (versus first 3 - 5 days in TAM)
    • May have a history of TAM
    • Dyserythropoiesis and bone marrow fibrosis more common (versus TAM)
    • Frequently negative for CD34, CD56, HLA-DR and positive for CD11b and CD13
    • Complex karyotype and other cytogenetic abnormalities (i.e. JAK-STAT pathway) in addition to +21 and GATA1 mutation
  • Acute myeloid leukemia not associated with Down syndrome:
    • Commonly adults; median age: 65 - 68 years
    • Poor prognosis compared with Down syndrome related AML (favorable prognosis)
    • AMKL subtype is rare (< 5% AML)
    • Absence of trisomy 21
Board review style question #1

A 4 day old boy with Down syndrome is diagnosed with a transient myeloproliferative condition associated with the cells seen in the above image. Which genetic abnormality would be expected in addition to the additional chromosome 21?

  1. GATA1 mutation
  2. NPM1 mutation
  3. PDGFRA mutation
  4. t(9;22) with p190 breakpoint
Board review style answer #1
Board review style question #2
Which immunophenotype is observed in AMKL associated with Down Syndrome?

  1. CD2+, CD4+, CD8- CD7-, CD10+, CXCL13+, BCL6+
  2. CD7+, CD3+, MPO-, CD4+, CD8+, TdT+, CD34+
  3. CD19+, Cd20+dim, CD5+, CD10-, FMC7-
  4. CD117+, CD34-, CD41+, CD7+, CD56-, CD61+, MPO-
Board review style answer #2
D. CD117+, CD34-, CD41+, CD7+, CD56-, CD61+, MPO-. Answer A is seen in AITL, answer B is seen in T-ALL and answer C is seen in CLL.

Comment Here

Reference: Transient abnormal myelopoiesis associated with Down syndrome

VEXAS (pending)
[Pending]

t(12;21)(p13;q22); ETV6::RUNX1
Definition / general
  • 20 - 30% of childhood preB ALL; most common translocation (Chin Med J (Engl) 2003;116:1298) but not infants; also 3% of adults
  • 5 different patterns of gene expression involving 14 genes, can detect with gene chip (BMC Genomics 2007;8:385)
  • Excellent prognosis due to good response to chemotherapy; 90% remissions; relapses occur later than other ALL
  • Persistence of TEL-AML1 transcripts is not necessarily related to relapse (Pediatr Int 2003;45:275)
Microscopic (histologic) description
  • No distinct morphology
Positive stains
Negative stains
Molecular / cytogenetics description
  • Fusion of TEL/ETV6 and AML1/RUNX1/CBFA2 genes
  • Detect with FISH or PCR; not found by conventional cytogenetics (i.e. are cryptic) because rearranged segments are too small
  • May have no other molecular abnormalities, almost never > 50 chromosomes
Molecular / cytogenetics images

Images hosted on other servers:
Missing Image

FISH: t(12;21)(p13;q22)


t(1;19)(q23;p13.3); TCF3::PBX1
Definition / general
  • 5 - 6% of ALL; commonly detected by conventional cytogenetics in children
  • E2A-PBX1 (TCF3-PBX1) is chimeric gene formed by t(1;19)(q23;p13.3) (Mol Cell Biol 1994;14:3938)
  • High WBC counts, frequent CNS involvement
  • In children (Blood 1984;63:721) and adults (Haematologica 2010;95:241), is no longer a poor prognostic factor due to intensive therapy and bone marrow transplantation
Case reports
Microscopic (histologic) description
  • No distinct morphologic findings
Positive stains
  • Typical immunophenotype cytoplasmic µ (cµ), CD10, CD19
  • If cytoplasmic µ negative, can be suspected if strong CD9 and CD34-
Molecular / cytogenetics description
  • Produces fusion transcript of PBX1 and E2A; occurs in balanced and unbalanced forms; unbalanced form is der(19)t(1;19)
  • The reciprocal product of der(1)t(1;19) is lost and the normal chromosome 1 is duplicated (Leukemia 2001;15:95)

Notes:
  • “Unbalanced translocations” means the exchange of chromosomal material is unequal resulting in extra or missing genes
  • “der” means derivative chromosome - term is used when only one chromosome from a translocation is present, or when one chromosome has two or more structural abnormalities

t(5;14)(q31;q32); IGH::IL3
Definition / general
  • Diagnosis based on immunophenotype and genetic findings, even with low blast count
  • Very rare; < 1% of all ALL; both children and adults (Blood 1989;73:2081)
  • Presents with variable eosinophilia; may be asymptomatic and lack any blasts (Medicine (Baltimore) 1990;69:232)
Case reports
Microscopic (histologic) description
  • CD19+ CD10+ lymphoblasts with circulating reactive eosinophils

t(9;22)(q34;q11); BCR::ABL1
Definition / general
  • ALL with BCR-ABL1 fusion transcript (Philadelphia chromosome)
  • 30% of adults with ALL, 4% of children but 80% of infants
  • Excluding infants, older age and higher WBC at presentation than other B ALL
  • May have more organomegaly or CNS involvement than other B ALL
  • Poor prognosis
Treatment
Microscopic (histologic) description
  • No defining morphology, but large blasts with prominent nucleoli and cytoplasmic granules are more common than other B ALL
Microscopic (histologic) images

AFIP images
Missing Image

Leukemic cells resemble early erythroblasts

Cytology images

Contributed by Julia Braza, M.D., M.S.
Missing Image Missing Image Missing Image

CSF

Positive stains
Flow cytometry images

Images hosted on other servers:
Missing Image

B-ALL associated with t(9;22); lymphoblasts express B-lineage
markers CD19, CD20, CD22, immature markers CD34, TdT, CD10,
and aberrant CD25; they are aberrantly dim for CD38 and CD81;
this pattern is useful in identifying minimal residual disease

Molecular / cytogenetics description
  • Translocation involves abl on #9q34 (tyrosine kinase) and bcr on #22q11 (breakpoint cluster region)
  • Most childhood cases associated with p190 kd BCR-ABL1 fusion protein
  • In 50% of adults, has p210 kd fusion protein that is present in CML, the rest is p190 kd; no definitive clinical difference
Molecular / cytogenetics images

Contributed by Julia Braza, M.D., M.S. and AFIP
Missing Image

FISH

Missing Image

Karyotype



Images hosted on other servers:
Missing Image

Drawing of translocation


t(v;11q23); KMT2A rearranged
Definition / general
  • Rearrangements of MLL gene
  • 20% of ALL overall (Anticancer Res 2005;25:1931) including 80% of infants (Leukemia 2007;21:633), 10% of older children and adults
  • May have bilineal lymphoblasts with monoblasts and promonocytes
  • Usually infants < 1 year with markedly increased WBC (> 100x109/L), CNS involvement; pure lymphomatous involvement not typical
  • Poor prognosis (30% event free survival)
Treatment
  • Intensive chemotherapy followed by hematopoietic stem cell transplantation
Microscopic (histologic) description
  • No defining morphology
Positive stains
Negative stains
Molecular / cytogenetics description
  • Over 75 genetic arrangements; most translocations at 11q23 involve MLL (mixed lineage leukemia) gene
  • t(4;11)(q21;q23) - MLL-AF4: occurs in 60% of infants, 2% of other children and 3 - 6% of adults
  • t(11;19)(q23;p13.3) - MLL-ENL and others
  • MLL translocation is associated with FLT3 overexpression
  • 11q23 deletions are NOT included in this group because different immunophenotype, clinical and prognostic features
Molecular / cytogenetics images

Images hosted on other servers:
Missing Image

ALL1 / MLL duplication


with DDX41 mutation
Definition / general
  • Autosomal dominant trait caused by pathogenic germline mutational variants in the DDX41 gene, increasing the risk of hematolymphoid neoplasms
Essential features
  • Pathogenic germline mutational variant on chromosome 5q35.3
  • Autosomal dominant inheritance
  • Familial myeloproliferative / lymphoproliferative neoplasms
Terminology
  • DEAD box helicase 41
ICD coding
  • ICD-10: C96.7 - other specified malignant neoplasms of lymphoid, hematopoietic and related tissue
Epidemiology
  • Germline DDX41 pathogenic variants are the most common cause for genetic predisposition to myeloid neoplasms and are identified in 1 - 4% of patients with myeloid neoplasms (Cancer Cell 2015;27:658, Blood 2019;134:1441, Am J Hematol 2019;94:757)
  • Lifelong risk of developing myeloid neoplasms from a pathogenic or likely pathogenic DDX41 variant is ~50%, starting at 40 years of age (Blood 2023;141:534)
  • Men with the gene variant have nearly double the risk of developing a myeloid neoplasm compared with female carriers (Blood 2022;140:716)
  • Certain DDX41 germline variants are more common in certain populations (e.g., p.Ala500Cysfs*9 in Japanese / Korean and p.D140Gfs*2 in those of Northern European descent) (Blood 2023;141:1544)
Sites
  • Bone marrow / peripheral blood
  • Lymph nodes (less common)
Pathophysiology
  • Precise mechanism by which germline DDX41 variants drive hematopoietic malignancies is still unknown
  • DDX41 gene is located on chromosome 5q35.3
  • DDX41 is expressed on hematopoietic cells and it encodes an ATP dependent RNA helicase that participates in RNA metabolism (Cancer Cell 2015;27:658)
  • Germline DDX41 variants are typically heterozygous and include frameshift, nonsense or splice site mutations whereas somatic mutations are usually missense heterozygous variants that affect the helicase 2 ATP binding domain (Blood 2016;127:1017)
  • DDX41 is found both in the cytoplasm and nucleus, where it plays a role in messenger RNA splicing, ribosomal RNA processing or small nucleolar RNA processing (Blood 2023;141:1544)
  • Disruption of the DDX41 protein contributes to inflammation and tumorigenesis (Blood 2023;141:1544)
Etiology
  • Germline DDX41 pathogenic mutations are passed on from a parent to child in an autosomal dominant inheritance pattern
  • Pathogenic variant is usually heterozygous
Diagrams / tables

Images hosted on other servers:
Proposed mechanisms for DDX41 involvement

Proposed mechanisms for DDX41 involvement

Clinical features
  • Individuals with a pathogenic DDX41 variant have an increased lifelong risk (~50%) of developing a myeloid neoplasm, including clonal cytopenia of undetermined significance (CCUS), myelodysplastic syndrome (MDS), myeloproliferative neoplasm (MPN), myelodysplastic / myeloproliferative neoplasm (MDS / MPN) and acute myeloid leukemia (AML) starting at age 40 (Blood 2023;141:534)
  • Germline DDX41 variants are found in 3 - 5% in MDS, 5 - 9% in secondary AML, 1 - 2% in MDS / MPNs and < 0.5% in MPNs (Blood 2023;141:534)
  • Aplastic anemia may occur but is rare
  • Some patients may develop a lymphoid neoplasm or a plasma cell neoplasm; however, this is much less common than myeloid neoplasms and includes follicular lymphoma, Hodgkin lymphoma, chronic lymphocytic leukemia, multiple myeloma and acute lymphoblastic leukemia (Blood 2016;127:1017)
Diagnosis
  • WHO 5th edition diagnostic category: myeloid neoplasms associated with germline predisposition
  • WHO 5th edition essential diagnostic criteria: detection of a pathogenic or likely pathogenic DDX41 variant
  • WHO 5th edition desirable diagnostic criteria: detection of pathogenic or likely pathogenic germline DDX41 variant at a variant allele frequency consistent with germline status from DNA derived from cultured skin fibroblasts or via segregation in other family members
  • Reference: Am J Hematol 2023;98:1780, Am J Hematol 2023;98:1780
Laboratory
Prognostic factors
  • Prognostic impact of germline DDX41 variants is unclear
  • Truncating germline DDX41 variants in patients who develop MDS are associated with faster transformation to AML compared with patients with nontruncating germline variants but no difference in overall survival is observed between these groups (Blood 2023;141:534)
Case reports
  • 9 year old boy presented with essential thrombocythemia-like features with a pathogenic germline variant in DDX41, JAK2 and a constitutional inv(7)(q11.2q22) (J Pediatr Hematol Oncol 2023;45:e609)
  • 48 year old man with Ph+ B cell lymphoblastic leukemia with biallelic DDX41 mutations (BMC Med Genomics 2022;15:46)
  • 62 year old man with myelodysplastic syndrome and DDX41 mutation treated successfully with lenalidomide (Am J Hematol 2020;95:227)
  • 70 year old man with an adult onset myeloid neoplasm with excess blasts without dysplasia and germline missense DDX41 variant (Leuk Lymphoma 2019;60:1337)
  • Caucasian family with inherited MDS / AML with 2 germline mutations in the DDX41 gene (Haematologica 2016;101:e228)
Treatment
  • No evidence based guidelines on the treatment for DDX41 associated familial MDS / AML have been published and patients are generally treated with standard neoplasm specific therapy
  • Patients with MDS associated with germline DDX41 pathogenic variants tend to have better response to hypomethylating agents and better outcomes compared to those with wild type DDX41 (Blood 2023;141:534)
  • Allogeneic hematopoietic stem cell transplant evaluation early in the course of hematologic malignancy may be appropriate based on the age of the individual, malignancy and health status
Microscopic (histologic) description
  • Hematolymphoid neoplasms arising in individuals with a DDX41 germline variant do not have distinctive morphologic features
  • Patients with a DDX41 variant have an increased likelihood of bone marrow hypocellularity and erythroid dysplasia (Am J Hematol 2019;94:757)
  • DDX41 mutation can lead to ineffective hematopoiesis resulting in cytopenia and macrocytosis in the peripheral blood (Front Oncol 2022:12:992340)
  • It is reported that patients with DDX41 variants can have persistent higher blast count in myelodysplastic syndrome or AML; among MDS subtypes, refractory anemia with excess blasts is the most frequent (Leuk Lymphoma 2019;60:1337)
Microscopic (histologic) images

Contributed by Jacob Armstrong, M.D.
Hypocellular bone marrow core

Hypocellular bone marrow core

Hyperchromatic megakaryocytes

Hyperchromatic megakaryocytes

Hypercellular bone marrow core

Hypercellular bone marrow core

Marrow with AML

Marrow with AML

CD34 with ~10% blasts

CD34 with ~10% blasts

CD61 with megakaryocyte atypia

CD61 with megakaryocyte atypia


Aspirate with increased blasts

Aspirate with increased blasts

nRBC with irregular nuclei

nRBC with irregular nuclei

nRBC with nuclear budding

nRBC with nuclear budding

nRBC with multinucleation

nRBC with multinucleation

Hypolobated megakaryocytes

Hypolobated megakaryocytes

Ring sideroblasts

Ring sideroblasts

Cytology description
  • Granulocytes
    • Can show dysplastic features (hypogranular or hyposegmented forms)
    • Increased blasts and immature myeloid precursors can be present
  • Erythrocytes
    • Can have dysplastic features including irregular nuclear contours, nuclear budding, binucleation, multinucleation, nuclear dyssynchrony, megaloblastoid changes and basophilic stippling
  • Megakaryocytes
    • Can have dysplastic features including large variation in sizes, small forms (micromegakaryocytes), hypolobated nuclei, widely spaced nuclei, hypersegmented (staghorn) nuclei and nuclear hyperchromasia
  • Blasts can be increased
  • Iron: ring sideroblasts can be present
  • Reference: GeneReviews: DDX41-Associated Familial Myelodysplastic Syndrome and Acute Myeloid Leukemia [Accessed 19 October 2023]
Peripheral smear description
  • Red blood cells (RBCs)
    • Normal RBC count, anemia or erythrocytosis can be present
    • Anemia including microcytic, normocytic or macrocytic
      • Macrocytosis is the most common finding
    • Anisopoikilocytosis, including teardrop cells (if there is myelofibrosis), etc.
    • Nucleated red blood cells (nRBC) can be present
  • White blood cells (WBCs)
    • Normal WBC count, leukocytosis or leukopenia can be present
    • Granulocytes can show dysplastic features (hypogranular or hyposegmented forms)
    • Circulating blasts and immature myeloid precursors can be present
  • Platelets
    • Normal platelet count, thrombocytopenia or thrombocytosis can be present
    • Morphology of platelets is usually unremarkable
  • Reference: GeneReviews: DDX41-Associated Familial Myelodysplastic Syndrome and Acute Myeloid Leukemia [Accessed 19 October 2023]
Peripheral smear images

Contributed by Jacob Armstrong, M.D.
Macrocytic anemia

Macrocytic anemia

Circulating blasts and nRBC

Circulating blasts and nRBC

Positive stains
  • There are no diagnostic stains for DDX41 mutation; helpful stains include
    • CD34: enumeration of CD34 positive blasts
    • CD117: enumeration of CD117 positive blasts and erythroid precursors
    • TdT: positive on some blasts
    • CD123: positive on some blasts
    • CD42b: highlights megakaryocytes
    • CD61: highlights megakaryocytes
    • Factor VIII: highlights megakaryocytes
    • CD15: highlights granulocytes
    • MPO: highlights the myeloid lineage
    • E-cadherin: highlights the erythroid lineage
    • CD71: highlights the erythroid lineage
    • Glycophorin A: highlights the erythroid lineage
    • CD4: highlights monocytes and a subset of T cells
    • CD68: highlights monocytes
    • Reticulin: highlights reticulin fibrosis
    • Trichrome: highlights collagen fibrosis
  • Reference: Am J Hematol 2023;98:1780
Flow cytometry description
  • No reported distinctive flow cytometry immunophenotypic pattern; however, individuals with DDX41 mutation can have an increased blast population or granulocytes with decreased side scatter as a reflection of hypogranularity
Flow cytometry images

Contributed by Zubaidah Al-Jumaili, M.D.
CD45 versus side scatter CD45 versus side scatter

CD45 versus side scatter

CD34 versus CD117 CD34 versus CD123

CD34 versus CD123

Molecular / cytogenetics description
  • Cytogenetics findings
    • DDX41 pathogenic variants are not associated with any karyotype abnormalities; while patients with DDX41 variants can have karyotype abnormalities, these changes are independent of the DDX41 mutation (Am J Hematol 2019;94:757)
  • Molecular findings
    • Deleterious germline DDX41 variants confer increased risk for myeloid neoplasms with an estimated prevalence of 1 - 4% among MNs
    • DDX41 can be mutated in the germline or in a somatic fashion
      • Germline mutations of DDX41 have been reported to predispose patients to somatic mutations and subsequent MDS or AML
      • ~50% of patients with germline mutations also harbor somatic mutations in the other allele of DDX41
    • The most frequent DDX41 mutation identified is a frameshift duplication causing a premature stop codon (p.D140Gfs*2); however, missense and splice variants are also described
    • The most frequent co-occurring somatic mutations involve ASXL1, CUX1, TP53, DNMT3A and TET2 and among these, CUX1 mutations are preferentially observed in DDX41 mutated cases compared with wild type ones (Am J Hematol 2019;94:757)
Molecular / cytogenetics images

Images hosted on other servers:
Distribution of DDX41 mutations along the gene

Distribution of DDX41 mutations along the gene

Germline and somatic DDX41 mutations

Germline and somatic DDX41 mutations

Deleterious DDX41 germline variants

Deleterious DDX41 germline variants

Videos

What is the clinical impact of DDX41 mutations in myeloid neoplasms?

Germline predisposition to myeloid malignancies

Sample pathology report
  • Bone marrow, biopsy:
    • Hypocellular (30%) bone marrow with dyserythropoiesis and ~10% blasts (see comment)
    • Comment: Bone marrow aspirate smears show erythroid dysplasia and a mild increase in blasts, comprising ~10% of the total nucleated cells on the aspirate smear. The bone marrow core biopsy and clot section are hypocellular for age with increased scattered blasts marked by CD34 immunostain. Flow cytometric analysis shows increased abnormal myeloblasts that comprise about 10.2% of total events with an atypical immunophenotype (CD2+, CD13+, CD33+, CD34+, CD38+, CD117+, CD123+, MPO+). These findings are compatible with a myelodysplastic neoplasm with increased blasts. NGS studies show a DDX41 mutation. This is consistent with a myeloid neoplasm associated with germline predisposition, more specifically, myelodysplastic neoplasm with increased blasts with DDX41 mutation.
Differential diagnosis
  • Myeloid neoplasms associated with a DDX41 pathogenic variant:
    • Clonal cytopenias of undetermined significance (CCUS):
      • Detection of 1 or more somatic mutations with variant allele frequency (VAF) ≥ 2% (≥ 4% for X linked gene mutations in men) in DNA from blood or bone marrow cells involving genes listed in table #32521 of the clonal hematopoiesis section of the WHO 5th edition or clonal chromosomal abnormalities in myeloid cells
      • 1 or more otherwise unexplained cytopenias that persist for > 4 months
      • Absence of diagnostic criteria for defined myeloid neoplasms on bone marrow examination
    • Myelodysplastic neoplasm (MDS):
      • Cytopenia in at least 1 hematopoietic lineage: Hb < 13 g/dL in men and < 12 g/dL in women for anemia, absolute neutrophil count < 1.8 x 109/L for leukopenia and platelets < 150 x 109/L for thrombocytopenia
      • Please see diagnostic criteria for MDS with defining genetic abnormalities or morphologically defined MDS
        • Defining genetic abnormalities include
          • MDS with low blasts and 5q deletion (MDS-5q)
          • MDS with low blasts and SF3B1 mutation (MDS-SF3B1)
          • MDS with biallelic TP53 inactivation (MDS-biTP53)
        • Morphologically defined MDS includes
          • MDS with low blasts (MDS-LB)
          • MDS, hypoplastic (MDS-h)
          • MDS with increased blasts (MDS-IB)
          • MDS with fibrosis (MDS-f)
    • Myelodysplastic neoplasm (MDS) / myeloproliferative neoplasm (MPN):
      • Please see diagnostic criteria for MDS / MPN entities
        • Chronic myelomonocytic leukemia
        • Myelodysplastic / myeloproliferative neoplasm with neutrophilia
        • Myelodysplastic / myeloproliferative neoplasm with SF3B1 mutation and thrombocytosis
        • Myelodysplastic / myeloproliferative neoplasm, NOS
    • Myeloproliferative neoplasm (MPN):
      • Please see diagnostic criteria for MPN entities
        • Chronic myeloid leukemia
        • Chronic neutrophilic leukemia
        • Chronic eosinophilic leukemia
        • Polycythemia vera
        • Essential thrombocythemia
        • Primary myelofibrosis
        • Juvenile myelomonocytic leukemia
        • Myeloproliferative neoplasm, NOS
    • Acute myeloid leukemia (AML):
      • Blasts comprising at least 20% or AML defining genetic features including t(8;21), t(15;17) or t(16;16)/inv(16)
    • Myeloid neoplasm postcytotoxic therapy:
      • Myeloid neoplasm meeting diagnostic criteria of any myelodysplastic neoplasm, myelodysplastic / myeloproliferative neoplasm or acute myeloid leukemia
      • History of prior exposure to cytotoxic therapy or large field radiation therapy for an unrelated disorder
      • Not meeting diagnostic criteria of myeloproliferative neoplasms
  • Other germline variants that cause predisposition to myeloid neoplasms:
  • Other syndromes that cause predisposition to myeloid neoplasms:
Board review style question #1

A 46 year old man presents with macrocytic anemia and thrombocytopenia. Bone marrow aspirate smears show the findings above. The bone marrow is hypercellular for age with hyperchromatic megakaryocytes. Cytogenetics findings reveal a 20q deletion and next genome sequencing shows a DDX41 pathogenic variant. The patient has a brother with similar findings. Which of the following is the diagnosis?

  1. Acute myeloid leukemia
  2. Chronic myeloid leukemia
  3. Chronic myelomonocytic leukemia
  4. Myeloid neoplasm postcytotoxic therapy
  5. Myeloid neoplasms associated with germline predisposition
Board review style answer #1
E. Myeloid neoplasms associated with germline predisposition. Myeloid neoplasms associated with germline predisposition include any myeloid neoplasms that are diagnosed in a patient with a DDX41 variant. The photo demonstrates several erythroids with binucleation, which is consistent with erythroid dysplasia and suggestive of MDS. The bicytopenia, erythroid dysplasia and hypercellular marrow along with a description of dysplastic megakaryocytes are consistent with MDS. Answer A is incorrect because the image does not display an increase in blasts. Answer B is incorrect because the image does not show myeloid hyperplasia with increased immature granulocytes; the cytogenetic and molecular findings also do not show a t(9;22) or BCR::ABL1. Answer C is incorrect because the image does not show myeloid hyperplasia with increased granulocytes and monocytes. Answer D is incorrect because there is no evidence in the question stem that the patient has received cytotoxic therapy.

Comment Here

Reference: with DDX41 mutation
Board review style question #2
Which of the following red blood cell (RBC) findings are most commonly seen in myeloid neoplasms with DDX41 mutation?

  1. Macrocytic anemia
  2. Microcytic anemia
  3. Normocytic anemia
  4. Normocytic normochromic red blood cells
Board review style answer #2
A. Macrocytic anemia. Macrocytic anemia is the most common finding in myeloid neoplasms with DDX41 mutation. Answer B is incorrect because microcytic anemia is typically seen in iron deficiency anemia or thalassemia and not myeloid neoplasms. Answer D is incorrect because while normocytic red blood cells (without anemia) can be seen in myeloid neoplasms, DDX41 is typically associated with macrocytic anemia due to marrow disruption of normal erythroipoiesis. Answer C is incorrect because normocytic anemia is the second typical finding in the red blood cells in myeloid neoplasms with DDX41 mutation.

Comment Here

Reference: with DDX41 mutation

with Down syndrome (pending)
[Pending]

with del(5q)
Definition / general
  • Myelodysplastic syndrome (MDS) characterized by anemia with or without other cytopenias or thrombocytosis in conjunction with chromosome 5q deletion with or without one additional cytogenetic abnormality apart from monosomy 7 or del(7q)
  • Myeloblasts represent < 5% of bone marrow cellularity and < 1% of peripheral blood leukocytes
Essential features
Terminology
  • MDS with 5q deletion
  • 5q minus syndrome
ICD coding
  • ICD-O: 9986/3 - myelodysplastic syndrome associated with isolated del(5q)
Epidemiology
  • F > M
  • Adults, median age approximately 60 - 70 years
Sites
  • Blood and bone marrow
Pathophysiology
Etiology
  • Unknown at this time
Clinical features
Diagnosis
Laboratory
  • Repetitive with clinical features and diagnosis headings
Prognostic factors
Case reports
Treatment
Microscopic (histologic) description
  • Blast count must be < 5% in bone marrow
  • Bone marrow cellularity typically hyper to normocellular (Leukemia 2004;18:113)
  • Megakaryocyte hyperplasia
  • Megakaryocytes with nonlobated and hypolobated nuclei
  • Erythroid hypoplasia
  • Dysgranulopoiesis is uncommon
Microscopic (histologic) images

Contributed by Natasha Iranzad, M.D.

Normocellular bone marrow

Megakaryocyte dysplasia

CD61

Cytology description
  • Ring sideroblasts may be present
  • Auer rods must be absent
Peripheral smear description
  • Blast percentage must be < 1%
  • Anemia common, often severe and typically macrocytic
  • Thrombocytosis in 33% of cases
  • Thrombocytopenia less commonly seen
Positive stains
  • CD61 highlights increased / dysplastic megakaryocytes
Flow cytometry description
  • No findings specific to this entity
  • Myeloblasts with aberrant phenotype may be detected
Molecular / cytogenetics description
  • Repetitive with diagnosis heading
Molecular / cytogenetics images

Images hosted on other servers:

Karyogram with del(5q)

Sample pathology report
  • Bone marrow, aspirate smear, touch imprint, clot and core biopsy:
    • Normocellular bone marrow (30%) with megakaryocyte dysplasia
    • 3% bone marrow blasts (see comment)
    • Comment: The presence of dysplastic micromegakaryocytes in this patient with longstanding macrocytic anemia is concerning for myelodysplastic syndrome with single lineage dysplasia (MDS SLD). However, a normocellular marrow would be highly unusual for this diagnosis as MDS typically presents with hypercellularity and rarely with hypocellularity. Thus, a definitive diagnosis of MDS cannot be made at this time without evidence of myeloid clonality, increased blasts or hyper / hypocellularity. Correlation with the pending chromosome analysis and FISH studies are required to establish clonality. In addition, myeloid NGS studies may be diagnostically useful to provide evidence of clonality with additional prognostic or potentially therapeutic information; this test may be performed on the peripheral blood if clinically indicated. The corresponding flow cytometric analysis did not detect any phenotypic abnormalities in myeloid or monocytic lineages or increase in blasts, in keeping with the morphologic findings.

FISH - MDS panel:
  • Interpretation
    • 18.5% of cells showed deletion of 5q (EGR1) by interphase FISH. No evidence of numeric abnormalities or deletions involving chromosomes 7, 8 or 20 observed by interphase FISH. Correlation with other laboratory and clinical information is recommended.
  • nuc ish (D5S721/D5S23x2,EGR1x1)[37/200]
  • nuc ish (D7Z1,D7S486)x2[200]
  • nuc ish (D8Z2,D20S108)x2[199]

Summary table:
Locus Result
Monosomy 5/5q- 18.5% deletion 5q
Monosomy 7/7q- within normal limits
Trisomy 8 within normal limits
Deletion of 20q within normal limits

Chromosome analysis:
  • Interpretation
    • 46,XX,del(5)(q12q33)[4]/46,XX[14]
    • Abnormal clone with deletion 5q in 4/18 cells, consistent with concurrent interphase FISH analysis showing 18.5% of cells with deletion 5q (EGR1). Please see the Objective Findings section for details. Evaluation for an acquired TP53 mutation is recommended in patients with MDS with isolated del(5q) to help identify an adverse prognostic subgroup in this generally favorable prognosis MDS entity. Correlation with other laboratory and clinical information is recommended. (Blood 2016;127:2391)
Differential diagnosis
  • Myelodysplastic syndrome (MDS), unclassifiable (Blood 2016;127:2391):
    • Diagnosis of exclusion
    • Cases otherwise meeting criteria for MDS with del(5q) but with 1% blasts on peripheral blood smear on ≥ 2 occasions or pancytopenia, defined as:
      • Hemoglobin < 10 g/dL
      • Absolute neutrophil count < 1.8 x 109/L
      • Platelet count < 100 x 10109/L
  • MDS with excess blasts (EB):
    • As the name suggests, with 2 distinct subcategories depending on the blast count:
      • MDS EB1
        • 5 - 9% blasts in bone marrow
        • 2 - 4% blasts in peripheral blood
      • MDS EB2
        • 10 - 19% blasts in bone marrow
        • 5 - 19% blasts in peripheral blood
  • MDS with single / multilineage dysplasia:
    • Lacks characteristic 5q deletion
  • MDS with ring sideroblasts:
    • Lacks characteristic 5q deletion
    • Ring sideroblasts can be seen in both
    • SF3B1 mutation can be seen in both
Board review style question #1

A 73 year old woman presents with macrocytic anemia and fatigue. Nutritional deficiency as a cause of her anemia has been ruled out clinically. Bone marrow biopsy shows a normocellular marrow. Aspirate smear shows the findings seen above. On chromosomal analysis, the patient is found to have isolated del(5q).

Which of the following is true about this disease?

  1. Approximately 40% of patients progress to AML
  2. Monosomy 7 is commonly seen in this entity
  3. SF3B1 mutation excludes the diagnosis
  4. TP53 mutation is associated with shorter overall survival
Board review style answer #1
D. TP53 mutation is associated with shorter overall survival

Comment Here

Reference: Isolated del(5q)

with hyperdiploidy
Definition / general
Microscopic (histologic) description
  • No distinct morphology
Molecular / cytogenetics description
  • Mutations in NRAS (10%), FLT3 (9%), PTPN11 (9%) and KRAS (6%, Genes Chromosomes Cancer 2008;47:26)
  • Most common extra copies are 21, X, 14 and 4; least often 1, 2, 3
  • Best prognosis of simultaneous trisomies: 4, 10 and 17

with hypodiploidy
Definition / general
  • Defined as fewer than 45 or 46 chromosomes
  • 5% of B ALL
  • 1% in both children and adults
  • Most have 45 chromosomes, fewer chromosomes is rare
Prognostic factors
  • Poorer prognosis: near haploid and low hypodiploid groups compared to those with:
  • Poorest prognosis: near haploid 23 - 29 chromosomes which appear limited to childhood
  • Overall 50% survival at 8 years
Microscopic (histologic) description
  • No distinct morphology
Molecular / cytogenetics images

Images hosted on other servers:
Missing Image

Karyotype of severe hypodiploidy


with iAMP21
Definition / general
  • B lymphoblastic leukemia / lymphoma (B ALL / LBL) with intrachromosomal amplification of RUNX1 gene at structurally abnormal chromosome 21 (iAMP21)
Essential features
  • B ALL / LBL with iAMP21 is characterized by amplification, which is defined as ≥ 5 copies of RUNX1 per cell and ≥ 3 copies of RUNX1 on a single abnormal chromosome 21 detected by fluorescence in situ hybridization (FISH) study
  • Typically occurs in older children with a median age of 9 years
  • Pediatric iAMP21 B ALL has a poor outcome with an increased rate of relapse when treated on standard treatment protocols (Hematol Rep 2019;11:7826)
Terminology
  • B ALL / LBL with iAMP21
ICD coding
  • ICD-O: 9811/3 - B lymphoblastic leukemia / lymphoma with iAMP21
  • ICD-11: 2A70.Y & XH0KD4 - other B lymphoblastic leukemia / lymphoma with recurrent genetic abnormalities & B lymphoblastic leukemia / lymphoma with iAMP21
Epidemiology
Sites
  • Peripheral blood and bone marrow are involved
Pathophysiology
  • iAMP21 is considered to be formed through breakage fusion bridge cycles followed by chromothripsis and other complex structural rearrangements of chromosome 21 (Blood 2015;125:1383)
  • Genetic studies demonstrated a common 5.1 Mb region that included RUNX1, which is part of the critical region consistently amplified (chr21:32.8 to 37.9 Mb, GRCh37/hg19) (Blood 2011;117:6848)
  • Although RUNX1 is present in the amplified region, there is no conclusive evidence which points towards RUNX1 as a critical event in the pathogenesis of the disease given that it is not overexpressed (J Clin Oncol 2012;30:1663, Blood 2011;117:6848)
Etiology
  • Higher risk of B ALL / LBL with iAMP21 occurrence is reported in patients with a constitutional Robertsonian translocation rob(15;21)(q10;q10) and presence of a constitutional ring chromosome 21, r(21)c (Eur J Med Genet 2016;59:162, Nature 2014;508:98)
Clinical features
Diagnosis
  • Presence of blasts of B cell lineage differentiation
  • FISH studies show ≥ 5 copies of RUNX1 per cell with ≥ 3 copies on a single abnormal chromosome 21
Laboratory
  • CBC frequently shows leukopenia or thrombocytopenia
Prognostic factors
  • Risk stratification includes older age at the time of diagnosis, initial white blood cell (WBC) count, associated genetic aberrations and minimal residual disease (MRD) (Hematol Rep 2019;11:7826, J Clin Oncol 2013;31:3397)
  • Older age (≥ 10 years), WBC count (< 50 K/uL), female sex and detectable MRD at day 29 are considered as poor prognostic factors (J Clin Oncol 2013;31:3397)
  • Deletion of SH2B3 is enriched in iAMP21 cases and confers a higher risk of relapse or death (Cancer Genet 2018:226:30, Leukemia 2019;33:1881)
  • Pediatric iAMP21 B ALL has a poor outcome with an increased rate of relapse when treated on standard protocols (Hematol Rep 2019;11:7826)
  • 4 year event free survival (EFS) and overall survival (OS) have been described as significantly worse for patients with iAMP21 with standard risk B ALL (SR BALL) (J Clin Oncol 2013;31:3397)
Case reports
  • 4 year old girl with Down syndrome who developed B ALL with iAMP21 due to a constitutional isodicentric chromosome 21 (Am J Med Genet A 2022;188:2325)
  • 9 year old girl with B ALL with iAMP21 development with constitutional ring chromosome 21, r(21)c (Cytogenet Genome Res 2022;162:231)
  • 12 year old girl with homozygous deletion of the SH2B3 gene, chromothripsis of chromosome 21 and a non-Robertsonian somatic t(15;21)(q25.3;q22.1) with NTRK3 gene rearrangement in iAMP21 B ALL (Cancer Genet 2022:262:16)
Treatment
Microscopic (histologic) description
  • Bone marrow core biopsy is usually hypercellular with extensive marrow replacement by sheets of atypical medium sized blasts, with round to oval nuclear contour, fine chromatin and pinpoint nucleoli
  • Lymph nodes show a diffuse or less commonly paracortical blast infiltrate with increased mitotic figures / apoptotic bodies and sometimes interspersed macrophages (starry sky pattern)
Microscopic (histologic) images

Contributed by Barina Aqil, M.D.
Increased blasts Increased blasts

Increased blasts

Atypical mononuclear cells

Atypical mononuclear cells

Blast morphology

Blast morphology


CD34+ blasts

CD34+ blasts

TDT positivity

TDT positivity

CD79a+ blasts

CD79a+ blasts

Cytology description
  • Aspirate smears and touch preparations show small to medium sized blasts with high N:C ratio, fine nuclear chromatin and prominent nucleoli
  • Some of the blasts show cytoplasmic vacuoles and rarely cytoplasmic granules
Peripheral smear description
  • Circulating blasts are seen, which are small to medium sized cells with high N:C ratio, fine nuclear chromatin and prominent nucleoli
Peripheral smear images

Contributed by Barina Aqil, M.D.
Circulating blasts

Circulating blasts

Positive stains
Negative stains
Flow cytometry description
Flow cytometry images

Contributed by Barina Aqil, M.D.
Blast phenotype

Blast phenotype

Molecular / cytogenetics description
  • FISH ETV6::RUNX1 fusion probe set is most commonly used to identify iAMP21
  • Conventional chromosome analysis further identifies the abnormalities of chromosome 21 along with additional chromosomal aberrations (Blood 2015;125:1383)
  • Chromosomal microarray (CMA) or next generation sequencing (NGS) based copy number analysis are being utilized for the identification of iAMP21 (Blood 2015;125:1383)
  • Secondary genomic abnormalities that have been reported include gain of chromosomes X, 10 and 14, loss of chromosomes 7/7q and 11q (including KMT2A) and deletions involving IKZF1, CDKN2A/B, PAX5, SH2B3, ETV6 and RB1 as well as rearrangements of P2RY8::CRLF2, ETV6:RUNX1 and BCR::ABL1 (Blood 2011;117:2129, Blood 2011;117:6848, Cancer Genet 2018:226:30, Mol Cytogenet 2016:9:84, Leukemia 2019;33:1881, Leukemia 2014;28:1015)
  • Given the unique nature and poor prognosis of the B ALL with iAMP21, cases with genomic lesions that may suggest different subcategory of B ALL / LBL classification should still be included as B ALL / LBL with iAMP21 (Semin Hematol 2019;56:90)
  • RAS signaling pathway mutations, including NRAS (45%), KRAS (18%), FLT3 (20%), PTPN11 (11%), BRAF (2%) and NF1 (2%), have been frequently described in cases with iAMP21 (Leukemia 2016;30:1824)
Molecular / cytogenetics images

Contributed by Barina Aqil, M.D. and Madina Sukhanova, Ph.D.
Complex karyotype and derivative chromosome 21

Complex karyotype and derivative chromosome 21

Sample pathology report
  • Peripheral blood, bone marrow aspirate and left posterior iliac crest bone marrow core biopsy:
    • B lymphoblastic leukemia with iAMP21 involving ~90% of a hypercellular (90% cellular) bone marrow (see comment)
    • Comment: The cytogenetic analysis showed complex karyotype including an ider(21) resulting in an amplification of the RUNX1 gene, which was confirmed by the concurrent FISH analysis. FISH study was positive for amplification of RUNX1 (84%).
    • Peripheral blood smear: The blood smear shows normocytic anemia with anisopoikilocytosis including ovalocytes, teardrop forms and rare schistocytes. Rare blasts are present, which are small to intermediate sized cells with round to cleaved nuclei, open chromatin, occasional small nucleoli and scant cytoplasm. There is absolute neutropenia. Platelets are decreased with unremarkable morphology.
    • Bone marrow aspirate: The bone marrow aspirate smear shows sheets of blasts with similar morphologic features as described above. Few scattered megakaryocytes and rare erythroid precursors are seen.
    • Bone marrow core biopsy (decalcified): The unilateral bone marrow core biopsy is hypercellular for age (~90% cellular). There is extensive replacement by sheets of atypical medium sized mononuclear cells, with round to irregular nuclear contour, fine chromatin and pinpoint nucleoli. Background hematopoiesis is markedly decreased. Few scattered megakaryocytes are present.
Differential diagnosis
Board review style question #1

Which of the following cytogenetic abnormalities leads to higher risk of development of AML with iAMP21?

  1. t(9;11)
  2. t(9;22)
  3. t(15;17)
  4. t(15;21)
Board review style answer #1
D. t(15;21). Robertsonian translocation between chromosomes 15 and 21, rob(15;21)(q10;q10)c, have a > 2,700 fold increased risk of developing iAMP21 ALL. Answers A and B are incorrect because they are different subcategories of B ALL, which are classified based on t(9;11) / KMT2A rearrangement and t(9;22) / BCR:ABL1 fusion. Answer C is incorrect because it is seen in acute promyelocytic leukemia, which is associated with t(15;17) / PML::RARA fusion.

Comment Here

Reference: with iAMP21
Board review style question #2
Which FISH probe set is utilized for diagnosing AML with iAMP21?

  1. 5' CRLF2 and 3' CRLF2 (Xp22.3 and Yp11.3), break apart
  2. 5' PDGFRB and 3' PDGFRB, break apart (5q33)
  3. ABL1 (9q34) and BCR (22q11.2), fusion
  4. ETV6 (12p13.2) and RUNX1 (21q22.3)
Board review style answer #2
D. ETV6 (12p13.2) and RUNX1 (21q22.3) shows amplification of RUNX1. Answer A is incorrect because CRLF2 break apart probe is used to identify BCR::ABL1-like B ALL. Answer C is incorrect because it is used to identify B ALL, BCR::ABL1 positive cases. Answer B is incorrect because PDGFRB rearrangement positivity helps to identify myeloid / lymphoid neoplasm with PDGFRB rearrangement.

Comment Here

Reference: with iAMP21
Back to top
Recent Bone marrow neoplastic Pathology books

Beck: 2021

Cherry: 2022

Dorfman: 2023

Hsi: 2017

Hudnall: 2019

Husain: 2021

IARC: 2017

Jaffe: 2016

Kantarjian: 2016

Lanzkowsky: 2016

Naeim: 2018

Wang: 2020



Find related Pathology books: hematopathology, immunology / transplant, lab medicine, oncology, pediatric
Image 01 Image 02